GMAT Official Advanced Questions

Download as pdf or txt
Download as pdf or txt
You are on page 1of 407
At a glance
Powered by AI
The document provides information about practice questions for the GMAT exam across quantitative and verbal reasoning. It includes hundreds of questions organized by topic as well as explanations for the answers.

The document consists of an introduction outlining what the GMAT Official Advanced Questions is and how it should be used, followed by sample practice questions and answers across multiple pages with page numbers listed at the top.

Information provided about the GMAT includes what it tests, who should use the practice material, and how the practice material should be used to prepare for the actual exam. Sample questions are also included to familiarize test takers with the format and difficulty of the real exam.

GMAT™ Official Advanced Questions

Copyright © 2020 by the Graduate Management Admission Council™. All rights reserved.
Published by John Wiley & Sons, Inc., Hoboken, New Jersey.
No part of this publication may be reproduced, stored in a retrieval system, or transmitted in any form or by any means,
electronic, mechanical, photocopying, recording, scanning, or otherwise, except as permitted under Section 107 or 108 of the
1976 United States Copyright Act, without either the prior written permission of the Publisher, or authorization through
payment of the appropriate percopy fee to the Copyright Clearance Center, Inc., 222 Rosewood Drive, Danvers, MA 01923,
(978) 750–8400, fax (978) 646–8600, or on the Web at www.copyright.com. Requests to the Publisher for permission
should be addressed to the Permissions Department, John Wiley & Sons, Inc., 111 River Street, Hoboken, NJ 07030, (201)
748–6011, fax (201) 748–6008, or online at http://www.wiley.com/go/permissions.
Limit of Liability/Disclaimer of Warranty: While the publisher and author have used their best efforts in preparing this book,
they make no representations or warranties with respect to the accuracy or completeness of the contents of this book and
specifically disclaim any implied warranties of merchantability or fitness for a particular purpose. No warranty may be
created or extended by sales representatives or written sales materials. The advice and strategies contained herein may not
be suitable for your situation. You should consult with a professional where appropriate. Neither the publisher nor author
shall be liable for any loss of profit or any other commercial damages, including but not limited to special, incidental,
consequential, or other damages.
Trademarks: Wiley, the Wiley Publishing logo, and related trademarks are trademarks or registered trademarks of John
Wiley & Sons, Inc. and/or its affiliates. The GMAT logo, GMAC™, GMASS™, GMAT™, Graduate Management Admission
Council™, and Graduate Management Admission Test™ are trademarks of GMAC in the United States and other countries.
All other trademarks are the property of their respective owners. Wiley Publishing, Inc. is not associated with any product or
vendor mentioned in this book.
For general information on our other products and services or for technical support, please contact our Customer Care
Department within the United States at (800) 762–2974, outside the United States at (317) 572–3993 or fax (317) 572–
4002.
Wiley publishes in a variety of print and electronic formats and by print-on-demand. Some material included with standard
print versions of this book may not be included in e-books or in print-on-demand. If this book refers to media such as a CD
or DVD that is not included in the version you purchased, you may download this material at http://booksupport.wiley.com.
For more information about Wiley products, visit www.wiley.com.
ISBN 978-1-119-62095-2 (pbk); ISBN 978-1-119-62097-6 (ePUB)
CONTENTS
Cover
Chapter 1 Introduction
I. What is the GMAT™ Official Advanced Questions?
II. Who should use the GMAT™ Official Advanced Questions?
III. How should I use the GMAT™ Official Advanced Questions?
Chapter 2 Quantitative Reasoning
Practice Questions Quantitative Reasoning
Answer Key Quantitative Reasoning
Answer Explanations Quantitative Reasoning
Problem Solving
Data Sufficiency
Chapter 3 Verbal Reasoning
Practice Questions Verbal Reasoning
Answer Key Verbal Reasoning
Answer Explanations Verbal Reasoning
Reading Comprehension
Critical Reasoning
Sentence Correction
Appendix Answer Sheet
GMAT™ Official Advanced Questions Online Index
End User License Agreement
Chapter 1 Introduction
Introduction
I. What is the GMAT™ Official Advanced Questions?
The GMAT™ Official Advanced Questions is a compilation of 300 hard Quantitative Reasoning and
Verbal Reasoning questions, similar in difficulty level to hard questions found in the GMAT Official
Guide series. Some of these questions have appeared in earlier editions of the GMAT Official Guide
series; others have never been published before.
Like all GMAT questions, hard GMAT questions test higher-order reasoning skills identified as key
factors of success in classrooms and job performances by educators and employers. They are unlike
questions on other types of achievement-based exams, which focus more on mastery of mechanical
cognitive skills and domain-specific knowledge. To solve GMAT questions, a test taker needs to
demonstrate higher-order reasoning skills by understanding the underlying relationships between
multiple concepts or arguments, identifying patterns, synthesizing information presented in various
formats, and/or evaluating relevant information from different sources. What makes some GMAT
questions harder than others is the level of complexity: hard GMAT questions typically involve
multiple reasoning tasks or concepts and require more than one step to solve. They do not require
mastery of skills and knowledge such as advanced-level mathematics, extensive calculations, or a
large vocabulary in order to be solved.
II. Who should use the GMAT™ Official Advanced Questions?
The GMAT™ Official Advanced Questions is designed for test takers who have studied the questions
in the GMAT Official Guide series and desire additional practice with hard questions.
For a simulation of the real exam experience, there are six GMAT Official Practice Exams—the first
two are free to all mba.com registrants—which are available on mba.com and feature questions from
past GMAT exams and use the same scoring algorithm as the actual exam.
III. How should I use the GMAT™ Official Advanced Questions?
Use the GMAT™ Official Advanced Questions for additional practice. The questions are organized by
fundamental skills tested on the GMAT exam to better help you locate the specific skill type that you
may want to improve. The fundamental skills are the same ones listed in the GMAT™ Enhanced
Score Report, a non-official score report that provides more detailed information about your
performance on the actual GMAT exam, which include:
Quantitative Reasoning
Geometry—3-dimensional geometry; angles in the plane/lines and segments; circle/area;
combo: circle/quadrangle/area/perimeter; coordinate geometry; polygon (5 or more sides);
Pythagorean Theorem; quadrilateral area
Rates/Ratio/Percent—conversion of fractions/decimals/percentages; graduated rate; percent
(basic); ratio, proportion
Value/Order/Factors—absolute value; computation-decimals; exponents; factors, multiples,
divisibles; number line and order; place value; positive and negative numbers; remainders
Equalities/Inequalities/Algebra—algebraic manipulation; applying formula; linear equation;
linear inequality; measurement conversion; newly defined functions; quadratic/other
equalities/inequalities; systems of equations/inequalities; translation to algebraic expression
Counting/Sets/Series—counting (combinatorics); estimation; series and sequences; sets
Verbal Reasoning
Critical Reasoning Analysis/Critique—ability to analyze given information and make reasoned
judgments by evaluating and breaking down an argument
Critical Reasoning Construction/Plan—ability to construct a plan of action to find the best
logical solution
Reading Comprehension Identify Inferred Idea—ability to comprehend what a passage implies
but does not explicitly mention
Reading Comprehension Identify Stated Idea—ability to recognize and understand the ideas
explicitly expressed in a passage
Sentence Correction Grammar—ability to identify the answer that will create a sentence which
is grammatically and structurally sound
Sentence Correction Communication—ability to identify the answer that will create an effective
sentence which expresses an idea or relationship clearly and concisely, as well as grammatically
For more information about the Enhanced Score Report and a description of each fundamental skill
area, go to www.mba.com/esr-demo. Note: The ratio of questions across different question types
(e.g., data sufficiency, problem solving, etc.) and fundamental skills in GMAT™ Official Advanced
Questions do not reflect the ratio of questions on the actual GMAT exam.
Spend time reviewing the answer explanations and understanding the methods discussed. Like those
in the GMAT Official Guide series, the answer explanations are written by GMAT subject matter
experts with decades of experience writing and editing GMAT questions. Review these explanations
carefully to understand the solution and identify techniques or approaches which may be applied to
other GMAT questions.
Lastly, go online to gmat.wiley.com and access the Online Question Bank. The GMAT exam is
administered on a computer, and it may be beneficial for you to practice answering on a computer.
The Online Question Bank contains all of the questions that are in this book and allows you to create
customized practice sets, use the timer, track your progress, and view performance metrics. You can
also study on-the-go by accessing the Online Question Bank through the Wiley Efficient Learning
mobile app, available in both the Apple App Store and the Google Play Store.
Chapter 2 Quantitative Reasoning

Practice Questions Quantitative Reasoning


Problem Solving
Counting/Sets/Series
PS54110.01

1. The letters C, I, R, C, L, and E can be used to form 6-letter strings such as CIRCLE or
CCIRLE. Using these letters, how many different 6-letter strings can be formed in which the
two occurrences of the letter C are separated by at least one other letter?
A. 96
B. 120
C. 144
D. 180
E. 240

PS24831.01

2. The map above shows the trails through a wilderness area. If travel is in the direction of the
arrows, how many routes along the marked trails are possible from point A to point B ?
A. 11
B. 18
C. 54
D. 108
E. 432
PS61551.01

3. In the figure above, X and Y represent locations in a district of a certain city where the streets
form a rectangular grid. In traveling only north or east along the streets from X to Y, how
many different paths are possible?
A. 720
B. 512
C. 336
D. 256
E. 56

PS92751.01

4. The figures above show a hexagonal nut that has a width of inches and a wrench that, in
order to fit the nut, must have a width of at least inches. Of all the wrenches that fit the
nut and have widths that are whole numbers of millimeters, the wrench that fits the nut most
closely has a width of how many millimeters?
(Note: 1 inch ≈ 25.4 millimeters)
A. 30
B. 31
C. 32
D. 33
E. 34
PS45461.01

5. Pat will walk from intersection X to intersection Y along a route that is confined to the square
grid of four streets and three avenues shown in the map above. How many routes from X to Y
can Pat take that have the minimum possible length?
A. Six
B. Eight
C. Ten
D. Fourteen
E. Sixteen
PS95302.01

6. Rita and Sam play the following game with n sticks on a table. Each must remove 1, 2, 3, 4 or
5 sticks at a time on alternate turns, and no stick that is removed is put back on the table. The
one who removes the last stick (or sticks) from the table wins. If Rita goes first, which of the
following is a value of n such that Sam can always win no matter how Rita plays?
A. 7
B. 10
C. 11
D. 12
E. 16
PS65402.01

7. When of the votes on a certain resolution have been counted, of those counted are in
favor of the resolution. What fraction of the remaining votes must be against the resolution
so that the total count will result in a vote of 2 to 1 against the resolution?
A.
B.

C.

D.

E.
PS85402.01

8. The sum of the first 100 positive integers is 5,050. What is the sum of the first 200 positive
integers?
A. 10,100
B. 10,200
C. 15,050
D. 20,050
E. 20,100

Month Average Price per Dozen


April $1.26
May $1.20
June $1.08
PS40502.01

9. The table above shows the average (arithmetic mean) price per dozen eggs sold in a certain
store during three successive months. If as many dozen were sold in April as in May, and
twice as many were sold in June as in April, what was the average price per dozen of the eggs
sold over the three-month period?
A. $1.08
B. $1.10
C. $1.14
D. $1.16
E. $1.18
PS96602.01

10. Each of the integers from 0 to 9, inclusive, is written on a separate slip of blank paper and
the ten slips are dropped into a hat. If the slips are then drawn one at a time without
replacement, how many must be drawn to ensure that the numbers on two of the slips drawn
will have a sum of 10 ?
A. Three
B. Four
C. Five
D. Six
E. Seven
PS15402.01

11. King School has an enrollment of 900 students. The school day consists of 6 class periods
during which each class is taught by one teacher. There are 30 students per class. Each
teacher teaches a class during 5 of the 6 class periods and has one class period free. No
students have a free class period. How many teachers does the school have?
A. 25
B. 30
C. 36
D. 60
E. 150
PS07602.01

12. Ben and Ann are among 7 contestants from which 4 semifinalists are to be selected. Of the
different possible selections, how many contain neither Ben nor Ann?
A. 5
B. 6
C. 7
D. 14
E. 21
Equalities/Inequalities/Algebra

PS03551.01

13. Let n and k be positive integers with k ≤ n. From an n × n array of dots, a k × k array of dots
is selected. The figure above shows two examples where the selected k × k array is enclosed in
a square. How many pairs (n, k) are possible so that exactly 48 of the dots in the n × n array
are NOT in the selected k × k array?
A. 1
B. 2
C. 3
D. 4
E. 5
PS41471.01

14. If there is a least integer that satisfies the inequality ≥ 2, what is that least integer?

A. 0
B. 1
C. 4
D. 5
E. There is not a least integer that satisfies the inequality.

x C(x)
0 25,000
10 24,919
20 24,846
30 24,781
40 24,724
50 24,675
PS18871.01

15. A certain manufacturer uses the function C(x) = 0.04x2 – 8.5x + 25,000 to calculate the cost,
in dollars, of producing x thousand units of its product. The table above gives values of this
cost function for values of x between 0 and 50 in increments of 10. For which of the following
intervals is the average rate of decrease in cost less than the average rate of decrease in cost
for each of the other intervals?
A. From x = 0 to x = 10
B. From x = 10 to x = 20
C. From x = 20 to x = 30
D. From x = 30 to x = 40
E. From x = 40 to x = 50
PS35302.01

16. On the day of the performance of a certain play, each ticket that regularly sells for less than
$10.00 is sold for half price plus $0.50, and each ticket that regularly sells for $10.00 or
more is sold for half price plus $1.00. On the day of the performance, a person purchases a
total of y tickets, of which x regularly sell for $9.00 each and the rest regularly sell for $12.00
each. What is the amount paid, in dollars, for the y tickets?
A. 7y – 2x
B. 12x – 7y

C.

D. 7y + 4x
E. 7y + 5x
PS47302.01

17. If N = , where T = and x = 5 – T, which of the following expresses y in terms of N and

T?

A.

B.

C.

D.

E.
PS78302.01

18. If 2x + 5y = 8 and 3x = 2y, what is the value of 2x + y ?


A. 4

B.

C.

D.

E.
PS79302.01

19. If kSn is defined to be the product of (n + k)(n – k + 1) for all positive integers k and n, which
of the following expressions represents k + 1Sn + 1 ?
A. (n + k)(n – k + 2)
B. (n + k)(n – k + 3)
C. (n + k + 1)(n – k + 2)
D. (n + k + 2)(n – k + 1)
E. (n + k + 2)(n – k + 3)
PS20502.01

20. There were 36,000 hardback copies of a certain novel sold before the paperback version was
issued. From the time the first paperback copy was sold until the last copy of the novel was
sold, 9 times as many paperback copies as hardback copies were sold. If a total of 441,000
copies of the novel were sold in all, how many paperback copies were sold?
A. 45,000
B. 360,000
C. 364,500
D. 392,000
E. 396,900
PS30502.01

21. In the formula w = , integers p and t are positive constants. If w = 2 when v = 1 and if w =

when v = 64, then t =

A. 1
B. 2
C. 3
D. 4
E. 16

PS03502.01

22. The figure above represents a network of one-way streets. The arrows indicate the direction
of traffic flow, and the numbers indicate the amount of traffic flow into or out of each of the
four intersections during a certain hour. During that hour, what was the amount of traffic
flow along the street from R to S if the total amount of traffic flow into P was 1,200 ? (Assume
that none of the traffic originates or terminates in the network.)
A. 200
B. 250
C. 300
D. 350
E. 400
PS23502.01

23. If C is the temperature in degrees Celsius and F is the temperature in degrees Fahrenheit,
then the relationship between temperatures on the two scales is expressed by the equation
9C = 5(F – 32). On a day when the temperature extremes recorded at a certain weather
station differed by 45 degrees on the Fahrenheit scale, by how many degrees did the
temperature extremes differ on the Celsius scale?

A.

B. 13
C. 25
D. 45
E. 81
PS93502.01

24. If d = , a = , and b = , what is the value of d in terms of c ?

A.

B.

C.

D.

E.
PS04502.01

25. A school supply store sells only one kind of desk and one kind of chair, at a uniform cost per
desk or per chair. If the total cost of 3 desks and 1 chair is twice that of 1 desk and 3 chairs,
then the total cost of 4 desks and 1 chair is how many times that of 1 desk and 4 chairs?
A. 5
B. 3
C.

D.

E.
PS35502.01

26. A certain truck traveling at 55 miles per hour gets 4.5 miles per gallon of diesel fuel
consumed. Traveling at 60 miles per hour, the truck gets only 3.5 miles per gallon. On a 500-
mile trip, if the truck used a total of 120 gallons of diesel fuel and traveled part of the trip at
55 miles per hour and the rest at 60 miles per hour, how many miles did it travel at 55 miles
per hour?
A. 140
B. 200
C. 250
D. 300
E. 360
PS45502.01

27. A merchant paid $300 for a shipment of x identical calculators. The merchant used two of
the calculators as demonstrators and sold each of the others for $5 more than the average
(arithmetic mean) cost of the x calculators. If the total revenue from the sale of the
calculators was $120 more than the cost of the shipment, how many calculators were in the
shipment?
A. 24
B. 25
C. 26
D. 28
E. 30
PS06502.01

28. A car traveled 462 miles per tankful of gasoline on the highway and 336 miles per tankful of
gasoline in the city. If the car traveled 6 fewer miles per gallon in the city than on the
highway, how many miles per gallon did the car travel in the city?
A. 14
B. 16
C. 21
D. 22
E. 27
PS56502.01

29. Machines X and Y run at different constant rates, and machine X can complete a certain job
in 9 hours. Machine X worked on the job alone for the first 3 hours and the two machines,
working together, then completed the job in 4 more hours. How many hours would it have
taken machine Y, working alone, to complete the entire job?
A. 18
B. 13

C. 7

D. 4

E. 3
PS77602.01
30. If , then the value of which of the

following can be determined?

A. I only
B. III only
C. I and II only
D. II and III only
E. I, II, and III
PS58602.01

31. If k ≠ 0 and k – , then x =

A. –3 – k2
B. k2 – 3
C. 3k2 – 3
D. k – 3 – 2k2
E. k – 3 + 2k2
PS68602.01

32. The sum of the ages of Doris and Fred is y years. If Doris is 12 years older than Fred, how
many years old will Fred be y years from now, in terms of y ?
A. y–6
B. 2y – 6
C. –6

D. –6

E. –6

Geometry
PS35461.01

33. The shaded region in the figure above represents a rectangular frame with length 18 inches
and width 15 inches. The frame encloses a rectangular picture that has the same area as the
frame itself. If the length and width of the picture have the same ratio as the length and width
of the frame, what is the length of the picture, in inches?
A. 9

B.

C.

D. 15(1 − )

E.
PS56271.01

34. The surface distance between 2 points on the surface of a cube is the length of the shortest
path on the surface of the cube that joins the 2 points. If a cube has edges of length 4
centimeters, what is the surface distance, in centimeters, between the lower left vertex on its
front face and the upper right vertex on its back face?
A. 8
B. 4
C. 8
D. 12
E. 4 +4
PS75571.01

35. The figure above shows 2 circles. The larger circle has center A, radius R cm, and is inscribed
in a square. The smaller circle has center C, radius r cm, and is tangent to the larger circle at
point B and to the square at points D and F. If points A, B, C, and E are collinear, which of
the following is equal to ?

A.

B.

C.

D.

E.

PS15302.01

36. The figure above shows the dimensions of a rectangular board that is to be cut into four
identical pieces by making cuts at points A, B, and C, as indicated. If x = 45, what is the
length AB ?
(1 foot = 12 inches)
A. 5 ft 6 in
B. 5 ft in
C. 5 ft 3 in
D. 5 ft
E. 4 ft 9 in

PS57302.01

37. In the figure above, the area of the shaded region is


A.
B.
C.
D.
E.

PS18302.01

38. If each side of ΔACD above has length 3 and if AB has length 1, what is the area of region
BCDE ?
A.

B.

C.

D.

E.

PS76402.01

39. In the figure above, three squares and a triangle have areas of A, B, C, and X as shown. If
A = 144, B = 81, and C = 225, then X =
A. 150
B. 144
C. 80
D. 54
E. 36
PS57402.01

40. In the figure above, PQ is a diameter of circle O, PR = SQ, and ΔRST is equilateral. If the
length of PQ is 2, what is the length of RT ?

A.

B.

C.

D.

E.

PS22502.01

41. The figure above shows some of the dimensions of a triangular plaza with an L-shaped walk
along two of its edges. If the width of the walk is 4 feet and the total area of the plaza and
walk together is 10,800 square feet, what is the value of x ?
A. 200
B. 204
C. 212
D. 216
E. 225
PS88602.01
42. A circular rim 28 inches in diameter rotates the same number of inches per second as a
circular rim 35 inches in diameter. If the smaller rim makes x revolutions per second, how
many revolutions per minute does the larger rim make in terms of x ?
A.

B. 75x
C. 48x
D. 24x
E.

Rates/Ratios/Percent
PS17302.01

43. The annual stockholders' report for Corporation X stated that profits were up 10 percent
over the previous year, although profits as a percent of sales were down 10 percent. Total
sales for that year were approximately what percent of sales for the previous year?
A. 78%
B. 90%
C. 110%
D. 122%
E. 190%
PS47402.01

44. A certain brand of house paint must be purchased either in quarts at $12 each or in gallons at
$18 each. A painter needs a 3-gallon mixture of the paint consisting of 3 parts blue and 2
parts white. What is the least amount of money needed to purchase sufficient quantities of
the two colors to make the mixture?
(4 quarts = 1 gallon)
A. $54
B. $60
C. $66
D. $90
E. $144

Month Change in sales from previous month


February +10%
March −15%
April +20%
May −10%
June +5%
PS43481.01
45. The table above shows the percent of change from the previous month in Company X's sales
for February through June of last year. A positive percent indicates that Company X's sales
for that month increased from the sales for the previous month, and a negative percent
indicates that Company X's sales for that month decreased from the sales for the previous
month. For which month were the sales closest to the sales in January?
A. February
B. March
C. April
D. May
E. June
PS56302.01

46. A store bought 5 dozen lamps at $30 per dozen and sold them all at $15 per lamp. The profit
on each lamp was what percent of its selling price?
A. 20%
B. 50%

C. 83 %

D. 100%
E. 500%
PS76302.01

47. Store N gives a 50 percent discount on the list price of all its items and Store W gives a 60
percent discount on the list price of all its items. If the list price of the same item is 20
percent higher in Store W, what percent (more or less) of the selling price in Store N is the
selling price of the item in Store W ?
A. 10% less
B. 4% less
C. 2% less
D. 10% more
E. 12% more
PS95402.01

48. A merchant purchased a jacket for $60 and then determined a selling price that equaled the
purchase price of the jacket plus a markup that was 25 percent of the selling price. During a
sale, the merchant discounted the selling price by 20 percent and sold the jacket. What was
the merchant's gross profit on this sale?
A. $0
B. $3
C. $4
D. $12
E. $15
PS12502.01

49. When a certain stretch of highway was rebuilt and straightened, the distance along the
stretch was decreased by 20 percent and the speed limit was increased by 25 percent. By
what percent was the driving time along this stretch reduced for a person who always drives
at the speed limit?
A. 16%
B. 36%
C. 37 %

D. 45%
E. 56 %

Value/Order/Factors
Number of components:
Components Monday Tuesday Wednesday
A 3 6 3
B 6 3 4
C 4 7 4
PS56441.01

50. A factory assembles Product X from three components, A, B, and C. One of each component
is needed for each Product X and all three components must be available when assembly of
each Product X starts. It takes two days to assemble one Product X. Assembly of each
Product X starts at the beginning of one day and is finished at the end of the next day. The
factory can work on at most five Product Xs at once. If components are available each day as
shown in the table above, what is the largest number of Product Xs that can be assembled
during the three days covered by the table?
A. 3
B. 5
C. 6
D. 9
E. 10
PS04851.01

51. How many positive integers n have the property that both 3n and are 4-digit integers?

A. 111
B. 112
C. 333
D. 334
E. 1,134
PS24851.01

52. If Whitney wrote the decimal representations for the first 300 positive integer multiples of 5
and did not write any other numbers, how many times would she have written the digit 5 ?
A. 150
B. 185
C. 186
D. 200
E. 201
PS01661.01

53. The difference 942 − 249 is a positive multiple of 7. If a, b, and c are nonzero digits, how
many 3-digit numbers abc are possible such that the difference abc − cba is a positive
multiple of 7 ?
A. 142
B. 71
C. 99
D. 20
E. 18
PS41661.01

54. Let S be the set of all positive integers having at most 4 digits and such that each of the digits
is 0 or 1. What is the greatest prime factor of the sum of all the numbers in S ?
A. 11
B. 19
C. 37
D. 59
E. 101

Age Tax only Tax and fees Fees only


18–39 20 30 30
≥40 10 60 100
PS43661.01

55. The table above shows the number of residents in each of two age groups who support the
use of each type of funding for a city initiative. What is the probability that a person
randomly selected from among the 250 residents polled is younger than 40, or supports a
type of funding that includes a tax, or both?

A.

B.

C.
D.

E.
PS55471.01

56. Which of the following describes the set of all possible values of the positive integer k such
that, for each positive odd integer n, the value of is midway between consecutive integers?

A. All positive integers greater than 2


B. All prime numbers
C. All positive even integers
D. All even prime numbers
E. All positive even multiples of 5
PS92981.01

57. A certain online form requires a 2-digit code for the day of the month to be entered into one
of its fields, such as 04 for the 4th day of the month. The code is valid if it is 01, 02, 03, …, 31
and not valid otherwise. The transpose of a code xy is yx. For example, 40 is the transpose of
04. If N is the number of valid codes having a transpose that is not valid, what is the value of
N?
A. 12
B. 13
C. 18
D. 19
E. 20
PS25302.01

58. If x < y < z and y − x > 5, where x is an even integer and y and z are odd integers, what is the
least possible value of z − x ?
A. 6
B. 7
C. 8
D. 9
E. 10
PS36302.01

59. An “Armstrong number” is an n-digit number that is equal to the sum of the nth powers of its
individual digits. For example, 153 is an Armstrong number because it has 3 digits and
13 + 53 + 33 = 153. What is the digit k in the Armstrong number 1,6k 4 ?
A. 2
B. 3
C. 4
D. 5
E. 6
PS30402.01

60. Five integers between 10 and 99, inclusive, are to be formed by using each of the ten digits
exactly once in such a way that the sum of the five integers is as small as possible. What is the
greatest possible integer that could be among these five numbers?
A. 98
B. 91
C. 59
D. 50
E. 37
PS66402.01

61. When the integer n is divided by 17, the quotient is x and the remainder is 5. When n is
divided by 23, the quotient is y and the remainder is 14. Which of the following is true?
A. 23x + 17y = 19
B. 17x − 23y = 9
C. 17x + 23y = 19
D. 14x + 5y = 6
E. 5x − 14y = −6
PS17402.01

62. Of the following, which is greatest?


A.
B.

C.

D.

E.

PS37402.01

63. If n = p2 and p is a prime number greater than 5, what is the units digit of n2 ?
A. 1
B. 3
C. 4
D. 7
E. 9
PS83502.01
64. A computer can perform 1,000,000 calculations per second. At this rate, how many hours
will it take this computer to perform the 3.6 × 1011 calculations required to solve a certain
problem?
A. 60
B. 100
C. 600
D. 1,000
E. 6,000
PS66602.01

65. In an auditorium, 360 chairs are to be set up in a rectangular arrangement with x rows of
exactly y chairs each. If the only other restriction is that 10 < x < 25, how many different
rectangular arrangements are possible?
A. Four
B. Five
C. Six
D. Eight
E. Nine
PS28602.01

66. If the product of the integers w, x, y, and z is 770, and if 1 < w < x < y < z, what is the value
of w + z ?
A. 10
B. 13
C. 16
D. 18
E. 21

PS78602.01

67. The addition problem above shows four of the 24 different integers that can be formed by
using each of the digits 1, 2, 3, and 4 exactly once in each integer. What is the sum of these 24
integers?
A. 24,000
B. 26,664
C. 40,440
D. 60,000
E. 66,660

Data Sufficiency
Counting/Sets/Series
DS19350.01

68. A country's per capita national debt is its national debt divided by its population. Is the per
capita national debt of Country G within $5 of $500 ?
1. Country G's national debt to the nearest $1,000,000,000 is $43,000,000,000.
2. Country G's population to the nearest 1,000,000 is 86,000,000.
A. Statement (1) ALONE is sufficient, but statement (2) alone is not sufficient.
B. Statement (2) ALONE is sufficient, but statement (1) alone is not sufficient.
C. BOTH statements TOGETHER are sufficient, but NEITHER statement ALONE is
sufficient.
D. EACH statement ALONE is sufficient.
E. Statements (1) and (2) TOGETHER are NOT sufficient.
DS06351.01

69. The cardinality of a finite set is the number of elements in the set. What is the cardinality of
set A ?
1. 2 is the cardinality of exactly 6 subsets of set A.
2. Set A has a total of 16 subsets, including the empty set and set A itself.
A. Statement (1) ALONE is sufficient, but statement (2) alone is not sufficient.
B. Statement (2) ALONE is sufficient, but statement (1) alone is not sufficient.
C. BOTH statements TOGETHER are sufficient, but NEITHER statement ALONE is
sufficient.
D. EACH statement ALONE is sufficient.
E. Statements (1) and (2) TOGETHER are NOT sufficient.
DS59851.01

70. For each positive integer k, let a k = . Is the product a1a2 … an an integer?

1. n + 1 is a multiple of 3.
2. n is a multiple of 2.
A. Statement (1) ALONE is sufficient, but statement (2) alone is not sufficient.
B. Statement (2) ALONE is sufficient, but statement (1) alone is not sufficient.
C. BOTH statements TOGETHER are sufficient, but NEITHER statement ALONE is
sufficient.
D. EACH statement ALONE is sufficient.
E. Statements (1) and (2) TOGETHER are NOT sufficient.
DS95491.01

71. Let S be a set of outcomes and let A and B be events with outcomes in S. Let ∼B denote the
set of all outcomes in S that are not in B and let P(A) denote the probability that event A
occurs. What is the value of P(A) ?
1. P(A B) = 0.7
2. P(A ∼B) = 0.9
A. Statement (1) ALONE is sufficient, but statement (2) alone is not sufficient.
B. Statement (2) ALONE is sufficient, but statement (1) alone is not sufficient.
C. BOTH statements TOGETHER are sufficient, but NEITHER statement ALONE is
sufficient.
D. EACH statement ALONE is sufficient.
E. Statements (1) and (2) TOGETHER are NOT sufficient.
DS41402.01

72. What is the number of integers that are common to both set S and set T ?
1. The number of integers in S is 7, and the number of integers in T is 6.
2. U is the set of integers that are in S only or in T only or in both, and the number of
integers in U is 10.
A. Statement (1) ALONE is sufficient, but statement (2) alone is not sufficient.
B. Statement (2) ALONE is sufficient, but statement (1) alone is not sufficient.
C. BOTH statements TOGETHER are sufficient, but NEITHER statement ALONE is
sufficient.
D. EACH statement ALONE is sufficient.
E. Statements (1) and (2) TOGETHER are NOT sufficient.
DS51402.01

73. What is the sum of 3 consecutive integers?


1. The sum of the 3 integers is less than the greatest of the 3 integers.
2. Of the 3 integers, the ratio of the least to the greatest is 3.
A. Statement (1) ALONE is sufficient, but statement (2) alone is not sufficient.
B. Statement (2) ALONE is sufficient, but statement (1) alone is not sufficient.
C. BOTH statements TOGETHER are sufficient, but NEITHER statement ALONE is
sufficient.
D. EACH statement ALONE is sufficient.
E. Statements (1) and (2) TOGETHER are NOT sufficient.
DS54402.01

74. How many people in Town X read neither the World newspaper nor the Globe newspaper?
1. Of the 2,500 people in Town X, 1,000 read no newspaper.
2. Of the people in Town X, 700 read the Globe only and 600 read the World only.
A. Statement (1) ALONE is sufficient, but statement (2) alone is not sufficient.
B. Statement (2) ALONE is sufficient, but statement (1) alone is not sufficient.
C. BOTH statements TOGETHER are sufficient, but NEITHER statement ALONE is
sufficient.
D. EACH statement ALONE is sufficient.
E. Statements (1) and (2) TOGETHER are NOT sufficient.
DS16402.01

75. Bowls X and Y each contained exactly 2 jelly beans, each of which was either red or black.
One of the jelly beans in bowl X was exchanged with one of the jelly beans in bowl Y. After
the exchange, were both of the jelly beans in bowl X black?
1. Before the exchange, bowl X contained 2 black jelly beans.
2. After the exchange, bowl Y contained 1 jelly bean of each color.
A. Statement (1) ALONE is sufficient, but statement (2) alone is not sufficient.
B. Statement (2) ALONE is sufficient, but statement (1) alone is not sufficient.
C. BOTH statements TOGETHER are sufficient, but NEITHER statement ALONE is
sufficient.
D. EACH statement ALONE is sufficient.
E. Statements (1) and (2) TOGETHER are NOT sufficient.
DS27602.01

76. All trainees in a certain aviator training program must take both a written test and a flight
test. If 70 percent of the trainees passed the written test, and 80 percent of the trainees
passed the flight test, what percent of the trainees passed both tests?
1. 10 percent of the trainees did not pass either test.
2. 20 percent of the trainees passed only the flight test.
A. Statement (1) ALONE is sufficient, but statement (2) alone is not sufficient.
B. Statement (2) ALONE is sufficient, but statement (1) alone is not sufficient.
C. BOTH statements TOGETHER are sufficient, but NEITHER statement ALONE is
sufficient.
D. EACH statement ALONE is sufficient.
E. Statements (1) and (2) TOGETHER are NOT sufficient.
Equalities/Inequalities/Algebra
DS06110.01

77. Each of the five divisions of a certain company sent representatives to a conference. If the
numbers of representatives sent by four of the divisions were 3, 4, 5, and 5, was the range of
the numbers of representatives sent by the five divisions greater than 2 ?
1. The median of the numbers of representatives sent by the five divisions was greater
than the average (arithmetic mean) of these numbers.
2. The median of the numbers of representatives sent by the five divisions was 4.
A. Statement (1) ALONE is sufficient, but statement (2) alone is not sufficient.
B. Statement (2) ALONE is sufficient, but statement (1) alone is not sufficient.
C. BOTH statements TOGETHER are sufficient, but NEITHER statement ALONE is
sufficient.
D. EACH statement ALONE is sufficient.
E. Statements (1) and (2) TOGETHER are NOT sufficient.
DS24931.01

78. An investment has been growing at a fixed annual rate of 20% since it was first made; no
portion of the investment has been withdrawn, and all interest has been reinvested. How
much is the investment now worth?
1. The value of the investment has increased by 44% since it was first made.
2. If one year ago $600 had been withdrawn, today the investment would be worth 12%
less than it is actually now worth.
A. Statement (1) ALONE is sufficient, but statement (2) alone is not sufficient.
B. Statement (2) ALONE is sufficient, but statement (1) alone is not sufficient.
C. BOTH statements TOGETHER are sufficient, but NEITHER statement ALONE is
sufficient.
D. EACH statement ALONE is sufficient.
E. Statements (1) and (2) TOGETHER are NOT sufficient.
DS53841.01

79. X, 81, 73, 71, 98, 73, 64


What is the value of X in the above list of 7 numbers?
1. The average (arithmetic mean) of these 7 numbers is 80.
2. The range of these 7 numbers is 36.
A. Statement (1) ALONE is sufficient, but statement (2) alone is not sufficient.
B. Statement (2) ALONE is sufficient, but statement (1) alone is not sufficient.
C. BOTH statements TOGETHER are sufficient, but NEITHER statement ALONE is
sufficient.
D. EACH statement ALONE is sufficient.
E. Statements (1) and (2) TOGETHER are NOT sufficient.
DS01451.01

80. In the first 2 hours after Meadow's self-service laundry opens, m large washing machines
and n small washing machines are in continual use. Including the time for filling and
emptying the washing machines, each load of laundry takes 30 minutes in a large washing
machine and 20 minutes in a small washing machine. What is the total number of loads of
laundry done at Meadow's self-service laundry during this 2-hour period?
1. n = 3m
2. 2m + 3n = 55
A. Statement (1) ALONE is sufficient, but statement (2) alone is not sufficient.
B. Statement (2) ALONE is sufficient, but statement (1) alone is not sufficient.
C. BOTH statements TOGETHER are sufficient, but NEITHER statement ALONE is
sufficient.
D. EACH statement ALONE is sufficient.
E. Statements (1) and (2) TOGETHER are NOT sufficient.
DS76851.01

81. A box of light bulbs contains exactly 3 light bulbs that are defective. What is the probability
that a sample of light bulbs picked at random from this box will contain at least 1 defective
light bulb?
1. The light bulbs in the sample will be picked 1 at a time without replacement.
2. The sample will contain exactly 20 light bulbs.
A. Statement (1) ALONE is sufficient, but statement (2) alone is not sufficient.
B. Statement (2) ALONE is sufficient, but statement (1) alone is not sufficient.
C. BOTH statements TOGETHER are sufficient, but NEITHER statement ALONE is
sufficient.
D. EACH statement ALONE is sufficient.
E. Statements (1) and (2) TOGETHER are NOT sufficient.
DS01951.01

82. Khalil drove 120 kilometers in a certain amount of time. What was his average speed, in
kilometers per hour, during this time?
1. If Khalil had driven at an average speed that was 5 kilometers per hour faster, his
driving time would have been reduced by 20 minutes.
2. If Khalil had driven at an average speed that was 25% faster, his driving time would
have been reduced by 20%.
A. Statement (1) ALONE is sufficient, but statement (2) alone is not sufficient.
B. Statement (2) ALONE is sufficient, but statement (1) alone is not sufficient.
C. BOTH statements TOGETHER are sufficient, but NEITHER statement ALONE is
sufficient.
D. EACH statement ALONE is sufficient.
E. Statements (1) and (2) TOGETHER are NOT sufficient.
DS70061.01

83. What is the median of the data set S that consists of the integers 17, 29, 10, 26, 15, and x ?
1. The average (arithmetic mean) of S is 17.
2. The range of S is 24.
A. Statement (1) ALONE is sufficient, but statement (2) alone is not sufficient.
B. Statement (2) ALONE is sufficient, but statement (1) alone is not sufficient.
C. BOTH statements TOGETHER are sufficient, but NEITHER statement ALONE is
sufficient.
D. EACH statement ALONE is sufficient.
E. Statements (1) and (2) TOGETHER are NOT sufficient.
DS47661.01

84. If n > 4, what is the value of the integer n ?

1.

2.

A. Statement (1) ALONE is sufficient, but statement (2) alone is not sufficient.
B. Statement (2) ALONE is sufficient, but statement (1) alone is not sufficient.
C. BOTH statements TOGETHER are sufficient, but NEITHER statement ALONE is
sufficient.
D. EACH statement ALONE is sufficient.
E. Statements (1) and (2) TOGETHER are NOT sufficient.
DS50571.01

85. Tami purchased several identically priced metal frames and several identically priced
wooden frames for a total pretax price of $144. What was the total pretax price of the metal
frames that Tami purchased?
1. The price of each metal frame was 60% greater than the price of each wooden frame.
2. Tami purchased twice as many wooden frames as metal frames.
A. Statement (1) ALONE is sufficient, but statement (2) alone is not sufficient.
B. Statement (2) ALONE is sufficient, but statement (1) alone is not sufficient.
C. BOTH statements TOGETHER are sufficient, but NEITHER statement ALONE is
sufficient.
D. EACH statement ALONE is sufficient.
E. Statements (1) and (2) TOGETHER are NOT sufficient.
DS02871.01

86. A $10 bill (1,000 cents) was replaced with 50 coins having the same total value. The only
coins used were 5-cent coins, 10-cent coins, 25-cent coins, and 50-cent coins. How many 5-
cent coins were used?
1. Exactly 10 of the coins were 25-cent coins and exactly 10 of the coins were 50-cent
coins.
2. The number of 10-cent coins was twice the number of 5-cent coins.
A. Statement (1) ALONE is sufficient, but statement (2) alone is not sufficient.
B. Statement (2) ALONE is sufficient, but statement (1) alone is not sufficient.
C. BOTH statements TOGETHER are sufficient, but NEITHER statement ALONE is
sufficient.
D. EACH statement ALONE is sufficient.
E. Statements (1) and (2) TOGETHER are NOT sufficient.
DS56971.01

87. Merle's spare change jar has exactly 16 U.S. coins, each of which is a 1-cent coin, a 5-cent
coin, a 10-cent coin, a 25-cent coin, or a 50-cent coin. If the total value of the coins in the jar
is 288 U.S. cents, how many 1-cent coins are in the jar?
1. The exact numbers of 10-cent, 25-cent, and 50-cent coins among the 16 coins in the jar
are, respectively, 6, 5, and 2.
2. Among the 16 coins in the jar there are twice as many 10-cent coins as 1-cent coins.
A. Statement (1) ALONE is sufficient, but statement (2) alone is not sufficient.
B. Statement (2) ALONE is sufficient, but statement (1) alone is not sufficient.
C. BOTH statements TOGETHER are sufficient, but NEITHER statement ALONE is
sufficient.
D. EACH statement ALONE is sufficient.
E. Statements (1) and (2) TOGETHER are NOT sufficient.
DS48391.01

88. At a certain university recreation center, a member can receive a 30-minute massage, a 60-
minute massage, or a 90-minute massage, and is charged $0.50 per minute for each
massage. A member receiving a massage is charged the same fixed amount for each
additional service, such as nutrition advice or a fitness evaluation. At this center, what is the
total charge to a member for a 60-minute massage and 3 additional services?
1. At this recreation center, Jordan, a member, had a massage and 3 additional services
for a total charge of $37.50.
2. At this recreation center, Ryan, a member, had a massage and 2 additional services for
a total charge of $60.00.
A. Statement (1) ALONE is sufficient, but statement (2) alone is not sufficient.
B. Statement (2) ALONE is sufficient, but statement (1) alone is not sufficient.
C. BOTH statements TOGETHER are sufficient, but NEITHER statement ALONE is
sufficient.
D. EACH statement ALONE is sufficient.
E. Statements (1) and (2) TOGETHER are NOT sufficient.
DS84302.01

89. If S is the sum of the first n positive integers, what is the value of n ?
1. S < 20
2. S2 > 220
A. Statement (1) ALONE is sufficient, but statement (2) alone is not sufficient.
B. Statement (2) ALONE is sufficient, but statement (1) alone is not sufficient.
C. BOTH statements TOGETHER are sufficient, but NEITHER statement ALONE is
sufficient.
D. EACH statement ALONE is sufficient.
E. Statements (1) and (2) TOGETHER are NOT sufficient.
DS48302.01

90. Is x2 − y2 a positive number?


1. x − y is a positive number.
2. x + y is a positive number.
A. Statement (1) ALONE is sufficient, but statement (2) alone is not sufficient.
B. Statement (2) ALONE is sufficient, but statement (1) alone is not sufficient.
C. BOTH statements TOGETHER are sufficient, but NEITHER statement ALONE is
sufficient.
D. EACH statement ALONE is sufficient.
E. Statements (1) and (2) TOGETHER are NOT sufficient.
DS89302.01

91. Alan and Sue have each been saving one dollar a day and will continue to do so for the next
month. If Sue began saving several days before Alan, in how many days from today will Alan
have saved one-half as much as Sue?
1. As of today, Alan has saved 7 dollars and Sue has saved 27 dollars.
2. Three days from today, Alan will have saved one-third as much as Sue.
A. Statement (1) ALONE is sufficient, but statement (2) alone is not sufficient.
B. Statement (2) ALONE is sufficient, but statement (1) alone is not sufficient.
C. BOTH statements TOGETHER are sufficient, but NEITHER statement ALONE is
sufficient.
D. EACH statement ALONE is sufficient.
E. Statements (1) and (2) TOGETHER are NOT sufficient.
DS64402.01

92. What is the value of x ?

1.

2.
A. Statement (1) ALONE is sufficient, but statement (2) alone is not sufficient.
B. Statement (2) ALONE is sufficient, but statement (1) alone is not sufficient.
C. BOTH statements TOGETHER are sufficient, but NEITHER statement ALONE is
sufficient.
D. EACH statement ALONE is sufficient.
E. Statements (1) and (2) TOGETHER are NOT sufficient.
DS26402.01

93. Is x less than y ?


1. x−y+1<0
2. x−y−1<0
A. Statement (1) ALONE is sufficient, but statement (2) alone is not sufficient.
B. Statement (2) ALONE is sufficient, but statement (1) alone is not sufficient.
C. BOTH statements TOGETHER are sufficient, but NEITHER statement ALONE is
sufficient.
D. EACH statement ALONE is sufficient.
E. Statements (1) and (2) TOGETHER are NOT sufficient.
DS08402.01

94. State X has a sales tax rate of k percent on all purchases and State Y has a sales tax rate of n
percent on all purchases. What is the value of k − n ?
1. The sales tax on a $15 purchase is 30 cents more in State X than in State Y.
2. The sales tax rate in State X is 1.4 times the sales tax rate in State Y.
A. Statement (1) ALONE is sufficient, but statement (2) alone is not sufficient.
B. Statement (2) ALONE is sufficient, but statement (1) alone is not sufficient.
C. BOTH statements TOGETHER are sufficient, but NEITHER statement ALONE is
sufficient.
D. EACH statement ALONE is sufficient.
E. Statements (1) and (2) TOGETHER are NOT sufficient.
DS28402.01

95. Is −3 ≤ x ≤ 3 ?
1. x2 + y2 = 9
2. x2 + y ≤ 9
A. Statement (1) ALONE is sufficient, but statement (2) alone is not sufficient.
B. Statement (2) ALONE is sufficient, but statement (1) alone is not sufficient.
C. BOTH statements TOGETHER are sufficient, but NEITHER statement ALONE is
sufficient.
D. EACH statement ALONE is sufficient.
E. Statements (1) and (2) TOGETHER are NOT sufficient.
DS69402.01

96. What is the value of x ?


1. 4x + 1 + 4x = 320
2. x2 = 9
A. Statement (1) ALONE is sufficient, but statement (2) alone is not sufficient.
B. Statement (2) ALONE is sufficient, but statement (1) alone is not sufficient.
C. BOTH statements TOGETHER are sufficient, but NEITHER statement ALONE is
sufficient.
D. EACH statement ALONE is sufficient.
E. Statements (1) and (2) TOGETHER are NOT sufficient.
DS50502.01

97. Three dice, each with faces numbered 1 through 6, were tossed onto a game board. If one of
the dice turned up 4, what was the sum of the numbers that turned up on all three dice?
1. The sum of two of the numbers that turned up was 10.
2. The sum of two of the numbers that turned up was 11.
A. Statement (1) ALONE is sufficient, but statement (2) alone is not sufficient.
B. Statement (2) ALONE is sufficient, but statement (1) alone is not sufficient.
C. BOTH statements TOGETHER are sufficient, but NEITHER statement ALONE is
sufficient.
D. EACH statement ALONE is sufficient.
E. Statements (1) and (2) TOGETHER are NOT sufficient.
DS81502.01

98. Of the numbers q, r, s, and t, which is greatest?


1. The average (arithmetic mean) of q and r is s.
2. The sum of q and r is t.
A. Statement (1) ALONE is sufficient, but statement (2) alone is not sufficient.
B. Statement (2) ALONE is sufficient, but statement (1) alone is not sufficient.
C. BOTH statements TOGETHER are sufficient, but NEITHER statement ALONE is
sufficient.
D. EACH statement ALONE is sufficient.
E. Statements (1) and (2) TOGETHER are NOT sufficient.

CAR RENTAL CHARGES AT THRIFTY AGENCY


Car Type Charge per day Charge per Week (7 days)
Economy $28 $100
Compact $30 $120
Midsize $32 $140
Standard $34 $160
Luxury $39 $200
DS94502.01

99. The table above shows the car rental charges at Thrifty Agency. The daily rate applies for
each day or fraction of a day in excess of any multiple of a 7-day week, up to the charge per
week. If Olga rented a car of one of the types indicated, which type was it?
1. Olga's total rental charge, based only on the rates specified, was $184.
2. Olga rented the car for 10 days.
A. Statement (1) ALONE is sufficient, but statement (2) alone is not sufficient.
B. Statement (2) ALONE is sufficient, but statement (1) alone is not sufficient.
C. BOTH statements TOGETHER are sufficient, but NEITHER statement ALONE is
sufficient.
D. EACH statement ALONE is sufficient.
E. Statements (1) and (2) TOGETHER are NOT sufficient.
DS76602.01

100. Is xy < 6 ?
1. x < 3 and y < 2.

2. and y2 < 64.

A. Statement (1) ALONE is sufficient, but statement (2) alone is not sufficient.
B. Statement (2) ALONE is sufficient, but statement (1) alone is not sufficient.
C. BOTH statements TOGETHER are sufficient, but NEITHER statement ALONE is
sufficient.
D. EACH statement ALONE is sufficient.
E. Statements (1) and (2) TOGETHER are NOT sufficient.
DS86602.01

101. What is the value of ?

1. and .

2. and .

A. Statement (1) ALONE is sufficient, but statement (2) alone is not sufficient.
B. Statement (2) ALONE is sufficient, but statement (1) alone is not sufficient.
C. BOTH statements TOGETHER are sufficient, but NEITHER statement ALONE is
sufficient.
D. EACH statement ALONE is sufficient.
E. Statements (1) and (2) TOGETHER are NOT sufficient.
DS47602.01

102. In a certain group of people, the average (arithmetic mean) weight of the males is 180
pounds and of the females, 120 pounds. What is the average weight of the people in the
group?
1. The group contains twice as many females as males.
2. The group contains 10 more females than males.
A. Statement (1) ALONE is sufficient, but statement (2) alone is not sufficient.
B. Statement (2) ALONE is sufficient, but statement (1) alone is not sufficient.
C. BOTH statements TOGETHER are sufficient, but NEITHER statement ALONE is
sufficient.
D. EACH statement ALONE is sufficient.
E. Statements (1) and (2) TOGETHER are NOT sufficient.
DS57602.01

103. If y = 2x + 1, what is the value of y − x ?


1. 22x +2 = 64
2. y = 22x − 1
A. Statement (1) ALONE is sufficient, but statement (2) alone is not sufficient.
B. Statement (2) ALONE is sufficient, but statement (1) alone is not sufficient.
C. BOTH statements TOGETHER are sufficient, but NEITHER statement ALONE is
sufficient.
D. EACH statement ALONE is sufficient.
E. Statements (1) and (2) TOGETHER are NOT sufficient.
DS67602.01

104. If x ≠ 1, is y equal to x + 1 ?
1.

2. y2 = (x + 1)2
A. Statement (1) ALONE is sufficient, but statement (2) alone is not sufficient.
B. Statement (2) ALONE is sufficient, but statement (1) alone is not sufficient.
C. BOTH statements TOGETHER are sufficient, but NEITHER statement ALONE is
sufficient.
D. EACH statement ALONE is sufficient.
E. Statements (1) and (2) TOGETHER are NOT sufficient.
DS18602.01

105. If x + y + z > 0, is z > 1 ?


1. z>x+y+1
2. x+y+1<0
A. Statement (1) ALONE is sufficient, but statement (2) alone is not sufficient.
B. Statement (2) ALONE is sufficient, but statement (1) alone is not sufficient.
C. BOTH statements TOGETHER are sufficient, but NEITHER statement ALONE is
sufficient.
D. EACH statement ALONE is sufficient.
E. Statements (1) and (2) TOGETHER are NOT sufficient.
Geometry
DS35210.01

106. In the rectangular coordinate system, line k passes through the point (n,−1). Is the slope of
line k greater than zero?
1. Line k passes through the origin.
2. Line k passes through the point (1,n + 2).
A. Statement (1) ALONE is sufficient, but statement (2) alone is not sufficient.
B. Statement (2) ALONE is sufficient, but statement (1) alone is not sufficient.
C. BOTH statements TOGETHER are sufficient, but NEITHER statement ALONE is
sufficient.
D. EACH statement ALONE is sufficient.
E. Statements (1) and (2) TOGETHER are NOT sufficient.
DS88111.01

107. In quadrilateral ABCD, is angle BCD a right angle?


1. Angle ABC is a right angle.
2. Angle ADC is a right angle.
A. Statement (1) ALONE is sufficient, but statement (2) alone is not sufficient.
B. Statement (2) ALONE is sufficient, but statement (1) alone is not sufficient.
C. BOTH statements TOGETHER are sufficient, but NEITHER statement ALONE is
sufficient.
D. EACH statement ALONE is sufficient.
E. Statements (1) and (2) TOGETHER are NOT sufficient.

DS29831.01

108. In the figure above, B is on , D is on , has the same length as , and has the
same measure as . What is the length of ?
1. The length of is 6.
2. The length of is 5.
A. Statement (1) ALONE is sufficient, but statement (2) alone is not sufficient.
B. Statement (2) ALONE is sufficient, but statement (1) alone is not sufficient.
C. BOTH statements TOGETHER are sufficient, but NEITHER statement ALONE is
sufficient.
D. EACH statement ALONE is sufficient.
E. Statements (1) and (2) TOGETHER are NOT sufficient.
DS92931.01

109. Sprinklers are being installed to water a lawn. Each sprinkler waters in a circle. Can the lawn
be watered completely by 4 installed sprinklers?
1. The lawn is rectangular and its area is 32 square yards.
2. Each sprinkler can completely water a circular area of lawn with a maximum radius of
2 yards.
A. Statement (1) ALONE is sufficient, but statement (2) alone is not sufficient.
B. Statement (2) ALONE is sufficient, but statement (1) alone is not sufficient.
C. BOTH statements TOGETHER are sufficient, but NEITHER statement ALONE is
sufficient.
D. EACH statement ALONE is sufficient.
E. Statements (1) and (2) TOGETHER are NOT sufficient.
DS18041.01

110. What is the length of the hypotenuse of ΔABC ?


1. The lengths of the three sides of ΔABC are consecutive even integers.
2. The hypotenuse of ΔABC is 4 units longer than the shorter leg.
A. Statement (1) ALONE is sufficient, but statement (2) alone is not sufficient.
B. Statement (2) ALONE is sufficient, but statement (1) alone is not sufficient.
C. BOTH statements TOGETHER are sufficient, but NEITHER statement ALONE is
sufficient.
D. EACH statement ALONE is sufficient.
E. Statements (1) and (2) TOGETHER are NOT sufficient.
DS37571.01

111. Patricia purchased x meters of fencing. She originally intended to use all of the fencing to
enclose a square region, but later decided to use all of the fencing to enclose a rectangular
region with length y meters greater than its width. In square meters, what is the positive
difference between the area of the square region and the area of the rectangular region?
1. xy = 256
2. y=4
A. Statement (1) ALONE is sufficient, but statement (2) alone is not sufficient.
B. Statement (2) ALONE is sufficient, but statement (1) alone is not sufficient.
C. BOTH statements TOGETHER are sufficient, but NEITHER statement ALONE is
sufficient.
D. EACH statement ALONE is sufficient.
E. Statements (1) and (2) TOGETHER are NOT sufficient.
DS45771.01

112. In the figure above, points A, B, C, and D are collinear and , , and are semicircles with
diameters d1 cm, d2 cm, and d3 cm, respectively. What is the sum of the lengths of , ,
and , in centimeters?
1. d1:d2:d3 is 3:2:1.
2. The length of is 48 cm.
A. Statement (1) ALONE is sufficient, but statement (2) alone is not sufficient.
B. Statement (2) ALONE is sufficient, but statement (1) alone is not sufficient.
C. BOTH statements TOGETHER are sufficient, but NEITHER statement ALONE is
sufficient.
D. EACH statement ALONE is sufficient.
E. Statements (1) and (2) TOGETHER are NOT sufficient.
DS16291.01

113. In the standard (x,y) coordinate plane, what is the slope of the line containing the distinct
points P and Q ?
1. Both P and Q lie on the graph of |x| + |y| = 1.
2. Both P and Q lie on the graph of |x + y| = 1.
A. Statement (1) ALONE is sufficient, but statement (2) alone is not sufficient.
B. Statement (2) ALONE is sufficient, but statement (1) alone is not sufficient.
C. BOTH statements TOGETHER are sufficient, but NEITHER statement ALONE is
sufficient.
D. EACH statement ALONE is sufficient.
E. Statements (1) and (2) TOGETHER are NOT sufficient.
DS61791.01

114. When opened and lying flat, a birthday card is in the shape of a regular hexagon. The card
must be folded in half along 1 of its diagonals before being placed in an envelope for mailing.
Assuming that the thickness of the folded card will not be an issue, will the birthday card fit
inside a rectangular envelope that is 4 inches by 9 inches?
1. Each side of the regular hexagon is 4 inches long.
2. The area of the top surface (which is the same as the area of the bottom surface) of the
folded birthday card is less than 36 square inches.
A. Statement (1) ALONE is sufficient, but statement (2) alone is not sufficient.
B. Statement (2) ALONE is sufficient, but statement (1) alone is not sufficient.
C. BOTH statements TOGETHER are sufficient, but NEITHER statement ALONE is
sufficient.
D. EACH statement ALONE is sufficient.
E. Statements (1) and (2) TOGETHER are NOT sufficient.

DS77302.01

115. In rectangular region PQRS above, T is a point on side PS. If PS = 4, what is the area of region
PQRS ?
1. ΔQTR is equilateral.
2. Segments PT and TS have equal length.
A. Statement (1) ALONE is sufficient, but statement (2) alone is not sufficient.
B. Statement (2) ALONE is sufficient, but statement (1) alone is not sufficient.
C. BOTH statements TOGETHER are sufficient, but NEITHER statement ALONE is
sufficient.
D. EACH statement ALONE is sufficient.
E. Statements (1) and (2) TOGETHER are NOT sufficient.
DS58302.01

116. The top surface area of a square tabletop was changed so that one of the dimensions was
reduced by 1 inch and the other dimension was increased by 2 inches. What was the surface
area before these changes were made?
1. After the changes were made, the surface area was 70 square inches.
2. There was a 25 percent increase in one of the dimensions.
A. Statement (1) ALONE is sufficient, but statement (2) alone is not sufficient.
B. Statement (2) ALONE is sufficient, but statement (1) alone is not sufficient.
C. BOTH statements TOGETHER are sufficient, but NEITHER statement ALONE is
sufficient.
D. EACH statement ALONE is sufficient.
E. Statements (1) and (2) TOGETHER are NOT sufficient.
DS49302.01

117. If the lengths of the legs of a right triangle are integers, what is the area of the triangular
region?
1. The length of one leg is the length of the other.

2. The length of the hypotenuse is 5.


A. Statement (1) ALONE is sufficient, but statement (2) alone is not sufficient.
B. Statement (2) ALONE is sufficient, but statement (1) alone is not sufficient.
C. BOTH statements TOGETHER are sufficient, but NEITHER statement ALONE is
sufficient.
D. EACH statement ALONE is sufficient.
E. Statements (1) and (2) TOGETHER are NOT sufficient.
DS12402.01

118. If cubical blocks in a display are stacked one on top of the other on a flat surface, what is the
volume of the stack of blocks in cubic centimeters?
1. The volume of the top block is 8 cubic centimeters.
2. The height of the stack of blocks is 10 centimeters.
A. Statement (1) ALONE is sufficient, but statement (2) alone is not sufficient.
B. Statement (2) ALONE is sufficient, but statement (1) alone is not sufficient.
C. BOTH statements TOGETHER are sufficient, but NEITHER statement ALONE is
sufficient.
D. EACH statement ALONE is sufficient.
E. Statements (1) and (2) TOGETHER are NOT sufficient.
DS53402.01

119. Is the perimeter of a certain rectangular garden greater than 50 meters?


1. The two shorter sides of the garden are each 15 meters long.
2. The length of the garden is 5 meters greater than the width of the garden.
A. Statement (1) ALONE is sufficient, but statement (2) alone is not sufficient.
B. Statement (2) ALONE is sufficient, but statement (1) alone is not sufficient.
C. BOTH statements TOGETHER are sufficient, but NEITHER statement ALONE is
sufficient.
D. EACH statement ALONE is sufficient.
E. Statements (1) and (2) TOGETHER are NOT sufficient.

DS34402.01
120. In the figure above, what is the perimeter of ΔPQR ?
1. The length of segment PT is 2.
2. The length of segment RS is .
A. Statement (1) ALONE is sufficient, but statement (2) alone is not sufficient.
B. Statement (2) ALONE is sufficient, but statement (1) alone is not sufficient.
C. BOTH statements TOGETHER are sufficient, but NEITHER statement ALONE is
sufficient.
D. EACH statement ALONE is sufficient.
E. Statements (1) and (2) TOGETHER are NOT sufficient.

DS07402.01

121. In the figure above, the shaded region represents the front of an upright wooden frame
around the entrance to an amusement park ride. If meters, what is the area of the
front of the frame?
1. x = 9 meters
2. meters
A. Statement (1) ALONE is sufficient, but statement (2) alone is not sufficient.
B. Statement (2) ALONE is sufficient, but statement (1) alone is not sufficient.
C. BOTH statements TOGETHER are sufficient, but NEITHER statement ALONE is
sufficient.
D. EACH statement ALONE is sufficient.
E. Statements (1) and (2) TOGETHER are NOT sufficient.
DS05502.01

122. In the figure above, if the shaded region is rectangular, what is the length of XY ?
1. The perimeter of the shaded region is 24.
2. The measure of XYZ is 45°.
A. Statement (1) ALONE is sufficient, but statement (2) alone is not sufficient.
B. Statement (2) ALONE is sufficient, but statement (1) alone is not sufficient.
C. BOTH statements TOGETHER are sufficient, but NEITHER statement ALONE is
sufficient.
D. EACH statement ALONE is sufficient.
E. Statements (1) and (2) TOGETHER are NOT sufficient.

DS87602.01

123. The figure above shows the dimensions of a square picture frame that was constructed using
four identical pieces of frame as shown. If w is the width of each piece of the frame, what is
the area of the front surface of each piece? (1 ft = 12 inches)
1. w = 3 inches
2. PQ = inches
A. Statement (1) ALONE is sufficient, but statement (2) alone is not sufficient.
B. Statement (2) ALONE is sufficient, but statement (1) alone is not sufficient.
C. BOTH statements TOGETHER are sufficient, but NEITHER statement ALONE is
sufficient.
D. EACH statement ALONE is sufficient.
E. Statements (1) and (2) TOGETHER are NOT sufficient.

DS48602.01

124. In the figure above, segments and represent two positions of the same ladder leaning
against the side of a wall. The length of is how much greater than the length of ?
1. The length of is 10 meters.
2. The length of is 5 meters.
A. Statement (1) ALONE is sufficient, but statement (2) alone is not sufficient.
B. Statement (2) ALONE is sufficient, but statement (1) alone is not sufficient.
C. BOTH statements TOGETHER are sufficient, but NEITHER statement ALONE is
sufficient.
D. EACH statement ALONE is sufficient.
E. Statements (1) and (2) TOGETHER are NOT sufficient.
Rates/Ratios/Percent
DS67410.01

125. A large flower arrangement contains 3 types of flowers: carnations, lilies, and roses. Of all the
flowers in the arrangement, are carnations, are lilies, and are roses. The total price of
which of the 3 types of flowers in the arrangement is the greatest?
1. The prices per flower for carnations, lilies, and roses are in the ratio 1:3:4, respectively.
2. The price of one rose is $0.75 more than the price of one carnation, and the price of one
rose is $0.25 more than the price of one lily.
A. Statement (1) ALONE is sufficient, but statement (2) alone is not sufficient.
B. Statement (2) ALONE is sufficient, but statement (1) alone is not sufficient.
C. BOTH statements TOGETHER are sufficient, but NEITHER statement ALONE is
sufficient.
D. EACH statement ALONE is sufficient.
E. Statements (1) and (2) TOGETHER are NOT sufficient.
DS34010.01
126. Town X has 50,000 residents, some of whom were born in Town X. What percent of the
residents of Town X were born in Town X ?
1. Of the male residents of Town X, 40 percent were not born in Town X.
2. Of the female residents of Town X, 60 percent were born in Town X.
A. Statement (1) ALONE is sufficient, but statement (2) alone is not sufficient.
B. Statement (2) ALONE is sufficient, but statement (1) alone is not sufficient.
C. BOTH statements TOGETHER are sufficient, but NEITHER statement ALONE is
sufficient.
D. EACH statement ALONE is sufficient.
E. Statements (1) and (2) TOGETHER are NOT sufficient.
DS29931.01

127. A bank account earned 2% annual interest, compounded daily, for as long as the balance was
under $1,000, starting when the account was opened. Once the balance reached $1,000, the
account earned 2.5% annual interest, compounded daily until the account was closed. No
deposits or withdrawals were made. Was the total amount of interest earned at the 2% rate
greater than the total amount earned at the 2.5% rate?
1. The account earned exactly $25 in interest at the 2.5% rate.
2. The account was open for exactly three years.
A. Statement (1) ALONE is sufficient, but statement (2) alone is not sufficient.
B. Statement (2) ALONE is sufficient, but statement (1) alone is not sufficient.
C. BOTH statements TOGETHER are sufficient, but NEITHER statement ALONE is
sufficient.
D. EACH statement ALONE is sufficient.
E. Statements (1) and (2) TOGETHER are NOT sufficient.
DS53541.01

128. A novelist pays her agent 15% of the royalties she receives from her novels. She pays her
publicist 5% of the royalties, plus a yearly fee. Did the novelist pay more to her agent last year
than she paid to her publicist?
1. The publicist's yearly fee is $2,000.
2. The novelist earned an average of $3,500 in royalties last year on each of her novels.
A. Statement (1) ALONE is sufficient, but statement (2) alone is not sufficient.
B. Statement (2) ALONE is sufficient, but statement (1) alone is not sufficient.
C. BOTH statements TOGETHER are sufficient, but NEITHER statement ALONE is
sufficient.
D. EACH statement ALONE is sufficient.
E. Statements (1) and (2) TOGETHER are NOT sufficient.
Value/Order/Factors
DS85100.01
129. If x and z are integers, is x + z2 odd?
1. x is odd and z is even.
2. x – z is odd.
A. Statement (1) ALONE is sufficient, but statement (2) alone is not sufficient.
B. Statement (2) ALONE is sufficient, but statement (1) alone is not sufficient.
C. BOTH statements TOGETHER are sufficient, but NEITHER statement ALONE is
sufficient.
D. EACH statement ALONE is sufficient.
E. Statements (1) and (2) TOGETHER are NOT sufficient.

DS95850.01

130. Each entry in the multiplication table above is an integer that is either positive, negative, or
zero. What is the value of a ?
1. h≠0
2. c=f
A. Statement (1) ALONE is sufficient, but statement (2) alone is not sufficient.
B. Statement (2) ALONE is sufficient, but statement (1) alone is not sufficient.
C. BOTH statements TOGETHER are sufficient, but NEITHER statement ALONE is
sufficient.
D. EACH statement ALONE is sufficient.
E. Statements (1) and (2) TOGETHER are NOT sufficient.
DS36141.01

131. Given a positive number N, when N is rounded by a certain method (for convenience, call it
Method Y), the result is 10n if and only if n is an integer and 5 × 10n − 1 ≤ N < 5 × 10n. In a
certain gas sample, there are, when rounded by Method Y, 1021 molecules of H2 and also 1021
molecules of O2. When rounded by Method Y, what is the combined number of H2 and O2
molecules in the gas sample?
1. The number of H2 molecules and the number of O2 molecules are each less than
3 × 1021.
2. The number of H2 molecules is more than twice the number of O2 molecules.
A. Statement (1) ALONE is sufficient, but statement (2) alone is not sufficient.
B. Statement (2) ALONE is sufficient, but statement (1) alone is not sufficient.
C. BOTH statements TOGETHER are sufficient, but NEITHER statement ALONE is
sufficient.
D. EACH statement ALONE is sufficient.
E. Statements (1) and (2) TOGETHER are NOT sufficient.
DS05541.01

132. If x is a positive integer, how many positive integers less than x are divisors of x ?
1. x2 is divisible by exactly 4 positive integers less than x2.
2. 2x is divisible by exactly 3 positive integers less than 2x.
A. Statement (1) ALONE is sufficient, but statement (2) alone is not sufficient.
B. Statement (2) ALONE is sufficient, but statement (1) alone is not sufficient.
C. BOTH statements TOGETHER are sufficient, but NEITHER statement ALONE is
sufficient.
D. EACH statement ALONE is sufficient.
E. Statements (1) and (2) TOGETHER are NOT sufficient.
DS33551.01

133. If m and n are positive integers, is n even?


1. m(m + 2) + 1 = mn
2. m(m + n) is odd.
A. Statement (1) ALONE is sufficient, but statement (2) alone is not sufficient.
B. Statement (2) ALONE is sufficient, but statement (1) alone is not sufficient.
C. BOTH statements TOGETHER are sufficient, but NEITHER statement ALONE is
sufficient.
D. EACH statement ALONE is sufficient.
E. Statements (1) and (2) TOGETHER are NOT sufficient.
DS65291.01

134. If m and n are positive integers, what is the value of + ?

(1) mn = 12

(2) is in lowest terms and is in lowest terms.

A. Statement (1) ALONE is sufficient, but statement (2) alone is not sufficient.
B. Statement (2) ALONE is sufficient, but statement (1) alone is not sufficient.
C. BOTH statements TOGETHER are sufficient, but NEITHER statement ALONE is
sufficient.
D. EACH statement ALONE is sufficient.
E. Statements (1) and (2) TOGETHER are NOT sufficient.
DS21891.01
135. The first four digits of the six-digit initial password for a shopper's card at a certain grocery
store is the customer's birthday in day-month digit form. For example, 15 August
corresponds to 1508 and 5 March corresponds to 0503. The 5th digit of the initial password
is the units digit of seven times the sum of the first and third digits, and the 6th digit of the
initial password is the units digit of three times the sum of the second and fourth digits. What
month, and what day of that month, was a customer born whose initial password ends in 16 ?
1. The customer's initial password begins with 21, and its fourth digit is 1.
2. The sum of the first and third digits of the customer's initial password is 3, and its
second digit is 1.
A. Statement (1) ALONE is sufficient, but statement (2) alone is not sufficient.
B. Statement (2) ALONE is sufficient, but statement (1) alone is not sufficient.
C. BOTH statements TOGETHER are sufficient, but NEITHER statement ALONE is
sufficient.
D. EACH statement ALONE is sufficient.
E. Statements (1) and (2) TOGETHER are NOT sufficient.
DS38302.01

136. If K is a positive integer less than 10 and N = 4,321 + K, what is the value of K ?
1. N is divisible by 3.
2. N is divisible by 7.
A. Statement (1) ALONE is sufficient, but statement (2) alone is not sufficient.
B. Statement (2) ALONE is sufficient, but statement (1) alone is not sufficient.
C. BOTH statements TOGETHER are sufficient, but NEITHER statement ALONE is
sufficient.
D. EACH statement ALONE is sufficient.
E. Statements (1) and (2) TOGETHER are NOT sufficient.
DS99302.01

137. If s is an integer, is 24 a divisor of s ?


1. Each of the numbers 3 and 8 is a divisor of s.
2. Each of the numbers 4 and 6 is a divisor of s.
A. Statement (1) ALONE is sufficient, but statement (2) alone is not sufficient.
B. Statement (2) ALONE is sufficient, but statement (1) alone is not sufficient.
C. BOTH statements TOGETHER are sufficient, but NEITHER statement ALONE is
sufficient.
D. EACH statement ALONE is sufficient.
E. Statements (1) and (2) TOGETHER are NOT sufficient.
DS32402.01

138. n = 24 ⋅ 32 ⋅ 52 and positive integer d is a divisor of n. Is d > ?


1. d is divisible by 10.
2. d is divisible by 36.
A. Statement (1) ALONE is sufficient, but statement (2) alone is not sufficient.
B. Statement (2) ALONE is sufficient, but statement (1) alone is not sufficient.
C. BOTH statements TOGETHER are sufficient, but NEITHER statement ALONE is
sufficient.
D. EACH statement ALONE is sufficient.
E. Statements (1) and (2) TOGETHER are NOT sufficient.
DS52402.01

139. Exactly 3 deposits have been made in a savings account and the amounts of the deposits are
3 consecutive integer multiples of $7. If the sum of the deposits is between $120 and $170,
what is the amount of each of the deposits?
1. The amount of one of the deposits is $49.
2. The amount of one of the deposits is $63.
A. Statement (1) ALONE is sufficient, but statement (2) alone is not sufficient.
B. Statement (2) ALONE is sufficient, but statement (1) alone is not sufficient.
C. BOTH statements TOGETHER are sufficient, but NEITHER statement ALONE is
sufficient.
D. EACH statement ALONE is sufficient.
E. Statements (1) and (2) TOGETHER are NOT sufficient.
DS44402.01

140. If x, y, and d are integers and d is odd, are both x and y divisible by d ?
1. x + y is divisible by d.
2. x − y is divisible by d.
A. Statement (1) ALONE is sufficient, but statement (2) alone is not sufficient.
B. Statement (2) ALONE is sufficient, but statement (1) alone is not sufficient.
C. BOTH statements TOGETHER are sufficient, but NEITHER statement ALONE is
sufficient.
D. EACH statement ALONE is sufficient.
E. Statements (1) and (2) TOGETHER are NOT sufficient.
DS06402.01

141. If x and y are integers, is xy + 1 divisible by 3 ?


1. When x is divided by 3, the remainder is 1.
2. When y is divided by 9, the remainder is 8.
A. Statement (1) ALONE is sufficient, but statement (2) alone is not sufficient.
B. Statement (2) ALONE is sufficient, but statement (1) alone is not sufficient.
C. BOTH statements TOGETHER are sufficient, but NEITHER statement ALONE is
sufficient.
D. EACH statement ALONE is sufficient.
E. Statements (1) and (2) TOGETHER are NOT sufficient.
DS00502.01

142. If x and y are integers between 10 and 99, inclusive, is an integer?

1. x and y have the same two digits, but in reverse order.


2. The tens' digit of x is 2 more than the units' digit, and the tens' digit of y is 2 less than
the units' digit.
A. Statement (1) ALONE is sufficient, but statement (2) alone is not sufficient.
B. Statement (2) ALONE is sufficient, but statement (1) alone is not sufficient.
C. BOTH statements TOGETHER are sufficient, but NEITHER statement ALONE is
sufficient.
D. EACH statement ALONE is sufficient.
E. Statements (1) and (2) TOGETHER are NOT sufficient.
DS85502.01

143. If b is the product of three consecutive positive integers c, c + 1, and c + 2, is b a multiple of


24 ?
1. b is a multiple of 8.
2. c is odd.
A. Statement (1) ALONE is sufficient, but statement (2) alone is not sufficient.
B. Statement (2) ALONE is sufficient, but statement (1) alone is not sufficient.
C. BOTH statements TOGETHER are sufficient, but NEITHER statement ALONE is
sufficient.
D. EACH statement ALONE is sufficient.
E. Statements (1) and (2) TOGETHER are NOT sufficient.
DS17602.01

144. If denotes a mathematical operation, does x y=y x for all x and y ?


1. For all x and y, x y = 2(x2 + y2).
2. For all y, 0 y = 2y2.
A. Statement (1) ALONE is sufficient, but statement (2) alone is not sufficient.
B. Statement (2) ALONE is sufficient, but statement (1) alone is not sufficient.
C. BOTH statements TOGETHER are sufficient, but NEITHER statement ALONE is
sufficient.
D. EACH statement ALONE is sufficient.
E. Statements (1) and (2) TOGETHER are NOT sufficient.
DS37602.01

145. If n is an integer, is an integer?


1. is an integer.

2. is an integer.

A. Statement (1) ALONE is sufficient, but statement (2) alone is not sufficient.
B. Statement (2) ALONE is sufficient, but statement (1) alone is not sufficient.
C. BOTH statements TOGETHER are sufficient, but NEITHER statement ALONE is
sufficient.
D. EACH statement ALONE is sufficient.
E. Statements (1) and (2) TOGETHER are NOT sufficient.
DS97602.01

146. If 1 < d < 2, is the tenths digit of the decimal representation of d equal to 9 ?
1. d + 0.01 < 2
2. d + 0.05 > 2
A. Statement (1) ALONE is sufficient, but statement (2) alone is not sufficient.
B. Statement (2) ALONE is sufficient, but statement (1) alone is not sufficient.
C. BOTH statements TOGETHER are sufficient, but NEITHER statement ALONE is
sufficient.
D. EACH statement ALONE is sufficient.
E. Statements (1) and (2) TOGETHER are NOT sufficient.
DS08602.01

147. The 9 participants in a race were divided into 3 teams with 3 runners on each team. A team
was awarded 6 – n points if one of its runners finished in nth place, where 1 ≤ n ≤ 5. If all of
the runners finished the race and if there were no ties, was each team awarded at least 1
point?
1. No team was awarded more than a total of 6 points.
2. No pair of teammates finished in consecutive places among the top five places.
A. Statement (1) ALONE is sufficient, but statement (2) alone is not sufficient.
B. Statement (2) ALONE is sufficient, but statement (1) alone is not sufficient.
C. BOTH statements TOGETHER are sufficient, but NEITHER statement ALONE is
sufficient.
D. EACH statement ALONE is sufficient.
E. Statements (1) and (2) TOGETHER are NOT sufficient.
DS38602.01

148. Can the positive integer n be written as the sum of two different positive prime numbers?
1. n is greater than 3.
2. n is odd.
A. Statement (1) ALONE is sufficient, but statement (2) alone is not sufficient.
B. Statement (2) ALONE is sufficient, but statement (1) alone is not sufficient.
C. BOTH statements TOGETHER are sufficient, but NEITHER statement ALONE is
sufficient.
D. EACH statement ALONE is sufficient.
E. Statements (1) and (2) TOGETHER are NOT sufficient.
DS73402.01

149. Is x an integer?
1. x2 is an integer.
2. is not an integer.

A. Statement (1) ALONE is sufficient, but statement (2) alone is not sufficient.
B. Statement (2) ALONE is sufficient, but statement (1) alone is not sufficient.
C. BOTH statements TOGETHER are sufficient, but NEITHER statement ALONE is
sufficient.
D. EACH statement ALONE is sufficient.
E. Statements (1) and (2) TOGETHER are NOT sufficient.
DS46402.01

150. If b is an integer, is an integer?


1. a2 + b2 is an integer.
2. a2 − 3b2 = 0
A. Statement (1) ALONE is sufficient, but statement (2) alone is not sufficient.
B. Statement (2) ALONE is sufficient, but statement (1) alone is not sufficient.
C. BOTH statements TOGETHER are sufficient, but NEITHER statement ALONE is
sufficient.
D. EACH statement ALONE is sufficient.
E. Statements (1) and (2) TOGETHER are NOT sufficient.
Answer Key Quantitative Reasoning
Problem Solving

Counting/Sets/Series
1. E
2. C
3. E
4. E
5. C
6. D
7. A
8. E
9. D
10. E
11. C
12. A

Equalities/Inequalities/Algebra
13. C
14. B
15. E
16. A
17. A
18. D
19. D
20. C
21. C
22. B
23. C
24. A
25. E
26. E
27. E
28. B
29. A
30. C
31. C
32. D

Geometry
33. A
34. B
35. D
36. C
37. D
38. B
39. D
40. D
41. A
42. C

Rates/Ratios/Percent
43. D
44. C
45. D
46. C
47. B
48. C
49. B

Value/Order/Factors
50. B
51. B
52. E
53. E
54. E
55. D
56. D
57. D
58. D
59. B
60. C
61. B
62. A
63. A
64. B
65. B
66. B
67. E
Data Sufficiency

Counting/Sets/Series
68. E
69. D
70. B
71. C
72. C
73. B
74. E
75. E
76. D

Equalities/Inequalities/Algebra
77. C
78. B
79. A
80. B
81. E
82. A
83. A
84. A
85. C
86. A
87. D
88. C
89. C
90. C
91. A
92. A
93. A
94. A
95. A
96. A
97. B
98. E
99. A
100. B
101. B
102. A
103. D
104. A
105. B

Geometry
106. C
107. E
108. A
109. E
110. A
111. B
112. B
113. E
114. A
115. A
116. D
117. B
118. E
119. A
120. A
121. D
122. C
123. D
124. D

Rates/Ratios/Percent
125. A
126. C
127. C
128. E

Value/Order/Factors
129. D
130. C
131. C
132. A
133. D
134. C
135. A
136. B
137. A
138. C
139. B
140. C
141. C
142. A
143. A
144. A
145. B
146. B
147. A
148. E
149. E
150. B
Answer Explanations Quantitative Reasoning
Problem Solving
Counting/Sets/Series
PS54110.01

1. The letters C, I, R, C, L, and E can be used to form 6-letter strings such as CIRCLE or
CCIRLE. Using these letters, how many different 6-letter strings can be formed in which the
two occurrences of the letter C are separated by at least one other letter?
A. 96
B. 120
C. 144
D. 180
E. 240
Arithmetic Elementary combinatorics
This can be solved by using the Multiplication Principle. The answer is m × n, where m is the
number of ways to choose the 2 suitable positions in which to place the C's and n is the
number of ways in which to place the 4 remaining letters in the 4 remaining positions.
The value of m can be found by a direct count of the number of suitable ways to choose the 2
positions in which to place the C's. In what follows, each * denotes one of the 4 remaining
positions.
There are 4 possibilities when a C is in the first position:
C*C*** C**C** C***C* C****C
There are 3 more possibilities when a C is in the second position:
*C*C** *C**C* *C***C
There are 2 more possibilities when a C is in the third position:
**C*C* **C**C
There is 1 more possibility when a C is in the fourth position:
***C*C
Therefore, m = 4 + 3 + 2 + 1 = 10.
Alternatively, the value of m can be found by subtracting the number of non-suitable ways to
place the C's (i.e., the number of consecutive positions in the string) from the number of all

possible ways to place the C's (suitable or not). This gives m = 15 − 5 = 10, where is

the number of all possible ways to place the C's (“6 choose 2”) and 5 is the number of non-
suitable ways to place the C's (shown below).
CC**** *CC*** **CC** ***CC* ****CC

Tip

The alternative approach for finding m is useful when a


direct count of the number of suitable ways is more
difficult than a direct count of the number of non-suitable
ways. An example is determining the number of 8-letter
strings that can be formed from the letters in REPEATED
in which there is at least one pair of E's having at least
one other letter between them. For this example, m =
= 56 − 6 = 50.

The value of n is equal to the number of ways to place the 4 remaining letters into 4
positions, where order matters and the letters are selected without replacement. Thus,
n = 4! = 24.
Therefore, the answer is m × n = 10 × 24 = 240.
The correct answer is E.

PS24831.01

2. The map above shows the trails through a wilderness area. If travel is in the direction of the
arrows, how many routes along the marked trails are possible from point A to point B ?
A. 11
B. 18
C. 54
D. 108
E. 432
Arithmetic Elementary combinatorics
It is clear that the number of routes from point A to point B that begin by going up from
point A (“up” relative to the orientation of the map) is the same as the number of routes from
point A to point B that begin by going down from point A. Therefore, we only need to
determine the number of routes from point A to point B that begin by going up and then
double the result.
Tip

If these two numbers of routes were not the same, or at


least if it was not clear whether they were the same, then
we would simply determine each of the numbers
separately and then add them.

To determine the number of routes that begin by going up from point A, we can apply the
Multiplication Principle. There are 3 locations at which branches occur. Moreover, at each of
these locations, there are 3 different trails that can be taken. Finally, the choices of which
trail to take at each location can be made independently. Therefore, the Multiplication
Principle applies and we get (3)(3)(3) = 27 for the number of routes that begin by going up
from point A. Hence, the number of routes from point A to point B is 2(27) = 54.
The correct answer is C.

PS61551.01

3. In the figure above, X and Y represent locations in a district of a certain city where the streets
form a rectangular grid. In traveling only north or east along the streets from X to Y, how
many different paths are possible?
A. 720
B. 512
C. 336
D. 256
E. 56
Arithmetic Elementary combinatorics
Each possible path will consist of traveling a total of 3 grid segments north and 5 grid
segments east. Thus, letting ‘N' represent traveling north by one grid segment and ‘E'
represent traveling east by one grid segment, each path can be uniquely represented by an
appropriate 8-character string of N's and E's. For example, as shown in the figure below,
NEENEENE represents grid segments traveled in the order north, east, east, north, east,
east, north, and east.
Therefore, the number of possible paths is equal to the number of appropriate 8-character

strings of N's and E's, which is = (7)(8) = 56, since

each appropriate string is determined when a specification is made for the 3 positions in the
string at which the N's are to be placed. Alternatively, the number of possible paths is equal
to the number of permutations of 8 objects in which 3 are identical (the N's) and the
remaining 5 are identical (the E's), and thus equal to .

Tip

The alternative approach has a well-known generalization


that can be used to calculate the number of permutations
of n objects when various subsets of those objects consist
of objects to be treated as identical. We give four
examples in which such a calculation can be used.

1. The number of 8-letter words that can be formed using


the letters of PEPPERER is equal to = 560.

2. Consider a 3-dimensional analog of the rectangular


grid above, with dimensions 2 by 3 by 3. The number of
paths from the front-left-down vertex to the back-
right-up vertex such that each path consists of traveling
only back, right, or up is equal to = 560.

3. The number of ways to distribute 8 different books to


David, Liam, and Sophia so that David is given 2 of the
books, Liam is given 3 of the books, and Sophia is given
3 of the books is equal to = 560.
4. The coefficient of a2b3c3 in the expansion of
(a + b + c)8 after like terms are combined is equal to
= 560.

The correct answer is E.

PS92751.01

4. The figures above show a hexagonal nut that has a width of inches and a wrench that, in
order to fit the nut, must have a width of at least inches. Of all the wrenches that fit the
nut and have widths that are whole numbers of millimeters, the wrench that fits the nut most
closely has a width of how many millimeters?
(Note: 1 inch ≈ 25.4 millimeters)
A. 30
B. 31
C. 32
D. 33
E. 34
Arithmetic Measurement conversion
The width of the nut in millimeters is nearly equal to

. Since

, it follows that the width of the nut is between 33


mm and 34 mm.
The correct answer is E.
PS45461.01

5. Pat will walk from intersection X to intersection Y along a route that is confined to the square
grid of four streets and three avenues shown in the map above. How many routes from X to Y
can Pat take that have the minimum possible length?
A. Six
B. Eight
C. Ten
D. Fourteen
E. Sixteen
Arithmetic Elementary combinatorics
Each minimum-length route will consist of traveling a total of 3 grid segments up and 2 grid
segments right. Thus, letting ‘U' represent traveling up by one grid segment and ‘R' represent
traveling right by one grid segment, each minimum-length route can be uniquely represented
by an appropriate 5-character string of U's and R's. For example, URUUR represents grid
segments traveled in the order up, right, up, up, and right. Therefore, the number of possible
minimum-length routes is equal to the number of appropriate 5-character strings of U's and

R's, which is = 10, since each appropriate string is determined when a

specification is made for the 3 positions in the string at which the U's are to be placed.

Tip

The ideas involved in counting the number of 5-character


strings of U's and R's can be extended to counting the
number of n-character strings when various characters
being used are the same. For example, MISSISSIPPI has
one M, four I's, four S's, and two P's, and thus the
number of 11-letter words that can be formed using the
letters of MISSISSIPPI is equal to

. (Each of

the two expressions we have given for 34,650 is intended


to suggest a method for counting the number of 11-letter
words.)

The correct answer is C.


PS95302.01

6. Rita and Sam play the following game with n sticks on a table. Each must remove 1, 2, 3, 4 or
5 sticks at a time on alternate turns, and no stick that is removed is put back on the table. The
one who removes the last stick (or sticks) from the table wins. If Rita goes first, which of the
following is a value of n such that Sam can always win no matter how Rita plays?
A. 7
B. 10
C. 11
D. 12
E. 16
Arithmetic Elementary combinatorics
Let Player A be either Rita or Sam, and let Player B be the other player. If, after one of Player
A's turns, there are exactly 6 sticks left, then Player A can win on his or her next turn. This is
because if 6 sticks are left after Player A's turn, then regardless of whether Player B removes
1, 2, 3, 4, or 5 sticks, it follows that Player A can win on his or her next turn by removing,
respectively, 5, 4, 3, 2, or 1 stick.
n = 7: If Rita begins by removing 1 stick, then there will be 6 sticks left after Rita's turn.
Therefore, by the remarks above, Rita can win. Hence, Sam cannot always win.
n = 10: If Rita begins by removing 4 sticks, then there will be 6 sticks left after Rita's turn.
Therefore, by the remarks above, Rita can win. Hence, Sam cannot always win.
n = 11: If Rita begins by removing 5 sticks, then there will be 6 sticks left after Rita's turn.
Therefore, by the remarks above, Rita can win. Hence, Sam cannot always win.
n = 12: If Rita begins by removing 1 stick, then Sam can win by removing 5 sticks on his next
turn, because 6 sticks will remain after Sam's turn. If Rita begins by removing 2 sticks, then
Sam can win by removing 4 sticks on his next turn, because 6 sticks will remain after Sam's
turn. By continuing in this manner, we see that if Rita begins by removing k sticks (where k is
one of the numbers 1, 2, 3, 4, or 5), then Sam can win by removing (6−k) sticks on his next
turn because 6 sticks will remain after Sam's turn. Therefore, no matter how many sticks Rita
removes on her first turn, Sam can win by removing appropriate numbers of sticks on his
next two turns. Hence, Sam can always win.
n = 16: If Rita removes 4 sticks on her first turn, then Sam will be in the same situation as
Rita for n = 12 above, and therefore Rita can win no matter what Sam does. Hence, Sam
cannot always win.
The correct answer is D.
PS65402.01

7. When of the votes on a certain resolution have been counted, of those counted are in
favor of the resolution. What fraction of the remaining votes must be against the resolution
so that the total count will result in a vote of 2 to 1 against the resolution?

A.

B.

C.

D.

E.

Arithmetic Operations on rational numbers


For this problem, by assigning carefully chosen numbers to quantities given in the problem,
it can be made more concrete and some of the computations with fractions can be avoided.
Since of all the votes have been counted and of them are for the resolution, 36 ( = 9 × 4)
would be a good number to use as the total number of votes cast. Since the total count must
result in a vote of 2 to 1 against the resolution, of all of the votes must be against the
resolution. This information can be summarized in the following table.
Total votes cast Counted so far Still to be counted

36 (36) = 8 36 − 8 = 28

For (8) = 6

Against (36) = 24 8−6=2 24 − 2 = 22

From the table, it is clear that of the 28 votes still to be counted, 22 must be against the
resolution. Therefore, the fraction of the votes still to be counted that must be against the
resolution is = .

In general, letting T represent the total number of votes cast, since the total count must result
in a vote of 2 to 1 against the resolution, T votes must be against the resolution. The
information is summarized in the following table.
Total votes cast Counted so far Still to be counted

T T T− T= T

For
= T

Against T = T T− T= T

From the table, it is clear that of the T votes still to be counted T must be against the
resolution. Therefore, the fraction of the votes still to be counted that must be against the

resolution is = .

The correct answer is A.


Alternative explanation:
Assign actual numbers to the problem to make the math more concrete. Since we are dealing
with of something and also of something, we will want our numbers to be convenient.

Look for multiples of 36 (9 times 4) for which and will result in whole numbers. A
number that will work well is 180.

Of the 180 votes, have been counted.

(180) = 40 votes counted. This means 140 votes have not been counted.

Of those 40 counted votes, are in favor. (40) = 30 votes in favor (of the 40 counted).

This means 10 votes are not in favor (of the 40 counted).

Looking ahead to the desired end result, in order to achieve a 2:1 ratio against, of the votes

will be for and will be against. Therefore we will need 120 votes against. So far we have 10
votes not in favor.
In order to reach a total of 120 uncounted votes, of the 140 uncounted votes, we will need 110
votes not in favor to combine with the 10 counted votes not in favor.

This is or . The correct answer is A.

PS85402.01

8. The sum of the first 100 positive integers is 5,050. What is the sum of the first 200 positive
integers?
A. 10,100
B. 10,200
C. 15,050
D. 20,050
E. 20,100
Arithmetic Sequences

The sum of the first n positive integers is given by , so the sum of the first

200 positive integers is = 20,100.

Alternatively, letting represent the sum of the first 100 positive integers, it is given that

= 5,050. Using this notation, = + = + + = +

= 2(5,050) + 10,000

= 10,100 + 10,000 = 20,100.


The correct answer is E.

Month Average Price per Dozen


April $1.26
May $1.20
June $1.08
PS40502.01

9. The table above shows the average (arithmetic mean) price per dozen eggs sold in a certain
store during three successive months. If as many dozen were sold in April as in May, and
twice as many were sold in June as in April, what was the average price per dozen of the eggs
sold over the three-month period?
A. $1.08
B. $1.10
C. $1.14
D. $1.16
E. $1.18
Arithmetic Statistics
Given that the numbers of eggs sold in each of the three months are in the ratio 2:3:4, it
follows that of the eggs sold in the three-month period were sold at an average of $1.26 per

dozen, were sold at $1.20 per dozen, and were sold at $1.08 per dozen. Therefore,
the average price per dozen of the eggs sold in the three-month period was
= $1.16.

The correct answer is D.


PS96602.01
10. Each of the integers from 0 to 9, inclusive, is written on a separate slip of blank paper and
the ten slips are dropped into a hat. If the slips are then drawn one at a time without
replacement, how many must be drawn to ensure that the numbers on two of the slips drawn
will have a sum of 10 ?
A. Three
B. Four
C. Five
D. Six
E. Seven
Arithmetic Elementary combinatorics
To simplify the discussion, we will refer to the drawing of the slip of paper with the integer n
written on it as “drawing the integer n.” The number of integers that must be drawn is at
least seven, because if the six integers 0 through 5 were drawn, then no two of the integers
drawn will have a sum of 10. In fact, it is easy to see that the sum of any two of these six
integers is less than 10.
0, 1, 2, 3, 4, 5
Of the answer choices, only seven is not eliminated.
Although it is not necessary to show that seven is the least number of integers that must be
drawn to ensure there exists a pair of the drawn integers that has a sum of 10, we provide a
proof that seven is the least such number. Thus, we will show that if seven integers were
drawn, then there exists a pair of the drawn integers that has a sum of 10. Since the integer 0
is such that none of the other integers can be paired with 0 to give a sum of 10, and similarly
for the integer 5, it will suffice to show that if five integers were drawn from the eight integers
1, 2, 3, 4, 6, 7, 8, and 9, then there exists a pair of the drawn integers that has a sum of 10.
Note that each of these eight integers differs from 5 by one of the numbers 1, 2, 3, or 4, as
shown below.
1=5−4 6=5+1
2=5−3 7=5+2
3=5−2 8=5+3
4=5−1 9=5+4
With these preliminaries out of the way, assume that five integers have been drawn from
these eight integers. Of the five integers that have been drawn, at least two must differ from 5
by the same number, say k, and since these two integers must be different, it follows that one
of these two integers is 5 + k and the other is 5 − k, and hence these two integers have a sum
of 10.
The correct answer is E.
PS15402.01

11. King School has an enrollment of 900 students. The school day consists of 6 class periods
during which each class is taught by one teacher. There are 30 students per class. Each
teacher teaches a class during 5 of the 6 class periods and has one class period free. No
students have a free class period. How many teachers does the school have?
A. 25
B. 30
C. 36
D. 60
E. 150
Algebra Statistics
If each teacher has 5 class periods a day and each class has 30 students, then each teacher
has (5)(30) students per day. Each of the 900 students has 6 classes per day, from which it
follows that all the teachers combined have a total of (6)(900) students per day. Therefore,
the school has or 36 teachers.

The correct answer is C.


PS07602.01

12. Ben and Ann are among 7 contestants from which 4 semifinalists are to be selected. Of the
different possible selections, how many contain neither Ben nor Ann?
A. 5
B. 6
C. 7
D. 14
E. 21
Arithmetic Elementary combinatorics
The number of possible selections of 4 semifinalists that do not contain Ben or Ann is equal
to the number of possible selections of 4 semifinalists from the remaining 7 − 2 = 5

contestants, which is equal to = 5. Alternatively, the number of possible

selections of 4 semifinalists from the remaining 5 contestants is equal to the number of


possible selections of exactly 1 non-semifinalist, which is equal to 5.
The correct answer is A.
Answer Explanations Quantitative Reasoning
Problem Solving
Equalities/Inequalities/Algebra

PS03551.01

13. Let n and k be positive integers with k ≤ n. From an n × n array of dots, a k × k array of dots is
selected. The figure above shows two examples where the selected k × k array is enclosed in a
square. How many pairs (n, k) are possible so that exactly 48 of the dots in the n × n array
are NOT in the selected k × k array?
A. 1
B. 2
C. 3
D. 4
E. 5
Algebra Factoring; Simultaneous equations
The n × n array has n2 dots and the k × k array has k2 dots. The number of dots in the n × n
array that are not in the k × k array is given by n2 – k2 = (n – k)(n + k).
Therefore, (n – k)(n + k) = 48 is a necessary condition for there to be 48 dots not in the k × k
array. This is also a sufficient condition, since it is clear that at least one k × k array of dots
can be selected for removal from an n × n array of dots when k ≤ n.
The equation (n – k)(n + k) = 48 represents two positive integers, namely n – k and n + k,
whose product is 48. Thus, the smaller integer n – k must be 1, 2, 3, 4, or 6, and the larger
integer n + k must be 48, 24, 16, 12, or 8. Rather than solving five pairs of simultaneous
equations (for example, n – k = 2 and n + k = 24 is one such pair), it is more efficient to
observe that the solution to the system n – k = a and n + k = b is (add the

equations, then divide by 2) and (substitute for n in either equation


and solve for k; or subtract the equations, then divide by 2). Therefore, the possible pairs (n,
k) arise exactly when 48 = ab and both a + b and b – a are divisible by 2. This occurs exactly
three times—48 = (2)(24), 48 = (4)(12), and 48 = (6)(8).
The correct answer is C.
PS41471.01

14. If there is a least integer that satisfies the inequality ≥ 2, what is that least integer?

A. 0
B. 1
C. 4
D. 5
E. There is not a least integer that satisfies the inequality.
Algebra Inequalities

It is clear that no negative integer satisfies the inequality (because is false) and

zero does not satisfy the inequality (because is undefined). Thus, the integers, if any, that

satisfy ≥ 2 must be among 1, 2, 3, 4, …. The least of these integers is 1, and it is easy to see

that x = 1 satisfies the inequality ≥ 2. Therefore, the least integer that satisfies the
inequality is 1.
Alternatively, the inequality can be solved algebraically. It will be convenient to consider
three cases according to whether x < 0, x = 0, and x > 0.
Case 1: Assume x < 0. Then multiplying both sides of the inequality by x, which is negative,
gives 9 ≤ 2x, or x ≥ 4.5. Because we are assuming x < 0, there are no solutions to x ≥ 4.5.
Therefore, no solutions exist in Case 1.

Case 2: Assume x = 0. Then is not defined, and thus x = 0 cannot be a solution.

Case 3: Assume x > 0. Then multiplying both sides of the inequality by x, which is positive,
gives 9 ≥ 2x, or x ≤ 4.5. Because we are assuming x > 0, the solutions that exist in Case 2 are
all real numbers x such that 0 < x ≤ 4.5.

The set of all solutions to the inequality ≥ 2 will be all solutions found in Cases 1, 2, and 3.
Therefore, the solutions to the inequality consist of all real numbers x such that 0 < x ≤ 4.5.
The least of these solutions that is an integer is 1.
The correct answer is B.
x C(x)
0 25,000
10 24,919
20 24,846
30 24,781
40 24,724
50 24,675
PS18871.01

15. A certain manufacturer uses the function C(x) = 0.04x2 – 8.5x + 25,000 to calculate the cost,
in dollars, of producing x thousand units of its product. The table above gives values of this
cost function for values of x between 0 and 50 in increments of 10. For which of the following
intervals is the average rate of decrease in cost less than the average rate of decrease in cost
for each of the other intervals?
A. From x = 0 to x = 10
B. From x = 10 to x = 20
C. From x = 20 to x = 30
D. From x = 30 to x = 40
E. From x = 40 to x = 50
Arithmetic Applied problems
Since the average rate of decrease of C(x) in the interval from x = a to x = a + 10 is
, we are to determine for which value of a, chosen from

the numbers 0, 10, 20, 30, and 40, the magnitude of is the least, or
equivalently, for which of these values of a the magnitude of C(a + 10) − C(a) is the least.
Probably the most straightforward method is to simply calculate or estimate the difference
C(a + 10) – C(a) for each of these values of a, as shown in the table below.
a to a + 10 C(a + 10) – C(a)
0 to 10 –81
10 to 20 –73
20 to 30 –65
30 to 40 –57
40 to 50 –49

Alternatively, since the graph of C(x) = 0.04x2 – 8.5x + 25,000 is a parabola with vertex at
x= = 100, it follows that the graph levels out as the value of x

approaches a number that is approximately equal to 100. Therefore, among the intervals
given, the least magnitude in the average rate of change of C(x) occurs for the interval closest
to the vertex, which is the interval from x = 40 to x = 50.
The correct answer is E.
PS35302.01
16. On the day of the performance of a certain play, each ticket that regularly sells for less than
$10.00 is sold for half price plus $0.50, and each ticket that regularly sells for $10.00 or
more is sold for half price plus $1.00. On the day of the performance, a person purchases a
total of y tickets, of which x regularly sell for $9.00 each and the rest regularly sell for $12.00
each. What is the amount paid, in dollars, for the y tickets?
A. 7y – 2x
B. 12x – 7y

C.

D. 7y + 4x
E. 7y + 5x
Algebra Applied problems
The amount paid for the y tickets is the sum of the amounts paid for two groups of tickets.
The first group consists of x tickets, each of which regularly sells for $9.00. The second group
consists of the remaining (y – x) tickets, each of which regularly sells for $12.00. The amount
paid for the first group was x($4.50 + $0.50) = $5x. The amount paid for the second group
was (y – x)($6.00 + $1.00) = $7(y – x), or $7y − $7x. Therefore, the amount paid for the y
tickets was $5x + ($7y – $7x) = $(7y – 2x).
The correct answer is A.
PS47302.01

17. If N = , where T = and x = 5 – T, which of the following expresses y in terms of N and

T?

A.

B.

C.

D.

E.

Algebra Simplifying algebraic expressions


To eliminate K and x in the first equation (the only equation in which y appears), use the
second and third equations to replace K and x with expressions involving only N and T. Then
solve for y in terms of N and T.
Alternatively, the algebraic manipulations involved in solving this type of problem as above
can often be replaced with numerical computations by assigning values to the variables. The
assigned values need to be consistent with all the constraints in the problem, and, for
efficiency, the assigned values should be chosen to minimize the numerical computations.
Letting K = 10, it follows from T = and x = 5 – T that T = 2 and x = 3. Using these
numerical values, the question can be rephrased as follows.

If N = , then which of the following expresses y in terms of N ?

A.

B.

C.

D.

E.
Letting y = 1, it follows that N = = 2. Plugging N = 2 into A, B, C, D, and E above gives,

respectively, 1, –1, , , and .

The correct answer is A.


PS78302.01

18. If 2x + 5y = 8 and 3x = 2y, what is the value of 2x + y ?


A. 4
B.

C.

D.

E.

Algebra Simultaneous equations

From 3x = 2y, it follows that y = , so 8 = 2x + 5 = . Then x = , 2x = , and

y= . Thus 2x + y = .

The correct answer is D.


PS79302.01

19. If kSn is defined to be the product of (n + k)(n – k + 1) for all positive integers k and n, which
of the following expressions represents k + 1Sn + 1 ?
A. (n + k)(n – k + 2)
B. (n + k)(n – k + 3)
C. (n + k + 1)(n – k + 2)
D. (n + k + 2)(n – k + 1)
E. (n + k + 2)(n – k + 3)
Algebra Substitution; Simplifying algebraic expressions
Substituting n + 1 for n and k + 1 for k in the definition gives k + 1Sn + 1
= (n + 1 + k + 1)(n + 1 – (k + 1) + 1)
= (n + k + 2)(n – k + 1).
The correct answer is D.
PS20502.01

20. There were 36,000 hardback copies of a certain novel sold before the paperback version was
issued. From the time the first paperback copy was sold until the last copy of the novel was
sold, 9 times as many paperback copies as hardback copies were sold. If a total of 441,000
copies of the novel were sold in all, how many paperback copies were sold?
A. 45,000
B. 360,000
C. 364,500
D. 392,000
E. 396,900
Algebra First-degree equations
Let h be the number of hardback copies of the novel that were sold after the paperback
version was issued. The following table summarizes the given information.
Hardbacks sold Paperbacks sold Total
Before paperbacks 36,000 0 36,000
After paperbacks h 9h 10h
Then, 441,000 = 36,000 + 10h or h = 40,500 and 9h = (9)(40,500) = 364,500.
The correct answer is C.
Alternate Solution
Arithmetic Ratios
From the moment the paperback version was issued, 441,000 – 36,000 = 405,000 copies of
the novel were sold. Test the answer choices to see whether the ratio of paperbacks sold to
hardbacks sold is 9:1. Start with the middle value, because the answer choices are usually
listed in numerical order. This way, if you pick an answer that does not give the 9:1 ratio, you
can learn whether it was too high or too low, thus allowing you to eliminate other answers
that are too high or too low.
Testing answer choice C, if 364,500 paperbacks were sold, then 405,000 – 364,500 = 40,500
hardbacks were sold after the initial 36,000 hardbacks were sold. Noting that 40,500
hardbacks is 10% of 405,000 and 364,500 paperbacks is 90% of 405,000, the ratio of
paperbacks to hardbacks is 9:1.
The correct answer is C.
PS30502.01

21. In the formula w = , integers p and t are positive constants. If w = 2 when v = 1 and if w =

when v = 64, then t =

A. 1
B. 2
C. 3
D. 4
E. 16
Algebra Exponents
It is given that w = and w = 2 when v = 1. Because 1 raised to any positive power is 1, it

follows that 2 = P. If w = when v = 64, then = or 4 = . Then = = since

64 = 43. So, 4 = and t = 3.


The correct answer is C.

PS03502.01

22. The figure above represents a network of one-way streets. The arrows indicate the direction
of traffic flow, and the numbers indicate the amount of traffic flow into or out of each of the
four intersections during a certain hour. During that hour, what was the amount of traffic
flow along the street from R to S if the total amount of traffic flow into P was 1,200 ? (Assume
that none of the traffic originates or terminates in the network.)
A. 200
B. 250
C. 300
D. 350
E. 400
Arithmetic Computation with integers
In the following, the notation A→B will be used to represent the amount of traffic flow from
A into B. Let x represent R→S. From the figure, the amount of traffic flow into P was 800
plus S→P. The total amount of traffic flow into P was 1,200, so S→P was 400. The amount of
traffic flow into S was 550 plus R→S, so 550 + x. The amount of traffic flow out of S was 400
plus S→P, or 400 + 400 = 800. Since the amount of traffic flow into S must equal the
amount of traffic flow out of S, 550 + x = 800. Therefore, x = 250, so R→S was 250.
The correct answer is B.
PS23502.01

23. If C is the temperature in degrees Celsius and F is the temperature in degrees Fahrenheit,
then the relationship between temperatures on the two scales is expressed by the equation
9C = 5(F – 32). On a day when the temperature extremes recorded at a certain weather
station differed by 45 degrees on the Fahrenheit scale, by how many degrees did the
temperature extremes differ on the Celsius scale?
A.

B. 13
C. 25
D. 45
E. 81
Algebra Formulas
Let F represent the larger extreme on the Fahrenheit scale. Then, F – 45 is the smaller
extreme. It follows that the difference in the temperatures on the Celsius scale is
= 25.

The correct answer is C.


PS93502.01

24. If d = , a = , and b = , what is the value of d in terms of c ?

A.

B.

C.

D.

E.

Algebra Simplifying algebraic expressions

First, a + b = + = and 1 + =1+ =1+ = . Then, d = = =

The correct answer is A.


PS04502.01

25. A school supply store sells only one kind of desk and one kind of chair, at a uniform cost per
desk or per chair. If the total cost of 3 desks and 1 chair is twice that of 1 desk and 3 chairs,
then the total cost of 4 desks and 1 chair is how many times that of 1 desk and 4 chairs?
A. 5
B. 3
C.

D.

E.

Algebra Simultaneous equations


Let d represent the cost of 1 desk and let c represent the cost of 1 chair. It is given that
3d + c = 2(d + 3c). It follows that d = 5c. Then 4d + c = 21c and d + 4c = 9c. Since ,

the total cost of 4 desks and 1 chair is times that of 1 desk and 4 chairs.

The correct answer is E.


PS35502.01

26. A certain truck traveling at 55 miles per hour gets 4.5 miles per gallon of diesel fuel
consumed. Traveling at 60 miles per hour, the truck gets only 3.5 miles per gallon. On a 500-
mile trip, if the truck used a total of 120 gallons of diesel fuel and traveled part of the trip at
55 miles per hour and the rest at 60 miles per hour, how many miles did it travel at 55 miles
per hour?
A. 140
B. 200
C. 250
D. 300
E. 360
Algebra Applied problems
Let m be the number of miles the truck traveled at 55 miles per hour. It follows that 500 – m
is the number of miles the truck traveled at 60 miles per hour. Then, is the number of

gallons the truck used while traveling at 55 miles per hour and is the number of

gallons the truck used while traveling at 60 miles per hour, so = 120. Solving
this equation gives m = 360.
The correct answer is E.
PS45502.01

27. A merchant paid $300 for a shipment of x identical calculators. The merchant used two of
the calculators as demonstrators and sold each of the others for $5 more than the average
(arithmetic mean) cost of the x calculators. If the total revenue from the sale of the
calculators was $120 more than the cost of the shipment, how many calculators were in the
shipment?
A. 24
B. 25
C. 26
D. 28
E. 30
Algebra Second-degree equations
The merchant paid $300 for a shipment of x calculators, so the average cost, in dollars, per
calculator was . The merchant sold (x – 2) of them at the price of 5 + dollars each, for

a total revenue of (x – 2) dollars, which was 120 + 300 = 420. Manipulating the

equation (x – 2) = 420 gives x 2 – 26x – 120 = 0 or (x – 30)(x + 4) = 0, which can


be solved by factoring. It follows that there were 30 calculators in the shipment.
The correct answer is E.
PS06502.01

28. A car traveled 462 miles per tankful of gasoline on the highway and 336 miles per tankful of
gasoline in the city. If the car traveled 6 fewer miles per gallon in the city than on the
highway, how many miles per gallon did the car travel in the city?
A. 14
B. 16
C. 21
D. 22
E. 27
Algebra Applied problems
Let g be the number of gallons of gasoline in 1 tankful. Then the number of miles per gallon
while traveling on the highway is and this number is 6 more than , which is the

number of miles per gallon while traveling in the city. Solving =6+ gives g = 21.

Therefore, the number of miles per gallon while the car was traveling in the city was
= 16.
The correct answer is B.
PS56502.01

29. Machines X and Y run at different constant rates, and machine X can complete a certain job
in 9 hours. Machine X worked on the job alone for the first 3 hours and the two machines,
working together, then completed the job in 4 more hours. How many hours would it have
taken machine Y, working alone, to complete the entire job?
A. 18

B. 13
C. 7

D. 4

E. 3

Algebra Applied problems

Machine X can complete the job in 9 hours, so it completed of the job before machine Y

started working. Since machine X worked 4 more hours, it completed of the job, which left

of the job for machine Y to do during the 4 hours it worked together with machine X to

complete the job. At this rate, machine Y would take = 18 hours to do the entire job

working alone.

Alternatively, machine X can complete the job in 9 hours, so it completed of the job before

machine Y started working. This leaves of the job, which took machines X and Y, working
together, 4 hours to complete. So, if working together the machines took 4 hours to complete
of the job, it would take them 6 hours to do the whole job working together. Furthermore,

rates are additive, so we should convert working times to rates. Machine X's rate is jobs per

hour. Both machines together have a rate of jobs per hour. We must determine machine Y's

individual rate for the whole job. First, add machine X's rate to machine Y 's rate to get the
combined rate. Then,

The correct answer is A.


PS77602.01

30. If , then the value of which of the following can be determined?


A. I only
B. III only
C. I and II only
D. II and III only
E. I, II, and III
Algebra Simplifying algebraic expressions
Substitute 2t for s in the expressions given in I, II, and III.

✓ I. Value can be determined: =1

✓ II. Value can be determined: =1

× III. Value cannot be determined: equals 0 if t = 1, and equals if t = 2

The correct answer is C.


PS58602.01

31. If k ≠ 0 and k – , then x =

A. –3 – k2
B. k2 – 3
C. 3k2 – 3
D. k – 3 – 2k2
E. k – 3 + 2k2
Algebra Simplifying algebraic expressions
Multiplying both sides of the equation by k gives k2 – (3 – 2k2) = x or x = 3k2 – 3.
The correct answer is C.
PS68602.01

32. The sum of the ages of Doris and Fred is y years. If Doris is 12 years older than Fred, how
many years old will Fred be y years from now, in terms of y ?
A. y–6
B. 2y – 6
C. –6

D. –6

E. –6

Algebra Applied problems


Let D and F represent Doris's and Fred's current ages, respectively. It is given that D + F = y
and D = F + 12. It follows that (F + 12) + F = y and F = = – 6. Therefore, Fred's age

y years from now will be y + = – 6.

The correct answer is D.


Answer Explanations Quantitative Reasoning
Problem Solving
Geometry

PS35461.01

33. The shaded region in the figure above represents a rectangular frame with length 18 inches
and width 15 inches. The frame encloses a rectangular picture that has the same area as the
frame itself. If the length and width of the picture have the same ratio as the length and width
of the frame, what is the length of the picture, in inches?
A. 9

B.

C.

D. 15(1 − )

E.

Geometry Rectangles; Area


Let k be the proportionality constant for the fractional decrease from the dimensions of the
frame to the dimensions of the picture. That is, let 18k be the length of the picture and let 15k
be the width of the picture. We are given that (18)(15) = 2(18k)(15k). Hence, k2 = and k =

. Therefore, the length of the picture is 18k = .

The correct answer is A.


PS56271.01

34. The surface distance between 2 points on the surface of a cube is the length of the shortest
path on the surface of the cube that joins the 2 points. If a cube has edges of length 4
centimeters, what is the surface distance, in centimeters, between the lower left vertex on its
front face and the upper right vertex on its back face?
A. 8
B. 4
C. 8
D. 12
E. 4 +4
Geometry Rectangular solids
The left figure below shows a cube with edge length 4, where P is the lower left vertex on its
front face and Q is the upper right vertex on its back face. It is clear that the shortest path on
the surface between P and Q consists of a path on the front face joined to a path on the top
face, or a path on the front face joined to a path on the right face. In fact, each of these two
approaches can be used in essentially the same way to give a path from P to Q, whose length
is the surface distance between P and Q. To simplify the exposition, we will consider the case
where the surface distance is the length of a certain path PF on the front face plus the length
of a certain path PT on the top face.
The middle figure below shows the top face of the cube lifted about 45 degrees, and the right
figure below shows only the front and top faces of the cube after the top face has been lifted
90 degrees. Since rotations of the top face about this “hinge” do not change the length of any
path on the front face or on the top face, this rotation by 90 degrees will not change the
length of PF or the length of PT, and hence this rotation will not change the sum of the
lengths of PF and PT.
In the right figure below, the shortest path from P to Q is the dashed segment shown in the
figure. Moreover, the length of this dashed segment is the surface distance between P and Q
because if PF and PT did not correspond to portions of this dashed segment on the front and
top faces, respectively, then the sum of the lengths of PF and PT would be greater than the
length of the dashed segment, since the shortest distance between P and Q in the right figure
below is the length of the line segment with endpoints P and Q.

From the discussion above, it follows that the surface distance between P and Q is the length
of the dashed segment in the right figure, which is easily found by using the Pythagorean
theorem: .

The correct answer is B.


PS75571.01

35. The figure above shows 2 circles. The larger circle has center A, radius R cm, and is inscribed
in a square. The smaller circle has center C, radius r cm, and is tangent to the larger circle at
point B and to the square at points D and F. If points A, B, C, and E are collinear, which of
the following is equal to ?

A.

B.

C.

D.

E.

Geometry Circles; Pythagorean theorem


Because is a diagonal of square CDEF, which has side length r, it follows from the
Pythagorean theorem that r2 + r2 = (CE)2, and hence CE = .

Tip

A sometimes useful shortcut is the fact that, for a square


we have d = , where d is the diagonal length and s is
the side length. This can be obtained by applying the
Pythagorean theorem as above or by using properties of a
45–45–90 triangle.
Therefore, BE = r + = r(1 + ) and AE = R + r(1 + ). Since 2(AE) is the diagonal
length of the large square, which has side length 2R, it follows from the above tip that
2(AE) = (2R) , or AE = R . Alternatively, an appropriate application of the Pythagorean
theorem gives R2 + R2 = (AE)2, or AE = R . Now substitute for AE and solve for .

From the last equation we get .

The correct answer is D.

PS15302.01

36. shows the dimensions of a rectangular board that is to be cut into four identical pieces by
making cuts at points A, B, and C, as indicated. If x = 45, what is the length AB ?
(1 foot = 12 inches)
A. 5 ft 6 in
B. 5 ft in
C. 5 ft 3 in
D. 5 ft
E. 4 ft 9 in
Geometry Rectangles; Triangles

The figure above shows the left side of the rectangular board with points E, F, G, and H
added and segment added. We are to determine the value of AB, which equals the value
of FH. Since ΔAFG is an isosceles triangle, it follows that FG = FA = 0.5 ft. Moreover,
EF = GH because the four pieces of the rectangular board have the same dimensions.
Therefore, since EH is half the length of the 20 ft board, EH = 10 ft and we have
EF + FG + GH = 10 ft, or 2(GH) + 0.5 ft = 10 ft, or GH = 4.75 ft. Hence,
AB = FH = FG + GH = 0.5 ft + 4.75 ft = 5.25 ft, which equals 5 ft 3 in.
The correct answer is C.

PS57302.01

37. In the figure above, the area of the shaded region is


A.
B.
C.
D.

E.
Geometry Triangles; Area
First, the diagonal of the square with sides of length 4 is . From this it follows that the
area of the shaded region consists of the area of an equilateral triangle with sides minus
the area of a square with sides of length 4. Thus, the area of the shaded region is

= .

The correct answer is D.


PS18302.01

38. If each side of ΔACD above has length 3 and if AB has length 1, what is the area of region
BCDE ?

A.

B.

C.

D.

E.
Geometry Triangles; Area
The area of region BCDE is the area of ΔACD minus the area of ΔABE. Since ΔACD is
equilateral, its area is . ΔABE is a 30–60–90 triangle with side lengths 1, ,

and 2 and area . Thus, the area of region BCDE is − = .

The correct answer is B.


PS76402.01

39. In the figure above, three squares and a triangle have areas of A, B, C, and X as shown. If
A = 144, B = 81, and C = 225, then X =
A. 150
B. 144
C. 80
D. 54
E. 36
Geometry Triangles; Quadrilaterals; Area
The side lengths of the squares with areas 144, 81, and 225 are 12, 9, and 15, respectively, so
the triangle with area X has sides 12, 9, and 15. Because 122 + 92 = 152, the triangle with area
X is a right triangle with legs of length 12 and 9. Thus, X = = 54.

The correct answer is D.

PS57402.01
40. In the figure above, PQ is a diameter of circle O, PR = SQ, and RST is equilateral. If the
length of PQ is 2, what is the length of RT ?

A.

B.

C.

D.

E.
Geometry Triangles
Since PR = SQ, it follows that RO = OS, so O is the midpoint of . Since ΔRST is equilateral
and O is the midpoint of , ΔROT is a 30°–60°–90° triangle, and since is a radius of the
circle with diameter 2, OT = 1. Using the ratios of the sides of a 30°–60°–90° triangle, it
follows that RT = .

The correct answer is D.

PS22502.01

41. The figure above shows some of the dimensions of a triangular plaza with an L-shaped walk
along two of its edges. If the width of the walk is 4 feet and the total area of the plaza and
walk together is 10,800 square feet, what is the value of x ?
A. 200
B. 204
C. 212
D. 216
E. 225
Geometry Polygons; Area

The area of the triangular plaza is given by , and the area of the walkway is
given by (x + 4)(4) + (96)(4). Since the total area of the plaza and walkway is 10,800, it
follows that 52x + 400 = 10,800 and x = 200.
The correct answer is A.
PS88602.01

42. A circular rim 28 inches in diameter rotates the same number of inches per second as a
circular rim 35 inches in diameter. If the smaller rim makes x revolutions per second, how
many revolutions per minute does the larger rim make in terms of x ?

A.

B. 75x
C. 48x
D. 24x
E.

Geometry Circles; Circumference


Since the smaller rim has diameter 28 inches and rotates x revolutions per second, it rotates
28πx inches per second. If y represents the number of revolutions the larger rim rotates per
second, then the larger rim rotates 35πy inches per second. Since the rims rotate the same
number of inches per second, it follows that 28πx = 35πy. Then y = inches per

second or = 48x inches per minute.

The correct answer is C.


Answer Explanations Quantitative Reasoning
Problem Solving
Rates/Ratios/Percent
PS17302.01

43. The annual stockholders' report for Corporation X stated that profits were up 10 percent over
the previous year, although profits as a percent of sales were down 10 percent. Total sales for
that year were approximately what percent of sales for the previous year?
A. 78%
B. 90%
C. 110%
D. 122%
E. 190%
Algebra Percents
Let P1 and S1 be the profit and sales for the previous year, and let P2 and S2 be the profit and

sales for the following year. It is given that P2 = 1.1P1 and . Substituting the first

equation into the second equation gives , or .

Therefore, S2 is approximately 122% of S1.


The correct answer is D.
PS47402.01

44. A certain brand of house paint must be purchased either in quarts at $12 each or in gallons at
$18 each. A painter needs a 3-gallon mixture of the paint consisting of 3 parts blue and 2
parts white. What is the least amount of money needed to purchase sufficient quantities of
the two colors to make the mixture?
(4 quarts = 1 gallon)
A. $54
B. $60
C. $66
D. $90
E. $144
Arithmetic Applied problems
To make 3 gallons of the mixture requires 12 quarts, and the least amount of money is
achieved by purchasing the greatest number of gallons and least number of quarts. Letting B
be the number of quarts of blue paint needed and W be the number of quarts of white paint
needed, it follows that B + W = 12, where B = since the mixture has a blue to white ratio

of 3 to 2. This gives B = and W = . Since the paint can be purchased in whole quarts
only, the painter must purchase 8 quarts or 2 gallons of blue, and 5 quarts or 1 gallon plus 1
quart of white for a total of (2 + 1)($18) + $12 = $66.
The correct answer is C.

Month Change in sales from previous month


February +10%
March −15%
April +20%
May −10%
June +5%
PS43481.01

45. The table above shows the percent of change from the previous month in Company X's sales
for February through June of last year. A positive percent indicates that Company X's sales
for that month increased from the sales for the previous month, and a negative percent
indicates that Company X's sales for that month decreased from the sales for the previous
month. For which month were the sales closest to the sales in January?
A. February
B. March
C. April
D. May
E. June
Arithmetic Percents
Explicit calculation incorporating a few numerical shortcuts gives the following, where J is
the January sales amount.
February sales: 1.1J
March sales: 0.85(1.1J) = 0.85J + 0.85(0.1J) = 0.85J + 0.085J = 0.935J
April sales: 1.2(0.935J) = 0.935J + 0.2(0.935J) = 0.935J + 0.187J = 1.122J
May sales: 0.9(1.122J) = 1.0098J
June sales: 1.05(1.0098J) (May is clearly closer)
Alternatively, from

it follows that a percent change of x% followed by a percent change of y% is equal to a


percent change of (x + y)% plus x% of y percentage points (equivalently, plus y% of
x percentage points).
Percent change from January through February: The percent change is given as +10%.
Percent change from January through March: Using the rule above for x = +10 and y = −15
gives (+10 − 15)% + (+0.10)(−15%), or −5% − 1.5% = −6.5%.
Percent change from January through April: This is equivalent to a −6.5% change (percent
change from January through March) followed by a +20% change, and hence using the rule
above for x = −6.5 and y = +20 gives (−6.5 + 20)% + (+0.20)(−6.5%), or
+13.5% − 1.3% = +12.2%.
Percent change from January through May: This is equivalent to a +12.2% change (percent
change from January through April) followed by a −10% change, and hence using the rule
above for x = +12.2 and y = −10 gives (+12.2 − 10)% + (−0.10)(+12.2%), or
+2.2% − 1.22% = +0.98%.
Percent change from January through June: This is clearly greater than +0.98% + 5%,
and hence greater in magnitude than the result for May.
From the results above, the least change in the magnitude of the percent change from
January occurred for May.
The correct answer is D.
PS56302.01

46. A store bought 5 dozen lamps at $30 per dozen and sold them all at $15 per lamp. The profit
on each lamp was what percent of its selling price?
A. 20%
B. 50%
C. 83 %

D. 100%
E. 500%
Arithmetic Applied problems; Percents
For this problem it is especially important to keep your focus on what is asked and to ignore
extraneous details.

The cost per lamp is and the selling price per lamp is $15, so the profit per lamp

is . Therefore, for each lamp the profit as a percent of the selling price is

The correct answer is C.


PS76302.01

47. Store N gives a 50 percent discount on the list price of all its items and Store W gives a 60
percent discount on the list price of all its items. If the list price of the same item is 20
percent higher in Store W, what percent (more or less) of the selling price in Store N is the
selling price of the item in Store W ?
A. 10% less
B. 4% less
C. 2% less
D. 10% more
E. 12% more
Arithmetic Percents
Let $P be the list price of the item in Store N. The table shows the list and selling prices of the
item in the two stores. For example, the selling price of the item in Store W is 60 percent less
than the item's list price of $1.2P, or (0.4)($1.2P) = $0.48P.
List price ($) Selling price ($)
Store N P 0.5P
Store W 1.2P 0.48P
The amount, in dollars, by which the selling price of the item in Store W is less than the
selling price of the item in Store N is 0.5P − 0.48P = 0.02P, which is = 4% less as a
percent of the selling price in Store N.
Alternatively, we can assign a specific value to P and carry out the computations using this
value. In percent problems, the computations are usually simpler when 100 is used, so let
P = 100. (If, say, of P had been involved, then P = 300 might be a better choice.)
Therefore, the list price of the item in Store N is $100, the discount price of the item in Store
N is (50%)($100) = $50, the list price of the item in Store W is (120%)($100) = $120, and the
discount price of the item in Store W is (40%)($120) = $48. Thus, the question becomes the
following. What percent (more or less) of 50 is 48 ? A simple computation shows that 48 is
less than 50.

The correct answer is B.


PS95402.01

48. A merchant purchased a jacket for $60 and then determined a selling price that equaled the
purchase price of the jacket plus a markup that was 25 percent of the selling price. During a
sale, the merchant discounted the selling price by 20 percent and sold the jacket. What was
the merchant's gross profit on this sale?
A. $0
B. $3
C. $4
D. $12
E. $15
Algebra Percents
The purchase price, in dollars, of the jacket was 60. If S represents the selling price, in
dollars, then S = 60 + 0.25S, from which S = 80. During the sale, the discounted selling price,
in dollars, was 0.8(80) = 64, so the merchant's gross profit, in dollars, was 64 − 60 = 4.
The correct answer is C.
Tip

Read carefully. Usually, markup is a percent of the


purchase price, but in this problem, it is a percent of the
selling price.

PS12502.01

49. When a certain stretch of highway was rebuilt and straightened, the distance along the
stretch was decreased by 20 percent and the speed limit was increased by 25 percent. By
what percent was the driving time along this stretch reduced for a person who always drives
at the speed limit?
A. 16%
B. 36%

C. 37 %

D. 45%
E. 56 %

Arithmetic Applied problems; Percents


Let D and r be the distance and speed limit, respectively, along the stretch of highway before
it was rebuilt. Then, the distance and speed limit along the stretch of highway after it was
rebuilt are given by 0.8D and 1.25r. It follows that the percent reduction in time is

= = 36%.

Alternatively, it helps to use actual numbers when calculating percent change, with 100 being
the most mathematically convenient number to use. For this problem, set the speed limit at
100 miles per hour and the distance at 100 miles. A 20 percent decrease in distance makes
the new distance 80 miles. A 25 percent increase in speed limit makes the new speed limit
125 miles per hour, which, of course, is unrealistic, but very convenient to work with. Then,
the new time is hours. The percent change in time is
= 36%.
The correct answer is B.
Answer Explanations Quantitative Reasoning
Problem Solving
Value/Order/Factors
Number of components:
Components Monday Tuesday Wednesday
A 3 6 3
B 6 3 4
C 4 7 4
PS56441.01

50. A factory assembles Product X from three components, A, B, and C. One of each component
is needed for each Product X and all three components must be available when assembly of
each Product X starts. It takes two days to assemble one Product X. Assembly of each
Product X starts at the beginning of one day and is finished at the end of the next day. The
factory can work on at most five Product Xs at once. If components are available each day as
shown in the table above, what is the largest number of Product Xs that can be assembled
during the three days covered by the table?
A. 3
B. 5
C. 6
D. 9
E. 10
Arithmetic Applied problems
We will determine the largest number of Product Xs that can be assembled during all three
days by considering separately the largest number that can be assembled if 0, 1, 2, or 3
Product Xs begin assembly on Monday.
0 Product Xs begin assembly on Monday: In this case, at most three Product Xs can begin
assembly on Tuesday (because only three units of Component A are available on
Wednesday), and hence at most 0 + 3 = 3 Product Xs could be assembled during the three
days.
1 Product X begins assembly on Monday: In this case, at most three Product Xs can begin
assembly on Tuesday (because only three units of Component A are available on
Wednesday), and hence at most 1 + 3 = 4 Product Xs could be assembled during the three
days.
2 Product Xs begin assembly on Monday: In this case, at most three Product Xs can begin
assembly on Tuesday (because only three units of Component A are available on
Wednesday), and hence at most 2 + 3 = 5 Product Xs could be assembled during the three
days.
3 Product Xs begin assembly on Monday: In this case, at most two Product Xs can begin
assembly on Tuesday (because the factory can work on at most five Product Xs at once), and
hence at most 3 + 2 = 5 Product Xs could be assembled during the three days.
Therefore, the largest number of Product Xs that can be assembled during the three days is 5.
The correct answer is B.
PS04851.01

51. How many positive integers n have the property that both 3n and are 4-digit integers?

A. 111
B. 112
C. 333
D. 334
E. 1,134
Arithmetic Inequalities
If n is an integer, then 3n is always an integer. Also, 3n will be a 4-digit integer only when
1,000 ≤ 3n ≤ 9,999. Therefore, n is an integer such that 333 ≤ n ≤ 3,333. Equivalently, n is
an integer such that 334 ≤ n ≤ 3,333.
If n is an integer, then is an integer only when n is a multiple of 3. Also, will be a 4-digit
integer only when 1,000 ≤ ≤ 9,999, or 3,000 ≤ n ≤ 29,997. Therefore, n is a multiple of 3
such that 3,000 ≤ n ≤ 29,997.
It follows that the values of n consist of all multiples of 3 between 3,000 = 3(1,000) and
3,333 = 3(1,111), inclusive. The number of such multiples of 3 is (1,111 − 1,000) + 1 = 112.

Tip

Be alert to possible easily overlooked constraints that


may exist in a problem. For example, in applying the
second requirement above, it is not sufficient to only
consider integer values of n such that 1,000 ≤ ≤ 9,999.
In addition, must also be an integer, and by applying
this constraint it follows that the values of n must be
multiples of 3.

The correct answer is B.


PS24851.01

52. If Whitney wrote the decimal representations for the first 300 positive integer multiples of 5
and did not write any other numbers, how many times would she have written the digit 5 ?
A. 150
B. 185
C. 186
D. 200
E. 201
Arithmetic Properties of integers
The number of times the digit 5 would be written is the number of times the digit 5 will
appear in the units place plus the number of times the digit 5 will appear in the tens place
plus the number of times the digit 5 will appear in the hundreds place.
Number of times the digit 5 will appear in the units place: This will be the number of terms
in the sequence 5, 15, 25, 35, …, 1485, 1495. Adding 5 to each member of this sequence does
not change the number of terms, and doing this gives the sequence 10, 20, 30, 40, …, 1490,
1500, which clearly has 150 terms (dividing the terms by 10 gives 1, 2, 3, 4, …, 149, 150).
Thus, the digit 5 appears 150 times in the units place.
Number of times the digit 5 will appear in the tens place: This will be the number of terms
in the sequence 50, 55, 150, 155, 250, 255, …, 1450, 1455. The digit 5 appears in the tens place
twice for each consecutive change in the hundreds digit. Thus, the digit 5 appears 2(15) = 30
times in the tens place.
Number of times the digit 5 will appear in the hundreds place: This will be the number of
terms in the sequence 500, 505, 510, 515, …, 590, 595, 1500. Thus, the digit 5 appears
20 + 1 = 21 times in the hundreds place.
Therefore, the number of times the digit 5 would be written is 150 + 30 + 21 = 201.

Tip

The method used above to count the number of terms in


the sequence 5, 15, 25, 35, …, 1,485, 1,495 can be applied
to any arithmetic sequence, and it avoids the necessity of
remembering certain formulas. For example, to
determine the number of terms in the sequence 13, 19,
25, 31, 37, 43, …, 301, we first observe that consecutive
differences are equal to 6, so we subtract from each term
a number chosen so that the first term becomes (1)
(6) = 6. Thus, we subtract 7 from each term and obtain
the sequence 6, 12, 18, 24, 30, 36, …, 294, which has the
same number of terms as the original sequence.The
number of terms in this new sequence is now easy to find
—divide each term of this new sequence by 6, and it will
be clear that the number of terms is 49.

Alternatively, in the 2-digit multiples of 5, namely the multiples of 5 in the interval 5−95,
there are twelve occurrences of the digit 5. The same number of occurrences of the digit 5
appear in the multiples of 5 in each of the intervals 100−195, 200−295, 300−395, and
400−495. For the multiples of 5 in the interval 500−595, there are the same corresponding
twelve occurrences of the digit 5 plus twenty more for the digit in hundreds place for each of
the twenty multiples of 5 in 500−595, for a total of thirty-two occurrences of the digit 5. For
the multiples of 5 in each of the intervals 600−695, 700−795, 800−895, 900−995,
1,000−1,095, 1,100−1,195, 1,200−1,295, 1,300−1,395, and 1,400–1,495, there are twelve
occurrences of the digit 5. Finally, there is one occurrence of the digit 5 in 1,500. Therefore,
the total number of occurrences of the digit 5 in the first 300 multiples of 5 is
14(12) + 32 + 1 = 201.
The correct answer is E.
PS01661.01

53. The difference 942 − 249 is a positive multiple of 7. If a, b, and c are nonzero digits, how
many 3-digit numbers abc are possible such that the difference abc − cba is a positive
multiple of 7 ?
A. 142
B. 71
C. 99
D. 20
E. 18
Arithmetic Place value
Since abc is numerically equal to 100a + 10b + c and cba is numerically equal to
100c + 10b + a, it follows that abc − cba is numerically equal to
(100 − 1)a + (10 − 10)b + (1 − 100)c = 99(a − c). Because 7 and 99 are relatively prime,
99(a − c) will be divisible by 7 if and only if a − c is divisible by 7. This leads to two choices
for the nonzero digits a and c, namely a = 9, c = 2 and a = 8, c = 1. For each of these two
choices for a and c, b can be any one of the nine nonzero digits. Therefore, there is a total of
2(9) = 18 possible 3-digit numbers abc.
The correct answer is E.
PS41661.01

54. Let S be the set of all positive integers having at most 4 digits and such that each of the digits
is 0 or 1. What is the greatest prime factor of the sum of all the numbers in S ?
A. 11
B. 19
C. 37
D. 59
E. 101
Arithmetic Properties of integers
By writing down all the positive integers in S, their sum can be found.
1 10 11 100 101
110 111 1,000 1,001 1,010
1,011 1,100 1,101 1,110 1,111
The sum of these integers is 8,888. Since this sum is 8 × 1,111 = 23 × 11 × 101 (note that
1,111 = (11 × 100) + 11), it follows that 101 is the largest prime factor of the sum.
Alternatively, we can simplify the description by letting the integers having fewer than four
digits be represented by four-digit strings in which one or more of the initial digits is 0. For
example, the two-digit number 10 can be written as
0010 = (0 × 103) + (0 × 102) + (1 × 101) + (0 × 100). Also, we can include 0 = 0000, since the
inclusion of 0 will not affect the sum. With these changes, it follows from the Multiplication
Principle that there are 24 = 16 integers to be added. Moreover, for each digit position (units
place, tens place, etc.) exactly half of the integers will have a digit of 1 in that digit position.
Therefore, the sum of the 16 integers will be (8 × 103) + (8 × 102) + (8 × 101) + (8 × 100), or
8,888. Note that this alternative method of finding the sum is much quicker than the other
method if “at most four digits” had been “at most seven digits.” In the case of “at most seven
digits,” there will be 27 = 128 integers altogether, and for each digit position, half of the
integers will have a digit of 1 in that digit position and the other half will have a digit of 0 in
that digit position. Thus, the sum will be (64 × 106) + (64 × 105) + … + (64 × 100) =
71,111,104. Incidentally, finding the greatest prime factor of 71,111,104 is not appropriate for a
GMAT problem, but in this case a different question about the sum could have been asked.
The correct answer is E.
Age Tax only Tax and fees Fees only
18–39 20 30 30
≥40 10 60 100
PS43661.01

55. The table above shows the number of residents in each of two age groups who support the
use of each type of funding for a city initiative. What is the probability that a person
randomly selected from among the 250 residents polled is younger than 40, or supports a
type of funding that includes a tax, or both?

A.

B.

C.

D.

E.

Arithmetic Probability
The requested probability is the number of residents described divided by the total number
(250) of residents. The number of residents described is equal to the number of residents of
age 18−39 (20 + 30 + 30 = 80) PLUS the number of residents of age ≥ 40 who support tax
only (10) PLUS the number of residents of age ≥ 40 who support tax and fees (60).
Therefore, the requested probability is .
Alternatively, the requested probability is 1 minus the number of residents NOT described
(100) divided by 250, or .

The correct answer is D.


PS55471.01

56. Which of the following describes the set of all possible values of the positive integer k such
that, for each positive odd integer n, the value of is midway between consecutive integers?

A. All positive integers greater than 2


B. All prime numbers
C. All positive even integers
D. All even prime numbers
E. All positive even multiples of 5
Arithmetic Properties of integers
The logical complexities involved in this question are lessened by testing individual values of
k.

Does k = 1 satisfy the condition? NO. If n = 1, then = 1 is not midway between


consecutive integers.

Does k = 2 satisfy the condition? YES. For each positive odd integer n, the value of is
an odd integer divided by 2, and hence is midway between consecutive integers.

Does k = 3 satisfy the condition? NO. If n = 1, then is not midway between consecutive
integers.
It is easy to see that no other positive integer satisfies the condition, by considering the value
of when n = 1. Therefore, 2 is the only positive integer value of k that satisfies the
condition.
The correct answer is D.
PS92981.01

57. A certain online form requires a 2-digit code for the day of the month to be entered into one
of its fields, such as 04 for the 4th day of the month. The code is valid if it is 01, 02, 03, …, 31
and not valid otherwise. The transpose of a code xy is yx. For example, 40 is the transpose of
04. If N is the number of valid codes having a transpose that is not valid, what is the value of
N?
A. 12
B. 13
C. 18
D. 19
E. 20
Arithmetic Operations with integers
It will be quicker to count the number of valid codes whose transposes are valid codes, and
then subtract the result from 31 (the number of valid codes) to obtain the value of N.
Three such codes begin with 0: 01, 02, 03. The rest are such that their transposes are invalid
codes.
Four such codes begin with 1: 10, 11, 12, 13. The rest are such that their transposes are
invalid codes.
Three such codes begin with 2: 20, 21, 22. The rest are such that their transposes are invalid
codes.
Two such codes begin with 3: 30, 31. The rest are invalid codes.
Therefore, the total number of valid codes whose transposes are valid codes is
3 + 4 + 3 + 2 = 12, and hence the total number of valid codes whose transposes are not valid
is 31 − 12 = 19.
The correct answer is D.
PS25302.01

58. If x < y < z and y − x > 5, where x is an even integer and y and z are odd integers, what is the
least possible value of z − x ?
A. 6
B. 7
C. 8
D. 9
E. 10
Algebra Inequalities
Since y − x > 5, it follows that y must be one of the integers
x + 6, x + 7, x + 8, x + 9, …
Also, because x is even and y is odd, y cannot be an even integer added to x, and thus y must
be one of the integers
x + 7, x + 9, x + 11, x + 13, …
Since z > y and both y and z are odd integers, it follows that z must be one of the integers
y + 2, y + 4, y + 6, y + 8, …
Therefore, the least possible value of z − x occurs when y is 7 greater than x and z is 2 greater
than y, which implies that z is 7 + 2 = 9 greater than x.
The correct answer is D.
PS36302.01

59. An “Armstrong number” is an n-digit number that is equal to the sum of the nth powers of its
individual digits. For example, 153 is an Armstrong number because it has 3 digits and
13 + 53 + 33 = 153. What is the digit k in the Armstrong number 1,6k 4 ?
A. 2
B. 3
C. 4
D. 5
E. 6
Arithmetic Operations with integers
If k = 1, then 1,6k4 = 1,614 and 14 + 64 + 14 + 44 is equal to 1 + 1,296 + 1 + 256 = 1,554 ≠
1,614. Therefore, k = 1 does not produce an Armstrong number.
If k = 2, then 1,6k4 = 1,624 and 14 + 64 + 24 + 44 is equal to
1 + 1,296 + 16 + 256 = 1,569 ≠ 1,624. Therefore, k = 2 does not produce an Armstrong
number.
If k = 3, then 1,6k4 = 1,634 and 14 + 64 + 34 + 44 is equal to 1 + 1,296 + 81 + 256 = 1,634.
Therefore, k = 3 produces an Armstrong number.
Alternatively, the condition that 1,6k4 is an Armstrong number can be expressed by the
equation 14 + 64 + k4 + 44 = 1,000 + 600 + 10k + 4, or 1,553 + k4 = 1,604 + 10k. This can be
rewritten as k(k3 − 10) = 51. Therefore, k must be a factor of 51, and 3 is the only answer
choice that is a factor of 51.
The correct answer is B.
PS30402.01

60. Five integers between 10 and 99, inclusive, are to be formed by using each of the ten digits
exactly once in such a way that the sum of the five integers is as small as possible. What is the
greatest possible integer that could be among these five numbers?
A. 98
B. 91
C. 59
D. 50
E. 37
Arithmetic Place value
Note that 0 cannot be the tens digit of an integer between 10 and 99, inclusive. In order for
the sum of the five integers to be as small as possible, their tens digits should be the five
smallest remaining digits (that is, 1, 2, 3, 4, and 5), leaving the digits 0, 6, 7, 8, and 9 to be
used as the ones digits. Let a, b, c, d, and e represent distinct digits chosen from the digits 0,
6, 7, 8, and 9. The sum of the five integers formed in this way is as small as possible and
equals (10 + a) + (20 + b) + (30 + c) + (40 + d) + (50 + e)
= 150 + (a + b + c + d + e) = 150 + 30 = 180 regardless of how the digits 0, 6, 7, 8, and 9 are
assigned to a, b, c, d, and e. By assigning 9 to e, it follows that one of the integers could be 59.
Therefore, 59 is the greatest possible integer among the five integers whose sum is 180.
The correct answer is C.
PS66402.01

61. When the integer n is divided by 17, the quotient is x and the remainder is 5. When n is
divided by 23, the quotient is y and the remainder is 14. Which of the following is true?
A. 23x + 17y = 19
B. 17x − 23y = 9
C. 17x + 23y = 19
D. 14x + 5y = 6
E. 5x − 14y = −6
Algebra Remainders; Simplifying algebraic expressions
It is given that n = 17x + 5 and n = 23y + 14. It follows that 17x + 5 = 23y + 14, so
17x − 23y = 9.
The correct answer is B.
PS17402.01

62. Of the following, which is greatest?


A.
B.

C.

D.

E.

Arithmetic Operations with radical expressions


Probably the easiest way to determine which of the given radical expressions is greatest is to
compare their squares, which are 18, 12, , , and . Clearly, 18 is the greatest of the
squares, so is the greatest of the given radical expressions.
The correct answer is A.
PS37402.01

63. If n = p2 and p is a prime number greater than 5, what is the units digit of n2 ?
A. 1
B. 3
C. 4
D. 7
E. 9
Arithmetic Properties of integers
First, all prime numbers greater than 5 are odd numbers with units digit 1, 3, 7, or 9. Note
that no prime number greater than 5 has units digit 5. The following table summarizes the
results for the possible cases.

Units digit of p Units digit of p2 = n Units digit of p4 = n2


1 1 1
3 9 1
7 9 1
9 1 1
The correct answer is A.
PS83502.01

64. A computer can perform 1,000,000 calculations per second. At this rate, how many hours
will it take this computer to perform the 3.6 × 1011 calculations required to solve a certain
problem?
A. 60
B. 100
C. 600
D. 1,000
E. 6,000
Arithmetic Measurement conversion

It will take this computer = 3.6 × 105 seconds to perform the calculations. Since

there are 3,600 seconds in 1 hour, this is equivalent to = 100 hours.

The correct answer is B.


PS66602.01

65. In an auditorium, 360 chairs are to be set up in a rectangular arrangement with x rows of
exactly y chairs each. If the only other restriction is that 10 < x < 25, how many different
rectangular arrangements are possible?
A. Four
B. Five
C. Six
D. Eight
E. Nine
Arithmetic Properties of integers
Because a total of 360 chairs are distributed in x rows of exactly y chairs each, it follows that
xy = 360. Also, 10 < x < 25, and so x can only be an integer factor of 360 = 23 × 32 × 5 that is
between 10 and 25. Below is a list of all integers from 11 through 24. Since 2, 3, and 5 are the
only prime factors of 360, any integer having a prime factor other than 2, 3, or 5 cannot be a
factor of 360 and has been crossed out. For example, 21 = 3 × 7 has a prime factor of 7, and
thus 21 has been crossed out.
11, 12, 13, 14, 15, 16, 17, 18, 19, 20, 21, 22, 23, 24
Of the six integers remaining, it is clear that each is a factor of 360 = 23 × 32 × 5 except for 16
= 24. Therefore, the number of possible rectangular arrangements is five.
The correct answer is B.
PS28602.01

66. If the product of the integers w, x, y, and z is 770, and if 1 < w < x < y < z, what is the value of
w+z?
A. 10
B. 13
C. 16
D. 18
E. 21
Arithmetic Properties of integers
The prime factorization of 770 is 2 × 5 × 7 × 11, so w = 2, z = 11, and w + z = 13.
The correct answer is B.

PS78602.01

67. The addition problem above shows four of the 24 different integers that can be formed by
using each of the digits 1, 2, 3, and 4 exactly once in each integer. What is the sum of these 24
integers?
A. 24,000
B. 26,664
C. 40,440
D. 60,000
E. 66,660
Arithmetic Place value
Each digit 1, 2, 3, and 4 will appear six times in each of 1,000s place, 100s place, 10s place,
and units place. Since 1 + 2 + 3 + 4 = 10, it follows that the sum of the 24 integers is (6)(10)
(1,000) + (6)(10)(100) + (6)(10)(10)
+ (6)(10)(1) = 66,660.
The correct answer is E.
Answer Explanations Quantitative Reasoning
Data Sufficiency
Counting/Sets/Series
DS19350.01

68. A country's per capita national debt is its national debt divided by its population. Is the per
capita national debt of Country G within $5 of $500 ?
1. Country G's national debt to the nearest $1,000,000,000 is $43,000,000,000.
2. Country G's population to the nearest 1,000,000 is 86,000,000.
A. Statement (1) ALONE is sufficient, but statement (2) alone is not sufficient.
B. Statement (2) ALONE is sufficient, but statement (1) alone is not sufficient.
C. BOTH statements TOGETHER are sufficient, but NEITHER statement ALONE is
sufficient.
D. EACH statement ALONE is sufficient.
E. Statements (1) and (2) TOGETHER are NOT sufficient.
Arithmetic Estimation
Briefly, the question is whether

495 ≤ ≤ 505 is true.

1. We are given that 42.5 billion ≤ debt < 43.5 billion. However, no information is given
about the population; NOT sufficient.
2. We are given that 85.5 million ≤ population < 86.5 million. However, no information is
given about the debt; NOT sufficient.

Given (1) and (2), then to determine whether lies between 495 and 505, we will

investigate the minimum possible value of and the maximum possible value of

. To simplify the discussion that follows, we will assume that the upper bounds

given in (1) and (2) can be achieved. This will not affect our conclusions because with this
assumption, the estimates we obtain will still justify our conclusions. The minimum possible

value of is × 1,000. Since

, which is less than , it follows that the

minimum possible value of is less than × 1,000 = 500 − 5 = 495. The


maximum possible value of is

. Since

, which is greater than , it follows that

the maximum possible value of is greater than


× 1,000 = 500 + 5 = 505.

Since the minimum possible value of is less than 495 and the maximum possible

value of is greater than 505, it is not possible to determine whether or not

lies between 495 and 505.

The correct answer is E;


both statements together are still not sufficient.
DS06351.01

69. The cardinality of a finite set is the number of elements in the set. What is the cardinality of
set A ?
1. 2 is the cardinality of exactly 6 subsets of set A.
2. Set A has a total of 16 subsets, including the empty set and set A itself.
A. Statement (1) ALONE is sufficient, but statement (2) alone is not sufficient.
B. Statement (2) ALONE is sufficient, but statement (1) alone is not sufficient.
C. BOTH statements TOGETHER are sufficient, but NEITHER statement ALONE is
sufficient.
D. EACH statement ALONE is sufficient.
E. Statements (1) and (2) TOGETHER are NOT sufficient.
Arithmetic Concepts of sets
Let n be the cardinality of the finite set A. What is the value of n ?
1. The number of 2-element subsets of A is equal to the number of unordered selections
without replacement of 2 objects from a collection of n distinct objects, or “n choose 2.”

Therefore, we have = 6, or equivalently, n2 – n – 12 = 0. Because this is a

quadratic equation that may have two solutions, we need to investigate further to
determine whether there is a unique value of n. Factoring leads to (n – 4)(n + 3) = 0, and
thus n = 4 or n = −3. Since n must be a nonnegative integer, we discard the solution
n = −3. Therefore, n = 4; SUFFICIENT.
2. The number of subsets of set A is 2n, because each subset of A corresponds to a unique
sequence of answers to yes-no questions about membership in the subset (one question
for each of the n elements). For example, let A = {1, 2, 3, 4, 5}, let Y represent “yes,” and
let N represent “no.” Then the sequence NYNNN corresponds to the subset {2}, since the
answers to “is 1 in the subset,” “is 2 in the subset,” “is 3 in the subset,” etc. are “no,” “yes,”
“no,” etc. Also, the subset {1, 3, 4} of A corresponds to the 5-letter sequence YNYYN.
Since the number of 5-letter sequences such that each letter is either N or Y is 25, it
follows that there are 25 = 32 subsets of {1, 2, 3, 4, 5}. For Statement (2), we are given that
2n = 16, and hence n = 4; SUFFICIENT.
Alternatively, observe that {1} has two subsets, {1, 2} has four subsets, and each addition of a
new element doubles the number of subsets, because the subsets after adding the new
element will consist of all the previous subsets along with the new element included in each
of the previous subsets. Thus, {1, 2, 3} has 2(4) = 8 subsets, {1, 2, 3, 4} has 2(8) = 16 subsets,
{1, 2, 3, 4, 5} has 2(16) = 32 subsets, etc.
The correct answer is D;
each statement alone is sufficient.
DS59851.01

70. For each positive integer k, let a k = . Is the product a1a2 … an an integer?

1. n + 1 is a multiple of 3.
2. n is a multiple of 2.
A. Statement (1) ALONE is sufficient, but statement (2) alone is not sufficient.
B. Statement (2) ALONE is sufficient, but statement (1) alone is not sufficient.
C. BOTH statements TOGETHER are sufficient, but NEITHER statement ALONE is
sufficient.
D. EACH statement ALONE is sufficient.
E. Statements (1) and (2) TOGETHER are NOT sufficient.
Algebra Series and sequences

Since , it follows that the product can be written as

. Therefore, the product a1a2 … an is an integer if and only if is an integer, or if


and only if n is an even integer.

1. If n = 2, then n + 1 = 3 is a multiple of 3 and the product is , which is an

integer. However, if n = 5, then n + 1 = 6 is a multiple of 3 and the product is


, which is not an integer; NOT sufficient.
2. If n is a multiple of 2, then by the remarks above it follows that the product is an
integer; SUFFICIENT.
The correct answer is B;
statement 2 alone is sufficient.
DS95491.01

71. Let S be a set of outcomes and let A and B be events with outcomes in S. Let ∼B denote the set
of all outcomes in S that are not in B and let P(A) denote the probability that event A occurs.
What is the value of P(A) ?
1. P(A B) = 0.7
2. P(A ∼B) = 0.9
A. Statement (1) ALONE is sufficient, but statement (2) alone is not sufficient.
B. Statement (2) ALONE is sufficient, but statement (1) alone is not sufficient.
C. BOTH statements TOGETHER are sufficient, but NEITHER statement ALONE is
sufficient.
D. EACH statement ALONE is sufficient.
E. Statements (1) and (2) TOGETHER are NOT sufficient.
Arithmetic Probability; Sets
The general addition rule for sets applied to probability gives the basic probability equation
P(A B) = P(A) + P(B) – P(A B).
1. Given that P(A B) = 0.7, it is not possible to determine the value of P(A) because
nothing is known about the relation of event A to event B. For example, if every outcome
in event B is an outcome in event A, then A B = A and we have P(A B) = P(A) = 0.7.
However, if events A and B are mutually exclusive (i.e., P(A B) = 0) and P(B) = 0.2,
then the basic probability equation above becomes 0.7 = P(A) + 0.2 – 0, and we have
P(A) = 0.5; NOT sufficient.
2. Given that P(A ∼B) = 0.9, it is not possible to determine the value of P(A) because
nothing is known about the relation of event A to event ∼B. For example, as indicated in
the first figure below, if every outcome in event ∼B is an outcome in event A, then A
∼B = A and we have P(A ∼B) = P(A) = 0.9. However, as indicated in the second figure
below, if events A and ∼B are mutually exclusive (i.e., P(A ∼B) = 0) and P(∼B) = 0.2,
then the basic probability equation above, with ∼B in place of B, becomes
0.9 = P(A) + 0.2 – 0, and we have P(A) = 0.7; NOT sufficient.

Given (1) and (2), if we can express event A as a union or intersection of events A B and A
∼B, then the basic probability equation above can be used to determine the value of P(A).
The figure below shows Venn diagram representations of events A B and A ∼B by the
shading of appropriate regions.

Inspection of the figure shows that the only portion shaded in both Venn diagrams is the
region representing event A. Thus, A is equal to the intersection of A B and A ∼B, and
hence we can apply the basic probability equation with event A B in place of event A and
event A ∼B in place of event B. That is, we can apply the equation
P(C D) = P(C) + P(D) – P(C D)
with C = A B and D = A ∼B. We first note that P(C) = 0.7 from (1), P(D) = 0.9 from (2),
and P(C D) = P(A). As for P(C D), inspection of the figure above shows that C D
encompasses all possible outcomes, and thus P(C D) = 1. Therefore, the equation above
involving events C and D becomes 1 = 0.7 + 0.9 – P(A), and hence P(A) = 0.6.
The correct answer is C;
both statements together are sufficient.
DS41402.01

72. What is the number of integers that are common to both set S and set T ?
1. The number of integers in S is 7, and the number of integers in T is 6.
2. U is the set of integers that are in S only or in T only or in both, and the number of
integers in U is 10.
A. Statement (1) ALONE is sufficient, but statement (2) alone is not sufficient.
B. Statement (2) ALONE is sufficient, but statement (1) alone is not sufficient.
C. BOTH statements TOGETHER are sufficient, but NEITHER statement ALONE is
sufficient.
D. EACH statement ALONE is sufficient.
E. Statements (1) and (2) TOGETHER are NOT sufficient.
Arithmetic Sets
In standard notation, and represent the intersection and union, respectively, of
sets S and T, and |S| represents the number of elements in a set S. Determine .
1. It is given that |S| = 7 and |T| = 6. If, for example, S = {1, 2, 3, 4, 5, 6, 7} and T =
{1, 2, 3, 4, 5, 6}, then = 6. However, if S = {1, 2, 3, 4, 5, 6, 7,} and T = {11, 12, 13, 14,
15, 16}, then = 0; NOT sufficient.
2. It is given that = 10. If, for example, S = {1, 2, 3, 4, 5, 6, 7} and T = {1, 2, 3, 8, 9,
10}, then = {1, 2, 3, 4, 5, 6, 7, 8, 9, 10}, = 10, and = 3. However, if
S = {1, 2, 3, 4, 5, 6, 7} and T = {11, 12, 13}, then = {1, 2, 3, 4, 5, 6, 7, 11, 12, 13},
= 10, and = 0; NOT sufficient.
Taking (1) and (2) together along with the general addition rule for two sets A and B
( = |A| + |B| – ) applied to sets S and T gives 10 = 7 + 6 – , from which
can be determined.
The correct answer is C;
both statements together are sufficient.
DS51402.01

73. What is the sum of 3 consecutive integers?


1. The sum of the 3 integers is less than the greatest of the 3 integers.
2. Of the 3 integers, the ratio of the least to the greatest is 3.
A. Statement (1) ALONE is sufficient, but statement (2) alone is not sufficient.
B. Statement (2) ALONE is sufficient, but statement (1) alone is not sufficient.
C. BOTH statements TOGETHER are sufficient, but NEITHER statement ALONE is
sufficient.
D. EACH statement ALONE is sufficient.
E. Statements (1) and (2) TOGETHER are NOT sufficient.
Algebra Sequences
Let k be the smallest of the three consecutive integers. It follows that k + 1 and k + 2 are the
other two integers and S = k + (k + 1) + (k + 2) = 3k + 3, where S is the sum of the three
consecutive integers. Determine S.
1. It is given that 3k + 3 < k + 2. It follows that
k< . If k = –1, then S = 0. However, if k = –2, then S = –3; NOT sufficient.

2. It is given that = 3. It follows that k = –3 and S can be determined; SUFFICIENT.

The correct answer is B;


statement 2 alone is sufficient.
DS54402.01

74. How many people in Town X read neither the World newspaper nor the Globe newspaper?
1. Of the 2,500 people in Town X, 1,000 read no newspaper.
2. Of the people in Town X, 700 read the Globe only and 600 read the World only.
A. Statement (1) ALONE is sufficient, but statement (2) alone is not sufficient.
B. Statement (2) ALONE is sufficient, but statement (1) alone is not sufficient.
C. BOTH statements TOGETHER are sufficient, but NEITHER statement ALONE is
sufficient.
D. EACH statement ALONE is sufficient.
E. Statements (1) and (2) TOGETHER are NOT sufficient.
Arithmetic Sets
Determine the number of people in Town X who read neither the World newspaper nor the
Globe newspaper.
1. It is given that 1,000 of the 2,500 people in Town X read no newspaper, from which it
follows that 1,500 people read at least one newspaper. It is possible that all of these 1,500
people read only the Earth, another newspaper that is read in Town X, so the number of
people who read neither the World nor the Globe could be 2,500. It is also possible that
all of the 1,500 people who read at least one newspaper read the World, so the number of
people who read neither the World nor the Globe could be 1,000; NOT sufficient.
2. It is given that 600 people read only the World and 700 people read only the Globe, but
there is no information about the number of people in Town X and no information about
other newspapers that might be read by the people in Town X; NOT sufficient.
Neither (1) nor (2) gives information as to whether other newspapers are read by people in
Town X, and without this information, the number of people who read neither the World nor
the Globe cannot be determined.
The correct answer is E;
both statements together are still not sufficient.

Tip

Just because the World and the Globe are the only
newspapers mentioned, do not assume that they are the
only newspapers read in Town X.

DS16402.01

75. Bowls X and Y each contained exactly 2 jelly beans, each of which was either red or black.
One of the jelly beans in bowl X was exchanged with one of the jelly beans in bowl Y. After
the exchange, were both of the jelly beans in bowl X black?
1. Before the exchange, bowl X contained 2 black jelly beans.
2. After the exchange, bowl Y contained 1 jelly bean of each color.
A. Statement (1) ALONE is sufficient, but statement (2) alone is not sufficient.
B. Statement (2) ALONE is sufficient, but statement (1) alone is not sufficient.
C. BOTH statements TOGETHER are sufficient, but NEITHER statement ALONE is
sufficient.
D. EACH statement ALONE is sufficient.
E. Statements (1) and (2) TOGETHER are NOT sufficient.
Arithmetic Sets
Determine after an exchange of one jelly bean if both jelly beans in bowl X are black.
1. Bowl X has two black jelly beans, but bowl Y could have two red or two black or one of
each color. If bowl Y has two red jelly beans, then bowl X will have one red and one black
jelly bean after the exchange, not two black. However, if bowl Y has two black jelly beans,
then bowl X will have two black after the exchange; NOT sufficient.
2. After the exchange, bowl Y has one red and one black. If, before the exchange, bowl X
had two black and bowl Y had two red, then after the exchange, each of bowl Y and bowl X
will have one of each color. However, if bowl Y had two black jelly beans, bowl X had one
of each color, and bowl X obtained a black in exchange for the red, then bowl Y will have
one of each color and bowl X will have two black; NOT sufficient.
Taking (1) and (2) together, bowl X had two black jelly beans to start with and bowl Y ended
up with one of each color after the exchange. bowl Y had to have had at least one red because
bowl X had no red jelly beans to give to bowl Y. If bowl Y has two red and exchanges a red for
one of bowl X's blacks, then, after the exchange, bowl Y will have one of each color, but bowl
X will not have two black. On the other hand, if bowl Y has one of each color and exchanges
its black for one of bowl X's blacks, then after the exchange, bowl Y will have one of each
color and bowl X will have two blacks.
The correct answer is E;
both statements together are still not sufficient.
DS27602.01

76. All trainees in a certain aviator training program must take both a written test and a flight
test. If 70 percent of the trainees passed the written test, and 80 percent of the trainees
passed the flight test, what percent of the trainees passed both tests?
1. 10 percent of the trainees did not pass either test.
2. 20 percent of the trainees passed only the flight test.
A. Statement (1) ALONE is sufficient, but statement (2) alone is not sufficient.
B. Statement (2) ALONE is sufficient, but statement (1) alone is not sufficient.
C. BOTH statements TOGETHER are sufficient, but NEITHER statement ALONE is
sufficient.
D. EACH statement ALONE is sufficient.
E. Statements (1) and (2) TOGETHER are NOT sufficient.
Algebra Sets
Let x be the percent of the trainees who passed both tests. The following Venn diagram
represents the information that is given as well as information that can be derived from what
is given. Note that x – 50 in the diagram can be found by using the requirement that the sum
of the four values in the diagram is 100. What is the value of x ?
1. Given that x – 50 = 10, it follows that x = 60; SUFFICIENT.
2. Given that 80 – x = 20, it follows that x = 60; SUFFICIENT.

Tip

A useful way to summarize the quantitative relations for a


two-circle Venn diagram is

Total = A + B − Both + Neither,

where A is the number of elements in Circle A, B is the


number of elements in Circle B, “Both” is the number of
elements in the intersection of the circles, and “Neither”
is the number of elements that do not belong to either of
the circles. If we think of A and B as the numbers of
elements in the two 1-way intersections (i.e., Circle A
alone and Circle B alone) and “Both” as the number of
elements in the single 2-way intersection (i.e., Circle A
intersects Circle B), then this equation can be written as

Total = (sum of 1-way) − (sum of 2-way) + None.

This second way of expressing the quantitative relations for a two-circle Venn diagram can be
modified to give a similar way of expressing the quantitative relations for a three-circle Venn
diagram:
Total = (sum of 1-way) − (sum of 2-way)
+ (sum of 3-way) + None.
Although Venn diagrams involving more than three circles will not likely be needed for the
GMAT, we recommend researching the inclusion-exclusion principle if the reader is
interested in further extensions of these ideas.
The correct answer is D;
each statement alone is sufficient.
Answer Explanations Quantitative Reasoning
Data Sufficiency
Equalities/Inequalities/Algebra
DS06110.01

77. Each of the five divisions of a certain company sent representatives to a conference. If the
numbers of representatives sent by four of the divisions were 3, 4, 5, and 5, was the range of
the numbers of representatives sent by the five divisions greater than 2 ?
1. The median of the numbers of representatives sent by the five divisions was greater
than the average (arithmetic mean) of these numbers.
2. The median of the numbers of representatives sent by the five divisions was 4.
A. Statement (1) ALONE is sufficient, but statement (2) alone is not sufficient.
B. Statement (2) ALONE is sufficient, but statement (1) alone is not sufficient.
C. BOTH statements TOGETHER are sufficient, but NEITHER statement ALONE is
sufficient.
D. EACH statement ALONE is sufficient.
E. Statements (1) and (2) TOGETHER are NOT sufficient.
Algebra Statistics
Let x be the unspecified number of representatives. By considering individual positive integer
values of x, the median of the numbers is found to be 4 when x = 1, 2, 3, or 4, and the median
of the numbers is found to be 5 when x ≥ 5. For example, the case in which x = 2 is shown
below.
2, 3, 4, 5, 5
1. In terms of x, the average of the numbers is . If x = 1, then by the

remarks above the median is 4, which is greater than (i.e., the median is greater
than the average), and the range is 5 − 1 = 4. If x = 5, then by the remarks above the
median is 5, which is greater than (i.e., the median is greater than the average),
and the range is 5 − 3 = 2; NOT sufficient.
2. Given the assumption that the median of the numbers is 4, it follows from the previous
remarks that x can be any one of the numbers 1, 2, 3, and 4. If x = 1, then the range is
5 − 1 = 4, which is greater than 2. If x = 4, then the range is 5 − 3 = 2, which is not greater
than 2; NOT sufficient.
Given (1) and (2), then from the previous remarks and (2) it follows that x must be among
the numbers 1, 2, 3, and 4. From (2) it follows that 4 > , or x < 3, and thus x is further
restricted to be among the numbers 1 and 2. However, for each of these possibilities the
range is greater than 2: If x = 1, then the range is 5 − 1 = 4 > 2; and if x = 2, then the range is
5 − 2 = 3 > 2.
The correct answer is C;
both statements together are sufficient.
DS24931.01

78. An investment has been growing at a fixed annual rate of 20% since it was first made; no
portion of the investment has been withdrawn, and all interest has been reinvested. How
much is the investment now worth?
1. The value of the investment has increased by 44% since it was first made.
2. If one year ago $600 had been withdrawn, today the investment would be worth 12%
less than it is actually now worth.
A. Statement (1) ALONE is sufficient, but statement (2) alone is not sufficient.
B. Statement (2) ALONE is sufficient, but statement (1) alone is not sufficient.
C. BOTH statements TOGETHER are sufficient, but NEITHER statement ALONE is
sufficient.
D. EACH statement ALONE is sufficient.
E. Statements (1) and (2) TOGETHER are NOT sufficient.
Algebra Applied problems
If the investment was initially worth $P, then after one year the investment was worth $1.2P,
after two years the investment was worth 1.2($1.2P) = $1.44P, after three years the
investment was worth 1.2($1.44P) = $1.728P, etc.
1. Given that the investment was worth $(P + 0.44P) = $1.44P, it follows from the
remarks above that the investment was first made two years ago. However, nothing is
known about the value of P; NOT sufficient.
2. Let $X be how much the investment was worth one year ago. Then the investment is
now worth $1.2X. However, if $600 had been withdrawn from the investment one year
ago, then the investment would have been worth $(X − 600) one year ago and the
investment would have been worth $1.2(X − 600) = $(1.2X − 720) today. It is given that
this amount, namely $(1.2X − 720), is 12 percent less than $1.2X. Therefore,
1.2X − 720 = (0.88)(1.2X). This equation can be solved for X, and using this value of X,
the value of 1.2X can be determined; SUFFICIENT.
The correct answer is B;
statement 2 alone is sufficient.
DS53841.01

79. X, 81, 73, 71, 98, 73, 64


What is the value of X in the above list of 7 numbers?
1. The average (arithmetic mean) of these 7 numbers is 80.
2. The range of these 7 numbers is 36.
A. Statement (1) ALONE is sufficient, but statement (2) alone is not sufficient.
B. Statement (2) ALONE is sufficient, but statement (1) alone is not sufficient.
C. BOTH statements TOGETHER are sufficient, but NEITHER statement ALONE is
sufficient.
D. EACH statement ALONE is sufficient.
E. Statements (1) and (2) TOGETHER are NOT sufficient.
Arithmetic Statistics
1. Since the average of the seven numbers is 80, it follows that the sum of the seven
numbers is 7(80) = 560. Therefore, X + 81 + 73 + 71 + 98 + 73 + 64 = 560, which can be
solved for a unique value of X; SUFFICIENT.
2. The range of the numbers when X is not included is 98 − 64 = 34. Therefore, if
X = 98 + 2 = 100, then the range of the seven numbers is 100 − 64 = 36, and if
X = 64 − 2 = 62, then the range of the seven numbers is 98 − 62 = 36. Therefore, more
than one value of X satisfies the given information and statement 2; NOT sufficient.
The correct answer is A;
statement 1 alone is sufficient.
DS01451.01

80. In the first 2 hours after Meadow's self-service laundry opens, m large washing machines
and n small washing machines are in continual use. Including the time for filling and
emptying the washing machines, each load of laundry takes 30 minutes in a large washing
machine and 20 minutes in a small washing machine. What is the total number of loads of
laundry done at Meadow's self-service laundry during this 2-hour period?
1. n = 3m
2. 2m + 3n = 55
A. Statement (1) ALONE is sufficient, but statement (2) alone is not sufficient.
B. Statement (2) ALONE is sufficient, but statement (1) alone is not sufficient.
C. BOTH statements TOGETHER are sufficient, but NEITHER statement ALONE is
sufficient.
D. EACH statement ALONE is sufficient.
E. Statements (1) and (2) TOGETHER are NOT sufficient.
Algebra Simultaneous equations
During the two-hour period each large washer does four loads (the number of 30-minute
periods in two hours is four) and each small washer does six loads (the number of 20-minute
periods in two hours is six). Therefore, the total number of loads done during the two-hour
period by m large washers and n small washers is 4m + 6n.
1. If m = 1 and n = 3, then n = 3m and the total number of loads of laundry done in the
two-hour period is 4(1) + 6(3) = 22. However, if m = 2 and n = 6, then n = 3m and the
total number of loads of laundry done in the two-hour period is 4(2) + 6(6) = 44; NOT
sufficient.
2. Since 2m + 3n = 55, it follows that 4m + 6n = 110. Therefore, by the remarks above, the
total number of loads done during the two-hour period is 110; SUFFICIENT.
The correct answer is B;
statement 2 alone is sufficient.
DS76851.01

81. A box of light bulbs contains exactly 3 light bulbs that are defective. What is the probability
that a sample of light bulbs picked at random from this box will contain at least 1 defective
light bulb?
1. The light bulbs in the sample will be picked 1 at a time without replacement.
2. The sample will contain exactly 20 light bulbs.
A. Statement (1) ALONE is sufficient, but statement (2) alone is not sufficient.
B. Statement (2) ALONE is sufficient, but statement (1) alone is not sufficient.
C. BOTH statements TOGETHER are sufficient, but NEITHER statement ALONE is
sufficient.
D. EACH statement ALONE is sufficient.
E. Statements (1) and (2) TOGETHER are NOT sufficient.
Arithmetic Statistics
It is clear that neither (1) alone nor (2) alone is sufficient.
Given (1) and (2), if the box contains 22 light bulbs, then a sample of 20 light bulbs must
contain at least one defective light bulb, and hence the desired probability is equal to 1.
However, if the box contains 22,000 light bulbs, then it is clear that the probability that a
sample of 20 light bulbs contains at least one defective light bulb is less than 1.
The correct answer is E;
both statements together are still not sufficient.
DS01951.01

82. Khalil drove 120 kilometers in a certain amount of time. What was his average speed, in
kilometers per hour, during this time?
1. If Khalil had driven at an average speed that was 5 kilometers per hour faster, his
driving time would have been reduced by 20 minutes.
2. If Khalil had driven at an average speed that was 25% faster, his driving time would
have been reduced by 20%.
A. Statement (1) ALONE is sufficient, but statement (2) alone is not sufficient.
B. Statement (2) ALONE is sufficient, but statement (1) alone is not sufficient.
C. BOTH statements TOGETHER are sufficient, but NEITHER statement ALONE is
sufficient.
D. EACH statement ALONE is sufficient.
E. Statements (1) and (2) TOGETHER are NOT sufficient.
Algebra Applied problems
Let r be Khalil's average speed in kilometers per hour and let t be Khalil's time in hours. Then
rt = 120, or t = . What is the value of r ?
1. If Khalil's speed had been (r + 5) kilometers per hour, then his driving time would have
been hours.

Although there are two possible values for r, namely r = −45 and r = 40, only the positive
value of r is consistent with the context. Therefore, r = 40; SUFFICIENT.
2. If Khalil's speed had been 1.25r kilometers per hour, then his driving time would have
been 0.8t hours. It follows that (1.25r)(0.8t) = 120, or rt = 120, which provides no
additional information to the given information; NOT sufficient.
The correct answer is A;
statement 1 alone is sufficient.
DS70061.01

83. What is the median of the data set S that consists of the integers 17, 29, 10, 26, 15, and x ?
1. The average (arithmetic mean) of S is 17.
2. The range of S is 24.
A. Statement (1) ALONE is sufficient, but statement (2) alone is not sufficient.
B. Statement (2) ALONE is sufficient, but statement (1) alone is not sufficient.
C. BOTH statements TOGETHER are sufficient, but NEITHER statement ALONE is
sufficient.
D. EACH statement ALONE is sufficient.
E. Statements (1) and (2) TOGETHER are NOT sufficient.
Arithmetic Statistics
1. Since the average of the six numbers is 17, it follows that the sum of the six numbers is
6(17). Therefore, 17 + 29 + 10 + 26 + 15 + x = 6(17), which can be solved for a unique
value of x, after which the median can be determined; SUFFICIENT.
2. The range of the numbers when x is not included is 29 − 10 = 19. Therefore, if
x = 29 + 5 = 34, then the range of the seven numbers (10, 15, 17, 26, 29, 34) is
34 − 10 = 24 and the median of the seven numbers is = 21.5.

However, if x = 10 − 5 = 5, then the range of the seven numbers (5, 10, 15, 17, 26, 29) is
29 − 5 = 24 and the median of the seven numbers is = 16; NOT sufficient.

The correct answer is A;


statement 1 alone is sufficient.
DS47661.01

84. If n > 4, what is the value of the integer n ?

1.

2.

A. Statement (1) ALONE is sufficient, but statement (2) alone is not sufficient.
B. Statement (2) ALONE is sufficient, but statement (1) alone is not sufficient.
C. BOTH statements TOGETHER are sufficient, but NEITHER statement ALONE is
sufficient.
D. EACH statement ALONE is sufficient.
E. Statements (1) and (2) TOGETHER are NOT sufficient.
Algebra Simplifying algebraic expressions
1. Because the numerators of the two fractions have several common factors, and
similarly for the denominators, a reasonable strategy is to begin by appropriately
canceling these common factors.
The manipulations above show that n = 7.
Alternatively, we could begin by reducing each of the fractions to lowest terms by using
identities such as n! = (n − 3)! × (n − 2)(n − 1)(n), and then performing operations on the
resulting equation; SUFFICIENT.
2. For the same reason given in (1) above, we begin by canceling factors that are common
on the left and right sides of the equality.

The manipulations above show that the original equation is identically true for all integers
greater than 4, and thus n can be any integer greater than 4.
Alternatively, we could begin by reducing each of the fractions to lowest terms by using
identities such as n! = (n − 3)! × (n − 2)(n − 1)(n), and then performing operations on the
resulting equation; NOT sufficient.
The correct answer is A;
statement 1 alone is sufficient.
DS50571.01
85. Tami purchased several identically priced metal frames and several identically priced
wooden frames for a total pretax price of $144. What was the total pretax price of the metal
frames that Tami purchased?
1. The price of each metal frame was 60% greater than the price of each wooden frame.
2. Tami purchased twice as many wooden frames as metal frames.
A. Statement (1) ALONE is sufficient, but statement (2) alone is not sufficient.
B. Statement (2) ALONE is sufficient, but statement (1) alone is not sufficient.
C. BOTH statements TOGETHER are sufficient, but NEITHER statement ALONE is
sufficient.
D. EACH statement ALONE is sufficient.
E. Statements (1) and (2) TOGETHER are NOT sufficient.
Algebra Simultaneous equations
Let m and w, respectively, be the number of metal frames and wooden frames that Tami
purchased, and let M and W, respectively, be their individual prices in dollars. We are given
that mM + wW = 144. What is the value of mM ?

1. Given that M = 1.6W = W, or W = M, we have mM + = 144. Although it

might seem there is not enough information to determine the value of mM, keep in mind
that we are only seeking the value of the product mM, and not the individual values of m
and M. Also, there are several implicit constraints involved, such as each of M and W
must be a positive number less than 144 and each of m and w must be a positive integer.
To see that more than one value of mM is possible, it will be convenient to specify a
particular value for w, for example, w = 16. Then mM + = 144 becomes

mM = 144 − 10M, and non-sufficiency is now straightforward. If w = 16 and M = 4, then


mM = 104. However, if w = 16 and M = 8, then mM = 64; NOT sufficient.
2. Given that w = 2m, we have mM + (2m)W = 144, or mM = 144 − 2mW. If m = 10 and
W = 4, then mM = 64. However, if m = 10 and W = 5, then mM = 44; NOT sufficient.

Given (1) and (2), we have W = M and w = 2m. Therefore, mM + wW = 144 becomes

mM + (2m) = 144, or mM = 144, and hence the value of mM can be

determined.
The correct answer is C;
both statements together are sufficient.
DS02871.01

86. A $10 bill (1,000 cents) was replaced with 50 coins having the same total value. The only
coins used were 5-cent coins, 10-cent coins, 25-cent coins, and 50-cent coins. How many 5-
cent coins were used?
1. Exactly 10 of the coins were 25-cent coins and exactly 10 of the coins were 50-cent
coins.
2. The number of 10-cent coins was twice the number of 5-cent coins.
A. Statement (1) ALONE is sufficient, but statement (2) alone is not sufficient.
B. Statement (2) ALONE is sufficient, but statement (1) alone is not sufficient.
C. BOTH statements TOGETHER are sufficient, but NEITHER statement ALONE is
sufficient.
D. EACH statement ALONE is sufficient.
E. Statements (1) and (2) TOGETHER are NOT sufficient.
Algebra Simultaneous equations
Let a, b, c, and d be the number, respectively, of 5-cent, 10-cent, 25-cent, and 50-cent coins.
We are given that a + b + c + d = 50 and 5a + 10b + 25c + 50d = 1,000, or
a + 2b + 5c + 10d = 200. Determine the value of a.
a + b + c + d = 50
a + 2b + 5c + 10d = 200

Tip

Note that each of a, b, c, and d must be a nonnegative


integer, and so care must be taken in deducing non-
sufficiency. For example, there are many real number
pairs (x,y) that satisfy the equation 2x + y = 1, but if each
of x and y must be a nonnegative integer, then x = 0 and
y = 1 is the only solution.

1. We are given that c = 10 and d = 10. Substituting c = 10 and d = 10 into the two
equations displayed above and combining terms gives a + b = 30 and a + 2b = 50.
Subtracting these last two equations gives b = 20, and hence it follows that a = 10;
SUFFICIENT.
2. We are given that b = 2a. Substituting b = 2a into the two equations displayed above
and combining terms gives a + 2a + c + d = 50 and a + 4a + 5c + 10d = 200, which are
equivalent to the following two equations.
3a + c + d = 50
a + c + 2d = 40
Subtracting these two equations gives 2a − d = 10, or 2a = d + 10. Since 2a is an even integer,
d must be an even integer. At this point it is probably simplest to choose various nonnegative
even integers for d to determine whether solutions for a, b, c, and d exist that have different
values for a. Note that it is not enough to find different nonnegative integer solutions to
2a = d + 10, since we must also ensure that c and d are nonnegative integers. If d = 8, then
2a = 8 + 10 = 18, and we have a = 9, b = 18, c = 15, and d = 8. However, if d = 10, then
2a = 10 + 10 = 20, and we have a = 10, b = 20, c = 10, and d = 10; NOT sufficient.
The correct answer is A;
statement 1 alone is sufficient.
DS56971.01

87. Merle's spare change jar has exactly 16 U.S. coins, each of which is a 1-cent coin, a 5-cent
coin, a 10-cent coin, a 25-cent coin, or a 50-cent coin. If the total value of the coins in the jar
is 288 U.S. cents, how many 1-cent coins are in the jar?
1. The exact numbers of 10-cent, 25-cent, and 50-cent coins among the 16 coins in the jar
are, respectively, 6, 5, and 2.
2. Among the 16 coins in the jar there are twice as many 10-cent coins as 1-cent coins.
A. Statement (1) ALONE is sufficient, but statement (2) alone is not sufficient.
B. Statement (2) ALONE is sufficient, but statement (1) alone is not sufficient.
C. BOTH statements TOGETHER are sufficient, but NEITHER statement ALONE is
sufficient.
D. EACH statement ALONE is sufficient.
E. Statements (1) and (2) TOGETHER are NOT sufficient.
Algebra Simultaneous equations
Let a, b, c, d, and e be the number, respectively, of 1-cent, 5-cent, 10-cent, 25-cent, and 50-
cent coins. We are given the two equations shown below. Determine the value of a.
a + b + c + d + e = 16
a + 5b + 10c + 25d + 50e = 288
1. We are given that c = 6, d = 5, and e = 2. Substituting these values into the two
equations displayed above and combining terms gives a + b = 3 and a + 5b = 3.
Subtracting these last two equations gives 4b = 0, and therefore b = 0 and a = 3;
SUFFICIENT.
2. We are given that c = 2a. Substituting c = 2a into the two equations displayed above
and combining terms gives the following two equations.
3a + b + d + e = 16
21a + 5b + 25d + 50e = 288
From the first equation above we have 3a = 16 − b − d − e. Therefore, 3a ≤ 16, and it follows
that the value of a must be among 0, 1, 2, 3, 4, and 5. From the second equation above we
have 5(b + 5d + 10e) = 288 − 21a, and thus the value of 288 − 21a must be divisible by 5.
a 288 − 21a
0 288
1 267
2 246
3 225
4 204
5 183
The table above shows that a = 3 is the only nonnegative integer less than or equal to 5 such
that 288 − 21a is divisible by 5; SUFFICIENT.
The correct answer is D;
each statement alone is sufficient.
DS48391.01

88. At a certain university recreation center, a member can receive a 30-minute massage, a 60-
minute massage, or a 90-minute massage, and is charged $0.50 per minute for each
massage. A member receiving a massage is charged the same fixed amount for each
additional service, such as nutrition advice or a fitness evaluation. At this center, what is the
total charge to a member for a 60-minute massage and 3 additional services?
1. At this recreation center, Jordan, a member, had a massage and 3 additional services
for a total charge of $37.50.
2. At this recreation center, Ryan, a member, had a massage and 2 additional services for
a total charge of $60.00.
A. Statement (1) ALONE is sufficient, but statement (2) alone is not sufficient.
B. Statement (2) ALONE is sufficient, but statement (1) alone is not sufficient.
C. BOTH statements TOGETHER are sufficient, but NEITHER statement ALONE is
sufficient.
D. EACH statement ALONE is sufficient.
E. Statements (1) and (2) TOGETHER are NOT sufficient.
Algebra First-degree equations
Let x be the charge, in dollars, of each additional service. Determine the value of (60)
(0.5) + 3x, or equivalently, determine the value of x.
1. The table below shows the value of x for each of the three possible massages that
Jordan had.
Massage Total charge x
30-minute 15 + 3x = 37.50 7.50
60-minute 30 + 3x = 37.50 2.50
90-minute 45 + 3x = 37.50 −2.50
From the table it follows that there are two possible values of x, namely x = 7.5 and x = 2.5;
NOT sufficient.
2. The table below shows the value of x for each of the three possible massages that Ryan
had.
Massage Total charge x
30-minute 15 + 2x = 60 22.50
60-minute 30 + 2x = 60 15.00
90-minute 45 + 2x = 60 7.50
From the table it follows that there are three possible values of x, namely x = 22.5, x = 15, and
x = 7.5; NOT sufficient.
Given (1) and (2), it follows that x = 7.5.
The correct answer is C;
both statements together are sufficient.
DS84302.01

89. If S is the sum of the first n positive integers, what is the value of n ?
1. S < 20
2. S2 > 220
A. Statement (1) ALONE is sufficient, but statement (2) alone is not sufficient.
B. Statement (2) ALONE is sufficient, but statement (1) alone is not sufficient.
C. BOTH statements TOGETHER are sufficient, but NEITHER statement ALONE is
sufficient.
D. EACH statement ALONE is sufficient.
E. Statements (1) and (2) TOGETHER are NOT sufficient.
Arithmetic Sequences and series

1. Given that S < 20, n could be 5 since = = 15 < 20. However, n could also be 4

since = = 10 < 20; NOT sufficient.

2. Given that S2 > 220, n could be 5 since = = 15 and 152 = 225 > 220. However,

n could also be 6 since = 21 and 212 = 441 > 220; NOT sufficient.

Given (1) and (2) together, it is clear from the following table that n = 5.

n S S2 allowed/not allowed
≤ 4 ≤ 10 ≤ 100 not allowed by (2)
5 15 225 allowed by (1) and (2)
≥ 6 ≥ 21 ≥ 441 not allowed by (1)
The correct answer is C;
both statements together are sufficient.
DS48302.01

90. Is x2 − y2 a positive number?


1. x − y is a positive number.
2. x + y is a positive number.
A. Statement (1) ALONE is sufficient, but statement (2) alone is not sufficient.
B. Statement (2) ALONE is sufficient, but statement (1) alone is not sufficient.
C. BOTH statements TOGETHER are sufficient, but NEITHER statement ALONE is
sufficient.
D. EACH statement ALONE is sufficient.
E. Statements (1) and (2) TOGETHER are NOT sufficient.
Algebra Factoring
Since x2 − y2 = (x + y)(x − y), it follows that x2 − y2 will be a positive number if both x + y
and x − y are positive numbers or if both are negative numbers.
1. It is given that x − y is a positive number. If, for example, x = 2 and y = −3, then x − y is
a positive number, but x + y is not a positive number, so x2 − y2 is not a positive number.
On the other hand, if x = 3 and y = 1, then x − y is a positive number and x + y is a
positive number, so x2 − y2 is a positive number; NOT sufficient.
2. It is given that x + y is a positive number. If, for example, x = −2 and y = 4, then x + y is
a positive number, but x − y is not a positive number, so x2 − y2 is not a positive number.
On the other hand, if x = 3 and y = 1, then x + y is a positive number and x − y is a
positive number, so x2 − y2 is a positive number; NOT sufficient.
Taking (1) and (2) together, both x + y and x − y are positive numbers, so x2 − y2 is a positive
number.
The correct answer is C;
both statements together are sufficient.
DS89302.01

91. Alan and Sue have each been saving one dollar a day and will continue to do so for the next
month. If Sue began saving several days before Alan, in how many days from today will Alan
have saved one-half as much as Sue?
1. As of today, Alan has saved 7 dollars and Sue has saved 27 dollars.
2. Three days from today, Alan will have saved one-third as much as Sue.
A. Statement (1) ALONE is sufficient, but statement (2) alone is not sufficient.
B. Statement (2) ALONE is sufficient, but statement (1) alone is not sufficient.
C. BOTH statements TOGETHER are sufficient, but NEITHER statement ALONE is
sufficient.
D. EACH statement ALONE is sufficient.
E. Statements (1) and (2) TOGETHER are NOT sufficient.
Algebra First-degree equations
Let A be the amount Alan has saved as of today. Let S be the amount Sue had already saved
when Alan started saving. Then, as of today, Sue has saved S + A dollars. Determine d, the
number of days from today that Alan will have saved half as much as Sue. That is, determine
d, where A + d = (S + A + d) or, after algebraic manipulation, determine d such that
d = S − A.
1. It is given that A = 7 and S = 27, so d = 20; SUFFICIENT.
2. It is given that three days from today, Alan will have saved one-third as much as Sue,
from which it follows that A + 3 = (S + A + 3) or, after algebraic manipulation,
S = 2A + 6. Then, d = S − A = (2A + 6) − A = A + 6. Since the value of A can vary, the
value of d cannot be determined; NOT sufficient.
The correct answer is A;
statement 1 alone is sufficient.
DS64402.01

92. What is the value of x ?


1.

2.
A. Statement (1) ALONE is sufficient, but statement (2) alone is not sufficient.
B. Statement (2) ALONE is sufficient, but statement (1) alone is not sufficient.
C. BOTH statements TOGETHER are sufficient, but NEITHER statement ALONE is
sufficient.
D. EACH statement ALONE is sufficient.
E. Statements (1) and (2) TOGETHER are NOT sufficient.
Algebra Simplifying algebraic expressions

1. If x4 + x2 + 1 = , then is its own reciprocal. The only numbers that


are their own reciprocals are −1 and 1. Since even powers of x are nonnegative,
x4 + x2 + 1 ≠ −1, so x4 + x2 + 1 = 1. It follows that x4 + x2 = 0 and x = 0; SUFFICIENT.
2. If x3 + x2 = 0, then x2(x + 1) = 0. It follows that x = 0 or x = −1; NOT sufficient.
The correct answer is A;
statement 1 alone is sufficient.
DS26402.01

93. Is x less than y ?


1. x−y+1<0
2. x−y−1<0
A. Statement (1) ALONE is sufficient, but statement (2) alone is not sufficient.
B. Statement (2) ALONE is sufficient, but statement (1) alone is not sufficient.
C. BOTH statements TOGETHER are sufficient, but NEITHER statement ALONE is
sufficient.
D. EACH statement ALONE is sufficient.
E. Statements (1) and (2) TOGETHER are NOT sufficient.
Algebra Inequalities
Determine if x < y is true.
1. If x − y + 1 < 0, then x < y − 1. Since y − 1 < y, it follows that x < y; SUFFICIENT.
2. If x = 1 and y = 2, then x − y − 1 = − 2 < 1 and x < y is true. However, if x = 1.5 and y = 1,
then x − y − 1 = −0.5 < 1 and x < y is not true; NOT sufficient.
The correct answer is A;
statement 1 alone is sufficient.

Tip

In (1), manipulating the given inequality leads to


x < y − 1, which leads directly to x < y since y − 1 < y.
This is not the case in (2), where manipulating the given
inequality leads to x < y + 1 but not to x < y since
y + 1 > y. Examples can then be used to verify that x < y
can be, but doesn't have to be, true.

DS08402.01
94. State X has a sales tax rate of k percent on all purchases and State Y has a sales tax rate of n
percent on all purchases. What is the value of k − n ?
1. The sales tax on a $15 purchase is 30 cents more in State X than in State Y.
2. The sales tax rate in State X is 1.4 times the sales tax rate in State Y.
A. Statement (1) ALONE is sufficient, but statement (2) alone is not sufficient.
B. Statement (2) ALONE is sufficient, but statement (1) alone is not sufficient.
C. BOTH statements TOGETHER are sufficient, but NEITHER statement ALONE is
sufficient.
D. EACH statement ALONE is sufficient.
E. Statements (1) and (2) TOGETHER are NOT sufficient.
Algebra Applied problems; Percents

1. It is given that . It follows that 15(k − n) = 30, from which the


value of k − n can be determined; SUFFICIENT.
2. It is given that k = 1.4n, from which k − n cannot be determined. For example, if n = 5,
then k = 7 and k − n = 2. However, if n = 10, then k = 14 and k − n = 4; NOT sufficient.
The correct answer is A;
statement 1 alone is sufficient.
DS28402.01

95. Is −3 ≤ x ≤ 3 ?
1. x2 + y2 = 9
2. x2 + y ≤ 9
A. Statement (1) ALONE is sufficient, but statement (2) alone is not sufficient.
B. Statement (2) ALONE is sufficient, but statement (1) alone is not sufficient.
C. BOTH statements TOGETHER are sufficient, but NEITHER statement ALONE is
sufficient.
D. EACH statement ALONE is sufficient.
E. Statements (1) and (2) TOGETHER are NOT sufficient.
Algebra Properties of numbers; Inequalities
1. Given that x2 + y2 = 9, if x < −3 or if x > 3, then x2 > 9 and y2 < 0, which is not
possible. Therefore, −3 ≤ x ≤ 3; SUFFICIENT.
2. Given that x2 + y ≤ 9, if x = 0 and y = 4, then x2 + y ≤ 9, and −3 ≤ x ≤ 3 is true.
However, if x = 4 and y = −7, then x2 + y ≤ 9, and −3 ≤ x ≤ 3 is not true; NOT sufficient.
The correct answer is A;
statement 1 alone is sufficient.
DS69402.01

96. What is the value of x ?


1. 4x +1 + 4x = 320
2. x2 = 9
A. Statement (1) ALONE is sufficient, but statement (2) alone is not sufficient.
B. Statement (2) ALONE is sufficient, but statement (1) alone is not sufficient.
C. BOTH statements TOGETHER are sufficient, but NEITHER statement ALONE is
sufficient.
D. EACH statement ALONE is sufficient.
E. Statements (1) and (2) TOGETHER are NOT sufficient.
Algebra Exponents
Determine the value of x.
1. If 4x + 1 + 4x = 320, then 4x(4 + 1) = 320, from which 4x = 64 and x = 3; SUFFICIENT.
2. If x2 = 9, then x = −3 or x = 3; NOT sufficient.
The correct answer is A;
statement 1 alone is sufficient.
DS50502.01

97. Three dice, each with faces numbered 1 through 6, were tossed onto a game board. If one of
the dice turned up 4, what was the sum of the numbers that turned up on all three dice?
1. The sum of two of the numbers that turned up was 10.
2. The sum of two of the numbers that turned up was 11.
A. Statement (1) ALONE is sufficient, but statement (2) alone is not sufficient.
B. Statement (2) ALONE is sufficient, but statement (1) alone is not sufficient.
C. BOTH statements TOGETHER are sufficient, but NEITHER statement ALONE is
sufficient.
D. EACH statement ALONE is sufficient.
E. Statements (1) and (2) TOGETHER are NOT sufficient.
Arithmetic Operations with integers
Determine the sum of the numbers that turned up when three dice were tossed, given that
one of the dice turned up 4.
1. Given that the sum of two of the numbers was 10, the numbers that turned up on the
three dice could be 5, 4, and 5 (5 + 5 = 10) for a total sum of 14. On the other hand, they
could be 2, 4, and 6 (4 + 6 = 10) for a total sum of 12; NOT sufficient.
2. The number 4 could not have been one of the two numbers whose sum was 11 because
11 − 4 = 7 and 7 is not a number that can turn up on the dice. Therefore, since 11 can be
obtained only when the other two numbers are 5 and 6, the numbers that turned up on
the three dice must be 4, 5, and 6 for a total sum of 15; SUFFICIENT.
The correct answer is B;
statement 2 alone is sufficient.
DS81502.01
98. Of the numbers q, r, s, and t, which is greatest?
1. The average (arithmetic mean) of q and r is s.
2. The sum of q and r is t.
A. Statement (1) ALONE is sufficient, but statement (2) alone is not sufficient.
B. Statement (2) ALONE is sufficient, but statement (1) alone is not sufficient.
C. BOTH statements TOGETHER are sufficient, but NEITHER statement ALONE is
sufficient.
D. EACH statement ALONE is sufficient.
E. Statements (1) and (2) TOGETHER are NOT sufficient.
Algebra Statistics
Determine which of the numbers q, r, s, and t is the greatest.

1. Given that , then s is halfway between q and r and q ≤ s ≤ r or r ≤ s ≤ q. So it is


possible that q is the greatest, it is possible that r is the greatest, and it is possible that t is
the greatest since no information is given about t; NOT sufficient.
2. Given that t = q + r, it is not possible to determine which of q, r, s, and t is greatest. For
example, if q = 1, r = 5, s = 3, and t = 6, then t is the greatest, but if q = 1, r = −5, s = −2,
and t = −4, then q is the greatest; NOT sufficient.
Taking (1) and (2) together, it is still not possible to determine which of q, r, s, and t is
greatest because the examples used to show that (2) is not sufficient satisfy (1) also.
The correct answer is E;
both statements together are still not sufficient.
CAR RENTAL CHARGES AT THRIFTY AGENCY
Car Type Charge per day Charge per Week (7 days)
Economy $28 $100
Compact $30 $120
Midsize $32 $140
Standard $34 $160
Luxury $39 $200
DS94502.01

99. The table above shows the car rental charges at Thrifty Agency. The daily rate applies for
each day or fraction of a day in excess of any multiple of a 7-day week, up to the charge per
week. If Olga rented a car of one of the types indicated, which type was it?
1. Olga's total rental charge, based only on the rates specified, was $184.
2. Olga rented the car for 10 days.
A. Statement (1) ALONE is sufficient, but statement (2) alone is not sufficient.
B. Statement (2) ALONE is sufficient, but statement (1) alone is not sufficient.
C. BOTH statements TOGETHER are sufficient, but NEITHER statement ALONE is
sufficient.
D. EACH statement ALONE is sufficient.
E. Statements (1) and (2) TOGETHER are NOT sufficient.
Arithmetic Interpretation of tables
Determine which type of car Olga rented.
1. Since Olga's total rental charge was $184, determine for which type of car (economy,
compact, midsize or standard) is an integer. Note that Olga did not

rent a luxury car because 184 < 200 and 184 is not a multiple of 39.
Economy: 184 − 100 = 84 = 3(28)
Compact: 184 − 120 = 64, not a multiple of 30
Midsize: 184 − 140 = 44, not a multiple of 32
Standard: 184 − 160 = 24, not a multiple of 34
Olga rented an economy car; SUFFICIENT.
2. Just knowing that Olga rented the car for 10 days is not enough information to
determine which type of car she rented; NOT sufficient.
The correct answer is A;
statement 1 alone is sufficient.
DS76602.01

100. Is xy < 6 ?
1. x < 3 and y < 2.
2. and y2 < 64.

A. Statement (1) ALONE is sufficient, but statement (2) alone is not sufficient.
B. Statement (2) ALONE is sufficient, but statement (1) alone is not sufficient.
C. BOTH statements TOGETHER are sufficient, but NEITHER statement ALONE is
sufficient.
D. EACH statement ALONE is sufficient.
E. Statements (1) and (2) TOGETHER are NOT sufficient.
Algebra Inequalities
1. Given that x < 3 and y < 2, it is not possible to determine whether or not xy < 6. For
example, if x = 1 and y = 1, then x < 3, y < 2, and xy = 1. However, if x = −3 and y = −3,
then x < 3, y < 2, and xy = 9; NOT sufficient.

2. Given that y2 < 64, then it easily follows that −8 < y < 8. Thus, we have and
−8 < y < 8. We consider two cases, according to the sign of y. Case 1: Suppose that
−8 < y ≤ 0. Since x > 0 and y ≤ 0, it follows that xy ≤ 0 < 6. Case 2: Suppose that
0 < y < 8. Then xy is the product of two positive quantities. Since the product of two
positive quantities is greatest when each of the quantities is greatest, it follows that xy <
< 6.

Since xy < 6 in each case, and the two cases include all possible values of x and y, we have
xy < 6; SUFFICIENT.
The correct answer is B;
statement 2 alone is sufficient.
DS86602.01

101. What is the value of ?

1. and .

2. and .

A. Statement (1) ALONE is sufficient, but statement (2) alone is not sufficient.
B. Statement (2) ALONE is sufficient, but statement (1) alone is not sufficient.
C. BOTH statements TOGETHER are sufficient, but NEITHER statement ALONE is
sufficient.
D. EACH statement ALONE is sufficient.
E. Statements (1) and (2) TOGETHER are NOT sufficient.
Algebra Simplifying algebraic expressions

1. Given that x = and z = , it follows that , which will have

different values for different nonzero values of x; NOT sufficient.

2. Given that and , it follows that ; SUFFICIENT.

The correct answer is B;


statement 2 alone is sufficient.
DS47602.01

102. In a certain group of people, the average (arithmetic mean) weight of the males is 180
pounds and of the females, 120 pounds. What is the average weight of the people in the
group?
1. The group contains twice as many females as males.
2. The group contains 10 more females than males.
A. Statement (1) ALONE is sufficient, but statement (2) alone is not sufficient.
B. Statement (2) ALONE is sufficient, but statement (1) alone is not sufficient.
C. BOTH statements TOGETHER are sufficient, but NEITHER statement ALONE is
sufficient.
D. EACH statement ALONE is sufficient.
E. Statements (1) and (2) TOGETHER are NOT sufficient.
Algebra Applied problems; Statistics
Let M and F, respectively, be the number of males and females in the group. Also, let
and , respectively, be the total weight, in pounds, of the males and females in the group.

We are given that = 180 and = 120. What is the value of ?

1. Given that F = 2M (equivalently, M = ), it follows that

= =

; SUFFICIENT.

2. We are given that F = M + 10. If M = 2 and F = 12, then there are six times as many
females as males, and hence the average weight of the people in the group will be strongly
skewed toward the average weight of the females. However, if M = 100,000 and
F = 100,010, then the ratio of females to males is close to 1, and hence the average weight
of the people in the group will be close to the average of 120 and 180.

Tip

In data sufficiency problems it is often helpful to consider


contrasting extreme scenarios when they exist. If a
variable can be any positive real number, then consider
the scenario when the variable is very close to 0 and the
scenario when the variable is very large. If a quadrilateral
can be any rectangle, then consider the scenario when the
rectangle is a square and the scenario when the rectangle
is very long with a small width.

Alternatively, , and by long division

(takes one step), this can be written as 150 − , which can clearly vary when the value of
M varies; NOT sufficient.
The correct answer is A;
statement 1 alone is sufficient.
DS57602.01

103. If y = 2x +1, what is the value of y − x ?


1. 22x +2 = 64
2. y = 22x −1
A. Statement (1) ALONE is sufficient, but statement (2) alone is not sufficient.
B. Statement (2) ALONE is sufficient, but statement (1) alone is not sufficient.
C. BOTH statements TOGETHER are sufficient, but NEITHER statement ALONE is
sufficient.
D. EACH statement ALONE is sufficient.
E. Statements (1) and (2) TOGETHER are NOT sufficient.
Algebra Equations; Exponents
1. Given that 22x + 2 = 64 = 26, it follows that 2x + 2 = 6. Therefore, x = 2 and y − x = 22 +
1 − 2 = 6; SUFFICIENT.

2. From the given information we have y = 2x + 1 and from (2) we have y = 22x−1.
Therefore, 2x + 1 = 22x−1, and hence x + 1 = 2x − 1. Solving for x gives x = 2, and thus
y − x = 22 + 1 − 2 = 6; SUFFICIENT.
The correct answer is D;
each statement alone is sufficient.
DS67602.01

104. If x ≠ 1, is y equal to x + 1 ?
1.

2. y2 = (x + 1)2
A. Statement (1) ALONE is sufficient, but statement (2) alone is not sufficient.
B. Statement (2) ALONE is sufficient, but statement (1) alone is not sufficient.
C. BOTH statements TOGETHER are sufficient, but NEITHER statement ALONE is
sufficient.
D. EACH statement ALONE is sufficient.
E. Statements (1) and (2) TOGETHER are NOT sufficient.
Algebra Simplifying algebraic expressions
Determine if y = x + 1.

1. Given = 1, then y − 2 = x − 1 and y = x + 1; SUFFICIENT.

2. Given y2 = (x + 1)2, then y = x + 1 or y = −(x + 1); NOT sufficient.


The correct answer is A;
statement 1 alone is sufficient.
DS18602.01

105. If x + y + z > 0, is z > 1 ?


1. z>x+y+1
2. x+y+1<0
A. Statement (1) ALONE is sufficient, but statement (2) alone is not sufficient.
B. Statement (2) ALONE is sufficient, but statement (1) alone is not sufficient.
C. BOTH statements TOGETHER are sufficient, but NEITHER statement ALONE is
sufficient.
D. EACH statement ALONE is sufficient.
E. Statements (1) and (2) TOGETHER are NOT sufficient.
Algebra Inequalities
Determine if z > 1 is true.
1. Given that z > x + y + 1, by adding z to both sides, it follows that 2z > x + y + z + 1. Also,
x + y + z + 1 > 1 because x + y + z > 0. Thus, 2z > 1 and z > . It is possible that z > 1 is
true and it is possible that z > 1 is not true. For example, if z = 1.1 and x = y = 0, then
x + y + z > 0 and z > x + y + 1 are both true, and z > 1 is true. However, if z = 1, x = −0.5
and y = −0.25, x + y + z > 0 and z > x + y + 1 are both true, and z > 1 is not true; NOT
sufficient.
2. Given that x + y + 1 < 0, it follows that 1 < −x − y. It is also given that x + y + z > 0, so
z > −x − y or −x − y < z. Combining 1 < −x − y and −x − y < z gives 1 < z or z > 1;
SUFFICIENT.
The correct answer is B;
statement 2 alone is sufficient.
Answer Explanations Quantitative Reasoning
Data Sufficiency
Geometry
DS35210.01

106. In the rectangular coordinate system, line k passes through the point (n,−1). Is the slope of
line k greater than zero?
1. Line k passes through the origin.
2. Line k passes through the point (1,n + 2).
A. Statement (1) ALONE is sufficient, but statement (2) alone is not sufficient.
B. Statement (2) ALONE is sufficient, but statement (1) alone is not sufficient.
C. BOTH statements TOGETHER are sufficient, but NEITHER statement ALONE is
sufficient.
D. EACH statement ALONE is sufficient.
E. Statements (1) and (2) TOGETHER are NOT sufficient.
Geometry Simple coordinate geometry

1. The slope of a line through (n,−1) and (0,0) is , which is greater than zero if n < 0
and less than zero if n > 0; NOT sufficient.
2. Given that line k passes through the points (n,−1) and (1,n + 2), then the slope of line k
(when it exists) is equal to . If n = 0, then the slope of line k is 3,
which is positive. However, if n = 2, then the slope of line k is −5, which is negative; NOT
sufficient.

Given (1) and (2), it follows that , which by cross-multiplying is equivalent to (−1)
(1 − n) = n(n + 3) when n is not equal to 0 or 1. This is a quadratic equation that can be
rewritten as n2 + 2n + 1 = 0, or (n + 1)2 = 0. Therefore, n = −1 and the slope of line k is
= 1, which is greater than zero.
The correct answer is C;
both statements together are sufficient.
DS88111.01

107. In quadrilateral ABCD, is angle BCD a right angle?


1. Angle ABC is a right angle.
2. Angle ADC is a right angle.
A. Statement (1) ALONE is sufficient, but statement (2) alone is not sufficient.
B. Statement (2) ALONE is sufficient, but statement (1) alone is not sufficient.
C. BOTH statements TOGETHER are sufficient, but NEITHER statement ALONE is
sufficient.
D. EACH statement ALONE is sufficient.
E. Statements (1) and (2) TOGETHER are NOT sufficient.
Geometry Quadrilaterals
1. The figure below shows two possibilities for a quadrilateral ABCD such that angle ABC
is a right angle. One quadrilateral is such that angle BCD is not a right angle (i.e., the
answer to the question can be NO), and the other quadrilateral is a square (i.e., the
answer to the question can be YES); NOT sufficient.

2. The figure below shows that relabeling the vertices of the examples used in (1) above
will give an example such that angle BCD is not a right angle and an example such that
angle BCD is a right angle; NOT sufficient.

Given (1) and (2), the figure below indicates how an example satisfying both (1) and (2) can
be constructed such that angle BCD is not a right angle. A right angle is constructed with
vertex B and another right angle, appropriately rotated with respect to the first right angle, is
constructed with vertex D. The rays of these two angles intersect at points A and C to form a
quadrilateral ABCD that satisfies both (1) and (2) and is such that angle BCD is not a right
angle (i.e., the answer to the question can be NO). For completeness, a square is also shown,
which satisfies both (1) and (2) and is such that angle BCD is a right angle (i.e., the answer to
the question can be YES).
The correct answer is E;
both statements together are still not sufficient.

DS29831.01

108. In the figure above, B is on , D is on , has the same length as , and has the
same measure as . What is the length of ?
1. The length of is 6.
2. The length of is 5.
A. Statement (1) ALONE is sufficient, but statement (2) alone is not sufficient.
B. Statement (2) ALONE is sufficient, but statement (1) alone is not sufficient.
C. BOTH statements TOGETHER are sufficient, but NEITHER statement ALONE is
sufficient.
D. EACH statement ALONE is sufficient.
E. Statements (1) and (2) TOGETHER are NOT sufficient.
Geometry Triangles
The given information implies that ΔABD is similar to ΔACE, since ABD and ACE have
the same measure and these two triangles share an angle at point A. Therefore, the lengths of
corresponding sides of these two triangles are proportional. Using AB = BC, it follows that
AC = 2(AB), and hence the lengths of the sides of ΔACE are twice the lengths of the
corresponding sides of ΔABD.

1. Given that EC = 6, it follows from the remarks above that ; SUFFICIENT.

2. The figure below shows two possibilities satisfying the given information and DE = 5
that have different values for DB; NOT sufficient.

The correct answer is A;


statement 1 alone is sufficient.
DS92931.01

109. Sprinklers are being installed to water a lawn. Each sprinkler waters in a circle. Can the lawn
be watered completely by 4 installed sprinklers?
1. The lawn is rectangular and its area is 32 square yards.
2. Each sprinkler can completely water a circular area of lawn with a maximum radius of
2 yards.
A. Statement (1) ALONE is sufficient, but statement (2) alone is not sufficient.
B. Statement (2) ALONE is sufficient, but statement (1) alone is not sufficient.
C. BOTH statements TOGETHER are sufficient, but NEITHER statement ALONE is
sufficient.
D. EACH statement ALONE is sufficient.
E. Statements (1) and (2) TOGETHER are NOT sufficient.
Geometry Circles; Rectangles
1. No information is given about the area of the region that can be completely covered by
four installed sprinklers; NOT sufficient.
2. No information is given about the area or the shape of the lawn; NOT sufficient.
Given (1) and (2), if the length of the rectangular lawn is sufficiently large, for example if the
length is 32 yards and the width is 1 yard, then it is clear that the four sprinklers cannot
completely water the lawn. However, if the lawn is in the shape of a square, then it is possible
that four sprinklers can completely water the lawn. To see this, we first note that the side
length of the square lawn is yards. To assist with the mathematical details, the
figure below shows the square lawn positioned in the standard (x,y) coordinate plane so that
the vertices of the lawn are located at (0,0), , , and . The two
diagonals of the square, each of length 8, are shown as dashed segments, and the four
sprinklers are at the four marked points located at the midpoints of the left and right halves
of the diagonals. For example, one of the sprinklers is located at the point . Using the
distance formula, it is straightforward to show that a circle centered at with radius 2
passes through each of the points (0,0), , , and . Therefore, the
interior of this circle covers the lower left square portion of the square lawn—that is, the
square portion having vertices (0,0), , , and . Hence, the four
sprinklers together, when located as described above, can completely water the square lawn.
Therefore, it is possible that the lawn cannot be completely watered by the four sprinklers,
and it is possible that the lawn can be completely watered by the four sprinklers.

The correct answer is E;


both statements together are still not sufficient.
DS18041.01

110. What is the length of the hypotenuse of ΔABC ?


1. The lengths of the three sides of ΔABC are consecutive even integers.
2. The hypotenuse of ΔABC is 4 units longer than the shorter leg.
A. Statement (1) ALONE is sufficient, but statement (2) alone is not sufficient.
B. Statement (2) ALONE is sufficient, but statement (1) alone is not sufficient.
C. BOTH statements TOGETHER are sufficient, but NEITHER statement ALONE is
sufficient.
D. EACH statement ALONE is sufficient.
E. Statements (1) and (2) TOGETHER are NOT sufficient.
Geometry Pythagorean theorem
1. Let n, n + 2, and n + 4 be the consecutive even integers. Using the Pythagorean
theorem, we have n2 + (n + 2)2 = (n + 4)2. Because this is a quadratic equation that may
have two solutions, we need to investigate further to determine whether there is a unique
hypotenuse length.

Therefore, n = 6 or n = −2. Since n = −2 corresponds to side lengths of −2, 0, and 2, we


discard n = −2. Therefore n = 6, the hypotenuse has length n + 4 = 10; SUFFICIENT.
2. Let the side lengths be a, b, and a + 4. Using the Pythagorean theorem, we have
a2 + b2 = (a + 4)2. Expanding and solving for b in terms of a will facilitate our search for
multiple hypotenuse length possibilities.

When a = 1, we obtain side lengths 1 and , and hypotenuse length 5. When a = 2, we


obtain side lengths 2 and , and hypotenuse length 6; NOT sufficient.
The correct answer is A;
statement 1 alone is sufficient.
DS37571.01

111. Patricia purchased x meters of fencing. She originally intended to use all of the fencing to
enclose a square region, but later decided to use all of the fencing to enclose a rectangular
region with length y meters greater than its width. In square meters, what is the positive
difference between the area of the square region and the area of the rectangular region?
1. xy = 256
2. y=4
A. Statement (1) ALONE is sufficient, but statement (2) alone is not sufficient.
B. Statement (2) ALONE is sufficient, but statement (1) alone is not sufficient.
C. BOTH statements TOGETHER are sufficient, but NEITHER statement ALONE is
sufficient.
D. EACH statement ALONE is sufficient.
E. Statements (1) and (2) TOGETHER are NOT sufficient.
Geometry Rectangles; Perimeter
The square's perimeter is x meters, and thus the square has adjacent sides of length meters
each. Since the rectangle's perimeter is also x meters, with adjacent side lengths that differ by
y meters, it follows that the rectangle's length is meters (i.e., lengthen two opposite

sides of the square by meters) and the rectangle's width is meters (i.e., shorten the

two other opposite sides of the square by meters). Alternatively, letting L and W be the
length and width, respectively and in meters, of the rectangle, then we can express each of L
and W in terms of x and y by algebraically eliminating L and W from the equations
2L + 2W = x and L = W + y.

Therefore, in square meters, the area of the square is , the area of the rectangle is

, and the positive difference between these two areas is .

Determine the value of .

1. Given xy = 256, it is clearly not possible to determine the value of ; NOT sufficient.

2. Given y = 4, the value of is equal to 4; SUFFICIENT.

The correct answer is B;


statement 2 alone is sufficient.

DS45771.01

112. In the figure above, points A, B, C, and D are collinear and , , and are semicircles with
diameters d1 cm, d2 cm, and d3 cm, respectively. What is the sum of the lengths of , ,
and , in centimeters?
1. d1:d2:d3 is 3:2:1.
2. The length of is 48 cm.
A. Statement (1) ALONE is sufficient, but statement (2) alone is not sufficient.
B. Statement (2) ALONE is sufficient, but statement (1) alone is not sufficient.
C. BOTH statements TOGETHER are sufficient, but NEITHER statement ALONE is
sufficient.
D. EACH statement ALONE is sufficient.
E. Statements (1) and (2) TOGETHER are NOT sufficient.
Geometry Circles; Circumference

Since the circumference of a semicircle is , it follows that has length

cm, has length cm, and has length cm. Therefore, the sum of the

lengths, in centimeters, of , , and is

. Determine the value of .

1. Given that d1:d2:d3 is 3:2:1, it is not possible to determine the value of


because d1, d2, and d3 could be 3, 2, and 1 (d1 + d2 + d3 = 6) or d1, d2, and d3 could be 6,
4, and 2 (d1 + d2 + d3 = 12); NOT sufficient.

2. Given that AD = 48 and AD = d1 + d2 + d3, it follows that ;


SUFFICIENT.
The correct answer is B;
statement 2 alone is sufficient.
DS16291.01

113. In the standard (x,y) coordinate plane, what is the slope of the line containing the distinct
points P and Q ?
1. Both P and Q lie on the graph of |x| + |y| = 1.
2. Both P and Q lie on the graph of |x + y| = 1.
A. Statement (1) ALONE is sufficient, but statement (2) alone is not sufficient.
B. Statement (2) ALONE is sufficient, but statement (1) alone is not sufficient.
C. BOTH statements TOGETHER are sufficient, but NEITHER statement ALONE is
sufficient.
D. EACH statement ALONE is sufficient.
E. Statements (1) and (2) TOGETHER are NOT sufficient.
Geometry Simple coordinate geometry
1. If P = (1,0) and Q = (0,1), then both P and Q lie on the graph of |x| + |y| = 1 and the
slope of the line containing P and Q is −1. However, if P = (1,0) and Q = (−1,0), then both
P and Q lie on the graph of |x| + |y| = 1 and the slope of the line containing P and Q is 0;
NOT sufficient.
2. If P = (1,0) and Q = (0,1), then both P and Q lie on the graph of |x + y| = 1 and the slope
of the line containing P and Q is −1. However, if P = (1,0) and Q = (−1,0), then both P and
Q lie on the graph of |x + y| = 1 and the slope of the line containing P and Q is 0; NOT
sufficient.
Taking (1) and (2) together is still not sufficient because the same examples used in (1) were
also used in (2).
Although it is not necessary to visualize the graphs of |x| + |y| = 1 and |x + y| = 1 to solve this
problem, some readers may be interested in their graphs. The graph of |x| + |y| = 1 is a
square with vertices at the four points (±1,0) and (0,±1). This can be seen by graphing
x + y = 1 in the first quadrant, which gives a line segment with endpoints (1,0) and (0,1), and
then reflecting this line segment about one or both coordinate axes for the other quadrants
(e.g., in the second quadrant, x < 0 and y > 0, and so |x| + |y| = 1 becomes −x + y = 1). The
graph of |x + y| = 1 is the union of two lines, one with equation x + y = 1 and the other with
equation x + y = −1.
The correct answer is E;
both statements together are still not sufficient.
DS61791.01

114. When opened and lying flat, a birthday card is in the shape of a regular hexagon. The card
must be folded in half along 1 of its diagonals before being placed in an envelope for mailing.
Assuming that the thickness of the folded card will not be an issue, will the birthday card fit
inside a rectangular envelope that is 4 inches by 9 inches?
1. Each side of the regular hexagon is 4 inches long.
2. The area of the top surface (which is the same as the area of the bottom surface) of the
folded birthday card is less than 36 square inches.
A. Statement (1) ALONE is sufficient, but statement (2) alone is not sufficient.
B. Statement (2) ALONE is sufficient, but statement (1) alone is not sufficient.
C. BOTH statements TOGETHER are sufficient, but NEITHER statement ALONE is
sufficient.
D. EACH statement ALONE is sufficient.
E. Statements (1) and (2) TOGETHER are NOT sufficient.
Geometry Polygons

As shown in the figure above, a regular hexagon with sides of length s can be partitioned into
six equilateral triangles. Using these triangles, it is possible to determine the length of each
diagonal (2s), the height, shown as a dashed line, of each triangle , the area of each

triangular region , and the area of the hexagonal region .

When the birthday card is folded in half along one of the diagonals it has the shape shown
below.

1. Given s = 4, the maximum width of the birthday card is 2s = 8, which is less than the
width of the envelope, and its height is = , which is less than the height of the

envelope because < = 4. Thus, the birthday card will fit in the envelope;
SUFFICIENT.
2. Given that the surface area of the card when folded is less than 36 square inches, it

follows that < 36, which simplifies to . If s = 4, then the birthday

card will fit in the envelope, as shown in (1) above. However, if s = 5, then s < (note
4
that 625 = 5 < ( 4
) = 768), but the maximum width of the birthday card will be
2s = 10, and the card will not fit in the envelope; NOT sufficient.
The correct answer is A;
statement 1 alone is sufficient.

DS77302.01

115. In rectangular region PQRS above, T is a point on side PS. If PS = 4, what is the area of region
PQRS ?
1. ΔQTR is equilateral.
2. Segments PT and TS have equal length.
A. Statement (1) ALONE is sufficient, but statement (2) alone is not sufficient.
B. Statement (2) ALONE is sufficient, but statement (1) alone is not sufficient.
C. BOTH statements TOGETHER are sufficient, but NEITHER statement ALONE is
sufficient.
D. EACH statement ALONE is sufficient.
E. Statements (1) and (2) TOGETHER are NOT sufficient.
Geometry Triangles
It is given that PQRS is a rectangle and PS = 4. The area of PQRS can be determined if and
only if PQ can be determined.

1.

It is given that ΔQTR is equilateral. If is the height of ΔQTR as shown above, then
ΔQUT is a 30−60−90° triangle with UR = 2. Using the ratios for 30−60−90 triangles,
TU = . Since PQ = TU, the area of PQRS can be determined; SUFFICIENT.
2. Given that PT = TS, PQ could be any positive number. Thus, it is not possible to
determine PQ and therefore not possible to determine the area of PQRS; NOT sufficient.
The correct answer is A;
statement 1 alone is sufficient.
DS58302.01

116. The top surface area of a square tabletop was changed so that one of the dimensions was
reduced by 1 inch and the other dimension was increased by 2 inches. What was the surface
area before these changes were made?
1. After the changes were made, the surface area was 70 square inches.
2. There was a 25 percent increase in one of the dimensions.
A. Statement (1) ALONE is sufficient, but statement (2) alone is not sufficient.
B. Statement (2) ALONE is sufficient, but statement (1) alone is not sufficient.
C. BOTH statements TOGETHER are sufficient, but NEITHER statement ALONE is
sufficient.
D. EACH statement ALONE is sufficient.
E. Statements (1) and (2) TOGETHER are NOT sufficient.
Geometry Quadrilaterals; First- and second-degree equations
Letting S be the side length of the square tabletop, the surface area before the change was S 2,
and the area after the change is (S − 1)(S + 2). Determine S 2.
1. It is given that (S − 1)(S + 2) = 70. Solving this equation gives S = −9 and S = 8.
Discarding S = −9, it follows that S = 8 and the surface area before the changes was 64
square inches; SUFFICIENT.
2. Since only one dimension was increased and that increase was 2 inches, it follows from
(2) that S + 2 = 1.25S. Solving this equation gives S = 8. Therefore, the surface area before
the changes was 64 square inches; SUFFICIENT.
The correct answer is D;
each statement alone is sufficient.
DS49302.01

117. If the lengths of the legs of a right triangle are integers, what is the area of the triangular
region?

1. The length of one leg is the length of the other.

2. The length of the hypotenuse is 5.


A. Statement (1) ALONE is sufficient, but statement (2) alone is not sufficient.
B. Statement (2) ALONE is sufficient, but statement (1) alone is not sufficient.
C. BOTH statements TOGETHER are sufficient, but NEITHER statement ALONE is
sufficient.
D. EACH statement ALONE is sufficient.
E. Statements (1) and (2) TOGETHER are NOT sufficient.
Geometry Triangles

1. Let x represent one leg of the triangle. Then the other leg is and the area is .
Since the value of x can be any positive multiple of 4, the area cannot be determined; NOT
sufficient.
2. Given that the lengths of the legs are the integers a and b and the length of the
hypotenuse is 5, it follows that a 2 + b 2 = 25, where 1 ≤ a ≤ 4 and 1 ≤ b ≤ 4. The only pair
of integers that meet these conditions are 3 and 4, so the area of the triangular region is 6;
SUFFICIENT.
The correct answer is B;
statement 2 alone is sufficient.
DS12402.01

118. If cubical blocks in a display are stacked one on top of the other on a flat surface, what is the
volume of the stack of blocks in cubic centimeters?
1. The volume of the top block is 8 cubic centimeters.
2. The height of the stack of blocks is 10 centimeters.
A. Statement (1) ALONE is sufficient, but statement (2) alone is not sufficient.
B. Statement (2) ALONE is sufficient, but statement (1) alone is not sufficient.
C. BOTH statements TOGETHER are sufficient, but NEITHER statement ALONE is
sufficient.
D. EACH statement ALONE is sufficient.
E. Statements (1) and (2) TOGETHER are NOT sufficient.
Geometry Volume
1. It is given that the volume of the top cube in the stack is 8 cubic centimeters, from
which it follows that the top block has edges of length 2 cm, but no information is given
about the size of the other blocks in the stack or how many blocks the stack contains;
NOT sufficient.
2. It is given that the height of the stack of blocks is 10 cm, but no information is given
about the size of any of the blocks in the stack or how many blocks are in the stack.
Taking (1) and (2) together gives no information about the size of the blocks below the top
block or how many blocks are in the stack. For example, there could be two blocks with edges
of lengths 2 cm and 8 cm. The volume of the top block would be 8 cubic centimeters, the
height of the stack would be 10 cm, and the volume of the stack of blocks would be 520 cubic
centimeters. But there could also be three blocks with edges of lengths 2 cm, 3 cm, and 5 cm.
The volume of the top block would be 8 cubic centimeters, the height of the stack would be 10
cm, and the volume of the stack of blocks would be 160 cubic centimeters.
The correct answer is E;
both statements together are still not sufficient.

Tip

Do not assume anything that is not explicitly stated in the


problem. In this problem, it is tempting to assume that all
of the blocks are identical, in which case there would be
five blocks, each with height 2 cm to give the whole stack
a height of 10 cm and a volume of 40 cubic centimeters.
Under the assumption that all of the blocks are identical,
the correct answer would be C.

DS53402.01

119. Is the perimeter of a certain rectangular garden greater than 50 meters?


1. The two shorter sides of the garden are each 15 meters long.
2. The length of the garden is 5 meters greater than the width of the garden.
A. Statement (1) ALONE is sufficient, but statement (2) alone is not sufficient.
B. Statement (2) ALONE is sufficient, but statement (1) alone is not sufficient.
C. BOTH statements TOGETHER are sufficient, but NEITHER statement ALONE is
sufficient.
D. EACH statement ALONE is sufficient.
E. Statements (1) and (2) TOGETHER are NOT sufficient.
Geometry Quadrilaterals; Perimeter
Let W represent the length of the shorter side (that is, the width) of the rectangular garden,
and let L represent the length of the longer side (that is, the length). The perimeter is then
2(L + W). Determine if 2(L + W) > 50 is true, or equivalently, if L +W > 25 is true.
1. It is given that W = 15, from which it follows that L > 15. Then, L + W > 15 + 15 > 25;
SUFFICIENT.
2. It is given that L = W + 5, from which it follows that L + W = 2W + 5. If, for example,
W = 1, then 2W + 5 is not greater than 25, but if W = 11, then 2W + 5 is greater than 25;
NOT sufficient.
The correct answer is A;
statement 1 alone is sufficient.

Tip

When determining whether a certain condition holds for


an expression involving two variables, avoid assuming
that the values of both variables must be known. In (1),
knowing that L represents the length of the longer side
and knowing the value of W, the length of the shorter
side, is enough information to determine whether L + W
meets the given condition.

DS34402.01

120. In the figure above, what is the perimeter of ΔPQR ?


1. The length of segment PT is 2.
2. The length of segment RS is .
A. Statement (1) ALONE is sufficient, but statement (2) alone is not sufficient.
B. Statement (2) ALONE is sufficient, but statement (1) alone is not sufficient.
C. BOTH statements TOGETHER are sufficient, but NEITHER statement ALONE is
sufficient.
D. EACH statement ALONE is sufficient.
E. Statements (1) and (2) TOGETHER are NOT sufficient.
Geometry Triangles; Perimeter
Determine the perimeter of ΔPQR by determining PQ + QR + PR.
1. It is given that PT = 2. Since ΔPTQ is a 45°−45°−90° triangle, it follows that QT = 2 and
PQ = . Since ΔQTR is a 30°−60°−90° triangle and QT = 2, it follows that QR = 4 and

TR = . PT + TR = PR, so PR, QR,

and PQ are known and the perimeter of ΔPQR can be determined; SUFFICIENT.
2. It is given that RS = , but no information is given to determine TS. If, for example,
TS = , then ΔQTR is a 30°−60°−90° triangle with TS + SR = TR = . It follows that
QT = 2 and QR = 4. Also, ΔPTQ is a 45°−45°−90° triangle with QT = 2. It follows that
PT = 2 (hence PR = 2 + ) and PQ = , so the perimeter of the triangle is
+ 4 + (2 + ). However, if TS = , then ΔQTR is a 30°−60°−90° triangle with
TS + SR = TR = . It follows that QT = 3, and QR = 6. Also, ΔPTQ is a 45°−45°−90°
triangle with QT = 3. It follows that PT = 3 (hence PR = 3 + ) and PQ = , so the
perimeter of the triangle is + 6 + (3 + ); NOT sufficient.
The correct answer is A;
statement 1 alone is sufficient.

DS07402.01

121. In the figure above, the shaded region represents the front of an upright wooden frame
around the entrance to an amusement park ride. If meters, what is the area of the
front of the frame?
1. x = 9 meters
2. meters
A. Statement (1) ALONE is sufficient, but statement (2) alone is not sufficient.
B. Statement (2) ALONE is sufficient, but statement (1) alone is not sufficient.
C. BOTH statements TOGETHER are sufficient, but NEITHER statement ALONE is
sufficient.
D. EACH statement ALONE is sufficient.
E. Statements (1) and (2) TOGETHER are NOT sufficient.
Geometry Triangles
The front of the frame is in the shape of an equilateral triangle with sides of length x with an
opening in the shape of an equilateral triangle with sides of length y, where y < x. The area of
the front of the frame is the area of the larger triangle minus the area of the smaller triangle.

Note that the height of an equilateral triangle with sides of length s is given by and the

area is given by , both of which are easily derived using the ratios for the sides of a 30°
−60°−90° triangle.

1. Given that x = 9, it follows that RT, the height of the large equilateral triangle, is .

Since RS = and RT = RS + ST, it follows that , where ST is the height


of the small equilateral triangle. Therefore, y = 4. With the side lengths of both triangles
known, the area of the front of the frame can be determined; SUFFICIENT.

2. Given that ST = , it follows that y = 4. Also, since RT = RS + ST and RS = , it

follows that RT = and x = 9. With the side lengths of both triangles known, the area

of the front of the frame can be determined; SUFFICIENT.


The correct answer is D;
each statement alone is sufficient.

DS05502.01

122. In the figure above, if the shaded region is rectangular, what is the length of XY ?
1. The perimeter of the shaded region is 24.
2. The measure of XYZ is 45°.
A. Statement (1) ALONE is sufficient, but statement (2) alone is not sufficient.
B. Statement (2) ALONE is sufficient, but statement (1) alone is not sufficient.
C. BOTH statements TOGETHER are sufficient, but NEITHER statement ALONE is
sufficient.
D. EACH statement ALONE is sufficient.
E. Statements (1) and (2) TOGETHER are NOT sufficient.
Geometry Triangles
Determine XY.
1. Letting L and W be the length and width of the shaded rectangular region, then from
(1), L + W = 12. This does not give enough information about ΔXZY to determine XY;
NOT sufficient.
2. From (2), ΔXZY is a 45°−45°−90° triangle, so XZ = YZ and XY = , but no
information is given to determine XZ; NOT sufficient.
Taking (1) and (2) together and using the figure below, from (2), ΔXRP is a 45°−45°−90°
triangle, so XR = PR = QZ = W. Likewise ΔPQY is a 45°−45°−90° triangle, so
PQ = YQ = RZ = L. It follows that XZ = XR + RZ = W + L and W + L= 12 from (1). Likewise,
YZ = YQ + QZ = L + W = 12. Since the length of the legs of ΔXZY are known, XY can be
determined.

The correct answer is C;


both statements together are sufficient.

DS87602.01

123. The figure above shows the dimensions of a square picture frame that was constructed using
four identical pieces of frame as shown. If w is the width of each piece of the frame, what is
the area of the front surface of each piece? (1 ft = 12 inches)
1. w = 3 inches
2. PQ = inches
A. Statement (1) ALONE is sufficient, but statement (2) alone is not sufficient.
B. Statement (2) ALONE is sufficient, but statement (1) alone is not sufficient.
C. BOTH statements TOGETHER are sufficient, but NEITHER statement ALONE is
sufficient.
D. EACH statement ALONE is sufficient.
E. Statements (1) and (2) TOGETHER are NOT sufficient.
Geometry Quadrilaterals; Area
Determine the area of the surface of each of the four pieces of wood used to construct the
square picture frame. In the following explanation, the larger square with one vertex labeled
P will be referred to as Square P and the smaller square with one vertex labeled Q will be
referred to as Square Q.
1. It is given that the width of each piece of wood is 3 inches. Then, the area one piece of
wood, in square inches, is or ; SUFFICIENT.

2. It is given that PQ = = inches. Since the pieces of the frame are identical, each
of the angles at P is 45° and ΔPRQ in the figure above is a 45°−45°−90° triangle. It
follows that RQ = 3. This is the same information given in (1), which was shown to be
sufficient; SUFFICIENT.
The correct answer is D;
each statement alone is sufficient.

DS48602.01

124. In the figure above, segments and represent two positions of the same ladder leaning
against the side of a wall. The length of is how much greater than the length of ?
1. The length of is 10 meters.
2. The length of is 5 meters.
A. Statement (1) ALONE is sufficient, but statement (2) alone is not sufficient.
B. Statement (2) ALONE is sufficient, but statement (1) alone is not sufficient.
C. BOTH statements TOGETHER are sufficient, but NEITHER statement ALONE is
sufficient.
D. EACH statement ALONE is sufficient.
E. Statements (1) and (2) TOGETHER are NOT sufficient.
Geometry Triangles
Note that TU = RS because they both represent the length of the same ladder. Determine
TV − RV.
1. It is given that TU = 10. Since ΔTVU is a 45°−45°−90° triangle, it follows that TV =
. Since RS = TU = 10 and ΔRVS is a 30°−60°−90° triangle, it follows that RV = 5.
Therefore, TV − RV can be determined; SUFFICIENT.
2. It is given that RV = 5. Then RS = 10 since ΔRVS is a 30°−60°−90° triangle. Since
RS = TU, it follows that TU = 10. This is the same information as in (1); SUFFICIENT.
The correct answer is D;
each statement alone is sufficient.
Answer Explanations Quantitative Reasoning
Data Sufficiency
Rates/Ratios/Percent
DS67410.01

125. A large flower arrangement contains 3 types of flowers: carnations, lilies, and roses. Of all the
flowers in the arrangement, are carnations, are lilies, and are roses. The total price of
which of the 3 types of flowers in the arrangement is the greatest?
1. The prices per flower for carnations, lilies, and roses are in the ratio 1:3:4, respectively.
2. The price of one rose is $0.75 more than the price of one carnation, and the price of one
rose is $0.25 more than the price of one lily.
A. Statement (1) ALONE is sufficient, but statement (2) alone is not sufficient.
B. Statement (2) ALONE is sufficient, but statement (1) alone is not sufficient.
C. BOTH statements TOGETHER are sufficient, but NEITHER statement ALONE is
sufficient.
D. EACH statement ALONE is sufficient.
E. Statements (1) and (2) TOGETHER are NOT sufficient.
Algebra Applied problems; Ratios
Let T be the total number of flowers, and let $C, $L, and $R be the cost, respectively, of one
carnation, one lily, and one rose.
1. We are given that L = 3C (because C:L is 1:3) and R = 4C (because C:R is 1:4). The table
below shows the total price for each type of flower.
Flower Number of flowers Price per flower Total price

Carnation C TC

Lily T 3C TC

Rose 4C TC

From the table it is clear that lilies have the greatest total cost; SUFFICIENT.
2. We are given that R = 0.75 + C and R = 0.25 + L. To simplify matters, we can use these
equations to express each of the variables C, L, and R in terms of a single fixed variable,
for example, C = R − and L = R − . This will allow us to replace all appearances of C, L,
and R with appearances of R only, thereby reducing by two the number of variables that
have to be dealt with. The table below shows, for two values of T and R, the total price for
each type of flower.
Flower Number of Price per Total Total price: T = 24, Total price: T = 24,
flowers flower price R=1 R = 10

Carnation T R− TR − 3 111
T

Lily T R− TR − 6 78

Rose T R TR 4 40

From the table it is clear that the type of flower having the greatest total cost can vary; NOT
sufficient.

Tip

Consider the expressions under “Total price” in the


previous table. Note that, for a fixed value of T, as the
value of R increases without bound, the total price for
carnation will eventually exceed the total price for each of
the other two types of flowers. Therefore, for non-
sufficiency of (2), it is only necessary to determine
whether there exist values for T and R such that
carnations do not have the greatest total price. This
suggests trying a small value for R, for example R = 1.
Also, note that T = 24 was chosen to avoid fractions in the
computations—24 is divisible by both 8 and 12.

The correct answer is A;


statement 1 alone is sufficient.
DS34010.01

126. Town X has 50,000 residents, some of whom were born in Town X. What percent of the
residents of Town X were born in Town X ?
1. Of the male residents of Town X, 40 percent were not born in Town X.
2. Of the female residents of Town X, 60 percent were born in Town X.
A. Statement (1) ALONE is sufficient, but statement (2) alone is not sufficient.
B. Statement (2) ALONE is sufficient, but statement (1) alone is not sufficient.
C. BOTH statements TOGETHER are sufficient, but NEITHER statement ALONE is
sufficient.
D. EACH statement ALONE is sufficient.
E. Statements (1) and (2) TOGETHER are NOT sufficient.
Algebra Applied problems
1. We are given that 40 percent of the male residents were NOT born in Town X, or
equivalently, that 60 percent of the male residents were born in Town X. However, no
information is given about the number of female residents or about the percent of female
residents born in Town X. By considering extreme cases, it is easy to see that the percent
of residents born in Town X cannot be determined. For example, if only 10 of the 50,000
residents were male and 0 percent of the female residents were born in Town X, then only
6 residents (i.e., close to 0 percent of the residents) would have been born in Town X.
However, if only 10 of the 50,000 residents were male and 100 percent of the female
residents were born in Town X, then 49,996 residents (i.e., close to 100 percent of the
residents) would have been born in Town X; NOT sufficient.
2. We are given that 60 percent of the female residents were born in Town X. However, no
information is given about the number of male residents or about the percent of male
residents born in Town X. By considering extreme cases in the same manner that was
done in (1), it is easy to see that the percent of residents born in Town X cannot be
determined; NOT sufficient.
Given (1) and (2), it follows that 60 percent of the male residents and 60 percent of the
female residents were born in Town X. Therefore, 60 percent of the residents were born in
Town X.
The correct answer is C;
both statements together are sufficient.
DS29931.01

127. A bank account earned 2% annual interest, compounded daily, for as long as the balance was
under $1,000, starting when the account was opened. Once the balance reached $1,000, the
account earned 2.5% annual interest, compounded daily until the account was closed. No
deposits or withdrawals were made. Was the total amount of interest earned at the 2% rate
greater than the total amount earned at the 2.5% rate?
1. The account earned exactly $25 in interest at the 2.5% rate.
2. The account was open for exactly three years.
A. Statement (1) ALONE is sufficient, but statement (2) alone is not sufficient.
B. Statement (2) ALONE is sufficient, but statement (1) alone is not sufficient.
C. BOTH statements TOGETHER are sufficient, but NEITHER statement ALONE is
sufficient.
D. EACH statement ALONE is sufficient.
E. Statements (1) and (2) TOGETHER are NOT sufficient.
Algebra Applied problems
Let P0, P1, and P2 be the initial balance, the balance after one year, and the balance after two
years.
1. Since $25 is the exact amount of interest earned in one year by an initial amount of
$1,000 earning 2.5 percent annual interest, compounded yearly, it follows that $25 is the
total amount of interest earned in slightly less than one year by an initial amount of
$1,000 earning 2.5 percent annual interest, compounded daily. However, the total
amount of interest earned at the 2 percent rate could be less than $25 (for example, if P0
= $990, then the interest earned at the 2 percent rate is $10) and the total amount of
interest earned at the 2 percent rate could be greater than $25 (for example, if P0 = $900,
then the interest earned at the 2 percent rate is $100); NOT sufficient.
2. Given that the account was open for exactly three years, then the total amount of
interest at the 2 percent rate could be less than the total amount of interest at the 2.5
percent rate (for example, if the balance reached $1,000 a few days after the account was
open). On the other hand, the total amount of interest at the 2 percent rate could also be
greater than the total amount of interest at the 2.5 percent rate (for example, if the
balance reached $1,000 a few days before the account was closed); NOT sufficient.
Given (1) and (2), it follows that the account earned interest at the 2.5 percent rate for
slightly less than one year and the account earned interest at the 2 percent rate for slightly
more than two years. Therefore, the balances of P1 and P2 were reached while the account
was earning interest at the 2 percent rate. Since P0(1.02) < P1 and P1(1.02) < P2
(compounding daily for one year produces a greater amount than compounding annually for
one year), the values of P0, P1, and P2 satisfy the following inequalities.
P0 < P0(1.02) < P1 < P1(1.02) < P2 < 1,000
Note that the difference 1,000 – P0 is the total amount of interest earned at the 2 percent
rate. Thus, using (2), we wish to determine whether this difference is greater than 25. From
P0(1.02) < P1 it follows that P0(1.02)2 < P1(1.02), and since P1(1.02) < 1,000, we have

P0(1.02)2 < 1,000. Therefore, P0 < , from which we can conclude the following

inequality.

Since 1,000 − > 25 (see below), it follows that 1,000 − P0 > 25 and hence the total

amount of interest earned at the 2 percent rate is greater than the total amount of interest
earned at the 2.5 percent rate.

One way to verify that 1,000 – > 25 is to verify that 1 – , or equivalently,

verify that , or 40 < 39(1.02)2. Now note that we can obtain this last inequality

from 40 < 39(1.04) (because 39 + 39(0.04) is greater than 39 + 1) and 1.04 < (1.02)2.
The correct answer is C;
both statements together are sufficient.
DS53541.01

128. A novelist pays her agent 15% of the royalties she receives from her novels. She pays her
publicist 5% of the royalties, plus a yearly fee. Did the novelist pay more to her agent last year
than she paid to her publicist?
1. The publicist's yearly fee is $2,000.
2. The novelist earned an average of $3,500 in royalties last year on each of her novels.
A. Statement (1) ALONE is sufficient, but statement (2) alone is not sufficient.
B. Statement (2) ALONE is sufficient, but statement (1) alone is not sufficient.
C. BOTH statements TOGETHER are sufficient, but NEITHER statement ALONE is
sufficient.
D. EACH statement ALONE is sufficient.
E. Statements (1) and (2) TOGETHER are NOT sufficient.
Arithmetic Applied problems
Let $R be the novelist's royalties last year, and let $Y be the yearly fee paid to the publicist.
Determine whether 0.15R > 0.05R + Y, or equivalently, whether R > 10Y.
1. No information is given that allows us to determine whether R is greater than
10Y = 10(2,000) = 20,000; NOT sufficient.
2. No information is given that allows us to determine whether 3,500n is greater than
10Y, where n is the number of novels; NOT sufficient.
Given (1) and (2) and letting n be the number of novels, we are to determine whether
3,500n > 20,000. If n = 1, then the answer is NO. However, if n = 10, then the answer is YES.
The correct answer is E;
both statements together are still not sufficient.
Answer Explanations Quantitative Reasoning
Data Sufficiency
Value/Order/Factors
DS85100.01

129. If x and z are integers, is x + z2 odd?


1. x is odd and z is even.
2. x – z is odd.
A. Statement (1) ALONE is sufficient, but statement (2) alone is not sufficient.
B. Statement (2) ALONE is sufficient, but statement (1) alone is not sufficient.
C. BOTH statements TOGETHER are sufficient, but NEITHER statement ALONE is
sufficient.
D. EACH statement ALONE is sufficient.
E. Statements (1) and (2) TOGETHER are NOT sufficient.
Arithmetic Properties of integers
1. We are given that x is odd and z is even. Therefore, z2 is even and hence x + z2 is odd,
because an odd integer added to an even integer is an odd integer; SUFFICIENT.
2. We are given that x – z is odd. Since there is not a readily apparent useful algebraic
relation between x – z and x + z2, we consider all possible cases.

x z z2 x − z x + z2
even even even even even
even odd odd odd odd
odd even even odd odd
odd odd odd even even

From the table it is clear that if x – z is odd, then x + z2 is odd; SUFFICIENT.


The correct answer is D;
each statement alone is sufficient.

DS95850.01
130. Each entry in the multiplication table above is an integer that is either positive, negative, or
zero. What is the value of a ?
1. h≠0
2. c=f
A. Statement (1) ALONE is sufficient, but statement (2) alone is not sufficient.
B. Statement (2) ALONE is sufficient, but statement (1) alone is not sufficient.
C. BOTH statements TOGETHER are sufficient, but NEITHER statement ALONE is
sufficient.
D. EACH statement ALONE is sufficient.
E. Statements (1) and (2) TOGETHER are NOT sufficient.
Arithmetic Properties of integers
1. If a, b, c equal 1, 2, 3 in this order, then each entry will be an integer, h ≠ 0, and a = 1.
However, if a, b, c equal 2, 3, 4 in this order, then each entry will be an integer, h ≠ 0, and
a = 2. Hence, the value of a cannot be determined; NOT sufficient.
2. The assumption c = f is equivalent to ac = c, or (a − 1)c = 0. Hence, c = f is equivalent to
a = 1 or c = 0. If a, b, c equal 1, 2, 3 in this order, then each entry will be an integer, c = f,
and a = 1. However, if a, b, c equal −2, −1, 0 in this order, then each entry will be an
integer, c = f, and a = −2. Hence, the value of a cannot be determined; NOT sufficient.
Given (1) and (2), then from (1) we have bc = h ≠ 0, and hence b ≠ 0 and c ≠ 0. From (2) we
have a = 1 or c = 0, but since c ≠ 0, it follows that a = 1. Hence, the value of a can be
determined.
The correct answer is C;
both statements together are sufficient.
DS36141.01

131. Given a positive number N, when N is rounded by a certain method (for convenience, call it
Method Y), the result is 10n if and only if n is an integer and 5 × 10n − 1 ≤ N < 5 × 10n. In a
certain gas sample, there are, when rounded by Method Y, 1021 molecules of H2 and also 1021
molecules of O2. When rounded by Method Y, what is the combined number of H2 and O2
molecules in the gas sample?
1. The number of H2 molecules and the number of O2 molecules are each less than
3 × 1021.
2. The number of H2 molecules is more than twice the number of O2 molecules.
A. Statement (1) ALONE is sufficient, but statement (2) alone is not sufficient.
B. Statement (2) ALONE is sufficient, but statement (1) alone is not sufficient.
C. BOTH statements TOGETHER are sufficient, but NEITHER statement ALONE is
sufficient.
D. EACH statement ALONE is sufficient.
E. Statements (1) and (2) TOGETHER are NOT sufficient.
Arithmetic Rounding
Let H be the number of H2 molecules and let O be the number of O2 molecules. We are given
that 0.5 × 1021 ≤ H < 5 × 1021 and 0.5 × 1021 ≤ O < 5 × 1021. When rounded by Method Y,
what is the value of H + O ?
1. If H = 2 × 1021 and O = 2 × 1021, then each of H and O equals 1021 when rounded by
Method Y, each of H and O is less than 3 × 1021, and H + O = 4 × 1021, which equals 1021
when rounded by Method Y. However, if H = 2.6 × 1021 and O = 2.6 × 1021, then each of
H and O equals 1021 when rounded by Method Y, each of H and O is less than 3 × 1021,
and H + O = 5.2 × 1021, which equals 1022 when rounded by Method Y; NOT sufficient.
2. If H = 2 × 1021 and O = 0.8 × 1021, then each of H and O equals 1021 when rounded by
Method Y, H > 2 × O, and H + O = 2.8 × 1021, which equals 1021 when rounded by
Method Y. However, if H = 4.5 × 1021 and O = 2 × 1021, then each of H and O equals 1021
when rounded by Method Y, H > 2 × O, and H + O = 6.5 × 1021, which equals 1022 when
rounded by Method Y; NOT sufficient.
Given (1) and (2), since O ≥ 0.5 × 1021 (given information), it follows from statement (2) that
H > 1 × 1021. Also, since H < 3 × 1021 (statement (1)), it follows from statement (2) that
O < 1.5 × 1021. Thus, 1 × 1021 < H < 3 × 1021 and 0.5 × 1021 ≤ O < 1.5 × 1021, and hence
1.5 × 1021 < H + O < 4.5 × 1021. Therefore, the value of H + O equals 1021 when rounded by
Method Y.
The correct answer is C;
both statements together are sufficient.
DS05541.01

132. If x is a positive integer, how many positive integers less than x are divisors of x ?
1. x2 is divisible by exactly 4 positive integers less than x2.
2. 2x is divisible by exactly 3 positive integers less than 2x.
A. Statement (1) ALONE is sufficient, but statement (2) alone is not sufficient.
B. Statement (2) ALONE is sufficient, but statement (1) alone is not sufficient.
C. BOTH statements TOGETHER are sufficient, but NEITHER statement ALONE is
sufficient.
D. EACH statement ALONE is sufficient.
E. Statements (1) and (2) TOGETHER are NOT sufficient.
Arithmetic Properties of integers

Tip

For problems that involve how many divisors an


unspecified integer has, it is sometimes useful to consider
separate cases based on the number of repeated prime
factors and the number of distinct prime factors the
integer has. For example, let p, q, and r be distinct prime
numbers. Then the factors of p3 are 1, p, q, and p2; the
factors of p2 are 1, p, q, r, p2, pr, qr, and pqr; the factors
of p3 are 1, p, p2, and p3; the factors of p2q are 1, p, p2, q,
pq, and p2q.

1. If x has at least two prime factors, say p and q, then among the factors of x2 are p, q, pq,
p2, q2, p2q, and pq2, each of which is less than x2 (because x2 ≥ p2q2). Thus, x cannot
have at least two prime factors, otherwise, x2 would have more than four divisors less
than x2. Therefore, x has the form x = pn for some prime number p and positive integer n.
There are 2n divisors of x2 = (pn)2 = p2n that are less than x2, namely 1, p, p2,
p3, …, p2n − 2, and p2n − 1. Statement (1) implies that 2n = 4, and hence n = 2. It follows
that x = p2 for some prime number p, and so x has exactly two divisors less than x,
namely 1 and p. Alternatively, the last part of this argument can be accomplished in a
more concrete way by separately considering the number of prime factors of p, p2, p3,
etc.; SUFFICIENT.
2. Probably the simplest approach is to individually consider the divisors of 2x that are
less than 2x for various values of x. If x = 1, then 2x = 2 has one such divisor, namely 1. If
x = 2, then 2x = 4 has two such divisors, namely 1 and 2. If x = 3, then 2x = 6 has three
such divisors, namely 1, 2, and 3. If x = 4, then 2x = 8 has three such divisors, namely 1, 2,
and 4. At this point we have two integers satisfying statement (2), x = 3 and x = 4. Since
x = 3 has one divisor less than x = 3 and x = 4 has two divisors less than x = 4; NOT
sufficient.
The correct answer is A;
statement 1 alone is sufficient.
DS33551.01

133. If m and n are positive integers, is n even?


1. m(m + 2) + 1 = mn
2. m(m + n) is odd.
A. Statement (1) ALONE is sufficient, but statement (2) alone is not sufficient.
B. Statement (2) ALONE is sufficient, but statement (1) alone is not sufficient.
C. BOTH statements TOGETHER are sufficient, but NEITHER statement ALONE is
sufficient.
D. EACH statement ALONE is sufficient.
E. Statements (1) and (2) TOGETHER are NOT sufficient.
Arithmetic Properties of integers
1. Given that m(m + 2) + 1 = mn, then m cannot be even, since if m were even, then we
would have an odd integer, namely m(m + 2) + 1, equal to an even integer, namely mn.
Therefore, m is odd. Hence, m(m + 2) is odd, being the product of two odd integers, and
thus m(m + 2) + 1 is even. Since m(m + 2) + 1 = mn, it follows that mn is even, and since
m is odd, it follows that n is even; SUFFICIENT.
Alternatively, the table below shows that m(m + 2) + 1 = mn is only possible when m is odd
and n is even.
m n m(m + 2) + 1 mn
even even odd even
even odd odd even
odd even even even
odd odd even odd
2. Since m(m + n) is odd, it follows that m is odd and m + n is odd. Therefore,
n = (m + n) − m is a difference of two odd integers and hence n is even; SUFFICIENT.
The correct answer is D;
each statement alone is sufficient.
DS65291.01

134. If m and n are positive integers, what is the value of + ?

1. mn = 12

2. is in lowest terms and is in lowest terms.

A. Statement (1) ALONE is sufficient, but statement (2) alone is not sufficient.
B. Statement (2) ALONE is sufficient, but statement (1) alone is not sufficient.
C. BOTH statements TOGETHER are sufficient, but NEITHER statement ALONE is
sufficient.
D. EACH statement ALONE is sufficient.
E. Statements (1) and (2) TOGETHER are NOT sufficient.
Arithmetic Fractions

1. If m = 1 and n = 12, then m and n are positive integers, mn = 12, and = 6.

However, if m = 3 and n = 4, then m and n are positive integers, mn = 12, and = 2;


NOT sufficient.

2. If m = 2 and n = 1, then m and n are positive integers, and are in lowest terms, and

. However, if m = 2 and n = 3, then m and n are positive integers, and

are in lowest terms, and ; NOT sufficient.

Given (1) and (2), the table below shows that only one possibility exists for the values of m
and n, and hence there is only one possible value of .

(m,n) lowest terms? lowest terms?

(1,12)
, YES , NO

(2,6) , YES , NO

(3,4) , NO , NO

(4,3) , YES , YES

(6,2) , NO , NO

(12,1) , NO , YES

The correct answer is C;


both statements together are sufficient.
DS21891.01

135. The first four digits of the six-digit initial password for a shopper's card at a certain grocery
store is the customer's birthday in day-month digit form. For example, 15 August
corresponds to 1508 and 5 March corresponds to 0503. The 5th digit of the initial password
is the units digit of seven times the sum of the first and third digits, and the 6th digit of the
initial password is the units digit of three times the sum of the second and fourth digits. What
month, and what day of that month, was a customer born whose initial password ends in 16 ?
1. The customer's initial password begins with 21, and its fourth digit is 1.
2. The sum of the first and third digits of the customer's initial password is 3, and its
second digit is 1.
A. Statement (1) ALONE is sufficient, but statement (2) alone is not sufficient.
B. Statement (2) ALONE is sufficient, but statement (1) alone is not sufficient.
C. BOTH statements TOGETHER are sufficient, but NEITHER statement ALONE is
sufficient.
D. EACH statement ALONE is sufficient.
E. Statements (1) and (2) TOGETHER are NOT sufficient.
Arithmetic Computation with integers
Let d1, d2, m1, and m2, respectively, represent the first four digits of the customer's initial
password. Then, the entire password has the form d1d2m1m216.
Because the maximum number of days per month is 31 and the number of months in a year
is 12, d1 can be only 0, 1, 2, or 3 and m1 can be only 0 or 1. The following summarizes the
possible values for d1, d2, m1, and m2.

d1 d2 m1 m2
0 1−9 0 1−9
1 0−9 1 0, 1, 2
2 0−9
3 0, 1
It is given that the fifth digit, which is 1, is the units digit of 7(d1 + m1). The only relevant
multiple of 7 with units digit 1 is (7)(3) = 21, from which it follows that d1 + m1 = 3.
Considering the restrictions on the values of the digits, then d1 = 2 and m1 = 1 or d1 = 3 and
m1 = 0. Also, it is given that the sixth digit, which is 6, is the units digit of 3(d2 + m2). The
only relevant multiples of 3 with units digit 6 are (3)(2) = 6 and (3)(12) = 36, from which it
follows that d2 + m2 = 2 or d2 + m2 = 12.
Considering the restrictions on the values of the digits, if d2 + m2 = 2, then the only
possibilities are d2 = 0 and m2 = 2 or d2 = 1 and m2 = 1 or d2 = 2 and m2 = 0. If d2 + m2 = 12,
each of d2 and m2 is at least 3 because if either of the digits is less than 3, then the sum of the
two digits cannot be 12. But if d1 = 2 and m1 = 1, which is one of the possibilities for d1 and
m1 above, then m2 can be only 0, 1, or 2; and if d1 = 3 and m1 = 0, which is the other
possibility above for d1 and m1, then d2 can be only 0 or 1. The table below summarizes the
first four digits of the passwords that meet all conditions thus far.
First digit d1 Second digit d2 Third digit m1 Fourth digit m2
2 0 1 2
2 1 1 1
2 2 1 0
3 0 0 2
3 1 0 1
3 2 0 0
1. It is given that the customer's password begins with 21 and the fourth digit is 1. In the
table above, only one possible password meets these conditions, so the first four digits of
the password are 2111 and the customer's birthday is the 21st day of November;
SUFFICIENT.
2. It is given that d1 + m1 = 3 and d2 = 1. In the table above, the possibilities for the first
four digits of the customer's password, where d2 = 1, are 3101 and 2111, so the customer's
birthday could be the 31st day of January or the 21st day of November; NOT sufficient.
The correct answer is A;
statement 1 alone is sufficient.
DS38302.01

136. If K is a positive integer less than 10 and N = 4,321 + K, what is the value of K ?
1. N is divisible by 3.
2. N is divisible by 7.
A. Statement (1) ALONE is sufficient, but statement (2) alone is not sufficient.
B. Statement (2) ALONE is sufficient, but statement (1) alone is not sufficient.
C. BOTH statements TOGETHER are sufficient, but NEITHER statement ALONE is
sufficient.
D. EACH statement ALONE is sufficient.
E. Statements (1) and (2) TOGETHER are NOT sufficient.
Arithmetic Computation with integers
1. Dividing 4,321 by 3 gives a quotient of 1,440 and a remainder of 1, so
4,321 = 3(1,440) + 1. It follows that N = [3(1,440) + 1] + K = 3(1,440) + (1 + K). It is given
that N is divisible by 3, from which it follows that 1 + K must be a multiple of 3. Therefore
K can be 2, 5, or 8 since K < 10.

Alternatively, a number is divisible by 3 if and only if the sum of its digits is divisible by 3.
If K ≠ 9, the sum of the digits of N = 4,321 + K is 4 + 3 + 2 + 1 + K = 10 + K = 1 + K, which
is divisible by 3 when K = 2, 5, or 8; NOT sufficient.
2. Dividing 4,321 by 7 gives a quotient of 617 and a remainder of 2, so 4,321 = 7(617) + 2.
It follows that N = [7(617) + 2] + K = 7(617) + (2 + K). It is given that N is divisible by 7
from which it follows that 2 + K must be a multiple of 7. Thus, K = 5 since K < 10;
SUFFICIENT.
The correct answer is B;
statement 2 alone is sufficient.
DS99302.01

137. If s is an integer, is 24 a divisor of s ?


1. Each of the numbers 3 and 8 is a divisor of s.
2. Each of the numbers 4 and 6 is a divisor of s.
A. Statement (1) ALONE is sufficient, but statement (2) alone is not sufficient.
B. Statement (2) ALONE is sufficient, but statement (1) alone is not sufficient.
C. BOTH statements TOGETHER are sufficient, but NEITHER statement ALONE is
sufficient.
D. EACH statement ALONE is sufficient.
E. Statements (1) and (2) TOGETHER are NOT sufficient.
Arithmetic Properties of integers
1. If each of the numbers 3 and 8 is a divisor of s, then using the prime factorization of 8
gives s = 23 × 3 × q = 24q, for some positive integer q. Thus, 24 is a divisor of s;
SUFFICIENT.
2. If each of the numbers 4 and 6 is a divisor of s, then s could be 24 and it follows that 24
is a divisor of s. On the other hand, s could be 12 because 4 and 6 are both divisors of 12
and 24 is not a divisor of s; NOT sufficient.
The correct answer is A;
statement 1 alone is sufficient.

Tip
If the integer n is divisible by each of the integers a and b
and the greatest common factor of a and b is 1, then n is
divisible by ab. However, if the greatest common divisor
of a and b is greater than 1, then n may or may not be
divisible by ab.

DS32402.01

138. n = 24 ⋅ 32 ⋅ 52 and positive integer d is a divisor of n. Is d > ?


1. d is divisible by 10.
2. d is divisible by 36.
A. Statement (1) ALONE is sufficient, but statement (2) alone is not sufficient.
B. Statement (2) ALONE is sufficient, but statement (1) alone is not sufficient.
C. BOTH statements TOGETHER are sufficient, but NEITHER statement ALONE is
sufficient.
D. EACH statement ALONE is sufficient.
E. Statements (1) and (2) TOGETHER are NOT sufficient.
Arithmetic Properties of integers
Given n = 24 ⋅ 32 ⋅ 52, then = (22)(3)(5) = 60. If d is a divisor of n,
determine whether d > 60 is true.
1. It is given that d is divisible by 10. If d = 10, then d is divisible by 10 and d is a divisor of
n since n = (10)(23 ⋅ 32 ⋅ 5), but d > 60 is not true. However, if d = 80, then d is divisible
by 10, d is a divisor of n since n = (80)(32 ⋅ 5), and d > 60 is true; NOT sufficient.
2. It is given that d is divisible by 36. If d = 36, then d is divisible by 36 and d is a divisor
of n since n = (36)(22 ⋅ 52), but d > 60 is not true. However, if d = 72, then d is divisible by
36, d is a divisor of n since n = (72)(2 ⋅ 52), and d > 80 is true; NOT sufficient.
Taking (1) and (2) together, it follows that d is divisible by 22 ⋅ 32 ⋅ 5 = 180 and every multiple
of 180 is greater than 60.
The correct answer is C;
both statements together are sufficient.
DS52402.01

139. Exactly 3 deposits have been made in a savings account and the amounts of the deposits are 3
consecutive integer multiples of $7. If the sum of the deposits is between $120 and $170,
what is the amount of each of the deposits?
1. The amount of one of the deposits is $49.
2. The amount of one of the deposits is $63.
A. Statement (1) ALONE is sufficient, but statement (2) alone is not sufficient.
B. Statement (2) ALONE is sufficient, but statement (1) alone is not sufficient.
C. BOTH statements TOGETHER are sufficient, but NEITHER statement ALONE is
sufficient.
D. EACH statement ALONE is sufficient.
E. Statements (1) and (2) TOGETHER are NOT sufficient.
Arithmetic Properties of integers
If k represents the least of the multiples of 7, then the three deposits, in dollars, are
represented by 7k, 7(k + 1), and 7(k + 2). The sum, in dollars, of the deposits is 21k + 21,
where 120 < 21k + 21 < 170. It follows that the value of the integer k is 5, 6, or 7. If k = 5, then
the deposits could be 35, 42, 49, with a sum of 126, which is between 120 and 170. If k = 6,
then the deposits could be 42, 49, 56 with a sum of 147, which is between 120 and 170. If
k = 7, then the deposits could be 49, 56, 63 with a sum of 168, which is between 120 and 170.
1. It is given that one of the deposits, in dollars, is 49. Since 49 is one of the amounts for
each value of k in the remarks above, the amounts of the three deposits cannot be
determined; NOT sufficient.
2. It is given that one of the deposits, in dollars, is 63. Since 63 occurs for exactly one
value of k in the remarks above, the amounts, in dollars, of the deposits are 49, 56, and
63; SUFFICIENT.
The correct answer is B;
statement 2 alone is sufficient.
DS44402.01

140. If x, y, and d are integers and d is odd, are both x and y divisible by d ?
1. x + y is divisible by d.
2. x − y is divisible by d.
A. Statement (1) ALONE is sufficient, but statement (2) alone is not sufficient.
B. Statement (2) ALONE is sufficient, but statement (1) alone is not sufficient.
C. BOTH statements TOGETHER are sufficient, but NEITHER statement ALONE is
sufficient.
D. EACH statement ALONE is sufficient.
E. Statements (1) and (2) TOGETHER are NOT sufficient.
Arithmetic Properties of integers
Determine whether both of the integers x and y are divisible by the odd integer d.
1. It is given that x + y is divisible by d. It is possible that both x and y are divisible by d,
and it is possible that they are not both divisible by d. For example, if x = 4, y = 2, and
d = 3, then 4 + 2 is divisible by 3, but neither 4 nor 2 is divisible by 3. On the other hand,
if x = 3, y = 6, and d = 3, then 3 + 6 is divisible by 3, and both 3 and 6 are divisible by 3;
NOT sufficient.
2. It is given that x – y is divisible by d. It is possible that both x and y are divisible by d,
and it is possible that they are not both divisible by d. For example, if x = 4, y = −2, and
d = 3, then 4 − (−2) is divisible by 3, but neither 4 nor –2 is divisible by 3. On the other
hand, if x = 3, y = −6, and d = 3, then 3 − (−6) is divisible by 3, and both 3 and −6 are
divisible by 3; NOT sufficient.
Taking (1) and (2) together, x + y is divisible by d, so is an integer and x – y is divisible

by d, so is an integer. It follows that + = is an integer and is an

integer because d is odd. Similarly, − = is an integer and is an integer


because d is odd.
The correct answer is C;
both statements together are sufficient.
DS06402.01

141. If x and y are integers, is xy + 1 divisible by 3 ?


1. When x is divided by 3, the remainder is 1.
2. When y is divided by 9, the remainder is 8.
A. Statement (1) ALONE is sufficient, but statement (2) alone is not sufficient.
B. Statement (2) ALONE is sufficient, but statement (1) alone is not sufficient.
C. BOTH statements TOGETHER are sufficient, but NEITHER statement ALONE is
sufficient.
D. EACH statement ALONE is sufficient.
E. Statements (1) and (2) TOGETHER are NOT sufficient.
Arithmetic Properties of integers
Determine whether xy + 1 is divisible by 3, where x and y are integers.
1. It is given that the remainder is 1 when x is divided by 3. It follows that x = 3q + 1 for
some integer q. So, xy + 1 = (3q + 1)y + 1. If y = 2, then xy + 1 = 6q + 3, which is divisible
by 3. However, if y = 1, then xy + 1 = 3q + 2, which is not divisible by 3; NOT sufficient.
2. It is given that the remainder is 8 when y is divided by 9. It follows that y = 9r + 8 for
some integer r. So, xy + 1 = (9r + 8)x + 1. If x = 1, then xy + 1 = 9r + 9, which is divisible
by 3. However, if x = 2, then xy + 1 = 18r + 17, which is not divisible by 3; NOT sufficient.
Taking (1) and (2) together gives x = 3q + 1 and y = 9r + 8, from which it follows that
xy + 1 = (3q + 1)(9r + 8) + 1 = 27qr + 9r + 24q + 9 = 3(9qr + 3r + 8q + 3), which is divisible
by 3.
The correct answer is C;
both statements together are sufficient.
DS00502.01

142. If x and y are integers between 10 and 99, inclusive, is an integer?

1. x and y have the same two digits, but in reverse order.


2. The tens' digit of x is 2 more than the units' digit, and the tens' digit of y is 2 less than
the units' digit.
A. Statement (1) ALONE is sufficient, but statement (2) alone is not sufficient.
B. Statement (2) ALONE is sufficient, but statement (1) alone is not sufficient.
C. BOTH statements TOGETHER are sufficient, but NEITHER statement ALONE is
sufficient.
D. EACH statement ALONE is sufficient.
E. Statements (1) and (2) TOGETHER are NOT sufficient.
Algebra Properties of integers
Determine whether is an integer, where x and y are 2-digit integers.

1. From (1), if x = 10a + b, then y = 10b + a. It follows that x – y = 9(a – b), so is an


integer; SUFFICIENT.
2. From (2), if x = 10a + b, then a = b + 2. If y = 10c + d, then c = d – 2. It is possible that
is an integer (for example, if x = 75 and y = 57), and it is possible that is not
an integer (for example, if x = 75 and y = 46); NOT sufficient.
The correct answer is A;
statement 1 alone is sufficient.
DS85502.01

143. If b is the product of three consecutive positive integers c, c + 1, and c + 2, is b a multiple of


24 ?
1. b is a multiple of 8.
2. c is odd.
A. Statement (1) ALONE is sufficient, but statement (2) alone is not sufficient.
B. Statement (2) ALONE is sufficient, but statement (1) alone is not sufficient.
C. BOTH statements TOGETHER are sufficient, but NEITHER statement ALONE is
sufficient.
D. EACH statement ALONE is sufficient.
E. Statements (1) and (2) TOGETHER are NOT sufficient.
Arithmetic Properties of integers
Since 24 = 23 × 3, and 1 is the only common factor of 2 and 3, any positive integer that is a
multiple of 24 must be a multiple of both 23 = 8 and 3. Furthermore, the product of any three
consecutive positive integers is a multiple of 3. This can be shown as follows. In b = c(c + 1)
(c + 2), when the positive integer c is divided by 3, the remainder must be 0, 1, or 2. If the
remainder is 0, then c itself is a multiple of 3. If the remainder is 1, then c = 3q + 1 for some
positive integer q and c + 2 = 3q + 3 = 3(q + 1) is a multiple of 3. If the remainder is 2, then
c = 3r + 2 for some positive integer r and c + 1 = 3r + 3 = 3(r + 1) is a multiple of 3. In all
cases, b = c(c + 1)(c + 2) is a multiple of 3.
1. It is given that b is a multiple of 8. It was shown above that b is a multiple of 3, so b is a
multiple of 24; SUFFICIENT.
2. It is given that c is odd. If c = 3, then b = (3)(4)(5) = 60, which is not a multiple of 24. If
c = 7, then b = (7)(8)(9) = (24)(7)(3), which is a multiple of 24; NOT sufficient.
The correct answer is A;
statement 1 alone is sufficient.
DS17602.01

144. If ⊛ denotes a mathematical operation, does x ⊛ y = y ⊛ x for all x and y ?

1. For all x and y, x ⊛ y = 2(x 2 + y 2).

2. For all y, 0 ⊛ y = 2y 2.
A. Statement (1) ALONE is sufficient, but statement (2) alone is not sufficient.
B. Statement (2) ALONE is sufficient, but statement (1) alone is not sufficient.
C. BOTH statements TOGETHER are sufficient, but NEITHER statement ALONE is
sufficient.
D. EACH statement ALONE is sufficient.
E. Statements (1) and (2) TOGETHER are NOT sufficient.
Algebra Functions
1. For all x and y, x y = 2(x2 + y2) is equal to y x = 2(y2 + x2); SUFFICIENT.
2. If x y = 2(x2 + y2) for all x and y, then 0 y = 2y2 for all y and x y = y x for all x
and y. However, if x y = 2(x3 + y2) for all x and y, then 0 y = 2y 2 for all y, but 1
2 = 2(13) + 2(22) = 10 and 2 1
= 2(23) + 2(12) = 18; NOT sufficient.
The correct answer is A;
statement 1 alone is sufficient.
DS37602.01

145. If n is an integer, is an integer?

1. is an integer.

2. is an integer.

A. Statement (1) ALONE is sufficient, but statement (2) alone is not sufficient.
B. Statement (2) ALONE is sufficient, but statement (1) alone is not sufficient.
C. BOTH statements TOGETHER are sufficient, but NEITHER statement ALONE is
sufficient.
D. EACH statement ALONE is sufficient.
E. Statements (1) and (2) TOGETHER are NOT sufficient.
Arithmetic Properties of integers

1. We are given that is an integer. If n = 15, then is an integer and is an

integer. However, if n = 5, then is an integer and is not an integer; NOT sufficient.


2. We are given that = k, where k is an integer. Since 8n = 15k, it follows that both 3
and 5 are factors of 8n. Therefore, the prime factorization of 8n = 23 × n includes at least
one factor of 3 and at least one factor of 5, and it is clear that each of these factors must be
among the prime factors of n. Thus, both 3 and 5 are factors of n, and hence n is divisible
by 15.
Alternatively, since 15 divides 8n, and 8 and 15 are relatively prime, then it follows that 15
divides n; SUFFICIENT.
The correct answer is B;
statement 2 alone is sufficient.
DS97602.01

146. If 1 < d < 2, is the tenths digit of the decimal representation of d equal to 9 ?
1. d + 0.01 < 2
2. d + 0.05 > 2
A. Statement (1) ALONE is sufficient, but statement (2) alone is not sufficient.
B. Statement (2) ALONE is sufficient, but statement (1) alone is not sufficient.
C. BOTH statements TOGETHER are sufficient, but NEITHER statement ALONE is
sufficient.
D. EACH statement ALONE is sufficient.
E. Statements (1) and (2) TOGETHER are NOT sufficient.
Arithmetic Place value
Determine if the tenths digit of d, where 1 < d < 2, is 9.
1. Given d + 0.01 < 2, it follows that d < 1.99. It is possible that the tenths digit of d is 9
(for example, 1.98 < 1.99, and the tenths digit of 1.98 is 9), and it is possible that the
tenths digit of d is not 9 (for example, 1.88 < 1.99, and the tenths digit of 1.98 is 8); NOT
sufficient.
2. Given d + 0.05 > 2, it follows that d > 1.95. Then 1.95 < d < 2, since d < 2, so the tenths
digit of d is 9; SUFFICIENT.
The correct answer is B;
statement 2 alone is sufficient.
DS08602.01

147. The 9 participants in a race were divided into 3 teams with 3 runners on each team. A team
was awarded 6 – n points if one of its runners finished in nth place, where 1 ≤ n ≤ 5. If all of
the runners finished the race and if there were no ties, was each team awarded at least 1
point?
1. No team was awarded more than a total of 6 points.
2. No pair of teammates finished in consecutive places among the top five places.
A. Statement (1) ALONE is sufficient, but statement (2) alone is not sufficient.
B. Statement (2) ALONE is sufficient, but statement (1) alone is not sufficient.
C. BOTH statements TOGETHER are sufficient, but NEITHER statement ALONE is
sufficient.
D. EACH statement ALONE is sufficient.
E. Statements (1) and (2) TOGETHER are NOT sufficient.
Arithmetic Operations with integers
Determine whether each team was awarded at least 1 point.
1. It is given that no team was awarded more than 6 points. Since there were no ties, one
of the nine runners had to have finished in first place. Say this runner was on Team A,
and Team A was awarded 5 points. Since Team A was awarded at most 6 points, the best
finish for one of the two other runners on Team A could be fifth place, leaving second
place to a runner on one of the other teams. Say a runner on Team B finished in second
place, and Team B was awarded 4 points. Since Team B was awarded at most 6 points, the
best finish for one of the two other runners on Team B could be fourth place, leaving third
place to a runner on the only team remaining, which would then be awarded 3 points.
Thus, each team was awarded at least 1 point; SUFFICIENT.
2. Given that no pair of teammates finished in consecutive places, it is possible that each
team was awarded at least 1 point and it is also possible that at least one team was not
awarded at least 1 point. For example, if the three runners on Team A placed first, third,
and fifth and two runners on Team B placed second and fourth, then no pair of
teammates finished in consecutive places, and Team C was awarded 0 points. On the
other hand, if the three runners on Team A placed first, third, and fifth, a runner on Team
B placed second, and a runner on Team C placed fourth, then no pair of teammates
finished in consecutive places and each team was awarded at least 1 point; NOT sufficient.
The correct answer is A;
statement 1 alone is sufficient.
DS38602.01

148. Can the positive integer n be written as the sum of two different positive prime numbers?
1. n is greater than 3.
2. n is odd.
A. Statement (1) ALONE is sufficient, but statement (2) alone is not sufficient.
B. Statement (2) ALONE is sufficient, but statement (1) alone is not sufficient.
C. BOTH statements TOGETHER are sufficient, but NEITHER statement ALONE is
sufficient.
D. EACH statement ALONE is sufficient.
E. Statements (1) and (2) TOGETHER are NOT sufficient.
Arithmetic Properties of integers
Determine if the positive integer n can be written as the sum of two different positive prime
numbers.
1. It is given that n > 3. If n = 5, then n = 2 + 3 and 2 and 3 are different positive prime
numbers. However, n = 11, then n can be written as the following sums of two different
positive numbers: 1 + 10, 2 + 9, 3 + 8, 4 + 7, and 5 + 6. In no case are the addends both
prime; NOT sufficient.
2. It is given that n is odd. The values of n in the examples used to show that (1) is not
sufficient also satisfy (2); NOT sufficient.
Taken together, (1) and (2) are not sufficient because the same examples used to show that
(1) is not sufficient also show that (2) is not sufficient.
The correct answer is E;
both statements together are still not sufficient.
DS73402.01

149. Is x an integer?
1. x2 is an integer.
2. is not an integer.

A. Statement (1) ALONE is sufficient, but statement (2) alone is not sufficient.
B. Statement (2) ALONE is sufficient, but statement (1) alone is not sufficient.
C. BOTH statements TOGETHER are sufficient, but NEITHER statement ALONE is
sufficient.
D. EACH statement ALONE is sufficient.
E. Statements (1) and (2) TOGETHER are NOT sufficient.
Algebra Properties of integers
Determine if x is an integer.
1. It is given that x2 is an integer. If, for example, x2 = 49, then x is an integer. However, if
x2 = 5, then x is not an integer; NOT sufficient.
2. It is given that is not an integer. If, for example, x = 7, then is not an integer, but x

is an integer. However, if x = , then is not an integer and neither is x; NOT sufficient.

Taken together, (1) and (2) are not sufficient because the same examples used to show that
(1) is not sufficient also show that (2) is not sufficient.
The correct answer is E;
both statements together are still not sufficient.
DS46402.01

150. If b is an integer, is an integer?


1. a 2 + b 2 is an integer.
2. a 2 − 3b 2 = 0
A. Statement (1) ALONE is sufficient, but statement (2) alone is not sufficient.
B. Statement (2) ALONE is sufficient, but statement (1) alone is not sufficient.
C. BOTH statements TOGETHER are sufficient, but NEITHER statement ALONE is
sufficient.
D. EACH statement ALONE is sufficient.
E. Statements (1) and (2) TOGETHER are NOT sufficient.
Algebra Operations on radical expressions
Given that b is an integer, determine if is an integer.
1. It is given that a 2 + b 2 is an integer. If a = 3 and b = 4, then = 5, which is an
integer. However, if a = 1 and b = 2, then = , which is not an integer; NOT
sufficient.
2. It is given that a 2 – 3b 2 = 0, from which it follows that a 2 = 3b 2. Then, =
= = , which is an integer; SUFFICIENT.
The correct answer is B;
statement 2 alone is sufficient.
Chapter 3 Verbal Reasoning
Practice Questions Verbal Reasoning
Reading Comprehension
RC62100.01

Line Anthropologists once thought that the ancestors


of modern humans began to walk upright because
it freed their hands to use stone tools, which they
had begun to make as the species evolved a brain of
(5) increased size and mental capacity. But discoveries
of the three-million-year-old fossilized remains of
our hominid ancestor Australopithecus have yielded
substantial anatomical evidence that upright walking
appeared prior to the dramatic enlargement of the
(10) brain and the development of stone tools.
Walking on two legs in an upright posture (bipedal
locomotion) is a less efficient proposition than walking
on all fours (quadrupedal locomotion) because several
muscle groups that the quadruped uses for propulsion
(15) must instead be adapted to provide the biped with
stability and control. The shape and configuration
of various bones must likewise be modified to allow
the muscles to perform these functions in upright
walking. Reconstruction of the pelvis (hipbones) and
(20) femur (thighbone) of “Lucy,” a three-million-year-old
skeleton that is the most complete fossilized skeleton
from the Australopithecine era, has shown that they
are much more like the corresponding bones of the
modern human than like those of the most closely
(25) related living primate, the quadrupedal chimpanzee.
Lucy's wide, shallow pelvis is actually better suited to
bipedal walking than is the rounder, bowl-like pelvis of
the modern human, which evolved to form the larger
birth canal needed to accommodate the head of a
(30) large-brained human infant. By contrast, the head of
Lucy's baby could have been no larger than that of a
baby chimpanzee.
If the small-brained australopithecines were not
toolmakers, what evolutionary advantage did they
(35) gain by walking upright? One theory is that bipedality
evolved in conjunction with the nuclear family:
monogamous parents cooperating to care for their
offspring. Walking upright permitted the father to
use his hands to gather food and carry it to his mate
(40) from a distance, allowing the mother to devote more
time and energy to nurturing and protecting their
children. According to this view, the transition to
bipedal walking may have occurred as long as ten
million years ago, at the time of the earliest hominids,
(45) making it a crucial initiating event in human evolution.

Questions 151–155 refer to the passage.


RC62100.01-10

151. The primary purpose of the passage is to


A. present an interpretation of the chronological relationship between bipedal locomotion
and certain other key aspects of human evolution
B. compare the evolutionary advantages and disadvantages of bipedal locomotion to those of
quadrupedal locomotion
C. argue that the transition to a nuclear family structure was a more crucial step in human
evolution than was the development of stone tools
D. analyze anatomical evidence of bipedal locomotion to show that the large brain of modern
humans developed at a later stage of evolution than was previously believed
E. use examples of muscle and bone structure to illustrate the evolutionary differences
between modern humans, australopithecines, and chimpanzees
RC62100.01-20

152. The passage suggests that proponents of the theory mentioned in lines 35–38 assume that
which of the following steps in human evolution occurred most recently?
A. Development of a nuclear family structure
B. Transition from walking on all fours to walking upright
C. Dramatic enlargement of the brain
D. Use of the hands to gather and carry food
E. Modification of propulsive muscles to provide stability and control in locomotion
RC62100.01-30

153. According to the passage, the hominid australopithecine most closely resembled a modern
human with respect to which of the following characteristics?
A. Brain size
B. Tool-making ability
C. Shape of the pelvis
D. Method of locomotion
E. Preference for certain foods
RC62100.01-40

154. The passage suggests that, in comparison with the hominid australopithecines, modern
humans are
A. less well adapted to large-group cooperation
B. less well adapted to walking upright
C. more agile in running and climbing
D. more well suited to a nuclear family structure
E. more well suited to cooperative caring for their offspring
RC62100.01-50

155. The theory mentioned in lines 35–38 suggests that which of the following was true for the
hominid ancestors of modern humans before they made the transition to walking upright?
A. Their brains were smaller than the brains of present-day chimpanzees.
B. They competed rather than cooperated in searching for food.
C. Their mating patterns and family structure were closer to those of present-day
chimpanzees than to those of modern humans.
D. Males played a more significant role in child rearing than they played after the transition
to walking upright.
E. Females' ability to nurture and protect their offspring was limited by the need to find food
for themselves.

RC04200.01

Line Recent feminist scholarship concerning the United


States in the 1920s challenges earlier interpretations
that assessed the 1920s in terms of the unkept
“promises” of the women's suffrage movement. This
(5) new scholarship disputes the long-held view that
because a women's voting bloc did not materialize
after women gained the right to vote in 1920,
suffrage failed to produce long-term political gains
for women. These feminist scholars also challenge
(10) the old view that pronounced suffrage a failure for
not delivering on the promise that the women's vote
would bring about moral, corruption-free governance.
Asked whether women's suffrage was a failure, these
scholars cite the words of turn-of-the-century social
(15) reformer Jane Addams, “Why don't you ask if suffrage
in general is failing?”
In some ways, however, these scholars still present
the 1920s as a period of decline. After suffrage, they
argue, the feminist movement lost its cohesiveness,
(20) and gender consciousness waned. After the mid-
1920s, few successes could be claimed by feminist
reformers: little could be seen in the way of legislative
victories.
During this decade, however, there was intense
(25) activism aimed at achieving increased autonomy for
women, broadening the spheres within which they
lived their daily lives. Women's organizations worked
to establish opportunities for women: they strove to
secure for women the full entitlements of citizenship,
(30) including the right to hold office and the right to serve
on juries.

Questions 156–161 refer to the passage.

RC04200.01-10

156. The passage is primarily concerned with


A. providing evidence indicating that feminist reformers of the 1920s failed to reach some of
their goals
B. presenting scholarship that contrasts suffragist “promises” with the historical realities of
the 1920s
C. discussing recent scholarship concerning the achievements of women's suffrage during
the 1920s and presenting an alternative view of those achievements
D. outlining recent findings concerning events leading to suffrage for women in the 1920s
and presenting a challenge to those findings
E. providing support for a traditional view of the success of feminist attempts to increase
gender consciousness among women during the 1920s

RC04200.01-20

157. It can be inferred that the author of the passage disagrees with the “new scholarship”
mentioned in line 5 regarding the
A. degree to which the “promises” of the suffrage movement remained unkept
B. degree to which suffrage for women improved the morality of governance
C. degree to which the 1920s represented a period of decline for the feminist movement
D. degree of legislative success achieved by feminist reformers during the 1920s
E. accuracy of the view that a women's voting bloc did not materialize once suffrage was
achieved

RC04200.01-30

158. The purpose of the second paragraph of the passage is to


A. suggest a reason why suffragist “promises” were not kept
B. contrast suffragist “promises” with the reality of the 1920s
C. deplore the lack of successful feminist reform in the 1920s
D. explain a view held by feminist scholars
E. answer the question asked by Jane Addams
RC04200.01-40

159. It can be inferred from the passage that recent scholars cite the words of Jane Addams
primarily in order to
A. suggest that women's achievement of suffrage brought about changes in government that
were not taken into account by early interpretations
B. point out contradictions inherent in the goals of the women's suffrage movement
C. show why a women's voting bloc was not formed when women won the right to vote
D. emphasize the place of social reform movements in the struggle for suffrage for women
E. suggest that the old view of women's suffrage was inappropriate
RC04200.01-50

160. It can be inferred that the analyses of the author of the passage and the scholars mentioned
in lines 20–23 differ with regard to which of the following?
A. The criteria they use to evaluate the success of the feminist movement during the 1920s
B. Their interpretations of the “promises” of the suffragist movement
C. The suggestions they make for achieving feminist goals
D. Their definitions of what constitutes a legislative victory
E. Their estimations of the obstacles preventing women's having achieved a voting bloc in
the 1920s

RC04200.01-60

161. The “new scholarship” mentioned in the first paragraph suggests which of the following
concerning the “promises” mentioned in lines 4–5?
A. Failure to keep these promises is not a measure of the success of the suffrage movement.
B. Failure to keep these promises caused the feminist movement to lose cohesiveness during
the 1920s.
C. Failure to keep these promises led recent feminist scholars to reevaluate the success of
the suffrage movement.
D. These promises included securing for women the right to hold office and the right to serve
on juries.
E. These promises were of little importance in winning suffrage for women.

RC60500.01

Line This passage is excerpted from material published


in 1997.
Is there a massive black hole at the center of our
galaxy, the Milky Way? The evidence is inconclusive.
(5) Just as the Sun's mass can be determined, given
knowledge of other variables, by the velocity at
which its planets orbit, the mass at the center of the
Milky Way can be revealed by the velocities of stars
and gas orbiting the galactic center. This dynamical
(10) evidence, based on recently confirmed assumptions
about the stars' velocities, argues for an extremely
compact object with a mass two to three million
times the mass of our Sun. Although according to
current theory this makes the mass at the center
(15) of the galaxy too dense to be anything but a black
hole, the relative lack of energy radiating from the
galactic center presents a serious problem. A black
hole's gravity attracts surrounding matter, which
swirls around the black hole, emitting some energy
(20) as it is engulfed. Scientists believe that the amount of
energy that escapes the black hole should be about
10 percent of the matter's rest energy (the energy
equivalent of its mass according to the equation
E=mc2). But when the energy coming from the
(25) galactic center is compared to widely held predictions
based on how much matter should be falling into a
theoretical central black hole, there is a discrepancy
by a factor of a few thousand.

Questions 162–165 refer to the passage.

RC60500.01-10

162. The primary purpose of the passage is to


A. present several theories that could account for a particular phenomenon
B. argue that a certain question needs to be reframed in light of new evidence
C. resolve an apparent inconsistency between two lines of evidence
D. explain why a certain issue remains unresolved
E. present evidence that calls into question certain assumptions of a current theory
RC60500.01-20

163. According to the passage, the dynamical evidence referred to in lines 9–10 supports which of
the following?
A. Recent assumptions about the velocities of stars
B. Widely held predictions about the amount of matter a black hole will engulf
C. The existence of an extremely dense object at the center of the Milky Way
D. The contention that too much energy is coming from the mass at the Milky Way's galactic
center for that mass to be a black hole
E. The conclusion that a compact object of two to three million times the mass of our Sun is
too dense to be anything but a black hole
RC60500.01-30

164. The “serious problem” referred to in line 17 could be solved if which of the following were
true?
A. Current assumptions about how much matter a black hole would engulf proved to be
several thousand times too high.
B. Current assumptions about how much matter a black hole would engulf proved to be a
few thousand times too low.
C. The object at the center of the Milky Way turned out to be far more dense than it is
currently estimated to be.
D. The object at the center of the Milky Way turned out to be far more massive than it is
currently estimated to be.
E. Matter being engulfed by a black hole radiated far more energy than is currently assumed.
RC60500.01-40

165. The “widely held predictions” mentioned in line 25 are predictions about the
A. compactness of objects whose mass is millions of times the mass of our Sun
B. velocities of stars orbiting the galactic center
C. amount of matter swirling around the object at the center of the Milky Way
D. amount of matter falling into a theoretical central black hole
E. amount of energy that should be coming from a black hole at the center of the Milky Way

RC39461.01

Line Despite their many differences of temperament and


of literary perspective, Emerson, Thoreau, Hawthorne,
Melville, and Whitman shared certain beliefs. Common
to all these writers is their humanistic perspective.
(5) Its basic premises are that humans are the spiritual
center of the universe and that in them alone is the
clue to nature, history, and ultimately the cosmos.
Without denying outright the existence of a deity, this
perspective explains humans and the world in terms
(10) of humanity.
This common perspective is almost always
universalized. It emphasizes the human as universal,
freed from the accidents of time, space, birth, and
talent. Thus, for Emerson, the “American Scholar”
(15) turns out to be simply “Man Thinking,” while, for
Whitman, the “Song of Myself” merges imperceptibly
into a song of all the “children of Adam,” where “every
atom belonging to me as good belongs to you.”
Also common to all five writers is the belief
(20) that self-realization depends on the harmonious
reconciliation of two universal psychological
tendencies: first, the self-asserting impulse of
the individual to be responsible only to himself or
herself, and second, the self-transcending impulse
(25) of the individual to know and become one with
that world. These conflicting impulses can be seen
in the democratic ethic. Democracy advocates
individualism, the preservation of the individual's
freedom and self-expression. But the democratic self
(30) is torn between the duty to self, which is implied by
the concept of liberty, and the duty to society, which
is implied by the concepts of equality and fraternity.
A third assumption common to the five writers is
that intuition and imagination offer a surer road to
(35) truth than does abstract logic or scientific method. It
is illustrated by their emphasis upon introspection—
their belief that the clue to external nature is to be
found in the inner world of individual psychology—and
by their interpretation of experience as, in essence,
(40) symbolic. Both these stresses presume an organic
relationship between the self and the cosmos of
which only intuition and imagination can properly take
account. These writers' faith in the imagination and
in themselves led them to conceive of the writer as a
(45) seer.

Questions 166–172 refer to the passage.

RC39461.01-10

166. The author's discussion of Emerson, Thoreau, Hawthorne, Melville, and Whitman is
primarily concerned with explaining
A. some of their beliefs about the difficulties involved in self-realization
B. some of their beliefs concerning the world and the place that humanity occupies in the
universal order
C. some of their beliefs concerning the relationship between humanism and democracy
D. the way some of their beliefs are shaped by differences in temperament and literary
outlook
E. the effects of some of their beliefs on their writings

RC39461.01-20

167. According to the passage, the five writers object to the scientific method primarily because
they think it
A. is not the best way to obtain an understanding of the relationship between the individual
and the cosmos
B. is so specialized that it leads to an understanding of separate parts of the universe but not
of the relationships among those parts
C. cannot provide an adequate explanation of intuition and imagination
D. misleads people into believing they have an understanding of truth, when they do not
E. prevents people from recognizing the symbolic nature of experience

RC39461.01-30

168. The author quotes Whitman primarily in order to


A. show that the poet does not agree with Emerson
B. indicate the way the poet uses the humanist ideal to praise himself
C. suggest that the poet adapts the basic premises of humanism to his own individual
outlook on the world
D. illustrate a way the poet expresses the relationship of the individual to the humanistic
universe
E. demonstrate that the poet is concerned with the well-being of all humans

RC39461.01-40

169. It can be inferred that intuition is important to the five writers primarily because it provides
them with
A. information useful for understanding abstract logic and scientific method
B. the discipline needed in the search for truth
C. inspiration for their best writing
D. clues to the interpretation of symbolic experience
E. the means of resolving conflicts between the self and the world
RC39461.01-50

170. The author discusses “the democratic ethic” (see lines 26–32) in order to
A. explain the relationship between external experience and inner imagination
B. support the notion that the self contains two conflicting and irreconcilable factions
C. illustrate the relationship between the self's desire to be individual and its desire to merge
with all other selves
D. elaborate on the concept that the self constantly desires to realize its potential
E. give an example of the idea that, in order to be happy, the self must reconcile its desires
with external reality

RC39461.01-60

171. It can be inferred that the idea of “an organic relationship between the self and the cosmos”
(see lines 40–41) is necessary to the thinking of the five writers because such a relationship
A. enables them to assert the importance of the democratic ethic
B. justifies their concept of the freedom of the individual
C. sustains their faith in the existence of a deity
D. is the foundation of their humanistic view of existence
E. is the basis for their claim that the writer is a seer

RC39461.01-70

172. The passage is most relevant to which of the following areas of study?
A. Aesthetics and logic
B. History and literature
C. Theology and sociology
D. Anthropology and political science
E. Linguistics and art

RC49461.01

Line The final quarter of the nineteenth century marked


a turning point in the history of biology—biologists
became less interested in applying an ideal of
historical explanation deductively to organic function
(5) and more interested in discerning the causes of vital
processes through experimental manipulation. But it
is impossible to discuss the history of biology in the
nineteenth century without emphasizing that those
areas of biology most in the public eye had depended
(10) on historical explanation. Wherever it was applied,
historical explanation was deemed causal explanation.
The biologist-as-historian and the general historian of
human events dealt with comparable phenomena and
assumed necessarily a common mode of explanation.
(15) Nineteenth-century biologists found a historical
explanation of organic function attractive partly
because their observation of the formation of a
new cell from a preexisting cell seemed to confirm
a historical explanation of cell generation. The
(20) same direct observation of continuous stages of
development was also possible when they examined
the complex sequence of events of embryogenesis.
In both cases, the observer received a concrete
impression that the daughter cell was brought into
(25) being, or caused, by the prior cell. The argument
that these scientists employed confuses temporal
succession and causal explanation, of course,
but such confusion is the heart of most historical
explanation.
(30) Not surprisingly, the evolutionary biologists of
the nineteenth century encountered a particularly
troublesome problem in their attempts to document
historical explanation convincingly: the factual record
of the history of life on earth (e.g., that provided by
(35) fossils) was incomplete. The temporal continuity of
living forms was convincing, but was an assumption
that was difficult to uphold when one compared
species or organisms forming any two stages of the
evolutionary record. Nineteenth-century biologists
(40) recognized this problem and attempted to resolve
it. Their solution today appears to be only verbal,
but was then regarded as eminently causal. The fact
of evolution demanded some connection between
all reproducing individuals and the species that they
(45) compose, as well as between living species and
their extinct ancestors. Their solution, the concept
of heredity, seemed to fill in an admittedly deficient
historical record and seemed to complete the
argument for a historical explanation of evolutionary
(50) events.

Questions 173–177 refer to the passage.

RC49461.01-10

173. The primary purpose of the passage is to


A. compare the information about organic function made available by historical explanation
with that made available by the experimental investigation of living organisms
B. assess the influence that theories of history had on developments in the field of biology in
the nineteenth century
C. discuss the importance of historical explanation in the thinking of nineteenth century
biologists
D. contrast biologists' use of historical explanation during the early nineteenth century with
its use during the final quarter of the nineteenth century
E. evaluate the way in which the concept of heredity altered the use of historical explanation
by nineteenth-century biologists

RC49461.01-20

174. According to information presented in the passage, which of the following is a true statement
about the methods of explanation used by biologists and historians in the nineteenth
century?
A. Neither biologists nor historians were able to develop methods of explanation that were
accepted by the majority of their colleagues.
B. The methods used by biologists to explain phenomena changed dramatically, whereas the
methods used by historians to explain events did not change as noticeably.
C. Biologists believed that they had refined the methods of explanation used by historians.
D. Biologists' and historians' methods of explaining what they believed to be comparable
phenomena were similar.
E. Although biologists and historians adopted similar methods of explanation, the biologists
were more apologetic about their use of these methods.
RC49461.01-30

175. Which of the following best summarizes the “turning point” mentioned in line 2?
A. The beginning of the conflict between proponents of the ideal of historical explanation
and the proponents of experimentation
B. The substitution of historical explanation for causal explanation
C. The shift from interest in historical explanation to interest in experimentation
D. The attention suddenly paid to problems of organic function
E. The growth of public awareness of the controversies among biologists

RC49461.01-40

176. The author implies that nineteenth-century biologists who studied embryogenesis believed
that they
A. had discovered physical evidence that supported their use of historical explanation
B. were the first biologists to call for systematic experimentation on living organisms
C. were able to use historical explanation more systematically than were biologists who did
not study embryogenesis
D. had inadvertently discovered an important part of the factual record of the history of
living organisms on earth
E. had avoided the logical fallacies that characterize the reasoning of most nineteenth-
century biologists

RC49461.01-50

177. The passage would be most likely to appear in which of the following?
A. An essay investigating the methodology used by historians of human events
B. A book outlining the history of biology in the nineteenth century
C. A seminar paper on the development of embryogenesis as a field of study in nineteenth-
century biology
D. A review of a book whose topic is the discovery of fossils in the nineteenth century
E. A lecture whose subject is the limitations of experimental investigation in modern biology

RC59461.01

Line Critics maintain that the fiction of Herman Melville


(1819–1891) has limitations, such as its lack
of inventive plots after Moby-Dick (1851) and its
occasionally inscrutable style. A more serious, yet
(5) problematic, charge is that Melville is a deficient
writer because he is not a practitioner of the “art of
fiction,” as critics have conceived of this art since the
late nineteenth-century essays and novels of Henry
James. Indeed, most twentieth-century commentators
(10) regard Melville not as a novelist but as a writer of
romance, since they believe that Melville's fiction
lacks the continuity that James viewed as essential
to a novel: the continuity between what characters
feel or think and what they do, and the continuity
(15) between characters' fates and their pasts or original
social classes. Critics argue that only Pierre (1852),
because of its subject and its characters, is close to
being a novel in the Jamesian sense.
However, although Melville is not a Jamesian
(20) novelist, he is not therefore a deficient writer. A more
reasonable position is that Melville is a different
kind of writer, who held, and should be judged
by, presuppositions about fiction that are quite
different from James's. It is true that Melville wrote
(25) “romances”; however, these are not the escapist
fictions this word often implies, but fictions that
range freely among very unusual or intense human
experiences. Melville portrayed such experiences
because he believed these best enabled him to
(30) explore moral questions, an exploration he assumed
was the ultimate purpose of fiction. He was content
to sacrifice continuity or even credibility as long
as he could establish a significant moral situation.
Thus Melville's romances do not give the reader
(35) a full understanding of the complete feelings and
thoughts that motivate actions and events that shape
fate. Rather, the romances leave unexplained the
sequence of events and either simplify or obscure
motives. Again, such simplifications and obscurities
(40) exist in order to give prominence to the depiction of
sharply delineated moral values, values derived from
a character's purely personal sense of honor, rather
than, as in a Jamesian novel, from the conventions of
society.

Questions 178–185 refer to the passage.

RC59461.01-10

178. The primary purpose of the passage is to


A. make a case for the importance of skillful psychological motivation in well-written novels
and romances
B. contrast the romantic and novelistic traditions and assert the aesthetic superiority of the
romantic tradition
C. survey some of the responses to Melville's fiction put forward by James and twentieth-
century literary critics
D. argue that the charges made against Melville's fiction by literary critics are suspect and
misleading
E. note several accusations made against Melville's fiction by literary critics and refute one
of these accusations
RC59461.01-20

179. The author draws which of the following conclusions about the fact that Melville's fiction
often does not possess the qualities of a Jamesian novel?
A. Literary critics should no longer use Jamesian standards to judge the value of novels.
B. Literary critics who have praised Melville's fiction at the expense of James's fiction should
consider themselves justified.
C. Literary critics should no longer attempt to place writers, including Melville and James,
in traditions or categories.
D. Melville and James should be viewed as different sorts of writers and one should not be
regarded as inherently superior to the other.
E. Melville and James nevertheless share important similarities and these should not be
overlooked or slighted when literary critics point out differences between the two writers.
RC59461.01-30

180. Which of the following would be the most appropriate title for the passage?
A. Melville's Unique Contribution to Romantic Fiction
B. Melville's Growing Reputation Among Twentieth-Century Literary Critics
C. Melville and the Jamesian Standards of Fiction: A Reexamination
D. Romantic and Novelistic: The Shared Assumptions of Two Traditions
E. The Art of Fiction: James's Influence on the Novelistic Tradition

RC59461.01-40

181. The author probably mentions Melville's Pierre to


A. refute those literary critics who have made generalizations about the quality of Melville's
fiction
B. argue that the portrayal of characters is one of Melville's more accomplished literary skills
C. give an example of a novel that was thought by James to resemble his own fiction
D. suggest that literary critics find few exceptions to what they believe is a characteristic of
Melville's fiction
E. reinforce the contention of literary critics
RC59461.01-50

182. Which of the following statements best describes the author's method of argumentation in
lines 24–31?
A. The author describes an important standard of evaluation used by critics of Melville and
then attacks that standard.
B. The author admits a contention put forward by critics of Melville but then makes a
countercharge against those critics.
C. The author describes a charge advanced by critics of Melville and then points out a logical
flaw in this charge.
D. The author provides evidence that seems to support a position held by critics of Melville
but then demonstrates that the evidence actually supports a diametrically opposed
position.
E. The author concedes an assertion made by critics of Melville but then mitigates the
weight of the assertion by means of an explanation.
RC59461.01-60
183. Which of the following can logically be inferred from the passage about the author's
application of the term “romance” to Melville's work?
A. The author uses the term in a broader way than did Melville himself.
B. The author uses the term in a different way than do many literary critics.
C. The author uses the term in a more systematic way than did James.
D. The author's use of the term is the same as the term's usual meaning for twentieth-
century commentators.
E. The author's use of the term is less controversial than is the use of the term “novel” by
many commentators.
RC59461.01-70

184. Which of the following can most logically be inferred about the author's estimation of the
romantic and novelistic traditions of fiction?
A. The romantic tradition should be considered at least as valuable as the novelistic tradition
in the examination of human experience.
B. The romantic tradition should be considered the more vital tradition primarily because
Melville is part of that tradition.
C. The romantic tradition should be considered the superior tradition because it is so
widespread.
D. The romantic tradition has had as much success in pleasing literary critics as has the
novelistic tradition.
E. The romantic and novelistic traditions have always made important contributions to
literature, but their most important contributions have been in the twentieth century.
RC59461.01-80

185. The author of the passage would be most likely to agree that a writer's fiction should be
evaluated by which of the following criteria?
A. How consistently that fiction establishes credibility with the reader
B. How skillfully that fiction supersedes the presuppositions or conventions of a tradition
C. How completely that fiction satisfies the standards of judgment held by most literary
critics
D. How well that fiction fulfills the premises about fiction maintained by the writer of the
fiction
E. How well that fiction exhibits a continuity of subject and style over the course of the
writer's career
RC69461.01

Line Behind every book review there are two key figures:
a book review editor and a reviewer. Editors decide
whether a book is reviewed in their publication, when
the review appears, how long it is, and who writes the
(5) review.
When many periodicals feature the same books,
this does not prove that the editors of different
periodicals have not made individual decisions.
Before publication, editors receive news releases and
(10) printer's proofs of certain books, signifying that the
publishers will make special efforts to promote these
books. They will be heavily advertised and probably
be among the books that most bookstores order in
quantity. Not having such books reviewed might give
(15) the impression that the editor was caught napping,
whereas too many reviews of books that readers will
have trouble finding in stores would be inappropriate.
Editors can risk having a few of the less popular
titles reviewed, but they must consider what will be
(20) newsworthy, advertised, and written about elsewhere.
If these were the only factors influencing editors,
few books that stand little chance of selling well would
ever be reviewed. But editors feel some concern
about what might endure, and therefore listen to
(25) literary experts. A generation ago, a newspaper used
a brilliant system of choosing which books to feature.
The book review editor sent out a greater number of
books than reviews he actually intended to publish.
If a review was unenthusiastic, he reasoned that the
(30) book was not important enough to be discussed
immediately, and if good reviews of enough other
books came in, the unenthusiastic review might never
be printed. The unenthusiastic reviewers were paid
promptly anyway, but they learned that if they wanted
(35) their material to be printed, it was advisable to be
kind.
Most editors print favorable and unfavorable
reviews; however, the content of the review may be
influenced by the editor. Some editors would actually
(40) feel that they had failed in their responsibility if they
gave books by authors they admired to hostile critics
or books by authors they disapproved of to critics
who might favor them. Editors usually can predict who
would review a book enthusiastically and who would
(45) tear it to shreds.

Questions 186–193 refer to the passage


RC69461.01-10

186. According to the passage, book review editors pay attention to all of the following in deciding
which books should be reviewed in their publications EXCEPT
A. news releases from publishers
B. sales figures compiled by bookstores
C. the opinions of literary experts
D. the probability that the books will be extensively advertised
E. the likelihood that the books will be reviewed in other publications

RC69461.01-20

187. The main idea of the second paragraph is that


A. decisions made by book review editors are influenced by the business of selling books
B. book review editors must be familiar with all aspects of the book trade
C. advertising is the most important factor influencing book sales
D. book reviews usually have no influence on what books are ordered in quantity by stores
E. publishers deliberately try to influence the decisions of book review editors
RC69461.01-30

188. According to the passage, a major concern of the unenthusiastic book reviewers mentioned
in line 33 was to
A. ensure prompt payment for their work
B. influence public opinion of books
C. confirm the opinions of other reviewers
D. promote new books by their favorite authors
E. have their reviews published in the newspaper
RC69461.01-40

189. The passage provides information to answer which of the following questions?
A. Would most book review editors prefer to have books reviewed without regard to the
probable commercial success of the books?
B. Are publishers' efforts to persuade bookstores to order certain books generally effective?
C. On what basis do literary experts decide that a book is or is not important?
D. What criteria are used to determine the length of a particular book review?
E. Have book review practices in general changed significantly since a generation ago?
RC69461.01-50

190. The passage suggests which of the following about book review readers?
A. They pay careful attention to reviewers' biases as they read reviews.
B. They disapprove of book review editors who try to influence what their reviewers write.
C. They use book reviews in order to gauge whether a book is likely to endure.
D. They expect to see timely reviews of widely publicized books in the periodicals they read.
E. They are usually willing to search in several stores for a highly recommended book that is
hard to find.
RC69461.01-60
191. Which of the following words, if substituted for “brilliant” in line 26, would LEAST change
the meaning of the sentence?
A. showy
B. articulate
C. literate
D. stingy
E. absurd

RC69461.01-70

192. Which of the following is an assumption made by the book review editor referred to in line
27?
A. A book of great worth will receive only good reviews.
B. An important book will endure despite possible bad reviews.
C. Reviewers might hide their true opinions in order to have their reviews published.
D. Book review editors should select reviewers whose opinions can be guessed in advance.
E. Book review editors have an obligation to print extensive reviews of apparently important
books.

RC69461.01-80

193. It can be inferred that, as a prospective book buyer, the author of the passage would generally
respond to highly enthusiastic reviews of new books with
A. resignation
B. amusement
C. condemnation
D. skepticism
E. disinterest
RC79461.01

Line There are two theories that have been used to


explain ancient and modern tragedy. Neither quite
explains the complexity of the tragic process or the
tragic hero, but each explains important elements
(5) of tragedy, and, because their conclusions are
contradictory, they represent extreme views. The first
theory states that all tragedy exhibits the workings
of external fate. Of course, the overwhelming
majority of tragedies do leave us with a sense of the
(10) supremacy of impersonal power and of the limitation
of human effort. But this theory of tragedy is an
oversimplification, primarily because it confuses
the tragic condition with the tragic process: the
theory does not acknowledge that fate, in a tragedy,
(15) normally becomes external to the hero only after
the tragic process has been set in motion. Fate, as
conceived in ancient Greek tragedy, is the internal
balancing condition of life. It appears as external
only after it has been violated, just as justice is an
(20) internal quality of an honest person, but the external
antagonist of the criminal. Secondarily, this theory
of tragedy does not distinguish tragedy from irony.
Irony does not need an exceptional central figure:
as a rule, the more ignoble the hero the sharper the
(25) irony, when irony alone is the objective. It is heroism
that creates the splendor and exhilaration that is
unique to tragedy. The tragic hero normally has an
extraordinary, often a nearly divine, destiny almost
within grasp, and the glory of that original destiny
(30) never quite fades out of the tragedy.
The second theory of tragedy states that the
act that sets the tragic process in motion must be
primarily a violation of moral law, whether human
or divine; in short, that the tragic hero must have a
(35) flaw that has an essential connection with sin. Again
it is true that the great majority of tragic heroes do
possess hubris, or a proud and passionate mind
that seems to make the hero's downfall morally
explicable. But such hubris is only the precipitating
(40) agent of catastrophe, just as in comedy the cause
of the happy ending is usually some act of humility,
often performed by a noble character who is meanly
disguised.

Questions 194–200 refer to the passage

RC79461.01-10

194. The primary purpose of the passage is to


A. compare and criticize two theories of tragedy
B. develop a new theory of tragedy
C. summarize the thematic content of tragedy
D. reject one theory of tragedy and offer another theory in its place
E. distinguish between tragedy and irony

RC79461.01-20

195. The author states that the theories discussed in the passage “represent extreme views” (see
line 6) because their conclusions are
A. unpopular
B. complex
C. paradoxical
D. contradictory
E. imaginative
RC79461.01-30

196. The author objects to the theory that all tragedy exhibits the workings of external fate
primarily because
A. fate in tragedies is not as important a cause of action as is the violation of a moral law
B. fate in tragedies does not appear to be something that is external to the tragic hero until
after the tragic process has begun
C. the theory is based solely on an understanding of ancient Greek tragedy
D. the theory does not seem to be a plausible explanation of tragedy's ability to exhilarate an
audience
E. the theory does not seem applicable to the large number of tragedies in which the hero
overcomes fate
RC79461.01-40

197. Which of the following comparisons of the tragic with the ironic hero is best supported by
information contained in the passage?
A. A tragic hero's fate is an external condition, but an ironic hero's fate is an internal one.
B. A tragic hero must be controlled by fate, but an ironic hero cannot be.
C. A tragic hero's moral flaw surprises the audience, but an ironic hero's sin does not.
D. A tragic hero and an ironic hero cannot both be virtuous figures in the same tragedy.
E. A tragic hero is usually extraordinary, but an ironic hero may be cowardly or even
villainous.

RC79461.01-50

198. The author contrasts an honest person and a criminal (see lines 19–21) primarily to
A. prove that fate cannot be external to the tragic hero
B. establish a criterion that allows a distinction to be made between irony and tragedy
C. develop the distinction between the tragic condition and the tragic process
D. introduce the concept of sin as the cause of tragic action
E. argue that the theme of omnipotent external fate is shared by comedy and tragedy
RC79461.01-60

199. The author suggests that the tragic hero's “original destiny never quite fades out of the
tragedy” (see lines 29–30) primarily to
A. qualify the assertion that the theory of tragedy as a display of external fate is inconsistent
B. introduce the discussion of the theory that tragedy is the consequence of individual sin
C. refute the theory that the tragic process is more important than the tragic condition
D. support the claim that heroism creates the splendor and exhilaration of tragedy
E. distinguish between fate as conceived in ancient Greek tragedy and fate in more recent
tragedy
RC79461.01-70

200. In the author's opinion, an act of humility in comedy is most analogous to


A. a catastrophe in tragedy
B. an ironic action in tragedy
C. a tragic hero's pride and passion
D. a tragic hero's aversion to sin
E. a tragic hero's pursuit of an unusual destiny

Critical Reasoning
Analysis/Critique
CR31410.01

201. Most of Western music since the Renaissance has been based on a seven-note scale known as
the diatonic scale, but when did the scale originate? A fragment of a bone flute excavated at a
Neanderthal campsite has four holes, which are spaced in exactly the right way for playing
the third through sixth notes of a diatonic scale. The entire flute must surely have had
more holes, and the flute was made from a bone that was long enough for these additional
holes to have allowed a complete diatonic scale to be played. Therefore, the Neanderthals
who made the flute probably used a diatonic musical scale.
In the argument given, the two portions in boldface play which of the following roles?
A. The first is presented as evidence that is confirmed by data presented elsewhere in the
argument given; the second states a hypothesis that this evidence is used to undermine.
B. The first is an opinion, for which no supporting evidence is presented in the argument
given, that is used to support the main conclusion of the argument; the second is that
main conclusion.
C. The first describes a discovery as undermining the position against which the argument is
directed; the second states the main conclusion of the argument.
D. The first is a preliminary conclusion drawn on the basis of evidence presented elsewhere
in the argument given; the second is the main conclusion that this preliminary conclusion
supports.
E. The first provides evidence to support the main conclusion of the argument; the second
states a subsidiary conclusion that is drawn in order to support the main conclusion
stated earlier in the argument.

CR53140.01

202. In a certain rural area, people normally dispose of household garbage by burning it. Burning
household garbage releases toxic chemicals known as dioxins. New conservation regulations
will require a major reduction in packaging—specifically, paper and cardboard packaging—
for products sold in the area. Since such packaging materials contain dioxins, one result of
the implementation of the new regulations will surely be a reduction in dioxin pollution in
the area.
Which of the following, if true, most seriously weakens the argument?
A. Garbage containing large quantities of paper and cardboard can easily burn hot enough
for some portion of the dioxins that it contains to be destroyed.
B. Packaging materials typically make up only a small proportion of the weight of household
garbage, but a relatively large proportion of its volume.
C. Per-capita sales of products sold in paper and cardboard packaging are lower in rural
areas than in urban areas.
D. The new conservation regulations were motivated by a need to cut down on the
consumption of paper products in order to bring the harvesting of timber into a healthier
balance with its regrowth.
E. It is not known whether the dioxins released by the burning of household garbage have
been the cause of any serious health problems.
CR45650.01

203. Suriland cannot both export wheat and keep bread plentiful and affordable in Suriland.
Accordingly, Suriland's wheat farmers are required to sell their crop to the government,
which pays them a dollar per bushel less than the price on the world market. Therefore, if the
farmers could sell their wheat on the world market, they would make a dollar per bushel
more, less any additional transportation and brokerage costs they would have to pay.
Which of the following, if true, most seriously weakens the argument?
A. Suriland's wheat farmers have higher production costs than do farmers in many other
wheat-producing countries.
B. Sale of a substantial proportion of Suriland's wheat crop on the world market would
probably depress the price of wheat.
C. The transportation and brokerage costs that Suriland's farmers would face if they sold
their wheat outside Suriland could amount to almost a dollar per bushel.
D. Suriland is surrounded by countries that do not import any wheat.
E. The price of a bushel of wheat on the world market occasionally drops below the average
cost of producing a bushel of wheat in Suriland.
CR30370.01

204. Sasha: It must be healthy to follow a diet high in animal proteins and fats. Human beings
undoubtedly evolved to thrive on such a diet, since our prehistoric ancestors ate large
amounts of meat.
Jamal: But our ancestors also exerted themselves intensely in order to obtain this food,
whereas most human beings today are much less physically active.
Jamal responds to Sasha by doing which of the following?
A. Refuting her statement about our prehistoric ancestors
B. Bringing forth a piece of information for the purpose of suggesting that she should qualify
her main conclusion
C. Citing additional evidence that indirectly supports her conclusion and suggests a way to
broaden it
D. Questioning whether her assumption about our prehistoric ancestors permits any
conclusions about human evolution
E. Expressing doubts about whether most human beings today are as healthy as our
prehistoric ancestors were
CR70870.01

205. Some theorists and critics insist that no aesthetic evaluation of a work of art is sound if it is
based even in part on data about the cultural background of the artist. This opinion is clearly
false. The only sound aesthetic evaluations of artists' works are those that take into account
factors such as the era and the place of the artists' births, their upbringing and education,
and the values of their societies—in sum, those factors that are part of their cultural
background.
The above argument is most vulnerable to which of the following objections?
A. The argument presupposes the conclusion for which it purports to provide evidence.
B. The argument cites evidence that undermines rather than supports the conclusion.
C. The argument draws its conclusion by means of an equivocal interpretation of key terms.
D. The argument assumes that the production of an effect is evidence of an intention to
produce that effect.
E. The argument assumes that evaluative disputes can be resolved by citing factual evidence.

CR53870.01

206. Banker: My country's laws require every bank to invest in its local community by lending
money to local businesses, providing mortgages for local home purchases, and so forth. This
is intended to revitalize impoverished local communities. But it is clear that the law will soon
entirely cease to serve its intended purpose. An increasing number of banks incorporated in
our country exist solely on the Internet and are not physically located in any specific
community.
The banker's argument is most vulnerable to criticism on which of the following grounds?
A. It overlooks the possibility that most banks that are physically located in specific
communities in the banker's country are not located in impoverished communities.
B. It takes for granted that a law that ceases to serve its originally intended purpose no
longer serves any other beneficial purpose, either.
C. It confuses a condition that would, if present, be likely to produce a given effect, with a
condition that would probably be the cause if that effect were present.
D. It overlooks the possibility that even if there is a strong correlation between two
phenomena, neither of those phenomena are necessarily causally responsible for the
other.
E. It fails to adequately address the possibility that an increase in the number of banks of
one kind in the banker's country will not lead to the complete elimination of banks of
another kind.
CR15380.01

207. The contingency-fee system, which allows lawyers and their clients to agree that the lawyer
will be paid only in the event of success, does not increase the number of medical malpractice
lawsuits brought against doctors. As attorneys must cover the costs for their time and
research, they want to be assured that any medical malpractice case they accept on a
contingency-fee basis has substantial merit. Consequently, attorneys turn away many people
who come to see them, for lack of a good case.
The argument above is most vulnerable to criticism on the grounds that it fails to
A. specify the criteria attorneys use to judge the merits of a medical malpractice case
B. consider whether, in the absence of a contingency-fee option, even people with
meritorious cases are much less likely to initiate litigation if they believe they might incur
large legal fees
C. note whether, in successful medical malpractice lawsuits, the average monetary award
after legal costs have been deducted is less under contingency-fee arrangements than
otherwise
D. consider the effect of the contingency-fee system on the number of lawsuits sought for
reasons other than medical malpractice
E. acknowledge the rising cost of medical malpractice insurance
CR66590.01

208. Shirla: In figure skating competitions that allow amateur and professional skaters to
compete against each other, the professionals are bound to have an unfair advantage. After
all, most of them became professional only after success on the amateur circuit.
Ron: But that means that it's been a long time since they've had to meet the more rigorous
technical standards of the amateur circuit.
Which of the following is most likely a point at issue between Shirla and Ron?
A. Whether there should be figure skating competitions that allow amateur and professional
skaters to compete against each other
B. Whether the scores of professional skaters competing against amateurs should be subject
to adjustment to reflect the special advantages of professionals
C. Whether figure skaters can successfully become professional before success on the
amateur circuit
D. Whether the technical standards for professional figure skating competition are higher
than those for amateur figure skating competition
E. Whether professional figure skaters have an unfair advantage over amateur figure skaters
in competitions in which they compete against each other

CR03001.01

209. Recent observations suggest that small, earthlike worlds form a very low percentage of the
planets orbiting stars in the galaxy other than the sun. Of over two hundred planets that
astronomers have detected around other stars, almost all are hundreds of times larger and
heavier than the earth and orbit stars much smaller than the sun.
Which of the following, if true, would most weaken the above justification of the claim that
earthlike worlds form a low percentage of the total number of planets?
A. There are millions of planets orbiting stars around which astronomers have not
attempted to detect planets.
B. The best current astronomical theories predict that almost all planets around other stars
are probably hundreds of times larger than the earth.
C. A planet orbiting a star similar to the sun would be more likely to be earthlike in size than
would a planet orbiting a much smaller star.
D. The smaller a planet is relative to the star it orbits, the more difficult it is for astronomers
to detect.
E. The observations would have detected any small, earthlike worlds orbiting the stars
around which larger planets have been detected.
CR61021.01

210. Researchers in City X recently discovered low levels of several pharmaceutical drugs in
public drinking water supplies. However, the researchers argued that the drugs in the water
were not a significant public health hazard. They pointed out that the drug levels were so low
that they could only be detected with the most recent technology, which suggested that the
drugs may have already been present in the drinking water for decades, even though they
have never had any discernible health effects.
Which of the following, if true, would most strengthen the researchers' reasoning?
A. If a drug found in drinking water is not a significant public health hazard, then its
presence in the water will not have any discernible health effects.
B. There is no need to remove low levels of pharmaceutical drugs from public drinking water
unless they present a significant public health hazard.
C. Even if a substance in drinking water is a public health hazard, scientists may not have
discerned which adverse health effects, if any, it has caused.
D. Researchers using older, less sensitive technology detected the same drugs several
decades ago in the public drinking water of a neighboring town but could find no
discernible health effects.
E. Samples of City X's drinking water taken decades ago were tested with today's most
recent technology, and none of the pharmaceutical drugs were found.

CR20521.01

211. Errors in the performance of repetitive or “boring” tasks—often attributed to a momentary


lapse in concentration—can be serious in such activities as flying a passenger aircraft. Is
there any method that would provide warning of such lapses—for example by monitoring
brain activity? Researchers scanned the brains of volunteers performing a repetitive task.
When the tasks were being performed correctly, the volunteers' brains showed activity in
cognitive-processing regions. However, these regions became less active several seconds
before some errors were made, and another brain region, region X, became active. The
researchers concluded that the monitoring of region X could provide warning of an
impending error.
Which of the following, if true, most supports the researchers' conclusion?
A. The cognitive effort required in performing a repetitive task diminishes significantly with
increases in the number of repetitions of the task performance.
B. Once a mistake was made and detected, brain activity in regions associated with cognitive
effort sometimes increased.
C. Other research found that whenever significant activity occurs in region X, it is generally
with repetitive tasks, soon before an error occurs.
D. The diminution of brain activity in cognitive processing regions and the increase of
activity in region X began at least 5 seconds before the errors occurred.
E. Reduced activity in brain regions associated with cognitive effort was accompanied by
increased activity in regions that become active during sleep.

CR46521.01

212. City resident: These new digital electronic billboards should be banned for light pollution
since they are much too bright.
Outdoor advertising spokesperson: No, that's not true. Testing with a sophisticated light
meter shows that at night they throw off less light than traditional billboards that are
reflectively lit. Your mistaken perception that they are brighter comes from looking directly
at the light source—the screen itself.
The underlying strategy of the spokesperson's response to the resident is most analogous to
the underlying strategy of which of the following?
A. A doctor dismisses a patient's claim to have had a heart attack, citing a cardiac enzyme
blood test.
B. A politician rejects an accusation of perjury by denying the credibility of witness
testimony.
C. An insurance agent rejects a claim, on the grounds that there is insufficient evidence to
support the claimant's testimony.
D. An investigator casts doubt on the results of a lie detector, citing the subject's report of
illness during the test.
E. A psychologist treats a mental illness by encouraging a patient to abandon inconsistent
beliefs.
CR02531.01

213. A fossil recently discovered in Marlandia, a chain of islands, proves that a present-day reptile
indigenous to Marlandia is descended from an ancient reptile species that lived on the
islands millions of years ago. The finding is surprising since the ancestral species was
thought to have become extinct when Marlandia was submerged in a global sea-level rise
twenty-five million years ago. Based on the new discovery, many scientists have concluded
that the sea-level rise in question left at least part of Marlandia unsubmerged.
Which of the following would, if true, provide the most additional support for the scientists'
conclusion?
A. Reptiles in Marlandia have adapted to many environmental changes since the sea-level
rise.
B. Marlandia separated from a much larger landmass about eighty million years ago.
C. No fossils that prove the relationship between the present-day species and the ancestral
species have been found anywhere other than Marlandia.
D. The present-day reptiles are able to thrive on very tiny Marlandia islands.
E. The ancestral reptiles could not have survived long at sea.

CR21041.01

214. Advertisement: Our competitors' computer salespeople are paid according to the value of the
products they sell, so they have a financial incentive to convince you to buy the most
expensive units—whether you need them or not. But here at Comput-o-Mart, our salespeople
are paid a salary that is not dependent on the value of their sales, so they won't try to tell you
what to buy. That means when you buy a computer at Comput-o-Mart, you can be sure you're
not paying for computing capabilities you don't need.
Which of the following would, if true, most weaken the advertisement's reasoning?
A. Some less-expensive computers actually have greater computing power than more
expensive ones.
B. Salespeople who have a financial incentive to make sales generally provide more attentive
service than do other salespeople.
C. Extended warranties purchased for less-expensive computers can cost nearly as much as
the purchase price of the computer.
D. Comput-o-Mart is open only limited hours, which makes it more difficult for many
shoppers to buy computers there than at other retail stores.
E. Comput-o-Mart does not sell any computers that support only basic computing.
CR36441.01

215. Consumer advocate: In our nation, food packages must list the number of calories per food
serving. But most of the serving sizes used are misleadingly small and should be updated.
The serving sizes were set decades ago, when our nation's people typically ate smaller
portions than they do today, and, as a result, people eating typical portions today consume
more calories than the package labeling appears to indicate that they will. It is time package
labeling reflected these changes.
Which of the following is the main point of the consumer advocate's argument?
A. The number of calories per serving listed on most food packages in the consumer
advocate's nation is misleadingly small.
B. Most serving sizes used on food packages in the consumer advocate's nation should be
increased to reflect today's typical portion sizes.
C. People eating typical portions today often consume far more calories than the number of
calories per serving listed on food packages in the consumer advocate's nation.
D. The serving sizes used on food packages in the consumer advocate's nation were set when
people ate smaller portions on average than they do today.
E. The use of misleadingly small serving sizes on food packages in the consumer advocate's
nation probably leads many people to consume more calories than they otherwise would.

CR05941.01
216. Columnist: Metro City has a lower percentage of residents with humanities degrees than any
other city of comparable size in our nation. Nationwide, university graduates generally earn
more than people who are not university graduates, but those with humanities degrees
typically earn less than do graduates with degrees in other disciplines. So the main reason
Metro City has higher income per capita than any other city of comparable size in our nation
must be its low percentage of residents with humanities degrees.
Which of the following, if true, would most strengthen the columnist's argument?
A. Metro City residents with humanities degrees have higher income per capita than do
people with humanities degrees in any other city of comparable size in the nation.
B. The percentage of residents with university degrees is lower in Metro City than in any
other city of comparable size in the nation.
C. Nationwide, university graduates without humanities degrees typically earn more than do
individuals without university degrees.
D. Metro City residents with degrees outside the humanities have per capita income no
higher than the per capita income of such residents of other cities of comparable size in
the nation.
E. In Metro City, a lower proportion of university graduates have humanities degrees than in
any other city of comparable size in the nation.
CR87051.01

217. Psychologist: In a survey, several hundred volunteers rated their own levels of self-control
and their overall life satisfaction. The volunteers who rated themselves as having better self-
control also reported greater satisfaction with their lives. This suggests that self-control is
one factor that helps people avoid situations likely to produce dissatisfaction.
In order to assess the strength of the psychologist's argument, it would be most helpful to
know whether
A. people typically rate themselves as having significantly better self-control than expert
psychological assessments would rate them as having
B. people's perceptions of how satisfied they are with their lives could be affected by factors
of which they are unaware
C. there is a high level of self-control that tends to reduce overall life satisfaction
D. people's ratings of their overall satisfaction with their lives tend to temporarily decrease
in situations likely to produce dissatisfaction
E. feelings of dissatisfaction significantly interfere with people's ability to exercise self-
control
CR03161.01

218. Mansour: We should both plan to change some of our investments from coal companies to
less polluting energy companies. And here's why. Consumers are increasingly demanding
nonpolluting energy, and energy companies are increasingly supplying it.
Therese: I'm not sure we should do what you suggest. As demand for nonpolluting energy
increases relative to supply, its price will increase, and then the more polluting energy will
cost relatively less. Demand for the cheaper, dirtier energy forms will then increase, as will
the stock values of the companies that produce them.
Therese responds to Mansour's proposal by doing which of the following?
A. Advocating that consumers use less expensive forms of energy
B. Implying that not all uses of coal for energy are necessarily polluting
C. Disagreeing with Mansour's claim that consumers are increasingly demanding
nonpolluting energy
D. Suggesting that leaving their existing energy investments unchanged could be the better
course
E. Providing a reason to doubt Mansour's assumption that supply of nonpolluting energy
will increase in line with demand

CR04161.01

219. Scientist: In an experiment, dogs had access to a handle they could pull to release food into a
nearby enclosure that contained a familiar dog and nothing else, contained an unfamiliar dog
and nothing else, or was empty. The dogs typically released more food to the familiar dog
than to the unfamiliar dog. This suggests that dogs are more motivated to help other dogs
they know than to help unfamiliar dogs.
The scientist's argument would be most strengthened if it were true that, in the experiment,
the dogs with access to the handle tended to release more food when
A. the behavior was being encouraged by a familiar person than when it was being
encouraged by an unfamiliar person
B. the enclosure was empty than when it contained an unfamiliar dog
C. an unfamiliar dog in the enclosure was displaying hostility toward them than when an
unfamiliar dog in the enclosure appeared friendly
D. a dog in the enclosure appeared uninterested in food already released into the enclosure
than when it appeared interested in that food
E. a familiar dog was in the enclosure than when a familiar dog was visible but the enclosure
was empty

CR09461.01

220. Most geologists believe oil results from chemical transformations of hydrocarbons derived
from organisms buried under ancient seas. Suppose, instead, that oil actually results from
bacterial action on other complex hydrocarbons that are trapped within the earth. As is well
known, the volume of these hydrocarbons exceeds that of buried organisms. Therefore, our
oil reserves would be greater than most geologists believe.
Which of the following, if true, gives the strongest support to the argument above about our
oil reserves?
A. Most geologists think optimistically about the earth's reserves of oil.
B. Most geologists have performed accurate chemical analyses on previously discovered oil
reserves.
C. Ancient seas are buried within the earth at many places where fossils are abundant.
D. The only bacteria yet found in oil reserves could have leaked down drill holes from
surface contaminants.
E. Chemical transformations reduce the volume of buried hydrocarbons derived from
organisms by roughly the same proportion as bacterial action reduces the volume of other
complex hydrocarbons.
CR66561.01

221. Meteorologists say that if only they could design an accurate mathematical model of the
atmosphere with all its complexities, they could forecast the weather with real precision. But
this is an idle boast, immune to any evaluation, for any inadequate weather forecast would
obviously be blamed on imperfections in the model.
Which of the following, if true, could best be used as a basis for arguing against the author's
position that the meteorologists' claim cannot be evaluated?
A. Certain unusual configurations of data can serve as the basis for precise weather
forecasts, even though the exact causal mechanisms are not understood.
B. Most significant gains in the accuracy of the relevant mathematical models are
accompanied by clear gains in the precision of weather forecasts.
C. Mathematical models of the meteorological aftermath of such catastrophic events as
volcanic eruptions are beginning to be constructed.
D. Modern weather forecasts for as much as a full day ahead are broadly correct about 80
percent of the time.
E. Meteorologists readily concede that the accurate mathematical model they are talking
about is not now in their power to construct.

CR38561.01

222. The lobbyists argued that because there is no statistical evidence that breathing other
people's tobacco smoke increases the incidence of heart disease or lung cancer in healthy
nonsmokers, legislation banning smoking in workplaces cannot be justified on health
grounds.
Of the following, which is the best criticism of the argument reported above?
A. It ignores causes of lung cancer other than smoking.
B. It neglects the damaging effects of smoke-filled air on nonsmokers who are not healthy.
C. It fails to mention the roles played by diet, exercise, and heredity in the development of
heart disease.
D. It does not consider the possibility that nonsmokers who breathe smoke-filled air at work
may become more concerned about their health.
E. It does not acknowledge that nonsmokers, even those who breathe smoke-filled air at
work, are in general healthier than smokers.

CR78561.01

223. Since 1978 when the copyright law was changed, books that are less than fifty years old must
not be photocopied without the publisher's permission. Thus, any book that has been
photocopied since 1978 without the publisher's permission must be at least fifty years old.
The reasoning above exhibits a flaw similar to one in which of the following?
A. Any member of the solar system must be either a planet or a moon, so if an asteroid is
neither a planet nor a moon, it must not be a member of the solar system.
B. Anyone who rides a city bus must buy a bus pass, and since Demetrios has a bus pass, he
must be riding on a city bus.
C. A driver who turns right must signal, so any driver who did not signal must not have
turned right.
D. Anyone who legally crosses a national boundary must have a passport; thus anyone who
does not have a passport cannot legally cross a national boundary.
E. Any wage earner residing in the state must pay state taxes, so since Blodwen pays state
taxes, she must be resident in the state.

CR10661.01

224. In the United States, injuries to passengers involved in automobile accidents are typically
more severe than in Europe, where laws require a different kind of safety belt. It is clear from
this that the United States needs to adopt more stringent standards for safety belt design to
protect automobile passengers better.
Each of the following, if true, weakens the argument above EXCEPT:
A. Europeans are more likely to wear safety belts than are people in the United States.
B. Unlike United States drivers, European drivers receive training in how best to react in the
event of an accident to minimize injuries to themselves and to their passengers.
C. Cars built for the European market tend to have more sturdy construction than do cars
built for the United States market.
D. Automobile passengers in the United States have a greater statistical chance of being
involved in an accident than do passengers in Europe.
E. States that have recently begun requiring the European safety belt have experienced no
reduction in the average severity of injuries suffered by passengers in automobile
accidents.
CR60661.01

225. A country's Aeronautics Board (AB) employs inspectors who make routine annual
inspections of all aircraft. On inspecting Azura Airlines' airplanes in December, they reported
considerably more violations of AB rules this year, compared to a year ago. This fact explains
why Azura had more accidents this year, compared to last year.
Which of the following, if true, would cast most doubt on the conclusion in the passage?
A. Some aviation experts in other countries consider certain AB rules to be too lax and too
easy to get around.
B. Azura's routes are no more dangerous than are those of most other airlines.
C. The AB increased the length and rigor of its inspections this year, compared to last year.
D. Prior to last year Azura had an excellent safety record with very few accidents.
E. In both years the AB report on Azura did not include violations on airplanes owned by
Azura but leased by another airline.
CR13661.01

226. The more frequently employees take time to exercise during working hours each week, the
fewer sick days they take. Even employees who exercise only once a week during working
hours take less sick time than those who do not exercise. Therefore, if companies started
fitness programs, the absentee rate in those companies would decrease significantly.
Which of the following, if true, most seriously weakens the argument above?
A. Employees who exercise during working hours occasionally fall asleep for short periods of
time after they exercise.
B. Employees who are frequently absent are the least likely to cooperate with or to join a
corporate fitness program.
C. Employees who exercise only once a week in their company's fitness program usually also
exercise after work.
D. Employees who exercise in their company's fitness program use their working time no
more productively than those who do not exercise.
E. Employees who exercise during working hours take slightly longer lunch breaks than
employees who do not exercise.

Construction/Plan
CR67370.01

227. Distressed by his own personal tragedies, the Roman philosopher Cicero once asked himself
whether a wise person should try to achieve the Stoic ideal of complete emotionlessness.
Cicero reasoned that, however desirable the goal may be, a wise person could never attain it,
since emotions are not simply irrational urges. They are, rather, a product of one's estimate
of the goodness and badness of the events, people, and actions one witnesses.
Which of the following is an assumption required by Cicero's reasoning?
A. Wise people inevitably evaluate at least some of the things they observe.
B. Irrationality makes evaluation of what one observes impossible.
C. Wisdom precludes attempting to attain what one cannot.
D. If evaluations are based only on reason, then they are inaccurate.
E. A wise person will not evaluate what cannot be directly observed.

CR49770.01

228. First discovered several years ago in North American lakes and rivers, the northern
snakehead is a nonnative fish with no local predators. To keep the northern snakehead's
population from growing, for the past three years wildlife officials have been paying
recreational fishers for each northern snakehead they catch. In this way, the officials hope to
stop the northern snakeheads from eliminating rare native fish species.
To evaluate the likelihood that the wildlife officials' plan will succeed, it would be most useful
to determine which of the following?
A. Whether the northern snakehead's population in local lakes and rivers could be reduced
by introducing predators from its native habitat
B. How local population numbers of rare native fish species have been changing since the
wildlife officials started paying recreational fishers to catch northern snakeheads
C. Whether the fish species on which the northern snakehead preys in regions to which it is
native and in which it is abundant have become significantly depleted in recent decades
D. What total number of northern snakehead have been caught by recreational fishers since
the wildlife officials began paying for them
E. Whether rare native fish species in the region face any threats to their survival other than
the proliferation of northern snakehead

CR51080.01

229. Scientist: A greenhouse gas, for example, carbon dioxide, forms a transparent layer that
traps solar heat beneath it in the earth's atmosphere. Atmospheric levels of carbon dioxide
are currently increasing, causing the climate to warm—an effect that is predicted by at least
one computer model of the greenhouse effect. But the warming that has occurred is a great
deal less than what would be expected based on the model. Therefore, _______.
Which of the following most logically completes the scientist's argument?
A. better measurements of atmospheric levels of carbon dioxide are needed
B. the definition of “greenhouse gas” should probably be reconsidered
C. there are factors besides the increase in greenhouse-gas emissions contributing to the
warming of the climate
D. the computer model of the greenhouse effect must be incorrect in some respect
E. the likely consequences of any warming of the climate are unlikely to be much less
damaging than predicted

CR09090.01

230. Beets and carrots are higher in sugar than many other vegetables. They are also high on the
glycemic index, a scale that measures the rate at which a food increases blood sugar levels.
But while nutritionists usually advise people to avoid high-sugar and high-glycemic-index
foods, despite any nutritional benefits they may confer, they are not very concerned about
the consumption of beets and carrots.
Which of the following, if true, would best explain the nutritionists' lack of concern?
A. Foods with added sugar are much higher in sugar, and have a larger effect on blood sugar
levels, than do beets and carrots.
B. Most consumption of beets and carrots occurs in combination with higher-protein foods,
which reduce blood sugar fluctuations.
C. Beets and carrots contain many nutrients, such as folate, beta-carotene, and vitamin C, of
which many people fail to consume optimal quantities.
D. The glycemic index measures the extent to which a food increases blood sugar levels as
compared to white bread, a food that is much less healthy than beets and carrots.
E. Nutritionists have only recently come to understand that a food's effect on blood sugar
levels is an important determinant of that food's impact on a person's health.
CR36601.01
231. Ozone in the stratosphere blocks deadly ultraviolet rays from the sun, but
chlorofluorocarbons (CFCs) in aerosols and other products have thinned this protective
layer. Evidence of this is the ozone hole that forms over the South Pole every Antarctic spring
as temperatures drop below –78°C, the temperature at which ozone depletion occurs.
Measurements of the ozone hole taken at various times this spring show that, compared with
the same times the previous year, its area diminished by four million square kilometers.
Nevertheless, scientists have not concluded that the ozone layer is recovering.
Which of the following would, if true, provide the strongest reason for the scientists' reaction
to the measurements?
A. The ozone hole has steadily grown in size every year for the past decade except this year.
B. The length of time that the ozone hole persists fluctuates from year to year.
C. As a result of international treaties, CFCs have been completely banned for several years.
D. Weather patterns allowed unusual amounts of warm air to mix into the polar regions this
year.
E. Human-made CFCs retain their ability to destroy ozone molecules for seventy-five to one
hundred years.

CR29111.01

232. The recycling of municipal solid waste is widely seen as an environmentally preferable
alternative to the prevailing practices of incineration and of dumping in landfills. Recycling is
profitable, as the recycling programs already in operation demonstrate. A state legislator
proposes that communities should therefore be required to adopt recycling and to reach the
target of recycling 50 percent of all solid waste within 5 years.
Which of the following, if true, most seriously calls into question the advisability of
implementing the proposal?
A. Existing recycling programs have been voluntary, with citizen participation ranging from
30 percent in some communities to 80 percent in others.
B. Existing recycling programs have been restricted to that 20 percent of solid waste that,
when reprocessed, can match processed raw materials in quality and price.
C. Existing recycling programs have had recurrent difficulties finding purchasers for their
materials, usually because of quantities too small to permit cost-effective pickup and
transportation.
D. Some of the materials that can be recycled are the very materials that, when incinerated,
produce the least pollution.
E. Many of the materials that cannot be recycled are also difficult to incinerate.

CR30721.01

233. Biologist: Species with broad geographic ranges probably tend to endure longer than species
with narrow ranges. The broader a species' range, the more likely that species is to survive
the extinction of populations in a few areas. Therefore, it is likely that the proportion of
species with broad ranges tends to gradually increase with time.
The biologist's conclusion follows logically from the above if which of the following is
assumed?
A. There are now more species with broad geographic ranges than with narrow geographic
ranges.
B. Most species can survive extinctions of populations in a few areas as long as the species'
geographic range is not very narrow.
C. If a population of a species in a particular area dies out, that species generally does not
repopulate that area.
D. If a characteristic tends to help species endure longer, then the proportion of species with
that characteristic tends to gradually increase with time.
E. Any characteristic that makes a species tend to endure longer will make it easier for that
species to survive the extinction of populations in a few areas.

CR79731.01

234. Letter to the editor: If the water level in the Searle River Delta continues to drop, the rising
sea level will make the water saltier and less suitable for drinking. Currently, 40 percent of
the water from upstream tributaries is diverted to neighboring areas. To keep the delta's
water level from dropping any further, we should end all current diversions from the
upstream tributaries. Neighboring water utilities are likely to see higher costs and
diminished water supplies, but these costs are necessary to preserve the delta.
Which of the following would, if true, indicate a serious potential weakness of the suggested
plan of action?
A. Desalination equipment would allow water from the delta to be used for drinking even it
if became saltier.
B. Water level is only one factor that affects salinity in the delta.
C. The upstream tributaries' water levels are controlled by systems of dams and reservoirs.
D. Neighboring areas have grown in population since the water was first diverted from
upstream tributaries.
E. Much of the recent drop in the delta's water level can be attributed to a prolonged
drought that has recently ended.

CR47931.01

235. Researchers conditioned a group of flies to associate a particular odor with a weak electric
shock. Twenty-four and forty-eight hours later the researchers conducted tests on the flies,
both individually and in groups, to determine whether the flies retained the conditioning.
When tested individually, the flies were significantly less likely to avoid areas marked with
the odor. The researchers hypothesized that in the presence of the odor, a fly that retains the
conditioned association gives off an alarm signal that arouses the attention of any
surrounding flies, retriggering the association in them and thereby causing them to avoid the
odor.
The researchers' hypothesis requires which of the following assumptions?
A. The flies do not give off odors as alarm signals.
B. Flies that did not avoid the odor when tested individually were not merely following other
flies' movements when tested in a group.
C. Flies that did not avoid the odor when tested individually were less likely than the other
flies to avoid the odor when tested in a group.
D. Prior to their conditioning, the flies would likely have found the odor used in the
experiment to be pleasant.
E. An electric shock was used during the flies' conditioning and during the later tests.

CR02741.01

236. Kayla: Many people are reluctant to shop in our neighborhood because street parking is
scarce. The city plans to address this by adding parking meters with time limits that ensure
that parking spaces are generally available. But this plan will surely backfire—shoppers
dislike paying at parking meters, so most will probably drive to other neighborhoods to shop
at malls with free parking.
Which of the following, if true, would be the most logically effective rebuttal a proponent of
the city's plan could make to Kayla's objection?
A. Most shoppers dislike hunting for scarce street parking spaces much more than they
dislike paying for metered parking spaces.
B. The city could post signs with street parking time limits to ensure that parking spaces
become available without forcing shoppers to pay at meters.
C. Currently, most shoppers in the neighborhood drive only occasionally to shop at malls in
other neighborhoods.
D. The neighborhood already contains a parking lot where shoppers must pay to park.
E. The nearby malls with free parking have no parking time limits to help ensure that
parking spaces in their lots become available.

CR78551.01

237. A new handheld device purports to determine the severity of concussions by reading the
brain's electrical signals and comparing them to a database of 15,000 scans compiled at a
brain research lab. The device is intended to help doctors decide whether an athlete who has
received a blow to the head during a competition should be sent back into the game.
Which of the following would it be most useful to establish in order to evaluate the
effectiveness of the device for its intended purpose?
A. Whether the database of brain scans will regularly be updated with new scans
B. Whether by use of this device doctors will be able to make a sound decision about
whether to allow an athlete back into the competition before it ends
C. Whether the device will be endorsed by a large number of medical professionals
D. Whether the database includes scans of non-injured athletes in the same game as the
injured athlete
E. Whether team doctors have until now been mistaken in their assessments of whether an
athlete can safely continue to play
CR30461.01

238. Mashika: We already know from polling data that some segments of the electorate provide
significant support to Ms. Puerta. If those segments also provide significant support to Mr.
Quintana, then no segment of the electorate that provides significant support to Mr.
Quintana provides significant support to Mr. Ramirez.
Salim: But actually, as the latest polling data conclusively shows, at least one segment of the
electorate does provide significant support to both Mr. Quintana and Mr. Ramirez.
Among the following statements, which is it most reasonable to infer from the assertions by
Mashika and Salim?
A. At least one segment of the electorate provides significant support neither to Mr.
Quintana nor to Mr. Ramirez.
B. At least one segment of the electorate provides significant support to Ms. Puerta but not
to Mr. Quintana.
C. Each segment of the electorate provides significant support to Ms. Puerta.
D. Each segment of the electorate provides significant support to Mr. Quintana.
E. Each segment of the electorate provides significant support to Mr. Ramirez.
CR98461.01

239. The proportion of manufacturing companies in Alameda that use microelectronics in their
manufacturing processes increased from 6 percent in 1979 to 66 percent in 1990. Many labor
leaders say that the introduction of microelectronics is the principal cause of the great
increase in unemployment during that period in Alameda. In actual fact, however, most of
the job losses were due to organizational changes. Moreover, according to new figures
released by the labor department, there were many more people employed in Alameda in the
manufacturing industry in 1990 than in 1979.
Which of the following, if true, best reconciles the discrepancy between the increase in
unemployment and the increase in jobs in the manufacturing industry of Alameda?
A. Many products that contain microelectronic components are now assembled completely
by machine.
B. Workers involved in the various aspects of the manufacturing processes that use
microelectronic technology need extensive training.
C. It is difficult to evaluate numerically what impact on job security the introduction of
microelectronics in the workplace had before 1979.
D. In 1990 over 90 percent of the jobs in Alameda's manufacturing companies were filled by
workers who moved to Alameda because they had skills for which there was no demand in
Alameda prior to the introduction of microelectronics there.
E. Many workers who have retired from the manufacturing industry in Alameda since 1979
have not been replaced by younger workers.

CR00561.01

240. The retinas of premature babies are not fully developed at birth. Because their retinas are so
delicate, premature babies sometimes lose their sight. Methods for preventing this
syndrome, which is called retinopathy of prematurity, have improved, but the proportion of
premature babies who lose their sight because of this syndrome has increased.
Which of the following, if true, best reconciles the apparent discrepancy described above?
A. When premature babies are born, their retinas are developmentally unprepared to deal
with light and air in the environment outside the womb.
B. The oxygen that must be administered to premature babies at birth can sometimes have a
damaging effect on the babies' retinas, but the oxygen is now administered in less
damaging concentrations than it used to be.
C. The effects of retinopathy of prematurity can be reduced by controlling the exposure of
premature babies to light and oxygen, but this method cannot completely prevent the
syndrome.
D. The improvement of methods to prevent retinopathy of prematurity has been a gradual
process, and there is still a need for further knowledge.
E. Improved medical technology is saving the lives of premature babies who would
previously have died, but these babies have even more delicate retinas than do other
premature babies and are more apt to lose their sight.
CR60561.01

241. Although elementary school children have traditionally received considerable instruction in
creating visual art, there has been no such instruction in music. Consequently, in contrast to
the situation for visual art, most people as adults do not recognize the artistic intentions of
composers. To remedy this situation, a few educators now recommend teaching elementary
school students to compose music.
Which of the following, if true, is the strongest basis for arguing that implementation of the
recommendation will not lead to the desired result?
A. Few elementary school students are likely to create superior compositions.
B. Traditional education facilitates the appreciation of visual art, but not the recognition of
the artistic intentions of artists.
C. More people report that they enjoy music than report that they enjoy visual art.
D. Some composers have had little formal instruction in composition.
E. The recommendation is based on the results of a controlled longitudinal study conducted
in three schools within a single city.

CR47561.01

242. Pharmaceutical companies spend more than ever on research and development; yet the
number of new drugs patented each year has dropped since 1963. At the same time, profits—
at constant 1963 dollars—for the industry as a whole have been steadily increasing.
Which of the following, if true, is the single factor most likely to explain, at least in part, the
three trends mentioned above for money spent, drugs patented, and profits made?
A. Government regulations concerning testing requirements for novel drugs have become
steadily more stringent.
B. Research competition among pharmaceutical companies has steadily intensified as a
result of a general narrowing of research targets to drugs for which there is a large
market.
C. Many pharmaceutical companies have entered into collaborative projects with leading
universities, while others have hired faculty members away from universities by offering
very generous salaries.
D. The number of cases in which one company's researchers duplicated work done by
another company's researchers has steadily grown.
E. The advertising budgets of the major pharmaceutical companies have grown at a higher
rate than their profits have.

CR97561.01

243. Under the agricultural policies of Country R, farmers can sell any grain not sold on the open
market to a grain board at guaranteed prices. It seems inevitable that, in order to curb the
resultant escalating overproduction, the grain board will in just a few years have to impose
quotas on grain production, limiting farmers to a certain flat percentage of the grain acreage
they cultivated previously.
Suppose an individual farmer in Country R wishes to minimize the impact on profits of the
grain quota whose eventual imposition is being predicted. If the farmer could do any of the
following and wants to select the most effective course of action, which should the farmer do
now?
A. Select in advance currently less profitable grain fields and retire them if the quota takes
effect.
B. Seek long-term contracts to sell grain at a fixed price.
C. Replace obsolete tractors with more efficient new ones.
D. Put marginal land under cultivation and grow grain on it.
E. Agree with other farmers on voluntary cutbacks in grain production.

CR69561.01

244. Exports of United States wood pulp will rise considerably during this year. The reason for the
rise is that the falling value of the dollar will make it cheaper for paper manufacturers in
Japan and Western Europe to buy American wood pulp than to get it from any other source.
Which of the following is an assumption made in drawing the conclusion above?
A. Factory output of paper products in Japan and Western Europe will increase sharply
during this year.
B. The quality of the wood pulp produced in the United States would be adequate for the
purposes of Japanese and Western European paper manufacturers.
C. Paper manufacturers in Japan and Western Europe would prefer to use wood pulp
produced in the United States if cost were not a factor.
D. Demand for paper products made in Japan and Western Europe will not increase sharply
during this year.
E. Production of wood pulp by United States companies will not increase sharply during this
year.
CR79561.01

245. A company's personnel director surveyed employees about their satisfaction with the
company's system for awarding employee performance ratings. The survey data indicated
that employees who received high ratings were very satisfied with the system. The personnel
director concluded from these data that the company's best-performing employees liked the
system.
The personnel director's conclusion assumes which of the following?
A. No other performance rating system is as good as the current system.
B. The company's best-performing employees received high ratings.
C. Employees who received low ratings were dissatisfied with the system.
D. Employees who receive high ratings from a performance-rating system will like that
system.
E. The company's best-performing employees were motivated to perform well by the
knowledge that they would receive performance ratings.

CR00661.01

246. There are fundamentally two possible changes in an economy that will each cause inflation
unless other compensating changes also occur. These changes are either reductions in the
supply of goods and services or increases in demand. In a pre-banking economy the quantity
of money available, and hence the level of demand, is equivalent to the quantity of gold
available.
If the statements above are true, then it is also true that in a pre-banking economy
A. any inflation is the result of reductions in the supply of goods and services
B. if other factors in the economy are unchanged, increasing the quantity of gold available
will lead to inflation
C. if there is a reduction in the quantity of gold available, then, other things being equal,
inflation must result
D. the quantity of goods and services purchasable by a given amount of gold is constant
E. whatever changes in demand occur, there will be compensating changes in the supply of
goods and services

CR20661.01

247. Clearbell Telephone provides slow-dialing (SD) service to customers for a low fee, and fast-
dialing (FD) service to other customers who pay a somewhat higher fee. FD technology,
however, is so efficient that it costs Clearbell substantially less per average call to provide
than does SD. Nonetheless, accountants have calculated that Clearbell's profits would drop if
it provided FD to all its customers at the current low-fee rate.
Assume that installation costs for FD are insignificant if the customer already has SD service.
Which of the following, if true about Clearbell, best explains the results of the accountants'
calculation?
A. The extra revenue collected from customers who pay the high fee is higher than the extra
cost of providing SD to customers who pay the low fee.
B. The low fee was increased by 6 percent last year, whereas the higher fee was not increased
last year.
C. Although 96 percent of customers regard FD service as reliable and more convenient than
SD, fewer than 10 percent of them choose to pay the higher fee for FD service.
D. The company's competitors generally provide business customers with FD service at low-
fee rates.
E. Profits rose slightly each month for the first three months after FD was first offered to
customers, then fell slightly each month for the succeeding three months.

CR23661.01

248. Manufacturers sometimes discount the price of a product to retailers for a promotion period
when the product is advertised to consumers. Such promotions often result in a dramatic
increase in amount of product sold by the manufacturers to retailers. Nevertheless, the
manufacturers could often make more profit by not holding the promotions.
Which of the following, if true, most strongly supports the claim above about the
manufacturers' profit?
A. The amount of discount generally offered by manufacturers to retailers is carefully
calculated to represent the minimum needed to draw consumers' attention to the
product.
B. For many consumer products the period of advertising discounted prices to consumers is
about a week, not sufficiently long for consumers to become used to the sale price.
C. For products that are not newly introduced, the purpose of such promotions is to keep the
products in the minds of consumers and to attract consumers who are currently using
competing products.
D. During such a promotion retailers tend to accumulate in their warehouses inventory
bought at discount; they then sell much of it later at their regular price.
E. If a manufacturer fails to offer such promotions but its competitor offers them, that
competitor will tend to attract consumers away from the manufacturer's product.

CR33661.01

249. Advertisement: Today's customers expect high quality. Every advance in the quality of
manufactured products raises customer expectations. The company that is satisfied with the
current quality of its products will soon find that its customers are not. At MegaCorp,
meeting or exceeding customer expectations is our goal.
Which of the following must be true on the basis of the statements in the advertisement
above?
A. MegaCorp's competitors will succeed in attracting customers only if those competitors
adopt MegaCorp's goal as their own.
B. A company that does not correctly anticipate the expectations of its customers is certain
to fail in advancing the quality of its products.
C. MegaCorp's goal is possible to meet only if continuing advances in product quality are
possible.
D. If a company becomes satisfied with the quality of its products, then the quality of its
products is sure to decline.
E. MegaCorp's customers are currently satisfied with the quality of its products.

Sentence Correction
Communication
SC21011.01

250. The prime lending rate is a key rate in the economy: not only are the interest rates on most
loans to small and medium-sized businesses tied to the prime, but also on a growing number
of consumer loans, including home equity loans.
A. not only are the interest rates on most loans to small and medium-sized businesses tied to
the prime, but also on
B. tied to the prime are the interest rates not only on most loans to small and medium-sized
businesses, but also on
C. the interest rates not only on most loans to small and medium-sized businesses are tied to
the prime, but also
D. not only the interest rates on most loans to small and medium-sized businesses are tied to
the prime, but also on
E. the interest rates are tied to the prime, not only on most loans to small and medium-sized
businesses, but also

SC83811.01

251. Lacking information about energy use, people tend to overestimate the amount of energy
used by equipment, such as lights, that are visible and must be turned on and off and
underestimate that used by unobtrusive equipment, such as water heaters.
A. equipment, such as lights, that are visible and must be turned on and off and
underestimate that
B. equipment, such as lights, that are visible and must be turned on and off and
underestimate it when
C. equipment, such as lights, that is visible and must be turned on and off and
underestimate it when
D. visible equipment, such as lights, that must be turned on and off and underestimate that
E. visible equipment, such as lights, that must be turned on and off and underestimate it
when

SC37561.01

252. Evidence of some shifts in the character of violence on television is emerging from a new
study of 500 television programs by the Center for Media and Public Affairs, a nonprofit
research center in Washington, D.C., a study that is underwritten by a number of educational
institutions.
A. programs by the Center for Media and Public Affairs, a nonprofit research center in
Washington, D.C., a study that is underwritten by a number of educational institutions
B. programs by the Center for Media and Public Affairs, a nonprofit research center in
Washington, D.C., and it is underwritten by a number of educational institutions
C. programs underwritten by a number of educational institutions and conducted by the
Center for Media and Public Affairs, a nonprofit research center based in Washington,
D.C.
D. programs, a study underwritten by a number of educational institutions and conducted
by the Center for Media and Public Affairs, a nonprofit research center in Washington,
D.C.
E. programs, a study conducted by the Center for Media and Public Affairs, a nonprofit
research center based in Washington, D.C., and it is underwritten by a number of
educational institutions

SC48461.01

253. Judge Lois Forer's study asks why do some litigants have a preferred status over others in the
use of a public resource, the courts, which in theory are available to all but in fact are
unequally distributed among rich and poor.
A. do some litigants have a preferred status over others in the use of a public resource, the
courts, which in theory are available to all but in fact are unequally distributed among
B. some litigants have a preferred status over others in the use of a public resource, the
courts, which in theory are available to all but in fact are unequally distributed between
C. do some litigants have a preferred status over another in the use of a public resource, the
courts, in theory available to all but in fact are unequally distributed among
D. some litigants have a preferred status to another in the use of a public resource, the
courts, in theory available to all but in fact not equally distributed between
E. does one litigant have a preferred status over the other in the use of a public resource, the
courts, in theory available to all but in fact they are not equally distributed among

SC30561.01

254. During an ice age, the buildup of ice at the poles and the drop in water levels near the
equator speed up the earth's rotation, like a spinning figure skater whose speed increases
when her arms are drawn in.
A. like a spinning figure skater whose speed increases when her arms are drawn in
B. like the increased speed of a figure skater when her arms are drawn in
C. like a figure skater who increases speed while spinning with her arms drawn in
D. just as a spinning figure skater who increases speed by drawing in her arms
E. just as a spinning figure skater increases speed by drawing in her arms
SC01561.01

255. Added to the increase in hourly wages requested last July, the railroad employees are now
seeking an expanded program of retirement benefits.
A. Added to the increase in hourly wages requested last July, the railroad employees are now
seeking an expanded program of retirement benefits.
B. Added to the increase in hourly wages which had been requested last July, the employees
of the railroad are now seeking an expanded program of retirement benefits.
C. The railroad employees are now seeking an expanded program of retirement benefits
added to the increase in hourly wages that were requested last July.
D. In addition to the increase in hourly wages that were requested last July, the railroad
employees are now seeking an expanded program of retirement benefits.
E. In addition to the increase in hourly wages requested last July, the employees of the
railroad are now seeking an expanded program of retirement benefits.

SC21561.01

256. The use of gravity waves, which do not interact with matter in the way electromagnetic waves
do, hopefully will enable astronomers to study the actual formation of black holes and
neutron stars.
A. in the way electromagnetic waves do, hopefully will enable
B. in the way electromagnetic waves do, will, it is hoped, enable
C. like electromagnetic waves, hopefully will enable
D. like electromagnetic waves, would enable, hopefully
E. such as electromagnetic waves do, will, it is hoped, enable

SC61561.01

257. Many of them chiseled from solid rock centuries ago, the mountainous regions of northern
Ethiopia are dotted with hundreds of monasteries.
A. Many of them chiseled from solid rock centuries ago, the mountainous regions of
northern Ethiopia are dotted with hundreds of monasteries.
B. Chiseled from solid rock centuries ago, the mountainous regions of northern Ethiopia are
dotted with many hundreds of monasteries.
C. Hundreds of monasteries, many of them chiseled from solid rock centuries ago, are
dotting the mountainous regions of northern Ethiopia.
D. The mountainous regions of northern Ethiopia are dotted with hundreds of monasteries,
many of which are chiseled from solid rock centuries ago.
E. The mountainous regions of northern Ethiopia are dotted with hundreds of monasteries,
many of them chiseled from solid rock centuries ago.

SC81561.01

258. Plausible though it sounds, the weakness of the hypothesis is that it does not incorporate all
relevant evidence.
A. Plausible though it sounds, the weakness of the hypothesis
B. Even though it sounds plausible, the weakness of the hypothesis
C. Though plausible, the hypothesis' weakness
D. Though the hypothesis sounds plausible, its weakness
E. The weakness of the hypothesis which sounds plausible

SC32561.01

259. In despite of the steady population flow out from rural areas into urban clusters, nearly 5
million farm households are still in Japan out of a total population of some 116 million
people.
A. In despite of the steady population flow out from rural areas into urban clusters, nearly 5
million farm households are still in Japan
B. In spite of the steady population flow out from rural areas into urban clusters, nearly 5
million farm households are still in Japan
C. Despite the steady population flow from rural areas into urban clusters, Japan's farm
households are still nearly 5 million
D. Despite the steady population flow from rural areas to urban clusters, there are still
nearly 5 million farm households in Japan
E. In Japan, despite the steady population flow out from rural areas into urban clusters, still
there are nearly 5 million farm households

SC52561.01

260. Financial uncertainties from the accident at Three Mile Island may prove even more
deterring to the nuclear industry than political opposition is.
A. from the accident at Three Mile Island may prove even more deterring to the nuclear
industry than political opposition is
B. from the accident at Three Mile Island may prove to be even more serious a deterrent to
the nuclear industry than political opposition
C. from the accident at Three Mile Island may prove to be an even more serious deterrent to
the nuclear industry than political opposition
D. resulting from the accident at Three Mile Island may prove to be an even more serious
deterrent to the nuclear industry than is political opposition
E. resulting from the accident at Three Mile Island may prove even more deterring to the
nuclear industry than political opposition

SC72561.01

261. Remembered almost as an epic among America's 12,000 Bosnian Muslims is the digging of
Chicago's subway tunnels in the early 1900s, one of the proudest of family legends.
A. Remembered almost as an epic among America's 12,000 Bosnian Muslims is the digging
of Chicago's subway tunnels in the early 1900s, one of the proudest of family legends.
B. Almost an epic among America's 12,000 Bosnian Muslims is the digging in the early
1900s of Chicago's subway tunnels, one of the proudest of family legends.
C. Digging Chicago's subway tunnels in the early 1900s, America's 12,000 Bosnian Muslims
remember it almost as an epic and it is the one of the proudest of family legends.
D. America's 12,000 Bosnian Muslims remember almost as an epic the digging of Chicago's
subway tunnels in the early 1900s, one of the proudest of family legends.
E. One of the proudest of family legends, remembered almost as an epic among America's
12,000 Bosnian Muslims, is the digging of Chicago's subway tunnels in the early 1900s.

SC92561.01

262. Like the one reputed to live in Loch Ness, also an inland lake connected to the ocean by a
river, inhabitants of the area around Lake Champlain claim sightings of a long and narrow
“sea monster.”
A. Like the one reputed to live in Loch Ness, also an inland lake connected to the ocean by a
river, inhabitants of the area around Lake Champlain claim sightings of a long and
narrow “sea monster.”
B. Inhabitants of the area around Lake Champlain claim sightings of a long and narrow “sea
monster” similar to the one reputed to live in Loch Ness, which, like Lake Champlain is an
inland lake connected to the ocean by a river.
C. Inhabitants of the area around Lake Champlain claim sightings of a long and narrow “sea
monster” similar to Loch Ness's, which, like Lake Champlain, is an inland lake connected
to the ocean by a river.
D. Like Loch Ness's reputed monster, inhabitants of the area around Lake Champlain, also
an inland lake connected to the ocean by a river, claim sightings of a long and narrow “sea
monster.”
E. Similar to that reputed to live in Loch Ness, inhabitants of the area around Lake
Champlain, also an inland lake connected to the ocean by a river, claim sightings of a long
and narrow “sea monster.”

SC43561.01

263. A star will compress itself into a white dwarf, a neutron star, or a black hole after it passes
through a red giant stage, depending on mass.
A. A star will compress itself into a white dwarf, a neutron star, or a black hole after it passes
through a red giant stage, depending on mass.
B. After passing through a red giant stage, depending on its mass, a star will compress itself
into a white dwarf, a neutron star, or a black hole.
C. After passing through a red giant stage, a star's mass will determine if it compresses itself
into a white dwarf, a neutron star, or a black hole.
D. Mass determines whether a star, after passing through the red giant stage, will compress
itself into a white dwarf, a neutron star, or a black hole.
E. The mass of a star, after passing through the red giant stage, will determine whether it
compresses itself into a white dwarf, a neutron star, or a black hole.

SC53561.01

264. Although many art patrons can readily differentiate a good debenture from an undesirable
one, they are much less expert in distinguishing good paintings and poor ones, authentic art
and fakes.
A. much less expert in distinguishing good paintings and poor ones, authentic art and
B. far less expert in distinguishing good paintings from poor ones, authentic art from
C. much less expert when it comes to distinguishing good paintings and poor ones, authentic
art from
D. far less expert in distinguishing good paintings and poor ones, authentic art and
E. far less the expert when it comes to distinguishing between good painting, poor ones,
authentic art, and
SC83561.01

265. A site once used as an observatory by the Anasazi, ancient pueblo dwellers of New Mexico,
has been recently discovered where patterns of light and shadow were employed to establish
the precise limits of the positions of the Sun and Moon over a nineteen-year cycle.
A. A site once used as an observatory by the Anasazi, ancient pueblo dwellers of New
Mexico, has been recently discovered where patterns of light and shadow were employed
to establish the precise limits of the positions of the Sun and Moon over a nineteen-year
cycle.
B. A recently discovered site was once used as an observatory by the Anasazi, ancient pueblo
dwellers of New Mexico, where patterns of light and shadow were employed to establish
the precise limits of the positions of the Sun and Moon over a nineteen-year cycle.
C. At a recently discovered site once used as an observatory by the Anasazi, ancient pueblo
dwellers of New Mexico, patterns of light and shadow were employed to establish the
precise limits of the positions of the Sun and Moon over a nineteen-year cycle.
D. Patterns of light and shadow were employed to establish the precise limits of the positions
of the Sun and Moon over a nineteen-year cycle at a site that was recently discovered and
was once used by the Anasazi, ancient pueblo dwellers of New Mexico.
E. Patterns of light and shadow were employed to establish the precise limits of the positions
of the Sun and Moon over a nineteen-year cycle at a recently discovered place that the
Anasazi, ancient pueblo dwellers of New Mexico, once used the site as an observatory.

SC93561.01

266. The cathedrals of the Middle Ages were community centers just as much as they were purely
religious edifices; and they were structures that represented a city's commitment to a public
realm, the opposite of being a private one.
A. community centers just as much as they were purely religious edifices; and they were
structures that represented a city's commitment to a public realm, the opposite of being a
private one
B. community centers as much as purely religious edifices; they were structures
representing a city's commitment to a public realm, as opposed to private
C. community centers as well as purely religious edifices; they were structures that
represented a city's commitment to a public realm, not private ones
D. as much community centers as purely religious edifices, structures that represented a
city's commitment to a public realm, as opposed to a private one
E. as much community centers as they were purely religious edifices, structures representing
a city's commitment to a public realm, opposite of a private one

SC14561.01

267. The newspaper story accurately recounted the history of the colonial mansion, that it
contained thirteen rooms, and that it had a reputation for being a haunted house.
A. mansion, that it contained thirteen rooms, and that it had a reputation for being a
haunted house
B. mansion, that it contained thirteen rooms, and that it had a reputation of being haunted
C. mansion, that the mansion contained thirteen rooms, and said that it had a reputation for
being haunted
D. mansion, said that it contained thirteen rooms and had a reputation for being a haunted
house
E. mansion and said that the mansion contained thirteen rooms and had the reputation of
being haunted
SC24561.01

268. An archaeological excavation at what might have been a workshop where statues were
reproduced yielded 1,532 fragments of human figures, including 7 intact statues.
A. what might have been a workshop where statues were reproduced yielded 1,532
fragments of human figures, including
B. what might have been a workshop where statues were reproduced yielded 1,532
fragments of human figures and
C. the site of a possible workshop where statues were reproduced yielded 1,532 fragments of
human figures and
D. the site of a possible workshop where statues were reproduced yielded 1,532 fragments of
human figures, including
E. the site of a possible workshop where statues might have been reproduced yielded 1,532
fragments of human figures, including

SC34561.01

269. Sophisticated laser-guided land graders can now flatten uneven farmland almost perfectly so
as not to waste rainwater in runoff down sloping fields.
A. so as not to waste rainwater
B. so that rainwater is not wasted
C. so that there is no wasted rainwater
D. and thereby not waste rainwater
E. and there is no rainwater wasted

SC54561.01

270. Because there is not a linguistic census in France, as there is for Britain, there is difficulty in
estimating the number of speakers of Breton, a Celtic language.
A. Because there is not a linguistic census in France, as there is for Britain, there is difficulty
in estimating
B. Because there is no linguistic census in France, unlike Britain, it is difficult to estimate
C. Unlike Britain, there is no linguistic census in France, and that fact makes for difficulty in
estimating
D. There is not a linguistic census in France, as there is for Britain, a fact making for
difficulty in the estimation of
E. There is no linguistic census in France, as there is in Britain, a fact that makes it difficult
to estimate

SC65561.01
271. When adjusted for body weight, children of various age groups in the United States have a
caffeine intake that ranges from 36 to 58 percent of the average amount consumed by adults.
A. children of various age groups in the United States have a caffeine intake that ranges from
36 to 58 percent of the average amount consumed by adults
B. the caffeine intake of children of various age groups in the United States ranges from 36
to 58 percent of the average amount consumed by adults
C. various age groups of children in the United States range in caffeine intake from 36 to 58
percent of that consumed by the average adult
D. in the United States, children of various age groups have a caffeine intake that ranges
from 36 to 58 percent of the average adult's consumption
E. in the United States, the caffeine intake of children in various age groups ranges from 36
to 58 percent of that consumed by the average adult
SC06561.01

272. When bitter managerial conflicts plague a small company, conflicts that in the past might
have led to dissolution of the business, executives are likely to turn to outside professional
counselors to help resolve disagreement.
A. conflicts plague a small company, conflicts that in the past might have led to dissolution
of the business, executives are likely to
B. conflicts plague a small company, conflicts that might have in the past led to its
dissolution, executives likely will
C. conflicts plague a small company, which in the past it might have led to the business's
dissolution, executives are liable to
D. conflicts, which in the past might have led to dissolution of the business, plague a small
company, executives are liable to
E. conflicts, which in the past might have led to its dissolution, plague a small company,
executives tend to

SC17561.01

273. A natural response of communities devastated by earthquake or flood is to rebuild on the


same site, overlooking the possibility that the forces that caused it could be repeated.
A. overlooking the possibility that the forces that caused it could be repeated
B. overlooking the possibility that the forces causing it could be repeated
C. overlooking that the forces that caused the disaster could also cause another one
D. without considering that the forces causing the disaster could be repeated
E. without considering that the forces that caused the disaster could also cause another such
disaster

SC67561.01

274. Avalanches at Rogers Pass in Glacier National Park killed more than 200 people between
1885 and 1910, but they are now controlled if not prevented; cannons are fired at the slopes
to make snow masses fall before they become dangerous.
A. Avalanches at Rogers Pass in Glacier National Park killed more than 200 people between
1885 to 1910, but they
B. More than 200 people have been killed by avalanches between 1885 and 1910 at Rogers
Pass in Glacier National Park, but they
C. Between 1885 and 1910, more than 200 people were killed by avalanches at Rogers Pass
in Glacier National Park, but they
D. More than 200 people have been killed by avalanches at Rogers Pass in Glacier National
Park between 1885 and 1910, but such avalanches
E. Avalanches at Rogers Pass in Glacier National Park killed more than 200 people between
1885 and 1910, but such avalanches

SC77561.01

275. Because 70 percent of the people of India use wood as their sole fuel, ten million acres of
forest have been lost there since 1960, resulting in wood now costing eight times as much to
collect and distribute than in 1960.
A. resulting in wood now costing eight times as much to collect and distribute than
B. resulting in wood now costing eight times as much to collect and distribute as
C. resulting in wood now costing eight times as much to collect and distribute than it did
D. and as a result wood now costs eight times as much to collect and distribute as it did
E. and wood now costs eight times as much as a result to collect and distribute than

SC09561.01

276. In a crowded, acquisitive world, the disappearance of lifestyles such as those once followed
by southern Africa's Bushmen and Australia's Aboriginal people, requiring vast wild spaces
and permitting little accumulation of goods, seem inevitably doomed.
A. requiring vast wild spaces and permitting little accumulation of goods, seem inevitably
doomed
B. requiring vast wild spaces and permitting little accumulation of goods, seems to be
inevitably doomed
C. which require vast wild spaces and permit little accumulation of goods, seem to be
inevitably doomed
D. lifestyles that require vast wild spaces and permit little accumulation of goods, seem
inevitable
E. lifestyles requiring vast wild spaces and permitting little accumulation of goods, seems
inevitable

SC42561.01

277. Before Colette, the female writers of France had been aristocrats, from Mme de Lafayette to
Anne de Noailles; there were no Jane Austens or Brontë sisters, perhaps because there were
almost no clergymen's daughters.
A. were no Jane Austens or
B. were not Jane Austens or
C. was not Jane Austen nor the
D. was not a Jane Austen or the
E. was no Jane Austen or no

SC73561.01

278. Chinese public buildings erected under a construction code of the Sung dynasty have
withstood earthquakes well because the white cedar used has four times the tensile strength
of steel and the timber frame, incorporating many joints and few nails, is flexible.
A. used has four times the tensile strength of steel and the timber frame, incorporating
B. used in them has four times the tensile strength of steel has and the timber frame,
incorporating
C. that was used in them has four times the tensile strength steel has, and the timber frame,
incorporating
D. that was used has four times as much tensile strength as steel, and the timber frame
incorporates
E. that was used has four times the tensile strength steel does, and the timber frame
incorporates

SC28561.01

279. Some historians of science have argued that science moves forward not so much because of
the insights of great thinkers but because of more mundane developments, such as improved
tools and technologies.
A. because of the insights of great thinkers but because of
B. because of the insights of great thinkers as the results of
C. because of the insights of great thinkers as because of
D. through the insights of great thinkers but through
E. through the insights of great thinkers but results from

Grammar
SC27561.01

280. Indoor air pollution can threaten the health of closely confined farm animals and the
workers who tend them and perhaps as well impairs the quality of such farm products like
eggs, poultry, and pork.
A. perhaps as well impairs the quality of such farm products like
B. perhaps as well impairs the quality of such farm products as
C. perhaps also impairs the quality of such farm products like
D. may also impair the quality of such farm products like
E. may also impair the quality of such farm products as

SC12811.01
281. Carbon-14 dating reveals that the megalithic monuments in Brittany are nearly 2,000 years
as old as any of their supposed Mediterranean predecessors.
A. as old as any of their supposed
B. older than any of their supposed
C. as old as their supposed
D. older than any of their supposedly
E. as old as their supposedly

SC68461.01

282. Some biographers have not only disputed the common notion that Edgar Allan Poe drank to
excess but also questioned whether he drank at all.
A. have not only disputed the common notion that Edgar Allan Poe drank to excess but also
questioned whether he drank
B. not only have disputed the common notion that Edgar Allan Poe drank to excess but also
over whether he drank
C. have disputed not only the common notion that Edgar Allan Poe drank to excess but also
whether he may not have drunk
D. not only have disputed the common notion that Edgar Allan Poe drank to excess but also
questioned whether or not he had drunk
E. have disputed the common notion not only that Edgar Allan Poe drank to excess but also
questioned whether he may not have drunk

SC90561.01

283. The large populations and impressive cultural achievements of the Aztecs, the Mayas, and
the Incas could not have come about without corn, which was not only nutritious but also
was able to be dried, transported, and stored for long periods.
A. which was not only nutritious but also was able to be
B. which not only was nutritious but also could be
C. which was not only nutritious but also it could be
D. not only nutritious but it could also be
E. not only nutritious but also able to be
SC91561.01

284. The Rorschach test is gaining new respect as a diagnostic tool because it takes only one hour
to expose behavior and thought processes that may be unlikely to emerge in other procedures
or weeks of ordinary interviewing.
A. that may be unlikely to emerge in other procedures or weeks of ordinary interviewing
B. whose emergence is unlikely in other procedures or weeks of ordinary interviews
C. that might not emerge in other procedures or in weeks of ordinary interviews
D. that may not emerge under other procedures or weeks of ordinary interviews
E. likely not to emerge during weeks of ordinary interviewing or in other procedures
SC82561.01

285. The overall slackening of growth in productivity is influenced less by government regulation,
although that is significant for specific industries like mining, than the coming to an end of a
period of rapid growth in agricultural productivity.
A. the coming to an end of
B. the ending of
C. by the coming to an end of
D. by ending
E. by the end of

SC13561.01

286. It may someday be worthwhile to try to recover uranium from seawater, but at present this
process is prohibitively expensive.
A. It may someday be worthwhile to try to recover uranium from seawater
B. Someday, it may be worthwhile to try and recover uranium from seawater
C. Trying to recover uranium out of seawater may someday be worthwhile
D. To try for the recovery of uranium out of seawater may someday be worthwhile
E. Recovering uranium from seawater may be worthwhile to try to do someday

SC23561.01

287. The spraying of pesticides can be carefully planned, but accidents, weather conditions that
could not be foreseen, and pilot errors often cause much larger deposits of spray than they
had anticipated.
A. weather conditions that could not be foreseen, and pilot errors often cause much larger
deposits of spray than they had
B. weather conditions that cannot be foreseen, and pilot errors often cause much larger
deposits of spray than
C. unforeseeable weather conditions, and pilot errors are the cause of much larger deposits
of spray than they had
D. weather conditions that are not foreseeable, and pilot errors often cause much larger
deposits of spray than
E. unforeseeable weather conditions, and pilot errors often cause much larger deposits of
spray than they had

SC33561.01

288. To read of Abigail Adams' lengthy separation from her family, her difficult travels, and her
constant battles with illness is to feel intensely how harsh life was even for the so-called
aristocracy of Revolutionary times.
A. To read of
B. Reading about
C. Having read about
D. Once one reads of
E. To have read of

SC74561.01

289. In the traditional Japanese household, most clothing could be packed flatly, and so it was
not necessary to have elaborate closet facilities.
A. flatly, and so it was not necessary to have elaborate closet facilities
B. flat, and so elaborate closet facilities were unnecessary
C. flatly, and so there was no necessity for elaborate closet facilities
D. flat, there being no necessity for elaborate closet facilities
E. flatly, as no elaborate closet facilities were necessary

SC75561.01

290. Many states, in search of industries that are clean, fast-growing, and pay good wages to
skilled workers, are trying to attract high-technology industries.
A. clean, fast-growing, and pay
B. clean, grow fast, and that pay
C. clean and fast-growing and that pay
D. clean and grow fast, paying
E. clean, fast-growing, and paying
SC95561.01

291. Much of the hope for continued improvement of the economy lies in the projection of
increasing consumer spending this year.
A. projection of increasing consumer spending
B. projection of consumers increasing spending for
C. projected consumer spending increase
D. consumer spending that is projected to increase
E. increase in consumer spending that is projected for

SC46561.01

292. Rejecting its argument that the Masters Dog Training Club's primary aim was to teach
people to train dogs, the court ruled the club ineligible for tax exemption as an educational
group.
A. Rejecting its argument that the Masters Dog Training Club's primary aim was to teach
people to train dogs, the court ruled the club ineligible for tax exemption as an
educational group.
B. In rejecting the Masters Dog Training Club's argument that their primary aim was to
teach people to train dogs, the court ruled the club ineligible to be exempted of taxes as an
educational group.
C. Rejecting the argument that the primary aim of the Masters Dog Training Club was to
teach people to train dogs, the court ruled that the club was ineligible for exemption from
taxes as an educational group.
D. The club was not to be exempted of taxes as an educational group, ruled the court by
rejecting its argument that the primary aim of the Masters Dog Training Club was
teaching people to train dogs.
E. The court ruled the Masters Dog Training Club not eligible to be exempted from taxes as
an educational group, rejecting the argument that the primary aim was to teach people to
train dogs.

SC56561.01

293. Sartre, an inadvertent guru, had an opinion on everything, painfully considered, elaborately
reasoned, often changed.
A. often changed
B. and it was usually changed
C. that was often changed
D. changing often
E. one he often changed

SC96561.01

294. One analyst of the liquor industry estimated that this year a few liquor stores have
experienced declining sales of up to fifty percent but predicted that the industry as a whole
will maintain a volume of sales fairly close to last year.
A. declining sales of up to fifty percent but predicted that the industry as a whole will
maintain a volume of sales fairly close to last year
B. declines in sales of up to fifty percent but predicted that the industry as a whole would
have maintained a volume of sales fairly close to last year
C. up to fifty percent in declining sales but predicted that the industry as a whole would
maintain a volume of sales fairly close to last year's
D. sales declines of up to fifty percent but predicted that the industry as a whole would
maintain a volume of sales fairly close to last year's
E. declines up to fifty percent of sales but predicted that the industry as a whole will have
maintained a volume of sales fairly close to last year's

SC08561.01

295. Television programs developed in conjunction with the marketing of toys, which was once
prohibited by federal regulations, are thriving in the free market conditions permitted by the
current Federal Communications Commission.
A. Television programs developed in conjunction with the marketing of toys, which was once
prohibited by federal regulations, are
B. Television programs developed in conjunction with the marketing of toys, a practice that
federal regulations once prohibited, is
C. Developing television programs in conjunction with the marketing of toys, as once
prohibited by federal regulations, is
D. Federal regulations once prohibited developing television programs in conjunction with
the marketing of toys, but they are
E. Federal regulations once prohibited developing television programs in conjunction with
the marketing of toys, but such programs are

SC98561.01

296. Statisticians studying the health effects of uranium mining on Navajo communities have
found others besides miners and millworkers to be affected; birth defects, children's cancer,
and altered birth ratios of males and females are much higher in mining than in non-mining
communities.
A. children's cancer, and altered birth ratios of males and females are much higher
B. cancer among children, and altered male and female ratios at birth are much higher
C. cancer among children, and altered birth ratios of males and females occur much more
frequently
D. altered birth ratios of males and females, and children's cancer are much higher
E. altered male and female ratios at birth, and cancer among children occur much more

SC19561.01

297. The effect of the earthquake that caused most of Port Royal to sink into the Caribbean was
like the eruption that buried ancient Pompeii: in each case a slice of civilization was instantly
frozen in time.
A. The effect of the earthquake that caused most of Port Royal to sink into the Caribbean
was
B. As the result of an earthquake, most of Port Royal sank into the Caribbean; the effect was
C. In its effects, the sinking of most of Port Royal into the Caribbean was the result of an
earthquake
D. The earthquake that caused most of Port Royal to sink into the Caribbean was, in its
effects,
E. Most of Port Royal sank into the Caribbean because of an earthquake, the effect of which
was

SC58461.01

298. Since the 1930s aircraft manufacturers have tried to build airplanes with frictionless wings,
shaped so smoothly and perfectly that the air passing over them would not become turbulent.
A. wings, shaped so smoothly and perfectly
B. wings, wings so smooth and so perfectly shaped
C. wings that are shaped so smooth and perfect
D. wings, shaped in such a smooth and perfect manner
E. wings, wings having been shaped smoothly and perfectly so
SC29561.01

299. A study of children of divorced parents found that ten years after the parents' divorce,
children who had been under six years of age at the time of the settlement were not
preoccupied, nor even very curious, about the reasons that led to their parents' divorces.
A. not preoccupied, nor even very curious, about the reasons that led to their parents'
divorces
B. not preoccupied with, or even very curious about, the reasons for their parents' divorce
C. neither preoccupied, nor even very curious, with the reasons that led to their parents
divorce
D. neither preoccupied with the reasons that led to their parents' divorces or even very
curious about them
E. neither preoccupied with the reasons that their parents divorced nor even very curious
about it

SC49561.01

300. When Medicare was enacted in 1965, it was aimed at the prevention of a catastrophic illness
from financially destroying elderly patients.
A. at the prevention of a catastrophic illness from financially destroying elderly patients
B. at being a preventive against catastrophic illness financially destroying elderly patients
C. at preventing a catastrophic illness from financially destroying the elderly patient
D. to prevent a catastrophic illness financially destroying an elderly patient
E. to prevent elderly patients being financially destroyed by a catastrophic illness
Answer Key Verbal Reasoning
Verbal Reasoning
Reading Comprehension
151. A
152. C
153. D
154. B
155. E
156. C
157. C
158. D
159. E
160. A
161. A
162. D
163. C
164. A
165. E
166. B
167. A
168. D
169. D
170. C
171. D
172. B
173. C
174. D
175. C
176. A
177. B
178. E
179. D
180. C
181. D
182. E
183. B
184. A
185. D
186. B
187. A
188. E
189. B
190. D
191. E
192. A
193. D
194. A
195. D
196. B
197. E
198. C
199. D
200. C
Critical Reasoning

Analysis/Critique
201. B
202. A
203. B
204. B
205. A
206. E
207. B
208. E
209. D
210. D
211. C
212. A
213. E
214. E
215. B
216. D
217. E
218. D
219. E
220. E
221. B
222. B
223. C
224. D
225. C
226. B

Construction/Plan
227. A
228. B
229. D
230. B
231. D
232. B
233. D
234. E
235. B
236. A
237. B
238. B
239. D
240. E
241. B
242. B
243. D
244. B
245. B
246. B
247. A
248. D
249. C
Sentence Correction

Communication
250. B
251. D
252. D
253. B
254. E
255. E
256. B
257. E
258. D
259. D
260. D
261. E
262. B
263. D
264. B
265. C
266. D
267. E
268. B
269. B
270. E
271. B
272. A
273. E
274. E
275. D
276. E
277. A
278. A
279. C

Grammar
280. E
281. B
282. A
283. B
284. C
285. C
286. A
287. B
288. A
289. B
290. C
291. E
292. C
293. A
294. D
295. E
296. C
297. D
298. B
299. B
300. C
Answer Explanations Verbal Reasoning
Reading Comprehension
Questions 151–155 refer to the passage on page 143.

RC62100.01-10

151. The primary purpose of the passage is to


A. present an interpretation of the chronological relationship between bipedal locomotion
and certain other key aspects of human evolution
B. compare the evolutionary advantages and disadvantages of bipedal locomotion to those of
quadrupedal locomotion
C. argue that the transition to a nuclear family structure was a more crucial step in human
evolution than was the development of stone tools
D. analyze anatomical evidence of bipedal locomotion to show that the large brain of modern
humans developed at a later stage of evolution than was previously believed
E. use examples of muscle and bone structure to illustrate the evolutionary differences
between modern humans, australopithecines, and chimpanzees
Main idea
To discern the primary purpose of a passage requires an understanding of the key messages
within the passage. The passage states that a once-favored view was that modern human
beings' ancestors began to walk upright around the same time that they began to use stone
tools.
However, this argument has been weakened by archaeological discoveries indicating that
these ancestors began to walk upright before the enlargement of the brain that led to the
development of such tools.
Furthermore, the passage indicates that these ancestors had a shallow pelvis that would
actually suit walking upright better than the bowl-shaped human pelvis, a development that
allows for children to be born with larger brains and therefore heads.
Given that walking upright is less efficient than walking on all fours, the passage suggests
that some sort of evolutionary advantage came from walking upright; the passage suggests
that this advantage may have been that it allowed monogamous parents to cooperate in the
care of offspring.
A. Correct. As the review above indicates, the passage is primarily concerned with the
chronological relationship between bipedal locomotion and certain other important
developments in human evolution, such as fitness for cooperative parental roles.
B. While the passage does mention the relative efficiency of bipedal and quadrupedal
locomotion, it does so primarily to motivate the larger purpose of the passage. That is, it
justifies the need to explain the development of bipedal locomotion. In other words, if
bipedal locomotion were more efficient rather than less efficient, this efficiency boost
would be sufficient to explain its evolution.
C. The passage does not discuss whether the development of the nuclear family or the
development of stone tools was the more crucial step in human evolution. Rather, it
simply rules out the development of stone tools as an explanation for the development of
bipedal locomotion and proposes the transition to a nuclear family as a possible
explanation.
D. The passage does present such evidence, but it does so merely as one step in pursuit of
the primary purpose of presenting the chronological relationships among the evolution of
bipedal locomotion and other key human developments.
E. The passage does use such examples, but it does so merely as one step in pursuit of the
overall, primary purpose of presenting the chronological relationships among the
evolution of bipedal locomotion and other key human developments.
The correct answer is A.
RC62100.01-20

152. The passage suggests that proponents of the theory mentioned in lines 35–38 assume that
which of the following steps in human evolution occurred most recently?
A. Development of a nuclear family structure
B. Transition from walking on all fours to walking upright
C. Dramatic enlargement of the brain
D. Use of the hands to gather and carry food
E. Modification of propulsive muscles to provide stability and control in locomotion
Inference
The passage states that fossilized remains provide anatomical evidence that upright walking,
which required a modification of propulsive muscles to provide stability and control in
locomotion, occurred before the dramatic enlargement of the brain.
Proponents of the theory mentioned in lines 35–38 argue that walking upright may have
evolved alongside the nuclear family structure because it allowed for cooperative caring for
infants, which would have required the use of hands to gather and carry food. Thus, the
dramatic enlargement of the brain was the most recent of the developments listed among the
answer options.
A. Proponents of the theory mentioned in lines 35–38 hold that the dramatic enlargement of
the brain occurred more recently than the development of walking upright, which
happened alongside the development of a nuclear family structure.
B. Proponents of the theory mentioned in lines 35–38 hold that the dramatic enlargement of
the brain occurred more recently than the transition from walking on all fours to walking
upright.
C. Correct. Proponents of the theory mentioned in lines 35–38 hold that the dramatic
enlargement of the brain was the most recent of these developments to occur.
D. Proponents of the theory mentioned in lines 35–38 hold that the dramatic enlargement of
the brain occurred more recently than the use of hands to gather and carry food, which
occurred alongside the development of walking upright.
E. Proponents of the theory mentioned in lines 35–38 hold that the dramatic enlargement of
the brain occurred more recently than the modification of propulsive muscles to provide
stability and control in locomotion, which is a key factor in the development of walking
upright.
The correct answer is C.
RC62100.01-30

153. According to the passage, the hominid australopithecine most closely resembled a modern
human with respect to which of the following characteristics?
A. Brain size
B. Tool-making ability
C. Shape of the pelvis
D. Method of locomotion
E. Preference for certain foods
Supporting idea
The passage directly states that australopithecines walked upright (used bipedal
locomotion), as human beings do.
A. The passage states that the dramatic enlargement of the brain among hominids occurred
after the australopithecine era; it follows that the size of the australopithecine brain did
not closely resemble that of the modern human brain.
B. The passage states that stone-tool-making ability did not develop until some time after
the australopithecine era. The passage suggests that study of Australopithecus indicates
that there is substantial evidence that upright walking appeared prior to . . . stone tools.
C. The passage specifies notable differences in shape between the australopithecine pelvis
and the pelvis of modern human beings.
D. Correct. As noted above, the passage indicates that australopithecines walked upright,
or used bipedal locomotion, as modern human beings do.
E. The passage does not indicate the types of foods preferred by australopithecines.
The correct answer is D.
RC62100.01-40

154. The passage suggests that, in comparison with the hominid australopithecines, modern
humans are
A. less well adapted to large-group cooperation
B. less well adapted to walking upright
C. more agile in running and climbing
D. more well suited to a nuclear family structure
E. more well suited to cooperative caring for their offspring
Inference
The information needed to answer this question is contained in the second paragraph. This is
the only place in the passage where comparisons between australopithecines and modern
human beings occur; the passage points out (1) that the pelvis and the femur of
australopithecines are more similar to those of modern humans than they are to those of
chimpanzees, the most closely related living primate, and (2) that the pelvis of
australopithecines is better suited for bipedal locomotion than is the pelvis of modern
humans.
A. The passage makes no mention of large-group cooperation.
B. Correct. As discussed above, the passage notes that the modern human pelvis is less
suited for bipedal locomotion than was the australopithecine pelvis. This suggests that, in
comparison with australopithecines, modern humans are less well adapted to walking
upright.
C. The fact that australopithecines were better suited for walking upright than modern
humans are would suggest if anything that australopithecines would also be better suited
than humans to running and climbing (rather than vice versa). Regardless, the passage
provides no clear evidence of whether modern humans or australopithecines were more
agile.
D. In the third paragraph, the passage suggests that australopithecines may have been
physically well suited to a nuclear family structure. The passage gives no information as
to whether australopithecines were more or less physically well suited to such a structure
than are modern humans.
E. In the third paragraph, the passage suggests that australopithecines may have been
physically well suited to cooperative caring for their offspring. The passage gives no
information as to whether they were more or less physically well suited to such caring
than modern humans.
The correct answer is B.
RC62100.01-50

155. The theory mentioned in lines 35–38 suggests that which of the following was true for the
hominid ancestors of modern humans before they made the transition to walking upright?
A. Their brains were smaller than the brains of present-day chimpanzees.
B. They competed rather than cooperated in searching for food.
C. Their mating patterns and family structure were closer to those of present-day
chimpanzees than to those of modern humans.
D. Males played a more significant role in child rearing than they played after the transition
to walking upright.
E. Females' ability to nurture and protect their offspring was limited by the need to find food
for themselves.
Inference
The theory mentioned in lines 35–38 holds that bipedality evolved among modern humans'
hominid ancestors specifically because it granted monogamous couples the ability to
cooperate in the care of their offspring.
According to this theory, because they could now walk upright, fathers were able to use their
hands to gather food and carry it to their mates from a distance. This in turn allowed mothers
to expend greater amounts of time and energy to the nurture and protection of their children.
This implies that prior to the development of walking upright, mothers had to spend more
time acquiring their own food, and therefore less time nurturing and protecting their
offspring.
A. According to the passage, the brains of baby australopithecine hominids were no larger
than the brains of baby chimpanzees. This in no way implies that the brains of these
ancestors were smaller than those of chimpanzees. Nothing in the passage indicates that
the theory would disagree with this.
B. The theory states that walking upright allowed for cooperation for food within a
monogamous couple; a simple lack of cooperation does not imply that these ancestors
necessarily competed for food. For example, these groups could have engaged in
cooperative hunting, just as many nonbipedal animals are now.
C. The theory does suggest that prior to the development of bipedality, these ancestors were
not as capable of cooperative care. However, this still does not imply that their mating
patterns and family structures were more similar to those of chimpanzees than to those of
modern humans.
D. The theory actually suggests the opposite: that bipedality developed because it allowed for
greater cooperative care among hominid parents.
E. Correct. As discussed above, the development of bipedality allowed fathers to assist
mothers in acquiring food. This, thereby, freed up time and energy for mothers to nurture
and protect their offspring.
The correct answer is E.

Questions 156–161 refer to the passage on page 145.

RC04200.01-10

156. The passage is primarily concerned with


A. providing evidence indicating that feminist reformers of the 1920s failed to reach some of
their goals
B. presenting scholarship that contrasts suffragist “promises” with the historical realities of
the 1920s
C. discussing recent scholarship concerning the achievements of women's suffrage during
the 1920s and presenting an alternative view of those achievements
D. outlining recent findings concerning events leading to suffrage for women in the 1920s
and presenting a challenge to those findings
E. providing support for a traditional view of the success of feminist attempts to increase
gender consciousness among women during the 1920s
Main idea
To understand the primary concern of the passage requires a clear understanding of the
passage as a whole. This passage discusses recent scholarship concerning the 1920s that
challenges an earlier interpretation in which the women's suffrage movement during the
1920s was considered a failure because it had not achieved long-term political goals. The
scholars responsible for this earlier interpretation present the 1920s as a period of decline,
and the passage presented here challenges this assessment.
A. The passage is primarily concerned with discussing how recent scholarship challenged an
earlier assessment of the women's suffrage movement. The passage provides no specific
evidence indicating that feminist reformers of the 1920s failed to reach their goals.
B. Although the passage refers to the unkept “promises” of the women's suffrage movement,
the passage does not present scholarship that contrasts such a “promise” with the
historical realities of the 1920s.
C. Correct. The passage discusses recent scholarship concerning the achievements of
women's suffrage. In the final paragraph, the passage presents an alternative view of
those achievements.
D. The passage does not outline findings regarding what led to suffrage for women. The
passage's focus is on scholarship regarding what followed women gaining suffrage.
E. The passage does not provide support for a traditional view of 1920s feminists' success at
increasing gender consciousness among women. In fact, the passage conflicts with the
traditional view by suggesting that, during the 1920s, there existed intense activism
aimed at increasing autonomy for women.
The correct answer is C.
RC04200.01-20

157. It can be inferred that the author of the passage disagrees with the “new scholarship”
mentioned in line 5 regarding the
A. degree to which the “promises” of the suffrage movement remained unkept
B. degree to which suffrage for women improved the morality of governance
C. degree to which the 1920s represented a period of decline for the feminist movement
D. degree of legislative success achieved by feminist reformers during the 1920s
E. accuracy of the view that a women's voting bloc did not materialize once suffrage was
achieved
Inference
In the final paragraph of the passage, the author offers several points to counter the claim
made by the new scholarship that the 1920s were still, in some ways, a period of decline.
A. Both the author and this new scholarship question the claims of the earlier interpretation
that the “promises” of the women's suffrage movement were unkept.
B. There is no indication of agreement or disagreement between the author of the passage
and this new scholarship about how much women improved the morality of governance
in the 1920s.
C. Correct. In the final paragraph, the author gives a counterpoint to the claim of the new
scholarship that the 1920s, in some ways, represented a period of decline for the feminist
movement.
D. Nothing in the passage supports the claim that the author disagrees with the new
scholarship regarding the degree of legislative success achieved by feminist reformers.
Note that none of the achievements discussed in the final paragraph are indicated to be
legislative successes.
E. While the author of the passage mentions the new scholarship's claim that a women's
voting bloc failed to materialize once suffrage was achieved, the author does not explicitly
disagree with this statement.
The correct answer is C.

RC04200.01-30

158. The purpose of the second paragraph of the passage is to


A. suggest a reason why suffragist “promises” were not kept
B. contrast suffragist “promises” with the reality of the 1920s
C. deplore the lack of successful feminist reform in the 1920s
D. explain a view held by feminist scholars
E. answer the question asked by Jane Addams
Evaluation
The second paragraph of the passage describes certain views presented in the recent feminist
scholarship discussed in the first paragraph. In the first paragraph, this new scholarship is
described as challenging earlier interpretations that suggested that “promises” of the
women's suffrage movement had not been kept.
Though this new scholarship argues that it is a mistake to view women's suffrage as a failure,
the scholarship nonetheless does, in some ways, present the 1920s as a period of decline. The
second paragraph of the passage explains precisely how the scholars present such a view.
A. Both the author and the new scholarship discussed in the passage reject the
interpretation that suffragist “promises” remained unkept.
B. The second paragraph does not present a contrast between the reality of the 1920s and
the suffragist “promises.” This paragraph does present ways in which the new scholarship
saw the 1920s as a period of decline. However, both the author and this new scholarship
argue that it is improper to interpret the results of women's suffrage in the 1920s as
unkept “promises.”
C. As noted in the third paragraph, the author sees successful feminist reform in the 1920s.
It is therefore incorrect to suggest that the author intends the second paragraph to
deplore a lack of successful feminist reform during this period.
D. Correct. As discussed above, the second paragraph explains how this new scholarship
presents the 1920s as a period of decline.
E. Jane Addams's question was rhetorical; there is no answer to this question in the passage.
The correct answer is D.
RC04200.01-40

159. It can be inferred from the passage that recent scholars cite the words of Jane Addams
primarily in order to
A. suggest that women's achievement of suffrage brought about changes in government that
were not taken into account by early interpretations
B. point out contradictions inherent in the goals of the women's suffrage movement
C. show why a women's voting bloc was not formed when women won the right to vote
D. emphasize the place of social reform movements in the struggle for suffrage for women
E. suggest that the old view of women's suffrage was inappropriate
Inference
The scholars cite the words of Jane Addams to suggest that it is no more appropriate to ask
whether women's suffrage was a failure than to ask whether suffrage in general is a failure.
The clear implication is that it is inappropriate to ask either question, presumably because
suffrage has value in and of itself.
A. The scholars do not dispute the claim that women's suffrage failed to bring about
significant changes in government. The point of Addams's statement is that it is
inappropriate to call women's suffrage into question simply because it does not bring
about all desired changes.
B. The scholars suggest no inherent contradictions in the goals of the women's suffrage
movement. Even if such a claim had been made, there is little relation between that claim
and Jane Addams's statement.
C. The scholars accept the earlier interpretation's assertion that a women's voting bloc was
not formed; however, the scholars offer no explanation as to why such a bloc was not
formed.
D. While it may be true that social reform movements played a significant role in the
struggle for suffrage for women, this particular quotation does not convey that idea.
E. Correct. The old view suggested that in many ways the women's suffrage movement was
a failure because it had failed to attain certain goals that had been associated with
women's suffrage. The scholars quote Addams to suggest that this view is inappropriate:
women's suffrage should no more be considered a failure for failing to reach all of its
goals than suffrage in general should be considered a failure for failing to reach all of its
goals.
The correct answer is E.

RC04200.01-50

160. It can be inferred that the analyses of the author of the passage and the scholars mentioned
in lines 20–23 differ with regard to which of the following?
A. The criteria they use to evaluate the success of the feminist movement during the 1920s
B. Their interpretations of the “promises” of the suffragist movement
C. The suggestions they make for achieving feminist goals
D. Their definitions of what constitutes a legislative victory
E. Their estimations of the obstacles preventing women's having achieved a voting bloc in
the 1920s
Inference
The author of the passage mentions ways in which the new scholarship mentioned in the
first paragraph sees the 1920s as a period of decline for the feminist movement. For example,
these scholars suggest that after the mid-1920s, few successes could be claimed by feminist
reformers: little could be seen in the way of legislative victories.
The author points out actual gains for women, for example, the establishment by women's
organizations of broader opportunities for women, striving to secure for women the full
entitlements of citizenship, including the right to hold office and the right to serve on juries.
The author thereby appears to counter these scholars' suggestion that the 1920s was a period
of decline by suggesting other criteria that can be used to evaluate the attainments of the
1920s feminist movement.
A. Correct. As discussed above, the author applies different criteria from those attributed
to the scholars mentioned in the second paragraph in evaluating the success of the
feminist movement during the 1920s.
B. The author presents these scholars' discussion of these “promises” in the first paragraph;
the author does not appear to disagree with the scholars' assessment.
C. The passage does not discuss any suggestions that these scholars have made about how to
achieve feminist goals.
D. The passage mentions legislative victories but offers no definition of the term.
E. Nowhere does the passage discuss obstacles preventing the formation of a voting bloc in
the 1920s.
The correct answer is A.

RC04200.01-60

161. The “new scholarship” mentioned in the first paragraph suggests which of the following
concerning the “promises” mentioned in lines 4–5?
A. Failure to keep these promises is not a measure of the success of the suffrage movement.
B. Failure to keep these promises caused the feminist movement to lose cohesiveness during
the 1920s.
C. Failure to keep these promises led recent feminist scholars to reevaluate the success of
the suffrage movement.
D. These promises included securing for women the right to hold office and the right to serve
on juries.
E. These promises were of little importance in winning suffrage for women.
Inference
The passage indicates that new scholarship disputed a long-held view that because certain
promises of the women's suffrage movement—e.g., that a women's voting bloc would form;
that the women's vote would bring about moral, corruption-free governance—went unkept,
the movement was a failure. This scholarship rejected the view that unkept “promises”
suggested a failure of the movement.
A. Correct. As indicated above, the new scholarship rejected the notion that the failure to
keep these promises was a reasonable measure of the success or failure of the suffrage
movement.
B. The second paragraph of the passage explains the new scholarship's view that the
feminist movement lost its cohesiveness in the 1920s. However, there is no suggestion
that a failure to keep the promises of the suffrage movement was actually a cause of this.
C. The new scholarship reevaluated the success of the suffrage movement. However, this
scholarship did not suggest that it did so because the suffrage movement failed to keep its
promises.
D. There is nothing in the passage to indicate that the new scholarship suggests that these
promises include securing the right to hold office and the right to serve on juries for
women.
E. There is nothing in the passage to suggest either that the promises were of little
importance in winning suffrage for women or even that this new scholarship believed
that they were.
The correct answer is A.

Questions 162–165 refer to the passage on page 147.

RC60500.01-10

162. The primary purpose of the passage is to


A. present several theories that could account for a particular phenomenon
B. argue that a certain question needs to be reframed in light of new evidence
C. resolve an apparent inconsistency between two lines of evidence
D. explain why a certain issue remains unresolved
E. present evidence that calls into question certain assumptions of a current theory
Main idea
Our goal is to determine which of the five options best expresses the primary purpose of the
passage. Note that this question regards the topic and how that topic is discussed: the issue
that the passage primarily focuses on concerns the mass at the center of our galaxy. The
passage primarily aims to explain why the specific nature of that mass had not—at least in
1997, when the passage was written—been adequately understood.
A. While the passage draws upon certain theoretical findings, it does not present multiple
theories.
B. The passage suggests that the author is puzzled by the enormous mass at the center of the
Milky Way, in light of evidence showing the relatively low level of energy radiating
outward. Nevertheless, the passage never attempts to convince us that the question needs
to be reframed.
C. The passage describes an apparent inconsistency—or discrepancy—between two lines of
evidence but does not seek to resolve that inconsistency.
D. Correct. As indicated above, this choice expresses the primary purpose of the passage;
that is, to explain why the nature of the mass at the center of our galaxy was not
adequately understood.
E. Although some of the evidence presented may cast doubt on one or more assumptions of
a theory that was current when the passage was written, the passage itself is not primarily
focused on identifying any such assumptions.
The correct answer is D.
RC60500.01-20

163. According to the passage, the dynamical evidence referred to in lines 9–10 supports which of
the following?
A. Recent assumptions about the velocities of stars
B. Widely held predictions about the amount of matter a black hole will engulf
C. The existence of an extremely dense object at the center of the Milky Way
D. The contention that too much energy is coming from the mass at the Milky Way's galactic
center for that mass to be a black hole
E. The conclusion that a compact object of two to three million times the mass of our Sun is
too dense to be anything but a black hole
Supporting idea
What does the passage claim the dynamical evidence mentioned supports? The passage
states that the dynamical evidence argues for an extremely compact object with a mass two
to three million times the mass of our Sun at the center of the Milky Way. An extremely
compact object with such a mass would, of course, be tremendously dense.
A. The passage in no way suggests that the dynamical evidence mentioned supports
assumptions about the velocities of stars. Rather, it states that the assumptions about the
velocities of the stars have been “recently confirmed” and that the dynamical evidence is
actually based on these assumptions. If the dynamical evidence is a product of these
assumptions, then it cannot be used to support them.
B. The passage suggests that the dynamical evidence is consistent with the existence in the
Milky Way of an extremely dense object that is likely a black hole. However, the evidence
in no way suggests that reliable predictions can be made about how much matter such a
hypothetical black hole would engulf.
C. Correct. As indicated above, this refers to the extremely compact object of immense
mass; this object would of course be “extremely dense.”
D. This choice presents a suggestion opposite to the information in the passage. The passage
suggests that the object at the center of the Milky Way is in fact radiating too little to be
easily identifiable as a black hole.
E. The passage acknowledges that the density of the mass at the center of the Milky Way is
consistent with the existence of a black hole there. However, the passage further suggests
that an unexpectedly small quantity of energy radiating outward from the center of the
Milky Way calls this into question.
The correct answer is C.

RC60500.01-30

164. The “serious problem” referred to in line 17 could be solved if which of the following were
true?
A. Current assumptions about how much matter a black hole would engulf proved to be
several thousand times too high.
B. Current assumptions about how much matter a black hole would engulf proved to be a
few thousand times too low.
C. The object at the center of the Milky Way turned out to be far more dense than it is
currently estimated to be.
D. The object at the center of the Milky Way turned out to be far more massive than it is
currently estimated to be.
E. Matter being engulfed by a black hole radiated far more energy than is currently assumed.
Application
Which one of the five options would, if true, indicate a possible solution to the serious
problem referred to? The serious problem is said to arise from the relative lack of energy
radiating from the galactic center compared with the expected value if a matter-engulfing
black hole were truly at the center of the Milky Way.
According to the passage, the radiated energy turned out to be a few thousand times less
than had been expected, on theoretical grounds, to radiate from mass engulfed by the
hypothetical black hole. However, if it were discovered that the matter engulfed were several
thousand times less than previously estimated, the relatively low level of radiated energy
observed would no longer seem at odds with the existence of the hypothesized black hole.
A. Correct. Reviewing the assumptions underlying the widely held predictions could lead
to revised predictions that harmonize with the observational evidence regarding radiated
energy from the galaxy's center.
B. The passage states that the assumed quantity of engulfed matter already seems to be
radiating too little; this answer choice would actually make the “serious problem” in the
passage worse. That is, assuming that there is actually more engulfed matter with such a
small amount of radiation would simply worsen the problem.
C. The passage does not specify what impact a greater-than-estimated density would have
on the quantity of energy that is radiated. That is, even if greater mass would change
predictions, greater density may not do so.
D. If the object were more massive, then presumably the matter engulfed by this
hypothetical more massive black hole would radiate a quantity of energy even greater
than the observed quantity. This again would simply make the “serious problem” worse.
E. Given that not enough radiated energy is currently being observed, finding that the
matter being engulfed radiates even more energy than expected would make the “serious
problem” significantly worse. That is, the relatively low quantity of radiated energy
observed would fall even farther short of the quantity expected.
The correct answer is A.

RC60500.01-40

165. The “widely held predictions” mentioned in line 25 are predictions about the
A. compactness of objects whose mass is millions of times the mass of our Sun
B. velocities of stars orbiting the galactic center
C. amount of matter swirling around the object at the center of the Milky Way
D. amount of matter falling into a theoretical central black hole
E. amount of energy that should be coming from a black hole at the center of the Milky Way
Evaluation
What do the “widely held predictions” mentioned in the passage's final sentence refer to?
Notice that the final sentence of the passage refers to a comparison between two things: the
energy coming from the galactic center and the quantity of energy widely predicted to be
radiated from matter being engulfed by a black hole. It follows that the best answer should
present an option that refers to a predicted quantity of energy.
A. This choice fails to refer to any predicted quantity of energy.
B. This choice fails to refer to any predicted quantity of energy.
C. This choice fails to refer to any predicted quantity of energy.
D. This choice fails to refer to any predicted quantity of energy.
E. Correct. This choice presents the only option that refers to a predicted quantity of
energy.
The correct answer is E.

Questions 166–172 refer to the passage on page 149.


RC39461.01-10

166. The author's discussion of Emerson, Thoreau, Hawthorne, Melville, and Whitman is
primarily concerned with explaining
A. some of their beliefs about the difficulties involved in self-realization
B. some of their beliefs concerning the world and the place that humanity occupies in the
universal order
C. some of their beliefs concerning the relationship between humanism and democracy
D. the way some of their beliefs are shaped by differences in temperament and literary
outlook
E. the effects of some of their beliefs on their writings
Main idea
Understanding the primary concern of the passage implies understanding the passage as a
whole. The passage starts by stating that these five American writers have many differences,
but that it will focus on those beliefs that they share. The passage suggests that all five of the
writers hold that humans are the spiritual center of the universe and that in them alone is
the clue to nature, history, and ultimately the cosmos.
The passage continues by providing more specific evidence for this assertion, outlining
beliefs that the writers hold in common. Note that the first sentences of each paragraph from
the second onward identify a particular perspective, belief, or assumption that the writers
hold in common.
A. The scope of this choice is too narrow. The third paragraph of the passage discusses two
apparently conflicting psychological tendencies that the writers suggest must be
reconciled in order to achieve self-actualization. However, this point is not of great
concern throughout the rest of the passage.
B. Correct. As discussed above, the passage is primarily concerned with beliefs the writers
share concerning the fundamental role of humanity in the universal order.
C. The scope of this choice is too narrow. In the third paragraph, the passage considers the
role of democracy in the five writers' thinking. However, it is only one of several such
issues that the passage discusses.
D. This choice presents an idea contrary to the primary concern of the passage: the passage
focuses not on the differences among these writers, but rather on the beliefs, concerns,
and assumptions they share.
E. The passage primarily focuses on the common beliefs of these writers, not on any
particular effects that these beliefs have on the writers' work.
The correct answer is B.
RC39461.01-20

167. According to the passage, the five writers object to the scientific method primarily because
they think it
A. is not the best way to obtain an understanding of the relationship between the individual
and the cosmos
B. is so specialized that it leads to an understanding of separate parts of the universe but not
of the relationships among those parts
C. cannot provide an adequate explanation of intuition and imagination
D. misleads people into believing they have an understanding of truth, when they do not
E. prevents people from recognizing the symbolic nature of experience
Evaluation
The final paragraph of the passage indicates that the five writers suggest that the scientific
method provides a less sure way to arrive at truth than do intuition and imagination. In
particular, these authors propose that intuition and imagination allow us to recognize, in a
way the scientific method cannot, the organic relationship between the self and the cosmos.
A. Correct. As indicated above, the passage indicates that these five writers believe that the
scientific method is not a sure way to obtain an understanding of how the individual self
relates to the cosmos.
B. The passage suggests that these five writers objected to use of the scientific method to
obtain understanding of how the individual relates to the cosmos, not because it is too
“specialized.”
C. The passage suggests that these five writers believed that intuition and imagination were
surer means of arriving at truth than the scientific method. This does not imply that the
scientific method is incapable of providing an adequate explanation of intuition and
imagination themselves.
D. The passage does not indicate whether the writers believed that, as a result of the
scientific method, people falsely believed that they understood the truth. The passage
does suggest that the writers believed that the scientific method cannot lead to certain
types of fundamental truths. However, this does not imply that these authors thought that
any beliefs arrived at through the scientific method are inherently false.
E. The passage indicates that the writers did not believe that the scientific method could
effectively lead to recognizing the symbolic nature of experience. Nevertheless, it does not
indicate that they believe that the scientific method prevents people from recognizing the
symbolic nature of experience.
The correct answer is A.
RC39461.01-30

168. The author quotes Whitman primarily in order to


A. show that the poet does not agree with Emerson
B. indicate the way the poet uses the humanist ideal to praise himself
C. suggest that the poet adapts the basic premises of humanism to his own individual
outlook on the world
D. illustrate a way the poet expresses the relationship of the individual to the humanistic
universe
E. demonstrate that the poet is concerned with the well-being of all humans
Evaluation
The second paragraph stresses that these five writers emphasize the human as universal,
freed from the accidents of time, space, birth, and talent; they see this idea as falling within
the general notion that humans are the spiritual center of the universe.
In this second paragraph the author presents two examples of writing, one from Emerson
and one from Whitman, which demonstrate this emphasis. Answering the question requires
recognizing that Whitman's statement illustrates the idea that each individual human has
such a general relationship with the universe.
A. The quotations from Emerson and Whitman are intended to illustrate the same idea—
namely, the human as universal—not to show how these two writers differ.
B. The author quotes Whitman to illustrate a general idea shared by the five writers rather
than to indicate how Whitman uses the humanist ideal to praise himself. Furthermore,
this choice is incorrect because the quotation itself indicates something true of all the
“children of Adam”—all of humanity—rather than something true of Whitman
specifically.
C. Once again, the author's quotation of Whitman is not meant to indicate anything
distinctive about Whitman, but rather to illustrate a perspective he shared with the other
four writers.
D. Correct. The author quotes Whitman to illustrate how Whitman expresses—as do each
of the other writers under consideration—the relationship of each individual to a
humanistic universe.
E. The quotation illustrates Whitman's view of the relationship of the individual to the
universe. Even though Whitman may in fact be concerned with the well-being of all
humans, the quotation itself in no way suggests this.
The correct answer is D.
RC39461.01-40

169. It can be inferred that intuition is important to the five writers primarily because it provides
them with
A. information useful for understanding abstract logic and scientific method
B. the discipline needed in the search for truth
C. inspiration for their best writing
D. clues to the interpretation of symbolic experience
E. the means of resolving conflicts between the self and the world
Inference
The final paragraph states that the five writers all held that intuition and imagination offer a
surer road to truth than does abstract logic or scientific method. Furthermore, the author
states that this is illustrated, in part, by their interpretation of experience as, in essence,
symbolic. This suggests that intuition's value is largely related to its ability to assist in
interpreting symbolic experience.
A. The passage suggests that the writers believed intuition to be a surer road to truth than
are abstract logic and the scientific method. Nevertheless, it does not indicate that these
writers believed intuition to be useful for understanding abstract logic or the scientific
method.
B. The passage suggests that the writers believed intuition to be helpful in arriving at the
truth. However, it does not indicate that this is because it provides them with any sort of
discipline needed in the search for the truth.
C. It may be true that intuition served as inspiration for the five writers' best writing.
However, the passage provides no information that allows us to make such an inference.
D. Correct. The passage suggests that these writers believed intuition to be important
because intuition (and imagination) assist in the interpretation of symbolic experience.
E. The passage nowhere indicates that intuition is involved in the resolution of conflicts
between the self and the world.
The correct answer is D.

RC39461.01-50

170. The author discusses “the democratic ethic” (see lines 26–32) in order to
A. explain the relationship between external experience and inner imagination
B. support the notion that the self contains two conflicting and irreconcilable factions
C. illustrate the relationship between the self's desire to be individual and its desire to merge
with all other selves
D. elaborate on the concept that the self constantly desires to realize its potential
E. give an example of the idea that, in order to be happy, the self must reconcile its desires
with external reality
Evaluation
The passage discusses the democratic ethic as a specific instance of two conflicting impulses:
the desire to be responsible to the self versus the desire to transcend the self and become one
with the world. In particular, the democratic ethic relates to the conflict between one's duty
to self and one's duty to society.
A. The passage's discussion of the democratic ethic relates to the conflict of the individual's
self-asserting and self-transcending impulses. However, the democratic ethic is brought
up to illustrate rather than explain this idea. Furthermore, the democratic ethic does not
deal specifically with inner imagination and external experience, but rather with
conflicting duties.
B. The passage nowhere indicates that the self has two irreconcilable factions. The
discussion of the democratic ethic is part of a discussion regarding the importance of the
harmonious reconciliation of two psychological tendencies.
C. Correct. As discussed above, the author's discussion of the democratic ethic is intended
to illustrate the relationship between one's duty to oneself and one's duty to be a part of
society; we can interpret the latter as one form of becoming one with the world.
D. The passage does not indicate that these five writers believed that the self constantly
desires to realize its own potential.
E. The passage does not indicate that these five writers believed that happiness requires a
reconciliation of the self's desires with external reality.
The correct answer is C.

RC39461.01-60

171. It can be inferred that the idea of “an organic relationship between the self and the cosmos”
(see lines 40–41) is necessary to the thinking of the five writers because such a relationship
A. enables them to assert the importance of the democratic ethic
B. justifies their concept of the freedom of the individual
C. sustains their faith in the existence of a deity
D. is the foundation of their humanistic view of existence
E. is the basis for their claim that the writer is a seer
Inference
The passage discusses the idea of an organic relationship between the self and the cosmos in
the context of discussion of the five writers' shared beliefs that the clue to external nature is
to be found in the inner world of individual psychology and that experience is, essentially,
symbolic.
The passage indicates that these views presume the idea that there is an organic relationship
between the self and the cosmos. This presumption of an organic relationship is, it seems, the
basis of their humanistic view of existence.
A. It is possible to assert the importance of the democratic ethic without holding to these
writers' view regarding an organic relationship between the self and the cosmos. In fact,
this view may have played a part in their promotion of the democratic ethic. Nevertheless,
the writers certainly could have asserted that this ethic was important without presuming
such a view.
B. The passage indicates tension between the idea of individual freedom and the individual's
duty to society, a view that these writers link to the organic relationship between the self
and the cosmos. It is therefore unclear specifically how this idea justifies these five
authors' conception of the freedom of the individual.
C. The passage indicates that these authors do not specifically deny the existence of a deity.
However, their belief in the humanistic view of existence stands in tension with a belief in
a deity. Therefore, it is unlikely that the idea of an organic relationship between the self
and the cosmos—a concept that underlies that humanistic view—would sustain whatever
faith they might have in the existence of a deity distinct from the universe itself.
D. Correct. As indicated above, these five writers' humanistic view of existence presupposes
an organic relationship between the self and the cosmos.
E. The passage indicates that the writers' faith in the imagination and in themselves
underlies their claim that the writer is a seer, not their idea that there is an organic
relationship between the self and the cosmos.
The correct answer is D.
RC39461.01-70

172. The passage is most relevant to which of the following areas of study?
A. Aesthetics and logic
B. History and literature
C. Theology and sociology
D. Anthropology and political science
E. Linguistics and art
Evaluation
The passage discusses five historically important literary figures; it follows that this passage
is most relevant to history and literature.
A. Because the passage discusses only thematic characteristics of these five writers, it has
little to do with aesthetics, that is, the study of beauty. Furthermore, it does not discuss
any general features of reasoning and argumentation, so it has little to do with the study
of logic.
B. Correct. Because the passage focuses on five historically important literary figures, it is
most relevant to the study of history and literature.
C. The passage does not focus on the study of religion or on the structure and functioning of
human society. Therefore, the passage has little to do with to either theology or sociology.
D. The passage does not focus on either human society and culture or on political activity
and behavior. Therefore, it has little to do with either anthropology or political science.
E. The passage does not engage in a study of language or the structure of language, so it has
little to do with linguistics. Furthermore, it is relevant to only one particular type of art,
literature, and so is of only limited relevance to the study of art in general.
The correct answer is B.

Questions 173–177 refer to the passage on page 151.

RC49461.01-10

173. The primary purpose of the passage is to


A. compare the information about organic function made available by historical explanation
with that made available by the experimental investigation of living organisms
B. assess the influence that theories of history had on developments in the field of biology in
the nineteenth century
C. discuss the importance of historical explanation in the thinking of nineteenth century
biologists
D. contrast biologists' use of historical explanation during the early nineteenth century with
its use during the final quarter of the nineteenth century
E. evaluate the way in which the concept of heredity altered the use of historical explanation
by nineteenth-century biologists
Main idea
This question requires that we understand the fundamental purpose of the passage, which is
a function of both its structure and content.
The first paragraph of the passage notes a turning point in the history of biology. In the late
nineteenth century, biologists made a shift away from historical explanation of biology.
However, as the passage continues, it addresses the importance of historical explanation in
biology throughout the nineteenth century.
A. The passage says little about the information about organic function that is made
available by the experimental investigation of living organisms.
B. The passage focuses on historical explanation in biology. It does not discuss different
theories of history. Therefore, it does not assess the influence of different theories of
history on the influence on developments in biology.
C. Correct. The primary purpose of the passage is to discuss the importance of historical
explanation in nineteenth-century biology.
D. The passage mentions the final quarter of the nineteenth century specifically to note the
turning point when biologists began to use experimental manipulation as a way to discern
the causes of vital processes. The passage does not contrast these biologists' use of
historical explanation during the final quarter of the nineteenth century with their use of
it earlier in the nineteenth century.
E. The final sentence of the passage suggests that the concept of heredity seemed to
complete the argument for a historical explanation of evolutionary events. However,
this is just one part of the passage's discussion of historical explanation in biology in the
nineteenth century.
The correct answer is C.
RC49461.01-20

174. According to information presented in the passage, which of the following is a true statement
about the methods of explanation used by biologists and historians in the nineteenth
century?
A. Neither biologists nor historians were able to develop methods of explanation that were
accepted by the majority of their colleagues.
B. The methods used by biologists to explain phenomena changed dramatically, whereas the
methods used by historians to explain events did not change as noticeably.
C. Biologists believed that they had refined the methods of explanation used by historians.
D. Biologists' and historians' methods of explaining what they believed to be comparable
phenomena were similar.
E. Although biologists and historians adopted similar methods of explanation, the biologists
were more apologetic about their use of these methods.
Supporting idea
The passage states The biologist-as-historian and the general historian of human events
dealt with comparable phenomena and assumed necessarily a common mode of
explanation. This indicates, in other words, that biologists' and historians' methods were
similar in explaining what they believed to be comparable phenomena.
A. The passage suggests that biologists through most of the nineteenth century tended to use
historical explanation. Presumably, it is not true that biologists were unable to develop
methods that were accepted by the majority of their colleagues.
B. The passage suggests that methods used by biologists changed significantly toward the
end of the nineteenth century. However, nothing in the passage indicates to what extent,
if at all, historians' methods changed.
C. The passage suggests that biologists believed that, in observing embryogenesis and cell
formation, there existed a clear justification for the utility of historical explanation in
biology. However, there is no indication that these biologists believed that they had
refined the historical methods used by historians.
D. Correct. The final sentence of the first paragraph indicates that the methods of
biologists and historians in the nineteenth century were similar.
E. The only evidence that biologists were apologetic about their methods in the nineteenth
century is that a turning point in their methods existed toward the end of the century.
This is of course only weak evidence that biologists were apologetic. And even if these
biologists were apologetic, nothing in the passage indicates whether historians were more
or less apologetic.
The correct answer is D.

RC49461.01-30

175. Which of the following best summarizes the “turning point” mentioned in line 2?
A. The beginning of the conflict between proponents of the ideal of historical explanation
and the proponents of experimentation
B. The substitution of historical explanation for causal explanation
C. The shift from interest in historical explanation to interest in experimentation
D. The attention suddenly paid to problems of organic function
E. The growth of public awareness of the controversies among biologists
Evaluation
The turning point mentioned in the passage refers to a shift among biologists during the last
quarter of the nineteenth century. Specifically, this shift was from favoring historical
explanation to a greater interest in experimentation.
A. The passage does not specify any sort of conflict between proponents of the ideal of
historical explanation and the proponents of experimentation. The passage only notes
that there was a shift in interest from historical explanation—applied deductively to
organic function—to experimentation.
B. The passage indicates, with some criticism, that historical explanation was seen as a type
of causal explanation.
C. Correct. As noted above, the turning point was a shift away from historical explanation
—the dominant thread in biology throughout most of the nineteenth century—toward
experimentation.
D. The passage suggests that, throughout the nineteenth century, biology had been
concerned with organic function: initially it explained organic function through historical
processes and then it began to explain organic function through experimentation.
E. The passage does not indicate whether public awareness of controversies among
biologists grew.
The correct answer is C.

RC49461.01-40

176. The author implies that nineteenth-century biologists who studied embryogenesis believed
that they
A. had discovered physical evidence that supported their use of historical explanation
B. were the first biologists to call for systematic experimentation on living organisms
C. were able to use historical explanation more systematically than were biologists who did
not study embryogenesis
D. had inadvertently discovered an important part of the factual record of the history of
living organisms on earth
E. had avoided the logical fallacies that characterize the reasoning of most nineteenth-
century biologists
Implication
The author indicates that nineteenth-century biologists used a historical explanation of
organic function. This use, they believed, was justified through observations of cell
generation and stages in embryogenesis.
A. Correct. The author suggests that nineteenth-century biologists who studied
embryogenesis believed that they had discovered physical evidence supporting the use of
historical explanation.
B. The passage does not imply that biologists who studied embryogenesis were the first to
call for systematic experimentation on living organisms.
C. The passage does not imply that biologists who studied embryogenesis were any more
successful in using historical explanation systematically than biologists who did not.
D. The passage states that the fossil record was incomplete, so it is likely that this means
many important things were missing. Any specific inadvertent discovery would have to be
mentioned directly in the passage. The passage does not suggest any sort of discovery.
E. The author, in the second paragraph, indicates that both those nineteenth-century
biologists who studied cell generation and those who studied embryology confused
temporal succession and causal explanation.
The correct answer is A.

RC49461.01-50

177. The passage would be most likely to appear in which of the following?
A. An essay investigating the methodology used by historians of human events
B. A book outlining the history of biology in the nineteenth century
C. A seminar paper on the development of embryogenesis as a field of study in nineteenth-
century biology
D. A review of a book whose topic is the discovery of fossils in the nineteenth century
E. A lecture whose subject is the limitations of experimental investigation in modern biology
Evaluation
The passage discusses a turning point in the late nineteenth century in the history of biology.
It then focuses primarily on the use of historical explanation in the field of biology during the
nineteenth century. Therefore, it seems that, among the five options here, the one this
passage would be most likely to appear in would be a book discussing the history of biology
in the nineteenth century.
A. The passage focuses on nineteenth-century biologists' use of historical explanation. It
presents no specific investigation of how historians used that methodology. Therefore, it
would not be of particular value in an essay investigating the methodology used by
historians of human events.
B. Correct. The passage focuses mainly on nineteenth-century biologists' use of historical
explanation. It also mentions a turning point in the history of biology. It references the
history of human events primarily as an analogy to help clarify the biologists' use of this
historical method. These facts about the passage suggest that, among the five options
here, the one the passage would be most likely to appear in would be a book outlining the
history of biology in the nineteenth century.
C. The passage spends only a portion of one paragraph saying anything about
embryogenesis. Even that portion is primarily offered only to give some explanation of
why nineteenth-century biologists were attracted to the use of historical explanation. It
does not seem that it would appropriate to use the entire passage in a seminar paper on
the topic of embryogenesis as a field of study in nineteenth-century biology.
D. There is no reference to a book whose topic is the discovery of fossils in the nineteenth
century. The passage only briefly mentions fossils. It seems unlikely that the passage
would be of any use to a review of such a book.
E. The passage says almost nothing about experimental investigation in modern biology,
and says absolutely nothing about its limitations.
The correct answer is B.
Questions 178–185 refer to the passage on page 153.
RC59461.01-10

178. The primary purpose of the passage is to


A. make a case for the importance of skillful psychological motivation in well-written novels
and romances
B. contrast the romantic and novelistic traditions and assert the aesthetic superiority of the
romantic tradition
C. survey some of the responses to Melville's fiction put forward by James and twentieth-
century literary critics
D. argue that the charges made against Melville's fiction by literary critics are suspect and
misleading
E. note several accusations made against Melville's fiction by literary critics and refute one
of these accusations
Main idea
Our goal is to find the primary purpose of the passage, which requires a firm
understanding of the structure of the passage and its objectives. The passage notes various
criticisms of Melville's fiction, but its main argument is to support the idea that Melville's
fiction is valid even though it does not follow Henry James's conception of the novel.
Furthermore, the passage defends Melville's fiction by arguing that Melville had an equally
valid conception of the purpose of fiction: one that differed fundamentally from that of
James. The passage indicates that Melville's strength does not derive from depictions of
character motivation. Rather, it lies in Melville establishing a strong moral situation;
occasionally this might be done at the expense of continuity or credibility.
A. The passage specifically refers to Melville's novels and does not suggest that well-written
novels or romances all share any particular characteristic.
B. The passage is specifically concerned with Melville's novels, not the general romantic or
novelistic traditions. Furthermore, the passage suggests that the romantic tradition in
fiction has its own literary validity, as has the novelistic tradition, but does not indicate
that one is superior to the other.
C. This passage does not simply give a survey of literary responses to Melville's fiction; the
passage goes further in arguing for the literary worth of Melville's work.
D. Because the passage argues against certain criticisms of Melville's work, it might be
argued that the author of the passage considers some criticisms of Melville “suspect.”
However, there is nothing in the passage to imply this applies to all criticisms of Melville's
work. Furthermore, there is absolutely nothing in the passage to indicate that these
critics' work is “misleading.”
E. Correct. The primary purpose of the passage is to counter one of several negative
evaluations of Melville's novels made by literary critics.
The correct answer is E.
RC59461.01-20
179. The author draws which of the following conclusions about the fact that Melville's fiction
often does not possess the qualities of a Jamesian novel?
A. Literary critics should no longer use Jamesian standards to judge the value of novels.
B. Literary critics who have praised Melville's fiction at the expense of James's fiction should
consider themselves justified.
C. Literary critics should no longer attempt to place writers, including Melville and James,
in traditions or categories.
D. Melville and James should be viewed as different sorts of writers and one should not be
regarded as inherently superior to the other.
E. Melville and James nevertheless share important similarities and these should not be
overlooked or slighted when literary critics point out differences between the two writers.
Supporting idea
Which among the answer choices is a conclusion drawn by the author of the passage
regarding the contrast between Melville's fiction and that of James? The second sentence of
the final paragraph states this: Melville is a different kind of writer, who held, and should be
judged by, presuppositions about fiction that are quite different from James's. In other
words, Melville held different standards regarding fiction and one needs to evaluate Melville
on these standards rather than on James's standards.
A. The passage suggests that Jamesian standards may be inappropriate for Melville's novels.
However, it does not suggest that Jamesian standards are necessarily invalid for judging
the value of other novels.
B. The passage mentions nothing about critics who have praised Melville's novels at the
expense of James's novels. In fact, the passage never mentions James's novels.
C. The passage does not suggest that critics should avoid categorization of writers. In fact,
the passage in lines 19–20 states that Melville is not a Jamesian novelist, which is in
itself such a categorization.
D. Correct. This accurately expresses a conclusion drawn by the author, namely that
Melville and James have valid, if different, approaches to fiction writing.
E. The author indicates no important similarities between Melville and James's writing. In
fact, the author emphasizes certain key differences between the two authors' work.
The correct answer is D.
RC59461.01-30

180. Which of the following would be the most appropriate title for the passage?
A. Melville's Unique Contribution to Romantic Fiction
B. Melville's Growing Reputation Among Twentieth-Century Literary Critics
C. Melville and the Jamesian Standards of Fiction: A Reexamination
D. Romantic and Novelistic: The Shared Assumptions of Two Traditions
E. The Art of Fiction: James's Influence on the Novelistic Tradition
Main idea
Given the content of the passage, which of these choices could most reasonably be used as a
title? The passage's main purpose is to counter the criticisms of those critics who describe
Melville's works of fiction as romances. These critics claim that Melville's works lack
significant literary value because they fail to satisfy James's criteria for literary worth in
novels, a standard that is widely accepted by literary critics. The passage argues that
Melville's novels would be more appropriately evaluated using the criteria that Melville
himself espoused; these criteria differ significantly from James's criteria.
A. This choice is inappropriate because nothing in the passage suggests that Melville's
approach was unique; that is, nothing in the passage indicates that Melville's contribution
is the only one of its kind.
B. This choice is inappropriate because the passage never states how Melville's literary
reputation among twentieth-century critics evolved.
C. Correct. This choice reflects the central idea of the passage that the literary worth of
Melville's fiction is not appropriately judged using the Jamesian standard; rather, it is
appropriately judged using Melville's own notion of the ultimate purpose of fiction.
D. Although the passage suggests that issues of morality figured in the fiction of both James
and Melville, the passage does not address any assumptions shared between James and
Melville.
E. Although the passage suggests that James had a significant influence on critical standards
for the novel, the passage centers on Melville's works rather than James' influence.
The correct answer is C.

RC59461.01-40

181. The author probably mentions Melville's Pierre to


A. refute those literary critics who have made generalizations about the quality of Melville's
fiction
B. argue that the portrayal of characters is one of Melville's more accomplished literary skills
C. give an example of a novel that was thought by James to resemble his own fiction
D. suggest that literary critics find few exceptions to what they believe is a characteristic of
Melville's fiction
E. reinforce the contention of literary critics
Evaluation
Why does the author of the passage mention Pierre? The final sentence of the first
paragraph reads: Critics argue that only Pierre (1852), because of its subject and characters,
is close to being a novel in the Jamesian sense.
This statement indicates that literary critics regard Pierre as the only possible exception to
their negative characterization of Melville's fiction as romance because Pierre has at least
some of the properties that James considered essential to novels of literary value.
A. This is not a purpose of mentioning Pierre. The passage does not cite any critics who
claim that most of Melville's novels satisfy Jamesian criteria.
B. This is not a purpose of mentioning Pierre. The author of the passage does not endorse
the view that character portrayal is a particular strength of Melville's novels. In fact, the
author suggests that Melville's novels tend to give questionable portrayals of what
characters feel or think.
C. The passage lacks any information indicating James's personal view of Pierre. Given the
information in the passage, it is entirely possible that James never even read Pierre.
D. Correct. The critics of Melville's novels generally characterize them as “romances” rather
than “novels” given that they lack certain properties Henry James regarded as essential to
novels. The mention of Pierre shows that the critics see it as the only one of Melville's
novels that might fit the Jamesian criteria.
E. In the sentence in which Pierre is mentioned, the author of the passage does not endorse
criticism suggesting that Melville's works are romances rather than novels. Furthermore,
the sentence is not intended to endorse a contention of any other type of literary critic.
The correct answer is D.
RC59461.01-50

182. Which of the following statements best describes the author's method of argumentation in
lines 24–31?
A. The author describes an important standard of evaluation used by critics of Melville and
then attacks that standard.
B. The author admits a contention put forward by critics of Melville but then makes a
countercharge against those critics.
C. The author describes a charge advanced by critics of Melville and then points out a logical
flaw in this charge.
D. The author provides evidence that seems to support a position held by critics of Melville
but then demonstrates that the evidence actually supports a diametrically opposed
position.
E. The author concedes an assertion made by critics of Melville but then mitigates the
weight of the assertion by means of an explanation.
Evaluation
Which answer choice best describes the reasoning in lines 24–32? The author suggests that
Melville's novels can reasonably be called “romances,” but also is careful to explain a sense of
this designation that still regards Melville's novels as valid works of literature.
A. The text in lines 24–31 does not describe any standard of evaluation used by Melville's
critics.
B. There is no countercharge against, or even mention of, Melville's critics in lines 24–31.
C. Again, there is no mention of criticisms of Melville's work in the lines 24–31; it follows
that the author does not point out a “logical flaw” in such criticisms within lines 24–31.
D. In lines 24–31, the author accepts that Melville's novels can be called “romances.” In lines
24–31, of course, the author presents no evidence to show that Melville's novels are not
romances.
E. Correct. In lines 24–31, the author concedes that Melville's novels are romances.
However, the author argues that this does not detract from the literary value of Melville's
work.
The correct answer is E.

RC59461.01-60

183. Which of the following can logically be inferred from the passage about the author's
application of the term “romance” to Melville's work?
A. The author uses the term in a broader way than did Melville himself.
B. The author uses the term in a different way than do many literary critics.
C. The author uses the term in a more systematic way than did James.
D. The author's use of the term is the same as the term's usual meaning for twentieth-
century commentators.
E. The author's use of the term is less controversial than is the use of the term “novel” by
many commentators.
Inference
Which of the following conclusions regarding the term “romance” as applied to Melville's
work is most strongly supported by the information in the passage? The author of the
passage uses the term without the negative connotation that the passage suggests the term
carried for many twentieth-century critics of Melville.
A. There is no information in the passage regarding how, or even whether, Melville used the
term “romance.”
B. Correct. The author of the passage will call Melville's novels romances provided this
term is used without the disparaging connotations of this term (particularly as used by
many critics of Melville's work).
C. Nothing in the passage specifies James's use of the term “romance.”
D. The author of the passage specifically applies the term “romance” in a way that avoids the
disparaging connotations of escapism present in the use of the term by some critics of
Melville's work.
E. The passage provides no information by which we can gauge how controversial the
author's use of “romance” as applied to Melville's works is, relative to other
commentators' use of the term “novel.”
The correct answer is B.

RC59461.01-70

184. Which of the following can most logically be inferred about the author's estimation of the
romantic and novelistic traditions of fiction?
A. The romantic tradition should be considered at least as valuable as the novelistic tradition
in the examination of human experience.
B. The romantic tradition should be considered the more vital tradition primarily because
Melville is part of that tradition.
C. The romantic tradition should be considered the superior tradition because it is so
widespread.
D. The romantic tradition has had as much success in pleasing literary critics as has the
novelistic tradition.
E. The romantic and novelistic traditions have always made important contributions to
literature, but their most important contributions have been in the twentieth century.
Inference
We must determine which statement regarding the author's evaluation of the romantic and
novelistic traditions of fiction is most strongly supported by the information in the passage.
The passage states: although Melville is not a Jamesian novelist, he is not therefore a
deficient writer. The author of the passage tells us that Melville sought to explore moral
questions, an exploration that Melville assumed to be the ultimate purpose of fiction. These
statements indicate that the author of the passage regards the romantic tradition's fictional
examination of human experience as at least equal in value to the novelistic tradition's
examination of it.
A. Correct. The passage suggests that both traditions are concerned with the examination
of human experience, yet their approaches to this examination differ. The passage argues
that Melville's body of fiction is no less valuable as literature than James's.
B. The passage provides nothing to indicate that Melville's position as a romantic writer
therefore implies that romance is “more vital” than the novelistic tradition. Note that such
a comparison would need to be clearly made for this to be a valid answer choice.
C. The passage provides no information regarding how widespread the romantic tradition is.
D. The passage provides no information to suggest that literary critics are more pleased by
romantic works than they are by novelistic works. Note that such a comparison would
need to be clearly made for this to be a valid answer choice.
E. The passage provides no information to suggest that the most important contributions of
the romantic and novelistic traditions have been during the twentieth century. In fact,
given that Melville and James both worked in the nineteenth century, it seems doubtful
that the passage would make such a claim.
The correct answer is A.

RC59461.01-80

185. The author of the passage would be most likely to agree that a writer's fiction should be
evaluated by which of the following criteria?
A. How consistently that fiction establishes credibility with the reader
B. How skillfully that fiction supersedes the presuppositions or conventions of a tradition
C. How completely that fiction satisfies the standards of judgment held by most literary
critics
D. How well that fiction fulfills the premises about fiction maintained by the writer of the
fiction
E. How well that fiction exhibits a continuity of subject and style over the course of the
writer's career
Application
Which criteria would the author of the passage most likely agree to be useful for evaluating
a writer's fiction? The passage argues that even though Melville's novels fail to satisfy Henry
James's criteria for literary value in a novel, they still have a different kind of literary value.
In particular, they match the criteria set out by Melville's own conception of fiction.
The author states that Melville sought in his fiction to explore moral questions, an
exploration he assumed was the ultimate purpose of fiction. Therefore, the author would
likely agree that the literary value of a writer's fiction is determined by the degree to which
the writer's fiction fulfills the writer's own conception of what fiction should accomplish.
A. The author of the passage would be unlikely to agree that fiction must establish credibility
with the reader. In the passage, the author suggests that Melville was prepared to sacrifice
some credibility in his fiction if doing so would help him to establish a significant moral
situation.
B. Nothing in the passage suggests that its author would agree with the idea that fiction
must “supersede presuppositions or conventions of a tradition.” While the author
indicates that Melville's novels do not satisfy James's criteria for a good literary novel,
this does not indicate that Melville's work supersedes or replaces any tradition.
C. The author of the passage clearly rejects this criterion and actually argues against the
standards of many critics. Note the author's position that criticisms based on widespread
acceptance by literary critics of James's standards for literary novels are not necessarily
valid for all fiction.
D. Correct. The author of the passage argues that Melville's fiction must be judged by
reference to Melville's own criteria rather than by those of James or the critics who
accepted James's criteria.
E. Nowhere does the passage mention continuity of style over a career. Therefore, nothing in
the passage suggests that the author would accept it.
The correct answer is D.

Questions 186–193 refer to the passage on page 156.


RC69461.01-10

186. According to the passage, book review editors pay attention to all of the following in deciding
which books should be reviewed in their publications EXCEPT
A. news releases from publishers
B. sales figures compiled by bookstores
C. the opinions of literary experts
D. the probability that the books will be extensively advertised
E. the likelihood that the books will be reviewed in other publications
Supporting idea
Which of the following is a factor that the passage does NOT indicate is considered by book
review editors when deciding which book reviews to publish? The passage indicates that
major decisions regarding which books will be reviewed in a given publication occur before
the relevant books have been published; it follows that sales data for those books would not
exist when initial decision making occurs.
This immediately suggests that answer choice B may be correct. But to verify that answer
choice B is correct, it will be important to quickly check two things: first, that the passage
neither states nor implies review editors consider sales data and, second, that for each of the
other factors listed, the passage either states or implies that review editors do consider that
factor.
A. The passage indicates that book review editors do in fact consider news releases from
publishers.
B. Correct. Nothing in the passage indicates that book review editors consider sales figures
compiled by bookstores. Furthermore, the passage indicates that such data would NOT be
available for certain books that the editors and book publishers consider most worth
publishing. Even so, the passage does not exclude the possibility that book review editors
may consider publishing reviews of books already in bookstores, based on a review of
bookstore sales data. However, it is important to note that the passage neither states nor
implies that this is so.
C. According to the passage, book review editors do in fact listen to literary experts
regarding books that may sell well over time despite slow initial sales.
D. According to the passage, book review editors receive news releases and printers' proofs
of certain books; this implies that the books will in fact be heavily advertised.
E. The passage states that book review editors must consider what will be newsworthy,
advertised, and written about elsewhere. In other words, other publications' writing
about the same books factor into the decisions of these editors.
The correct answer is B.
RC69461.01-20

187. The main idea of the second paragraph is that


A. decisions made by book review editors are influenced by the business of selling books
B. book review editors must be familiar with all aspects of the book trade
C. advertising is the most important factor influencing book sales
D. book reviews usually have no influence on what books are ordered in quantity by stores
E. publishers deliberately try to influence the decisions of book review editors
Evaluation
What idea is most central to the second paragraph? A good strategy here is to identify the
two most plausible answer candidates by quickly eliminating the three least plausible.
Looking through the options, answer choices B, C, and D can be reasonably easily eliminated,
leaving the remaining two options as the most plausible candidates. Note that the paragraph
focuses on the idea that review editors' decisions are influenced by the business of selling
books (answer choice A).
A. Correct. As indicated, this idea is the main theme of the paragraph.
B. The paragraph is focused on how editors' decisions are influenced by the business of
selling books, not on how much knowledge book review editors must have.
C. Although the paragraph suggests that advertising may significantly influence book orders
by bookstores, the paragraph does not indicate that advertising is the most important
factor.
D. The paragraph does not indicate that book reviews usually have no influence on book
orders by bookstores.
E. The paragraph does not indicate that publishers' pre-publication outreach to review
editors is a deliberate effort to influence the editors' decisions.
The correct answer is A.

RC69461.01-30

188. According to the passage, a major concern of the unenthusiastic book reviewers mentioned
in line 33 was to
A. ensure prompt payment for their work
B. influence public opinion of books
C. confirm the opinions of other reviewers
D. promote new books by their favorite authors
E. have their reviews published in the newspaper
Supporting idea
What does the passage suggest was a major concern of the unenthusiastic book reviewers
mentioned? The necessary information is found in the final sentence of the third paragraph.
Given that the reviewers were paid promptly anyway, it follows that prompt payment
would not be a major concern for them. This sentence goes on to suggest that having their
material published in the newspaper was of concern, specifically implying that this concern
may have induced some of them to provide more favorable reviews in order to ensure
publication. While this is not directly stated, this line of reasoning is confirmed by the
information that the review editor's brilliant system tended to publish only positive reviews.
A. The passage suggests that the practice was to promptly pay all reviewers—even those
whose reviews were not published.
B. Nothing in the passage suggests that influencing public opinion was part of the agenda of
any of the reviewers, even those occasionally writing unenthusiastic reviews.
C. Nothing in the passage suggests that writers of unenthusiastic book reviews were
necessarily influenced by the opinions of other reviewers.
D. Nothing in the passage suggests that unenthusiastic book reviewers attempted to promote
any book, let alone ones from their favorite authors.
E. Correct. The final sentence of the third paragraph indicates that this may have been a
major concern of the unenthusiastic reviewers.
The correct answer is E.

RC69461.01-40

189. The passage provides information to answer which of the following questions?
A. Would most book review editors prefer to have books reviewed without regard to the
probable commercial success of the books?
B. Are publishers' efforts to persuade bookstores to order certain books generally effective?
C. On what basis do literary experts decide that a book is or is not important?
D. What criteria are used to determine the length of a particular book review?
E. Have book review practices in general changed significantly since a generation ago?
Inference
Which one of the five questions can be answered based on the information given in the
passage? This implies that four of the answer choices will be questions that we cannot
answer based on the passage. Certain among these choices are easily eliminated: the question
regarding literary experts' evaluation and the question regarding criteria for length of
reviews.
The question of whether book review practices in general have changed over the last
generation is also fairly easily eliminated: the information about such practices a generation
ago only concerns the system used by one review editor in one newspaper.
Thus we can narrow down our answer candidates to the following: the question about how
expected commercial success relates to review editors' preferences, and the question about
whether book publishers' outreach efforts for certain books succeed in persuading bookstores
to order large quantities of those books. Which one of these two questions does the passage
help answer?
A. The passage provides neither a definitive “yes” nor a definitive “no” answer to this
question, at least as concerns “most” book editors. The second paragraph of the passage
indicates that expectations regarding the books that bookstores will order in quantity
does figure prominently in decision making regarding publication of reviews. However,
this does not necessarily indicate what most editors would prefer. It is possible, if not
likely, that book review editors differ widely in their preferences. This does not of course
answer whether book review editors listen to their personal preferences when making
decisions about publication of book reviews. Ultimately, we do not have the information
to address either this question or its nuances.
B. Correct. The passage tells us that the books that publishers heavily advertise will
probably be among the books that most bookstores order in quantity. This provides a
“yes” answer to the question.
C. The passage provides no information regarding how literary experts decide whether a
book is “important.” Note that the passage specifically discusses book review editors. It is
unclear whether this is the same thing as “literary expert,” but we are given no reason to
think so.
D. The passage provides no information regarding how the length of a book review is
determined.
E. The passage indicates that a generation ago, a review editor for a newspaper had a
brilliant system of publishing only the reviews that were enthusiastic about particular
books. However, the passage does not indicate that this was common editorial practice at
the time. Rather, the passage tells us that most editors currently publish both positive and
negative reviews. Nothing in the passage indicates that this practice was different a
generation ago.
The correct answer is B.

RC69461.01-50

190. The passage suggests which of the following about book review readers?
A. They pay careful attention to reviewers' biases as they read reviews.
B. They disapprove of book review editors who try to influence what their reviewers write.
C. They use book reviews in order to gauge whether a book is likely to endure.
D. They expect to see timely reviews of widely publicized books in the periodicals they read.
E. They are usually willing to search in several stores for a highly recommended book that is
hard to find.
Inference
What does the passage convey about readers of book reviews? Note that the question does
not concern what is most likely true of at least some book review readers; rather, the passage
most strongly suggests what is true of book review readers in general.
Nothing in the passage implies that any of the first three answer choices given is true of book
review readers in general. This leaves the final two answer choices as possible candidates. Of
the two, D more clearly describes something that the passage suggests is generally true of
book review readers.
A. No doubt some book review readers pay attention to reviewers' biases, but the passage
does not imply that this is generally true of readers of book reviews.
B. The passage does not tell us that readers of book reviews generally know whether review
content is influenced by book review editors. Furthermore, it does not tell us whether
these readers generally approve or disapprove of such editors.
C. The passage nowhere suggests that readers of book reviews generally make judgments as
to whether a book is likely to be a long-lasting cultural legacy.
D. Correct. The passage indicates that book review editors in general feel responsible to
meet their readers' expectations by providing reviews of books that are newsworthy,
advertised, and written about elsewhere.
E. The passage nowhere suggests that most book review readers will search in several stores
for a highly recommended book. The passage refers to books that readers will have
trouble finding in stores; however, this is more plausibly read as readers of books rather
than readers of book reviews. Furthermore, it does not suggest that most readers—or
most readers of book reviews—will go to several stores to search for a book.
The correct answer is D.

RC69461.01-60

191. Which of the following words, if substituted for “brilliant” in line 26, would LEAST change
the meaning of the sentence?
A. showy
B. articulate
C. literate
D. stingy
E. absurd
Evaluation
Which word, if substituted for “brilliant” in the sentence, would least alter the meaning of
the sentence? In other words, we must pick the word we could substitute for brilliant in the
sentence without significantly changing the meaning of the sentence.
Note that neither articulate nor literate would be an apt modifier of the word system. Stingy
indicates a financial use of the system, which seems inappropriate.
Eliminating these options leaves only showy and absurd as candidates. Given these choices,
it is fairly clear that brilliant is meant ironically. We are told that the unenthusiastic
reviewers quickly learned that if they wanted their material to be printed, it was advisable
to be kind.
A. Nothing in the passage conveys that the system, ironically designated brilliant, was
showy, that is, eye-catching.
B. Articulate could describe fluent or coherent speech, description, or writing, but it does
not appropriately modify the noun system in this context.
C. The adjective literate does not appropriately modify system; furthermore, it fails to
convey a clear meaning that would match the ironic meaning of the modifier brilliant.
D. The system does not seem to have a monetary purpose, so stingy would not fit. Note that
the final sentence of the paragraph suggests that the system self-selects for books that are
likely to be successful, potentially at the expense of writers who write “unenthusiastic
reviews.” Nevertheless, the passage notes that the unpublished writers would still be paid,
so their risk lies in not having their work read rather than in not being paid for their work.
E. Correct. Among the answer choices, absurd best captures the ironic use of brilliant and
preserves the meaning of the sentence as a whole.
The correct answer is E.
RC69461.01-70

192. Which of the following is an assumption made by the book review editor referred to in line
27?
A. A book of great worth will receive only good reviews.
B. An important book will endure despite possible bad reviews.
C. Reviewers might hide their true opinions in order to have their reviews published.
D. Book review editors should select reviewers whose opinions can be guessed in advance.
E. Book review editors have an obligation to print extensive reviews of apparently important
books.
Evaluation
Which one of the five options is an assumption the book review editor made in following the
book review system described? The editor's policy was to publish the reviews that gave books
a sufficiently positive evaluation while often refusing to publish unenthusiastic reviews; that
is, reviews that did not rate books highly.
Given the information in the passage, it is likely that this editor followed other editors in
feeling some concern about what might endure. That is, these editors would presumably not
wish to risk ignoring a book of great worth. Therefore, this editor may have believed that his
brilliant system of not publishing negative reviews would guard against such a risk. In other
words, the editor assumed that unenthusiastic reviews accurately reflected the value of the
books reviewed. By this logic, a book of great worth would, of course, receive only positive
reviews.
A. Correct. The editor, in applying the system described, assumed that a book of great
worth would receive only positive reviews.
B. If the editor assumed this, he would be justified in publishing bad reviews given that a
good book would eventually be vindicated.
C. There is nothing to suggest that the editor thought the reviewers to be dishonest in
representing their views of the books they review; that is, that the reviewers would lie in
order to get their reviews published. While this may have been the case, the passage
provides no evidence to suggest that the editor assumed this.
D. Nothing in the passage suggests that the editor's practice was governed by this sort of rule
regarding how book review editors should select reviewers.
E. The passage suggests the book review editor wanted books of significant worth reviewed
in his newspaper. However, it does not suggest that the editor assumed this rule should
apply to all book review editors.
The correct answer is A.

RC69461.01-80

193. It can be inferred that, as a prospective book buyer, the author of the passage would generally
respond to highly enthusiastic reviews of new books with
A. resignation
B. amusement
C. condemnation
D. skepticism
E. disinterest
Inference
What does the passage suggest about how its author might respond to highly enthusiastic
reviews of new books? Nothing in the passage suggests the response would be resignation,
amusement, or condemnation.
This narrows down the possibilities to skepticism or disinterest. The intended meaning of
disinterest is “lack of interest.” It is unlikely that the author of the passage would be generally
uninterested in highly enthusiastic reviews of new books.
This of course leaves skepticism as the best candidate. The passage does suggest that this
might be the passage author's attitude: the passage discusses the impact of commercial
considerations—the business of selling books—on review editors' choices regarding which
books to review. It follows that highly enthusiastic reviews may, at least in some cases, be
influenced by such considerations. With this in mind, the author of the passage is likely to
approach such reviews with skepticism. In other words, the author holds a certain amount of
doubt as to whether such reviews accurately reflect the true worth of the books reviewed.
A. Given the critical approach to reviewing practices conveyed in the passage, it is unlikely
that the response of the author would be resignation.
B. The author of the passage might, for various reasons, be amused at some highly
enthusiastic reviews. Nevertheless, the passage does not suggest that amusement would
be the author's response to any highly enthusiastic review.
C. Nothing in the passage suggests that the author's general response to enthusiastic reviews
would be condemnation.
D. Correct. As explained above, skepticism is most likely to be the author's most usual
response.
E. Some book reviews, even highly enthusiastic ones, might not interest the author of the
passage. However, nothing in the passage indicates that the author's general response
would be to show no interest.
The correct answer is D.

Questions 194–200 refer to the passage on page 158.

RC79461.01-10

194. The primary purpose of the passage is to


A. compare and criticize two theories of tragedy
B. develop a new theory of tragedy
C. summarize the thematic content of tragedy
D. reject one theory of tragedy and offer another theory in its place
E. distinguish between tragedy and irony
Main idea
What is the primary purpose of the passage? The hypothesis that the primary purpose of the
passage is to compare and criticize two theories of tragedy is attractive. After all, the
substance of the passage consists mainly of a discussion of the two theories.
To confirm that answer choice A is the best choice, however, we need to quickly review the
other options. Does the passage develop a new theory of tragedy? No. Does it summarize the
thematic content of tragedy? This would be clear if so; it is not clear. Does it offer a theory of
tragedy to replace a theory it rejects? Definitely not. Does it distinguish between tragedy and
irony? This is discussed, but only in a manner incidental to the main idea of the passage.
Given the unsuitability of the other answer choices, we are left with answer choice A: that the
main purpose of the passage is to compare and criticize two theories of tragedy.
A. Correct. This best describes the main purpose of the passage, based on the analysis
above.
B. The passage does not develop a new theory of tragedy.
C. While the passage summarizes the thematic content of tragedy in a very general way, it
does so only as a means to providing a critical analysis of the two theories of tragedy
discussed.
D. The passage does not advance a new theory of tragedy; it simply considers two existing
theories.
E. The passage distinguishes between tragedy and irony; it does so only in service of the
larger analysis that is the main focus of the passage.
The correct answer is A.

RC79461.01-20

195. The author states that the theories discussed in the passage “represent extreme views” (see
line 6) because their conclusions are
A. unpopular
B. complex
C. paradoxical
D. contradictory
E. imaginative
Supporting idea
What reason does the author of the passage state for the claim that the two theories of
tragedy discussed represent extreme views?
A careful reading of the first few sentences of the passage provides a quick answer to this
question. In the second sentence of the passage, the author states that the two theories
represent extreme views because their conclusions are contradictory.
It may be that the author has other reasons for this conclusion. However, no other such
reasons are stated.
A. The author neither explicitly nor implicitly characterizes either of the theories as
unpopular.
B. The author refers to the complexity of the tragic process, but this statement is not used to
complain that the theories are extreme.
C. The author nowhere states that either of the theories—or their juxtaposition—is
paradoxical.
D. Correct. The author presents this as reason for concluding that the theories are extreme.
E. The author nowhere refers to imaginativeness as a reason for the claim that the theories
are extreme.
The correct answer is D.

RC79461.01-30

196. The author objects to the theory that all tragedy exhibits the workings of external fate
primarily because
A. fate in tragedies is not as important a cause of action as is the violation of a moral law
B. fate in tragedies does not appear to be something that is external to the tragic hero until
after the tragic process has begun
C. the theory is based solely on an understanding of ancient Greek tragedy
D. the theory does not seem to be a plausible explanation of tragedy's ability to exhilarate an
audience
E. the theory does not seem applicable to the large number of tragedies in which the hero
overcomes fate
Supporting idea
What is the author's main reason for objecting to the theory that all tragedy exhibits the
workings of external fate?
This view is attributed, in the passage's third sentence, to the first of the two theories
considered. The author characterizes this view as an oversimplification, primarily because it
confuses the tragic condition with the tragic process. In other words, the tragic hero's
relationship with fate changes as the tragic process continues: fate is used to balance the
tragic hero's life, and becomes an external condition as the hero's life becomes unbalanced.
The author elaborates that in ancient Greek tragedy fate normally becomes external to the
hero only after the tragic process has begun. Therefore, the correct answer will likely
mention either the tragic “process” or the tragic “condition.”
A. This option mentions neither the tragic condition nor the tragic process.
B. Correct. This option mentions the tragic process and accurately captures the reason
presented by the author to support the objection raised.
C. The author invokes ancient Greek tragedy to illustrate and support the objection raised; it
is not invoked to indicate a flaw in the theory objected to.
D. The author does not cite tragedy's ability to exhilarate an audience as a primary reason
for the objection raised.
E. The author does not cite the large number of tragedies in which the hero overcomes fate
as the primary reason for the objection raised.
The correct answer is B.
RC79461.01-40

197. Which of the following comparisons of the tragic with the ironic hero is best supported by
information contained in the passage?
A. A tragic hero's fate is an external condition, but an ironic hero's fate is an internal one.
B. A tragic hero must be controlled by fate, but an ironic hero cannot be.
C. A tragic hero's moral flaw surprises the audience, but an ironic hero's sin does not.
D. A tragic hero and an ironic hero cannot both be virtuous figures in the same tragedy.
E. A tragic hero is usually extraordinary, but an ironic hero may be cowardly or even
villainous.
Inference
Which one of the comparisons between the tragic hero and the ironic hero is most strongly
supported by the information in the passage?
To eliminate some of the choices, note the following: first, the passage indicates that the
tragic hero's fate is initially internal, but the passage does not apply this to the ironic hero's
fate. Second, the passage does not suggest that an ironic hero cannot be controlled by fate.
Third, the passage does not attribute a sin to the ironic hero, yet it does indicate that the
character of the ironic hero tends to be ignoble.
In any case, the passage is silent as to whether a tragedy can feature two heroes, one tragic
and the other ironic. Finally, the passage implies that tragedy, unlike irony, needs an
exceptional central figure. Furthermore, the passage suggests the following about an ironic
hero: the more ignoble the hero the sharper the irony.
A. The passage indicates that the tragic hero's fate is initially internal but becomes external
as part of the tragic process. The passage does consider whether an ironic hero can be
controlled by fate but also does not exclude that possibility. Therefore, the passage does
not suggest that externalized fate is a factor that distinguishes the tragic hero from the
ironic hero.
B. Nothing in the passage indicates that an ironic hero cannot be controlled by fate.
C. The passage indicates that the character of the ironic hero tends to be ignoble. However,
it does not imply that a sin by the ironic hero is essential to the development of the irony.
D. The passage does not address whether a tragedy can feature two heroes, one of whom is
tragic and one of whom is ironic.
E. Correct. Among the choices provided, this comparison is best supported: tragedy
requires an exceptional central figure, while for irony, the more ignoble the hero the
better.
The correct answer is E.

RC79461.01-50

198. The author contrasts an honest person and a criminal (see lines 19–21) primarily to
A. prove that fate cannot be external to the tragic hero
B. establish a criterion that allows a distinction to be made between irony and tragedy
C. develop the distinction between the tragic condition and the tragic process
D. introduce the concept of sin as the cause of tragic action
E. argue that the theme of omnipotent external fate is shared by comedy and tragedy
Evaluation
For what reason does the author draw a contrast between an honest person and a
criminal? The contrast is presented during the discussion of the first of the two theories of
tragedy; in particular, it is introduced to question whether fate is necessarily external in the
tragic hero. The author suggests that fate, as conceived in ancient Greek tragedy, is initially
the internal balancing condition of life. However, fate becomes external once the tragic
process is unleashed. The tragic process begins when the theory violates this internal
balance, ultimately leading to the tragic condition.
From this perspective, fate is both internal and external during the tragic process.
Attributing sin to the tragic hero pertains only to the discussion of the second theory of
tragedy. Note that no reference to comedy occurs in the context of the contrast drawn
between an honest person and a criminal.
A. The passage suggests that fate can be external as well as internal in ancient Greek
tragedy.
B. The distinction between tragedy and irony is offered as a critique of the first theory of
tragedy; it is not presented as derived from the preceding discussion about fate.
C. Correct. As explained above, the contrast between the internal and external forms of fate
is presented to distinguish between the tragic process itself and the tragic condition that
is the outcome of the tragic process.
D. The attribution of sin to the tragic hero figures only in the discussion of the second theory
of tragedy; therefore it is not associated with the mentioned contrast.
E. While the passage briefly mentions comedy, it is not in association with the contrast
mentioned.
The correct answer is C.

RC79461.01-60

199. The author suggests that the tragic hero's “original destiny never quite fades out of the
tragedy” (see lines 29–30) primarily to
A. qualify the assertion that the theory of tragedy as a display of external fate is inconsistent
B. introduce the discussion of the theory that tragedy is the consequence of individual sin
C. refute the theory that the tragic process is more important than the tragic condition
D. support the claim that heroism creates the splendor and exhilaration of tragedy
E. distinguish between fate as conceived in ancient Greek tragedy and fate in more recent
tragedy
Evaluation
Which one of the five answer choices best describes the primary purpose of the author's
claim that the glory of the tragic hero's “original destiny never quite fades out of the
tragedy”? In the sentence immediately preceding this claim, we read: It is heroism that
creates the splendor and exhilaration that is unique to tragedy.
Tragedy persistently reminds us of the extraordinary destiny that could have been attained
by the hero. This reminds the audience that this glorious destiny has been tragically lost. The
final sentence of the first paragraph, therefore, serves to support the claim in the sentence
that precedes it.
A. Nothing suggests that the author regards this claim as lessening the flaw that the author
sees in the theory of tragedy first discussed.
B. This claim does not introduce the discussion of the second theory in the sense of creating
a meaningful transition to it.
C. Nowhere does the passage address a theory that the tragic process is more important than
the tragic condition; the author suggests that both are inherent in tragedy.
D. Correct. The context indicates that the author's suggestion is presented to support the
claim expressed in the sentence that precedes it.
E. Nowhere does the passage mention a distinction between ancient Greek tragedy and
more recent tragedy.
The correct answer is D.

RC79461.01-70

200. In the author's opinion, an act of humility in comedy is most analogous to


A. a catastrophe in tragedy
B. an ironic action in tragedy
C. a tragic hero's pride and passion
D. a tragic hero's aversion to sin
E. a tragic hero's pursuit of an unusual destiny
Inference
What would the author regard as most analogous to an act of humility in comedy? The
author writes that a proud and passionate mind is the precipitating agent of catastrophe,
just as in comedy the cause of the happy ending is usually some act of humility. In other
words, in tragedy, the hero's hubris leads to his or her downfall.
A. A catastrophe is an external event rather than the quality of a character, whereas both
hubris and humility are qualities of human characters.
B. Nowhere does the passage associate an ironic action in tragedy with an act of humility.
C. Correct. The author compares how hubris leads to catastrophe in tragedy with how an
act of humility leads to a happy ending in comedy.
D. The author refers to how the second theory associates the tragic hero with sin. However,
the author makes no reference to the tragic hero's aversion to sin as analogous to an act of
humility in comedy.
E. The author does not propose any similarity between the extraordinary aspiration of the
tragic hero—that is, the hero's unusual destiny—and an act of humility in comedy.
The correct answer is C.
Answer Explanations Verbal Reasoning
Critical Reasoning
Analysis/Critique
CR31410.01

201. Most of Western music since the Renaissance has been based on a seven-note scale known as
the diatonic scale, but when did the scale originate? A fragment of a bone flute excavated at a
Neanderthal campsite has four holes, which are spaced in exactly the right way for playing
the third through sixth notes of a diatonic scale. The entire flute must surely have had
more holes, and the flute was made from a bone that was long enough for these additional
holes to have allowed a complete diatonic scale to be played. Therefore, the Neanderthals
who made the flute probably used a diatonic musical scale.
In the argument given, the two portions in boldface play which of the following roles?
A. The first is presented as evidence that is confirmed by data presented elsewhere in the
argument given; the second states a hypothesis that this evidence is used to undermine.
B. The first is an opinion, for which no supporting evidence is presented in the argument
given, that is used to support the main conclusion of the argument; the second is that
main conclusion.
C. The first describes a discovery as undermining the position against which the argument is
directed; the second states the main conclusion of the argument.
D. The first is a preliminary conclusion drawn on the basis of evidence presented elsewhere
in the argument given; the second is the main conclusion that this preliminary conclusion
supports.
E. The first provides evidence to support the main conclusion of the argument; the second
states a subsidiary conclusion that is drawn in order to support the main conclusion
stated earlier in the argument.
Argument Construction
To determine what roles the two portions in boldface play, it is useful to look first for certain
“inference indicator” words: words that indicate that what follows is a premise (words like
because and since) or a conclusion (words like thus and therefore).
Here, there is only one, therefore; it immediately precedes the second boldfaced portion.
This indicates that that portion is a conclusion. Because of this, we can effectively rule out
answer choice A.
However, we must investigate the rest of the argument to determine whether this is the main
conclusion. If it is, we can rule out answer choice E as well. Alternatively, it may be an
intermediate conclusion, in which case E would be the correct answer.
To determine which sort of conclusion it is, ask whether this conclusion is used in support of
another claim in the argument. This conclusion is not, which makes it the main conclusion.
This rules out answer choice E.
To make a correct choice among options B, C, and D, we must determine the role of the first
highlighted portion.
A. This choice is incorrect because the second boldfaced portion is a conclusion drawn in the
argument. It is not, of course, a hypothesis that the first boldfaced portion is used to
undermine. Furthermore, the argument presents no data to confirm the first boldfaced
portion.
B. Correct. The first boldfaced portion is not a conclusion; it is merely an assertion that is
not supported by any claims presented in the argument. This portion, along with the
statement immediately following it, are offered in support of the second boldfaced
portion. This second boldfaced portion is the argument's main conclusion.
C. The first boldfaced portion does not undermine a position that the argument is directed
against. In fact, the argument is not explicitly directed against any position. Note that the
argument is rhetorically positive, arguing for a specific position rather than against one.
D. The argument provides no evidence in support of the first boldfaced position.
E. As noted above, this cannot be the correct answer because the second boldfaced portion is
in fact the main conclusion of the argument.
The correct answer is B.
CR53140.01

202. In a certain rural area, people normally dispose of household garbage by burning it. Burning
household garbage releases toxic chemicals known as dioxins. New conservation regulations
will require a major reduction in packaging—specifically, paper and cardboard packaging—
for products sold in the area. Since such packaging materials contain dioxins, one result of
the implementation of the new regulations will surely be a reduction in dioxin pollution in
the area.
Which of the following, if true, most seriously weakens the argument?
A. Garbage containing large quantities of paper and cardboard can easily burn hot enough
for some portion of the dioxins that it contains to be destroyed.
B. Packaging materials typically make up only a small proportion of the weight of household
garbage, but a relatively large proportion of its volume.
C. Per-capita sales of products sold in paper and cardboard packaging are lower in rural
areas than in urban areas.
D. The new conservation regulations were motivated by a need to cut down on the
consumption of paper products in order to bring the harvesting of timber into a healthier
balance with its regrowth.
E. It is not known whether the dioxins released by the burning of household garbage have
been the cause of any serious health problems.
Argument Evaluation
This question requires us to identify a claim that seriously weakens the argument that new
conservation regulations that require a major reduction in paper and cardboard packaging
will reduce dioxin pollution in a certain rural area.
Dioxins are released when household garbage is burned. It seems reasonable to think that
reducing packaging material that contains dioxins would help reduce dioxin pollution.
Nevertheless, suppose for some reason burning large amounts of paper and cardboard
containing dioxins actually—however counterintuitively—leads to a reduction in the amount
of dioxins that pollute the environment. This would indicate a major weakness in the
argument.
A. Correct. This claim tells us that garbage containing large quantities of paper and
cardboard burns at such a high temperature that a portion of the dioxins in the garbage is
destroyed. If so, then reducing quantities of paper and cardboard in burned garbage
might in fact increase dioxin pollution, not reduce it, despite the fact that paper and
cardboard packaging contains dioxins.
B. This choice does not weaken the argument: the amount of dioxin pollution could still be
reduced by reducing the amount of dioxin-containing packaging.
C. This choice is not relevant to the question. Even if per-capita sales of products sold in
paper and cardboard packaging are relatively low in the area in question, it could still be
the case that the amount of dioxin pollution in the area would be reduced if the amount of
cardboard and paper packaging was reduced.
D. This choice provides an additional reason for the regulations; it thus does nothing to
weaken the argument.
E. Health problems caused by burning dioxins are outside the scope of the argument.
Remember, the argument is about whether the regulations will reduce dioxin pollution.
Determining whether burning household garbage is harmful might be relevant to
deciding whether the plan should be implemented. It is not relevant, though, to deciding
whether the plan would work.
The correct answer is A.
CR45650.01

203. Suriland cannot both export wheat and keep bread plentiful and affordable in Suriland.
Accordingly, Suriland's wheat farmers are required to sell their crop to the government,
which pays them a dollar per bushel less than the price on the world market. Therefore, if the
farmers could sell their wheat on the world market, they would make a dollar per bushel
more, less any additional transportation and brokerage costs they would have to pay.
Which of the following, if true, most seriously weakens the argument?
A. Suriland's wheat farmers have higher production costs than do farmers in many other
wheat-producing countries.
B. Sale of a substantial proportion of Suriland's wheat crop on the world market would
probably depress the price of wheat.
C. The transportation and brokerage costs that Suriland's farmers would face if they sold
their wheat outside Suriland could amount to almost a dollar per bushel.
D. Suriland is surrounded by countries that do not import any wheat.
E. The price of a bushel of wheat on the world market occasionally drops below the average
cost of producing a bushel of wheat in Suriland.
Argument Evaluation
To keep bread affordable in Suriland, the country's government requires that wheat farmers
in the country sell their wheat to the government for one dollar per bushel less than the
world market price.
This question requires us to identify a statement that seriously weakens the argument. The
conclusion of the argument is that, if these wheat farmers could legally sell their wheat on the
world market, they would make a dollar per bushel more, minus additional transportation
and brokerage costs.
However, this argument assumes that the world market price for wheat is fixed and would
not be affected by introducing Suriland's wheat supply. That is, it is possible that the world
market price per bushel for wheat might decline as a result of an increase in the wheat supply
available on that market. If this were the case, it would severely weaken the argument.
A. This choice is outside the scope of the argument: the argument is about Suriland's wheat
farmers increasing how much money that can receive per bushel, not about how these
farmers' costs compare to the costs of farmers in other countries. In any case, this claim
does not indicate that Suriland's wheat farmers could make a dollar more, minus any
additional transportation and brokerage costs, than they do now.
B. Correct. An increase in supply on the world market with no increased demand could
easily depress the price of wheat on the market. It is still possible that Suriland's farmers
would make more than they do now, even after subtracting additional transportation and
brokerage costs. But the argument specifically says that the farmers would make a dollar
more, minus those costs, and that does not follow if the claim made in this answer choice
is true.
C. Note that the argument suggests that these farmers would make a dollar per bushel
more, less any additional transportation and brokerage costs they would have to pay.
That could still be true no matter what those costs are.
D. Whether Suriland's wheat is sold to near or distant countries is immaterial to the
argument; note that the conclusion includes the qualification less any additional
transportation costs.
E. This indicates that Suriland wheat farmers might sometimes lose money on their wheat if
selling on the world market. This does not indicate, however, that they would not lose less
on the world wheat market than they would selling to the government. Note that the
government's price for wheat is pegged to the world market, so the price the world market
offers and the price the government offers will always differ by the exact same amount:
the government pays one dollar less.
The correct answer is B.
CR30370.01

204. Sasha: It must be healthy to follow a diet high in animal proteins and fats. Human beings
undoubtedly evolved to thrive on such a diet, since our prehistoric ancestors ate large
amounts of meat.
Jamal: But our ancestors also exerted themselves intensely in order to obtain this food,
whereas most human beings today are much less physically active.
Jamal responds to Sasha by doing which of the following?
A. Refuting her statement about our prehistoric ancestors
B. Bringing forth a piece of information for the purpose of suggesting that she should qualify
her main conclusion
C. Citing additional evidence that indirectly supports her conclusion and suggests a way to
broaden it
D. Questioning whether her assumption about our prehistoric ancestors permits any
conclusions about human evolution
E. Expressing doubts about whether most human beings today are as healthy as our
prehistoric ancestors were
Argument Construction
Sasha argues that, because our prehistoric ancestors consumed large amounts of meat,
humans must have evolved to thrive on a diet of animal proteins and fats. She concludes
from this that it must be good for us to consume such a diet.
Jamal responds by pointing out that our ancestors exerted themselves much more than we
do, primarily to catch this food.
This question asks for the best characterization, among the answer choices, of Jamal's
response.
A. Jamal implicitly accepts Sasha's statement that our prehistoric ancestors consumed large
amounts of meat. Therefore, Jamal does not refute this statement made by Sasha.
B. Correct. Jamal's response indicates that our prehistoric ancestors were far more active
than we are. This suggests not that Sasha is incorrect in her statement, but that her
conclusion would benefit from this qualification: a diet high in animal protein and fat is
healthy when one engages in intense physical activity at a level that is higher than what is
normal for modern human beings.
C. Jamal's response suggests that Sasha make her conclusion more restrictive rather than
more broad.
D. Jamal suggests a qualification of Sasha's argument based on the behavior of prehistoric
human beings versus modern human beings. This in no way suggests that her assumption
about our prehistoric ancestors tells us anything about evolution.
E. Jamal does not question whether most modern human beings are as healthy as our
prehistoric ancestors. However, he does question whether a diet high in animal proteins
and fats is healthy without adjusting one's level of intense physical activity.
The correct answer is B.
CR70870.01

205. Some theorists and critics insist that no aesthetic evaluation of a work of art is sound if it is
based even in part on data about the cultural background of the artist. This opinion is clearly
false. The only sound aesthetic evaluations of artists' works are those that take into account
factors such as the era and the place of the artists' births, their upbringing and education,
and the values of their societies—in sum, those factors that are part of their cultural
background.
The above argument is most vulnerable to which of the following objections?
A. The argument presupposes the conclusion for which it purports to provide evidence.
B. The argument cites evidence that undermines rather than supports the conclusion.
C. The argument draws its conclusion by means of an equivocal interpretation of key terms.
D. The argument assumes that the production of an effect is evidence of an intention to
produce that effect.
E. The argument assumes that evaluative disputes can be resolved by citing factual evidence.
Argument Evaluation
This question asks us to identify which of the objections listed among the answer choices the
argument is most vulnerable to.
The argument's conclusion is that the opinion, expressed by some theorists and critics, that
no aesthetic evaluation of a work of art is sound if it is based even in part on data about the
cultural background of the artist, is false.
The only reason given for this conclusion is essentially a reiteration of the conclusion: that
the cultural background is in fact vital to aesthetic evaluation of the artist. Given that the only
reason given against the theorists' and critics' opinion is an opposite opinion, the argument is
circular.
Therefore, the answer to this question must identify this: that is, that the argument
presupposes the truth of the conclusion for which it claims to provide evidence.
A. Correct. As indicated above, the argument is vulnerable to this criticism.
B. The argument does not cite evidence that undermines rather than supports the
conclusion. In fact, the argument's “evidence” for its conclusion is simply a reiteration of
the conclusion itself.
C. The argument does not equivocate on any key terms.
D. The argument does not assume that the production of an effect means that one intended
to produce that effect.
E. The argument does not assume that facts will resolve evaluative disputes.
The correct answer is A.

CR53870.01

206. Banker: My country's laws require every bank to invest in its local community by lending
money to local businesses, providing mortgages for local home purchases, and so forth. This
is intended to revitalize impoverished local communities. But it is clear that the law will soon
entirely cease to serve its intended purpose. An increasing number of banks incorporated in
our country exist solely on the Internet and are not physically located in any specific
community.
The banker's argument is most vulnerable to criticism on which of the following grounds?
A. It overlooks the possibility that most banks that are physically located in specific
communities in the banker's country are not located in impoverished communities.
B. It takes for granted that a law that ceases to serve its originally intended purpose no
longer serves any other beneficial purpose, either.
C. It confuses a condition that would, if present, be likely to produce a given effect, with a
condition that would probably be the cause if that effect were present.
D. It overlooks the possibility that even if there is a strong correlation between two
phenomena, neither of those phenomena are necessarily causally responsible for the
other.
E. It fails to adequately address the possibility that an increase in the number of banks of
one kind in the banker's country will not lead to the complete elimination of banks of
another kind.
Argument Evaluation
This item requires us to identify a criticism to which this argument is vulnerable.
The argument is as follows: an increasing number of banks in a particular country exist in no
specific community, but only on the Internet. The banker believes the country's law requiring
banks to invest in their specific communities will soon cease to serve its purpose, namely, to
help revitalize impoverished communities.
Note that the banker's conclusion is quite strong: The law will soon entirely cease to serve
its intended purpose. The only reasons to believe that the law would entirely cease to serve
its … purpose would be: (1) if circumstances led to the complete disappearance of banks that
do exist in specific communities; (2) if somehow circumstances caused investment in
impoverished local communities to fail to revitalize those communities.
A. To propose that most banks that are physically located in specific communities are not
located in impoverished communities would, if anything, help support the banker's
argument. That is, these banks would not have local communities in need of help, so the
law would have little real effect on the impoverished communities it is intended to serve.
B. First, it is not clear that the argument does take for granted that the law will serve no
other beneficial purpose even if it fails to serve its intended purpose. Even if the argument
did take this statement as given, it does nothing to call into question the banker's
argument.
C. The argument does not confuse cause and effect.
D. The argument does not confuse correlation with causation.
E. Correct. The argument's only reason given for its conclusion is that the number of
Internet-only banks is growing. This does not mean, however, that banks within local
communities will no longer exist, or that those banks' investments will fail to help
revitalize impoverished communities. Even if the number of local banks declines, as long
as some local banks remain, the law will not necessarily cease to serve its intended
purpose.
The correct answer is E.

CR15380.01

207. The contingency-fee system, which allows lawyers and their clients to agree that the lawyer
will be paid only in the event of success, does not increase the number of medical malpractice
lawsuits brought against doctors. As attorneys must cover the costs for their time and
research, they want to be assured that any medical malpractice case they accept on a
contingency-fee basis has substantial merit. Consequently, attorneys turn away many people
who come to see them, for lack of a good case.
The argument above is most vulnerable to criticism on the grounds that it fails to
A. specify the criteria attorneys use to judge the merits of a medical malpractice case
B. consider whether, in the absence of a contingency-fee option, even people with
meritorious cases are much less likely to initiate litigation if they believe they might incur
large legal fees
C. note whether, in successful medical malpractice lawsuits, the average monetary award
after legal costs have been deducted is less under contingency-fee arrangements than
otherwise
D. consider the effect of the contingency-fee system on the number of lawsuits sought for
reasons other than medical malpractice
E. acknowledge the rising cost of medical malpractice insurance
Argument Evaluation
This question asks us to identify the best criticism of this argument among the given answer
choices.
Based on the premise that attorneys will turn away many potential clients who are not likely
to win their cases, the argument concludes that a contingency-fee system does not increase
the number of medical malpractice lawsuits brought against doctors.
In order to understand the argument more fully, we would need to consider whether the
alternative to a contingency-fee system—a system wherein a client pays the attorney's fees
regardless of outcome—makes it less likely that a potential client would bring a medical
malpractice lawsuit against a doctor.
A. While it is true that the argument fails to specify the criteria that attorneys use to judge
the merits of a malpractice case, this in no way suggests that such criteria do not exist.
B. Correct. This claim suggests that in the absence of a contingency-fee option, potential
clients might hesitate to bring to court even lawsuits with merit. This suggests that there
might actually be fewer meritorious malpractice lawsuits against doctors without the
contingency-fee option. This would result in an overall reduction in the number of
malpractice lawsuits against doctors, which substantially weakens the conclusion.
C. This implies that the contingency-fee option would reduce costs. Therefore, there would
be more incentive to bring cases under the contingency-fee option. This strengthens the
argument.
D. This is outside the scope of the argument. Note that the conclusion is about medical
malpractice lawsuits, so this criticism is immaterial to the argument.
E. Note that medical malpractice insurance is a cost paid by doctors themselves. Therefore,
the rising cost of medical malpractice insurance has no effect on the likelihood that a
medical malpractice case will be brought to court.
The correct answer is B.

CR66590.01

208. Shirla: In figure skating competitions that allow amateur and professional skaters to
compete against each other, the professionals are bound to have an unfair advantage. After
all, most of them became professional only after success on the amateur circuit.
Ron: But that means that it's been a long time since they've had to meet the more rigorous
technical standards of the amateur circuit.
Which of the following is most likely a point at issue between Shirla and Ron?
A. Whether there should be figure skating competitions that allow amateur and professional
skaters to compete against each other
B. Whether the scores of professional skaters competing against amateurs should be subject
to adjustment to reflect the special advantages of professionals
C. Whether figure skaters can successfully become professional before success on the
amateur circuit
D. Whether the technical standards for professional figure skating competition are higher
than those for amateur figure skating competition
E. Whether professional figure skaters have an unfair advantage over amateur figure skaters
in competitions in which they compete against each other
Argument Construction
To answer this question, we must identify a point on which Shirla and Ron disagree.
Shirla argues that because professional figure skaters who compete against amateur skaters
will already have had success on the amateur circuit, the professionals will have an unfair
advantage over the amateurs.
Ron, on the other hand, points out that this means that it has been a long time since the
professionals have had to meet the more rigorous technical standards of the amateur circuit.
What that indicates is that Ron is countering Shirla's reason for thinking that professional
figure skaters would have an unfair advantage. However, it is not clear whether Ron is
suggesting that amateurs now have an advantage or whether professionals and amateurs
would be on a relatively equal plane. It is only clear that Ron disagrees with Shirla's claim
about professionals having an unfair advantage.
A. Shirla believes that professionals would have an unfair advantage. It therefore seems
reasonable to conclude that she would oppose figure skating competitions that allow
amateur and professional skaters to compete against one another. Nevertheless, she does
not specify this and so we cannot be certain. That said, it is even less clear what Ron
would say about such competitions. Note that it is unclear where Ron stands with regard
to which group actually holds an advantage, if either.
B. It is unclear whether Shirla would say that the scores of professional skaters competing
against amateur skaters must be adjusted to reflect the professionals' advantage. It is
possible, after all, that Shirla would instead argue that there should be no such
competition at all. Ron, on the other hand, simply disagrees with the claim that
professionals have an unfair advantage.
C. Shirla believes that professional figure skaters will have initially competed on the amateur
circuit before becoming professional. There is no indication that Ron disagrees with this.
D. Ron believes that the technical standards for professional figure skating competitions are
lower, not higher, than those for amateur figure skating. There is no indication that Shirla
disagrees with this.
E. Correct. Shirla believes that professional figure skaters would have an unfair advantage
in competitions with amateur figure skaters; Ron disagrees with this belief.
The correct answer is E.
CR03001.01

209. Recent observations suggest that small, earthlike worlds form a very low percentage of the
planets orbiting stars in the galaxy other than the sun. Of over two hundred planets that
astronomers have detected around other stars, almost all are hundreds of times larger and
heavier than the earth and orbit stars much smaller than the sun.
Which of the following, if true, would most weaken the above justification of the claim that
earthlike worlds form a low percentage of the total number of planets?
A. There are millions of planets orbiting stars around which astronomers have not
attempted to detect planets.
B. The best current astronomical theories predict that almost all planets around other stars
are probably hundreds of times larger than the earth.
C. A planet orbiting a star similar to the sun would be more likely to be earthlike in size than
would a planet orbiting a much smaller star.
D. The smaller a planet is relative to the star it orbits, the more difficult it is for astronomers
to detect.
E. The observations would have detected any small, earthlike worlds orbiting the stars
around which larger planets have been detected.
Argument Evaluation
This question asks you to identify a claim that would, if true, weaken the justification for the
conclusion that only a small percentage of the total number of planets in our galaxy are
formed by earthlike worlds.
The only justification given for this conclusion is that, of the over 200 planets that
astronomers have detected around other stars, almost all are hundreds of times larger and
heavier than the earth and orbit stars much smaller than the sun.
Any evidence suggesting that the planets the astronomers have detected may be
unrepresentative of planets in general would weaken the justification this claim gives to the
conclusion.
A. This may look like it weakens the justification. After all, if the total number of planets
were significantly smaller than millions, then the sample size of over 200 planets would
allow us to make the inference with more confidence. Nevertheless, particularly when
accounting for the vagueness of the conclusion, the size of the sample is large enough to
give us a reasonable degree of certainty. More importantly, though, note that the
conclusion is restricted to planets orbiting stars in our galaxy. There is nothing in this
answer choice to suggest that the planets it refers to are actually in our galaxy.
B. This choice strengthens the justification for the conclusion.
C. This choice does not weaken the justification for the conclusion. The only way it might do
so is if it provided information showing that astronomers have mainly looked at planets
orbiting an unrepresentative sample of stars, that is, a sample that is more heavily
populated with planets orbiting stars that are smaller than most stars in the total star
population. We are given no reason to believe this is the case.
D. Correct. This gives us a reason to think that the sample may be unrepresentative.
Planets more earthlike in size may be less likely to be detected than the much larger stars
that astronomers have detected.
E. This claim strengthens, rather than weakens, the argument. It implies that the
astronomers' detection methods would not have inadvertently underrepresented the
number of earthlike worlds.
The correct answer is D.
CR61021.01

210. Researchers in City X recently discovered low levels of several pharmaceutical drugs in
public drinking water supplies. However, the researchers argued that the drugs in the water
were not a significant public health hazard. They pointed out that the drug levels were so low
that they could only be detected with the most recent technology, which suggested that the
drugs may have already been present in the drinking water for decades, even though they
have never had any discernible health effects.
Which of the following, if true, would most strengthen the researchers' reasoning?
A. If a drug found in drinking water is not a significant public health hazard, then its
presence in the water will not have any discernible health effects.
B. There is no need to remove low levels of pharmaceutical drugs from public drinking water
unless they present a significant public health hazard.
C. Even if a substance in drinking water is a public health hazard, scientists may not have
discerned which adverse health effects, if any, it has caused.
D. Researchers using older, less sensitive technology detected the same drugs several
decades ago in the public drinking water of a neighboring town but could find no
discernible health effects.
E. Samples of City X's drinking water taken decades ago were tested with today's most
recent technology, and none of the pharmaceutical drugs were found.
Argument Evaluation
This question asks us to find the answer choice that would most strengthen this argument.
Researchers in City X reason that because the levels of certain pharmaceutical drugs that
have been found in the city's drinking water are so low—detectable only by use of the most
recent technology—these drugs may well have been in the drinking water for decades.
Furthermore, the researchers point out that there have been no discernible health effects
from the use of the drugs. They conclude that the drugs are probably not a significant
concern.
As it stands, the argument is quite weak. The researchers conclude only that the drugs may
have . . . been present for decades. This leaves open the possibility that they were not present
for that long. If they were not, then obviously the current lack of discernible health effects
does not imply that there will be no such effects in the future.
We can strengthen the argument if we find solid information indicating that these drugs can
be present in a city's drinking water at the levels found in City X's drinking water, or higher,
for a long time without presenting any ill health effects.
A. This choice does not strengthen the argument. Note that there have not been any
discernible health effects from drinking the water; this fact is compatible with this
statement as well as with the drug being a significant public health hazard. Perhaps the
reason there have been no discernible health effects is that the drugs have only recently
entered the water supply.
B. This choice does not strengthen the argument's reasoning. Until we can establish that
there is no significant health hazard—what the argument sets out to prove—we cannot
know whether there is a need to remove these drugs from the drinking water.
C. This claim weakens the argument. It introduces the possibility that there may have been
adverse health effects resulting from these drugs, yet the researchers have not been able
to discern these effects, or have not been able to determine that they were effects of the
drugs.
D. Correct. Researchers several decades ago, using less sensitive technology, were able to
detect the same drugs in another town's public drinking water. This implies that the drug
levels in that town were higher than those recently detected in City X's drinking water.
Given that there have been no discernible health effects in this previous case, this lends
support to the researchers' reasoning regarding City X.
E. This claim weakens the argument; it suggests that the drugs are a relatively new presence
in the water. Therefore, the effects of these drugs might not have had time to arise.
The correct answer is D.

CR20521.01

211. Errors in the performance of repetitive or “boring” tasks—often attributed to a momentary


lapse in concentration—can be serious in such activities as flying a passenger aircraft. Is
there any method that would provide warning of such lapses—for example by monitoring
brain activity? Researchers scanned the brains of volunteers performing a repetitive task.
When the tasks were being performed correctly, the volunteers' brains showed activity in
cognitive-processing regions. However, these regions became less active several seconds
before some errors were made, and another brain region, region X, became active. The
researchers concluded that the monitoring of region X could provide warning of an
impending error.
Which of the following, if true, most supports the researchers' conclusion?
A. The cognitive effort required in performing a repetitive task diminishes significantly with
increases in the number of repetitions of the task performance.
B. Once a mistake was made and detected, brain activity in regions associated with cognitive
effort sometimes increased.
C. Other research found that whenever significant activity occurs in region X, it is generally
with repetitive tasks, soon before an error occurs.
D. The diminution of brain activity in cognitive processing regions and the increase of
activity in region X began at least 5 seconds before the errors occurred.
E. Reduced activity in brain regions associated with cognitive effort was accompanied by
increased activity in regions that become active during sleep.
Argument Evaluation
This question requires us to find a statement that would provide additional support for the
researchers' conclusion that monitoring region X can provide warning that an error is about
to be made by someone engaged in a repetitive task.
Note that researchers had observed during brain scans that cognitive-processing regions of
the brain remained active when a repetitive task was performed correctly. These regions
became less active, and brain region X became active, several seconds before errors were
made.
Certainly, further research showing such errors being preceded by the onset of activity in
region X would strengthen the researchers' conclusion—this would help rule out that the
researchers had simply noticed an unusual coincidence.
However, what would be even more helpful would be to indicate that whenever significant
activity in someone's region X occurs, this person is definitively engaged in repetitive tasks
and is about to make an error.
To see why this would be helpful, consider: if such activity in region X frequently happened,
even when no errors were about to be made, monitoring such activity would not be helpful as
a warning that an error was impending. Therefore, ruling this out would support the
conclusion.
A. Note that this does not indicate that cognitive effort diminishes; it merely indicates that
the amount of such effort required diminishes. Even more important, it tells us nothing
about activity in region X.
B. This indicates what sometimes happens after errors are made. However, it gives us no
information about what happens soon before an error. Information about that, of course,
is what we need if we are trying to determine whether something can provide warning of
an impending error.
C. Correct. As indicated above, the conclusion would be well supported by research
suggesting that whenever region X has significant activity, this is usually during repetitive
tasks and soon before an error occurs.
D. This does not provide additional support for the claim that monitoring region X will be
useful as a warning of an impending error. After all, the statement that activity in region
X began at least 5 seconds before the errors occurred rules out only that the increase in
activity in region X occurred less than 5 seconds before the errors occurred. This
statement does not rule out the possibility that the increase came, for example, many
hours before the error occurred.
E. Such a discovery may help researchers discover why the errors occurred. However, it does
not help support the claim that monitoring region X could provide a warning of
impending error.
The correct answer is C.

CR46521.01

212. City resident: These new digital electronic billboards should be banned for light pollution
since they are much too bright.
Outdoor advertising spokesperson: No, that's not true. Testing with a sophisticated light
meter shows that at night they throw off less light than traditional billboards that are
reflectively lit. Your mistaken perception that they are brighter comes from looking directly
at the light source—the screen itself.
The underlying strategy of the spokesperson's response to the resident is most analogous to
the underlying strategy of which of the following?
A. A doctor dismisses a patient's claim to have had a heart attack, citing a cardiac enzyme
blood test.
B. A politician rejects an accusation of perjury by denying the credibility of witness
testimony.
C. An insurance agent rejects a claim, on the grounds that there is insufficient evidence to
support the claimant's testimony.
D. An investigator casts doubt on the results of a lie detector, citing the subject's report of
illness during the test.
E. A psychologist treats a mental illness by encouraging a patient to abandon inconsistent
beliefs.
Argument Construction
This question requires us to identify the answer choice that that has an underlying reasoning
strategy that is most analogous to the spokesperson's strategy.
The outdoor advertising spokesperson responds to the city resident by citing an objective test
that shows the factual claims of the resident to be false.
A. Correct. This choice is the most closely analogous to the spokesperson's strategy: the
doctor uses an objective test to show the factual claims of the patient to be false.
B. The politician does not use an objective test to reject the accusation of perjury. The
politician merely denies the credibility of the witness; the basis for this denial is not
stated.
C. The insurance agent does not use an objective test as justification for rejecting a claim or
for suggesting that there is sufficient evidence for the claimant's testimony.
D. In this choice, the investigator rejects the results of what some might see as an objective
test. However, the investigator does not use the results of an objective test to prove the
factual claims of the subject to be incorrect.
E. This choice does not involve rejecting a claim, nor does it involve any sort of objective
test.
The correct answer is A.
CR02531.01

213. A fossil recently discovered in Marlandia, a chain of islands, proves that a present-day reptile
indigenous to Marlandia is descended from an ancient reptile species that lived on the
islands millions of years ago. The finding is surprising since the ancestral species was
thought to have become extinct when Marlandia was submerged in a global sea-level rise
twenty-five million years ago. Based on the new discovery, many scientists have concluded
that the sea-level rise in question left at least part of Marlandia unsubmerged.
Which of the following would, if true, provide the most additional support for the scientists'
conclusion?
A. Reptiles in Marlandia have adapted to many environmental changes since the sea-level
rise.
B. Marlandia separated from a much larger landmass about eighty million years ago.
C. No fossils that prove the relationship between the present-day species and the ancestral
species have been found anywhere other than Marlandia.
D. The present-day reptiles are able to thrive on very tiny Marlandia islands.
E. The ancestral reptiles could not have survived long at sea.
Argument Construction
This question requires that we identify the answer choice that lends the most support for the
scientists' conclusion. This conclusion states that the islands of Marlandia must not have
been completely submerged during a global rise in sea level as had previously been thought.
Scientists arrived at this conclusion after being surprised to find that a present-day reptile
indigenous to Marlandia is descended from an ancient reptile species that lived on the
islands millions of years ago. They had believed that this prehistoric species had become
extinct millions of years ago when Marlandia was submerged due to a global rise in sea level.
What are some ways that we can strengthen support for the conclusion? Information that
rules out the possibility that the reptile could have survived even if the islands had been
completely submerged would strengthen support. So would information that rules out the
possibility that the reptile had migrated to somewhere other than Marlandia but traveled
back to Marlandia after the islands were no longer submerged.
A. Even if it is true that reptiles on Marlandia have adapted to environmental changes since
the sea-level increase, that does not support the belief that part of Marlandia never
became submerged when the sea level rose. It does not rule out either of the two
alternative explanations discussed immediately above, for instance.
B. The argument's conclusion is about whether some part of Marlandia never became
submerged. Information indicating that Marlandia separated from a much larger
landmass many millions of years before the global sea-level rise is not relevant to that
conclusion.
C. It might seem that this choice supports the conclusion. If the ancestral species never lived
anywhere other than Marlandia, then the present-day species could not have descended
from this ancestral species elsewhere and only later, after the sea-level rise, migrated to
Marlandia. That would help rule out an alternative explanation of how the present-day
species survived the global sea-level rise. However, note that all we are told is that no
fossils have been found anywhere other than Marlandia. Simply because no such fossils
have yet been found does not indicate that these ancestral species never existed
elsewhere. Fossils are often deeply buried and hard to find. After all, the fossils on
Marlandia that are discussed in the argument were only recently found; perhaps other
such fossils will soon be found elsewhere. Thus, this answer choice does not support the
conclusion.
D. This does not provide any particular support for the argument. Note that it is not the
present-day reptile species that is presumed to have survived on Marlandia when sea
levels rose. Rather, it is the ancestral species that is presumed to have survived.
E. Correct. This rules out that the ancestral species could have survived the sea-level rise
simply by living at sea. It also reduces the possibility that the ancestral species had also
lived elsewhere than Marlandia and had only later—after the sea-level rise—migrated to
Marlandia.
The correct answer is E.

CR21041.01

214. Advertisement: Our competitors' computer salespeople are paid according to the value of the
products they sell, so they have a financial incentive to convince you to buy the most
expensive units—whether you need them or not. But here at Comput-o-Mart, our salespeople
are paid a salary that is not dependent on the value of their sales, so they won't try to tell you
what to buy. That means when you buy a computer at Comput-o-Mart, you can be sure you're
not paying for computing capabilities you don't need.
Which of the following would, if true, most weaken the advertisement's reasoning?
A. Some less-expensive computers actually have greater computing power than more
expensive ones.
B. Salespeople who have a financial incentive to make sales generally provide more attentive
service than do other salespeople.
C. Extended warranties purchased for less-expensive computers can cost nearly as much as
the purchase price of the computer.
D. Comput-o-Mart is open only limited hours, which makes it more difficult for many
shoppers to buy computers there than at other retail stores.
E. Comput-o-Mart does not sell any computers that support only basic computing.
Argument Evaluation
This question asks us to weaken the argument's reasoning. The advertisement makes the
following argument: because the salespeople at Comput-o-Mart are on salary rather than
paid a commission for products they sell, the store's customers will not pay for computers
that are more powerful than those that the customers need.
To weaken this reasoning, we need to drive a wedge between the given premises and the
conclusion: we need to show that it is not necessarily true that, simply because salespeople
do not have an incentive to sell more powerful computers, customers will not buy computers
that exceed their own needs.
For example, consider a case where customers' computing needs are basic, but Comput-o-
Mart sells only advanced computers. In this scenario, customers purchasing from Comput-o-
Mart would almost certainly be paying for computing capabilities that they do not need.
A. The argument hinges on the fact that a customer may pay for computing power that he or
she does not need. This statement simply notes that high computing power may in at least
some cases not cost more than low computing power. In this case, if anything, it might be
more likely that a customer would buy a computer more powerful than he or she needs.
Even so, the statement is a general statement about computers rather than a statement
specifically about those sold at Comput-o-Mart. We are not told whether Comput-o-Mart
even sells any of these computers. If not, then this statement is irrelevant to the
argument.
B. This statement suggests that the salespeople at Comput-o-Mart may be less attentive to
customers than salespeople at Comput-o-Mart's competitors. That clearly does not give
us a reason to think that a customer at Comput-o-Mart may end up paying for computing
power that he or she does not need.
C. The argument discusses whether customers at Comput-o-Mart pay for computing power
that they do not need. The costs of extended warranties are irrelevant to this discussion.
D. Again, this is irrelevant to the argument: Comput-o-Mart's hours, however limited, do not
affect whether its customers pay for computing power that they do not need.
E. Correct. If Comput-o-Mart's customers require only basic computing and Comput-o-
Mart sells only advanced computers, then it follows that Comput-o-Mart's customers are
likely to pay for computing power that they do not need. That is, regardless of Comput-o-
Mart's salespeople's payment structure (salary versus commission), if Comput-o-Mart
sells only more advanced, more expensive models, then any customer at Comput-o-Mart
who requires only basic computing would in fact be paying for unnecessary computing
power.
The correct answer is E.
CR36441.01

215. Consumer advocate: In our nation, food packages must list the number of calories per food
serving. But most of the serving sizes used are misleadingly small and should be updated.
The serving sizes were set decades ago, when our nation's people typically ate smaller
portions than they do today, and, as a result, people eating typical portions today consume
more calories than the package labeling appears to indicate that they will. It is time package
labeling reflected these changes.
Which of the following is the main point of the consumer advocate's argument?
A. The number of calories per serving listed on most food packages in the consumer
advocate's nation is misleadingly small.
B. Most serving sizes used on food packages in the consumer advocate's nation should be
increased to reflect today's typical portion sizes.
C. People eating typical portions today often consume far more calories than the number of
calories per serving listed on food packages in the consumer advocate's nation.
D. The serving sizes used on food packages in the consumer advocate's nation were set when
people ate smaller portions on average than they do today.
E. The use of misleadingly small serving sizes on food packages in the consumer advocate's
nation probably leads many people to consume more calories than they otherwise would.
Argument Construction
Of the answer choices given, which one most accurately states the consumer advocate's
main point, that is, the advocate's main conclusion?
We are told that nutritional information on food packages currently is outdated and
misleading. That is, the serving sizes that these packages show tend to understate the calories
people will consume because people now typically consume greater amounts than people did
when the serving sizes were set decades ago.
The consumer advocate's main conclusion is to recommend that serving sizes be updated to
reflect the changes in food-consumption trends.
A. The fact that serving sizes are misleadingly small is a premise of the argument rather than
its conclusion.
B. Correct. This accurately expresses the consumer advocate's main conclusion.
C. The fact that people consume more calories per serving than the amount indicated on the
packaging is a premise of the argument rather than its conclusion.
D. The fact that serving sizes were set during a time when people consumed smaller portions
is a premise of the argument rather than its conclusion.
E. The fact that people consume more calories than they believe they consume because of
misleading packaging is a premise of the argument rather than its conclusion.
The correct answer is B.
CR05941.01

216. Columnist: Metro City has a lower percentage of residents with humanities degrees than any
other city of comparable size in our nation. Nationwide, university graduates generally earn
more than people who are not university graduates, but those with humanities degrees
typically earn less than do graduates with degrees in other disciplines. So the main reason
Metro City has higher income per capita than any other city of comparable size in our nation
must be its low percentage of residents with humanities degrees.
Which of the following, if true, would most strengthen the columnist's argument?
A. Metro City residents with humanities degrees have higher income per capita than do
people with humanities degrees in any other city of comparable size in the nation.
B. The percentage of residents with university degrees is lower in Metro City than in any
other city of comparable size in the nation.
C. Nationwide, university graduates without humanities degrees typically earn more than do
individuals without university degrees.
D. Metro City residents with degrees outside the humanities have per capita income no
higher than the per capita income of such residents of other cities of comparable size in
the nation.
E. In Metro City, a lower proportion of university graduates have humanities degrees than in
any other city of comparable size in the nation.
Argument Evaluation
Which one of the five answer options provides the information that most strengthens the
columnist's argument?
The columnist's reasoning seeks to explain why Metro City has higher per capita earnings
than any city of comparable size. It attributes this to the fact that Metro City has a lower
percentage of residents with humanities degrees than do these other comparable cities.
Suppose the residents with university degrees outside the humanities had a higher per capita
income than such residents in the comparable cities nationwide. If that were the case, then
that would indicate that the higher per capita income of such residents is sufficient to explain
Metro City's divergence in per capita income from the comparable cities.
That is, if we have information to indicate that is NOT the case, the case for the explanation
offered—a lower percentage of residents with humanities degrees—is strengthened.
A. This weakens the columnist's explanation by offering a potential alternative explanation:
the higher per capita incomes of those with humanities degrees in Metro City might
explain the higher per capita income of Metro City residents.
B. This does not lend additional support to the columnist's explanation. It suggests other
possible explanations: first, the holders of degrees outside of humanities may have
extraordinarily high incomes even for such graduates nationwide; second, Metro City may
be unusual in having workers without university degrees who have an unusually high per
capita income; third, both of these groups may have unusually high per capita incomes.
C. This does not lend additional support to the columnist's explanation. However, it is well-
supported by the information offered in support of the argument's conclusion: university
graduates generally earn more than others, and among university graduates, humanities
graduates earn less than others.
D. Correct. As explained above, this information, by eliminating an alternative explanation
to that offered in the argument's conclusion, strengthens the argument.
E. This information is consistent with the information provided in support of the argument's
conclusion. It does not, however, provide additional support for the argument's
conclusion. This information is consistent with alternative explanations for the higher per
capita income of Metro City's residents. That is, for example, that the holders of non-
humanities degrees, or those with no degrees at all, have particularly high per capita
income.
The correct answer is D.

CR87051.01

217. Psychologist: In a survey, several hundred volunteers rated their own levels of self-control
and their overall life satisfaction. The volunteers who rated themselves as having better self-
control also reported greater satisfaction with their lives. This suggests that self-control is
one factor that helps people avoid situations likely to produce dissatisfaction.
In order to assess the strength of the psychologist's argument, it would be most helpful to
know whether
A. people typically rate themselves as having significantly better self-control than expert
psychological assessments would rate them as having
B. people's perceptions of how satisfied they are with their lives could be affected by factors
of which they are unaware
C. there is a high level of self-control that tends to reduce overall life satisfaction
D. people's ratings of their overall satisfaction with their lives tend to temporarily decrease
in situations likely to produce dissatisfaction
E. feelings of dissatisfaction significantly interfere with people's ability to exercise self-
control
Argument Evaluation
Which one of the further pieces of information given in the answer choices would most help
us evaluate the psychologist's argument?
A psychologist tells us that people surveyed who reported high levels of self-control reported
high levels of life satisfaction also. The psychologist infers from this that self-control is one
factor that helps people avoid situations likely to produce dissatisfaction with their lives.
Note that only answer options C and E directly address a possible causal link between degree
of life satisfaction and degree of self-control. Answer option C tells us that a high degree of
self-control can actually reduce life satisfaction.
However, this information, if accurate, does little to weaken support for the conclusion that
self-control helps people avoid situations likely to create dissatisfaction. Furthermore, it
provides no support for that conclusion.
That leaves E as the most likely candidate for the correct response. Suppose E were correct;
that is, feelings of dissatisfaction make effective self-control less likely. This is compatible
with the correlation between self-control and feelings of satisfaction—but also strongly
suggests that self-control is the effect rather than the cause of feelings of satisfaction. And
this undermines the conclusion of the argument.
A. This suggests that the self-ascribed levels of self-control of the participants in the survey
may have been exaggerated. This weakens the psychologists' conclusion without
completely invalidating this evidence.
B. This implies that survey participants were not fully aware of all the factors that affected
their perceptions of life satisfaction. However, the conclusion does not depend on the
participants' degree of awareness of the factors affecting their perceptions of life
satisfaction.
C. This may be true, but even if true in some cases, this information does not necessarily
invalidate the psychologist's conclusion in general.
D. This implies that people's ratings of their life satisfaction can vary, and be temporarily
lowered, by encountering a situation likely to produce dissatisfaction. But such possible
sources of random error would be allowed for in the statistical analysis of the survey
results and do not necessarily invalidate the study's results.
E. Correct. As explained, this information undermines the psychologist's conclusion by
providing an alternative explanation for the survey results.
The correct answer is E.
CR03161.01

218. Mansour: We should both plan to change some of our investments from coal companies to
less polluting energy companies. And here's why. Consumers are increasingly demanding
nonpolluting energy, and energy companies are increasingly supplying it.
Therese: I'm not sure we should do what you suggest. As demand for nonpolluting energy
increases relative to supply, its price will increase, and then the more polluting energy will
cost relatively less. Demand for the cheaper, dirtier energy forms will then increase, as will
the stock values of the companies that produce them.
Therese responds to Mansour's proposal by doing which of the following?
A. Advocating that consumers use less expensive forms of energy
B. Implying that not all uses of coal for energy are necessarily polluting
C. Disagreeing with Mansour's claim that consumers are increasingly demanding
nonpolluting energy
D. Suggesting that leaving their existing energy investments unchanged could be the better
course
E. Providing a reason to doubt Mansour's assumption that supply of nonpolluting energy
will increase in line with demand
Argument Construction
Which one of the answer options best describes the response of Therese to Mansour?
Mansour advocates that Therese and he should replace their investments in coal companies
with investments in companies producing less polluting energy. He suggests this because
there is both an increasing demand for and increasing supply of such energy.
However, Therese responds that as demand for clean energy increases, its prices will
increase. These higher prices will increase demand for cheaper, dirtier energy. This will boost
the stock prices of companies producing such dirty energy, for example, coal. The point of
Therese's response is that since the stock prices of coal companies and other companies
producing dirty energy will likely increase, investments in these stocks will increase in value.
This provides a financial reason not to do what Mansour advocates.
A. Therese does not advocate that consumers use polluting energy; she simply predicts that
the stock values of producers of polluting energy are likely to increase as the prices of
polluting energy decrease relative to the prices of non-polluting energy.
B. Therese does not imply that there exist non-polluting uses of coal; in fact, she appears to
agree with Mansour that coal is a dirty form of energy.
C. There is nothing to indicate that Therese disagrees with Mansour regarding whether
consumers are increasingly demanding non-polluting energy.
D. Correct. This accurately captures the main point of Therese's response to Mansour: she
provides a particular reason that maintaining their current investments could be a better
option.
E. Therese does not directly address the question of whether this assumption of Mansour's
is correct. Even if she implies that Mansour's assumption is incorrect, this is not the main
point of her response to Mansour.
The correct answer is D.
CR04161.01

219. Scientist: In an experiment, dogs had access to a handle they could pull to release food into a
nearby enclosure that contained a familiar dog and nothing else, contained an unfamiliar dog
and nothing else, or was empty. The dogs typically released more food to the familiar dog
than to the unfamiliar dog. This suggests that dogs are more motivated to help other dogs
they know than to help unfamiliar dogs.
The scientist's argument would be most strengthened if it were true that, in the experiment,
the dogs with access to the handle tended to release more food when
A. the behavior was being encouraged by a familiar person than when it was being
encouraged by an unfamiliar person
B. the enclosure was empty than when it contained an unfamiliar dog
C. an unfamiliar dog in the enclosure was displaying hostility toward them than when an
unfamiliar dog in the enclosure appeared friendly
D. a dog in the enclosure appeared uninterested in food already released into the enclosure
than when it appeared interested in that food
E. a familiar dog was in the enclosure than when a familiar dog was visible but the enclosure
was empty
Argument Evaluation
Which one of the five experimental outcomes, if added to the information given, would most
strengthen the evidence for the scientist's conclusion?
There were three enclosures, two of which contained a dog. Only one of these contained a
familiar dog. The dogs released more food to familiar dogs than to unfamiliar dogs. The
scientists thereby concluded that dogs are more motivated to help familiar dogs than they are
to help unfamiliar dogs.
However, it is possible that releasing the food to the familiar dog could have been motivated
by other reasons. For example, the dog could simply be trying to communicate with the
familiar dog rather than necessarily trying specifically to help this dog.
If the dogs released more food to a familiar dog while it was contained in an enclosure than
when it was not enclosed yet nearby and visible, this would strengthen the idea of trying to
“help” the other dog.
A. This information would weaken the scientist's argument. It introduces information
suggesting the presence of a confounding variable in the experimental setup. That is, if
the behavior was encouraged by a familiar person, we would not be able to tell whether it
was this person's presence or the presence of the other dog that increased the behavior.
B. This would suggest that the dog's activation of the lever was not contingent on providing
food to another animal. In other words, if the dog provides food even when there is no
animal to provide food for, then it follows that the presence of the other dog is irrelevant
to this behavior.
C. The experimental setup described here introduces the factor of friendliness. Adding this
extra factor could easily confound testing of the original hypothesis, which suggested that
simple familiarity increased the behavior.
D. The experimental setup described here introduces a factor of food interest. Adding this
extra factor could easily confound testing of the original hypothesis, which suggested that
simple familiarity increased the behavior.
E. Correct. This information would strengthen the hypothesis that a desire to help a
familiar dog was operative in the dog's behavior.
The correct answer is E.

CR09461.01

220. Most geologists believe oil results from chemical transformations of hydrocarbons derived
from organisms buried under ancient seas. Suppose, instead, that oil actually results from
bacterial action on other complex hydrocarbons that are trapped within the earth. As is well
known, the volume of these hydrocarbons exceeds that of buried organisms. Therefore, our
oil reserves would be greater than most geologists believe.
Which of the following, if true, gives the strongest support to the argument above about our
oil reserves?
A. Most geologists think optimistically about the earth's reserves of oil.
B. Most geologists have performed accurate chemical analyses on previously discovered oil
reserves.
C. Ancient seas are buried within the earth at many places where fossils are abundant.
D. The only bacteria yet found in oil reserves could have leaked down drill holes from
surface contaminants.
E. Chemical transformations reduce the volume of buried hydrocarbons derived from
organisms by roughly the same proportion as bacterial action reduces the volume of other
complex hydrocarbons.
Argument Evaluation
What new information, if added to the argument, would strengthen it?
The argument sets forth a novel hypothesis about how oil reserves are created. That is, oil
reserves are created through bacterial action on complex hydrocarbons within the earth
rather than through chemical transformation of hydrocarbons derived from organisms
buried under ancient seas.
The argument notes that the volume of the hydrocarbons that bacteria transform to yield oil
is greater than the volume of hydrocarbons derived from the buried organisms and concludes
that total oil reserves are greater than most geologists believe them to be.
A. This suggests that most geologists might, if anything, be inclined to overestimate oil
reserves. However, this consideration has little bearing on the chemical origin of oil or
how much oil may remain buried in the earth.
B. This does not tell us whether the chemical analyses can identify whether the oil
originated from hydrocarbons derived from buried organisms.
C. The existence of buried ancient seas has little, if any, relevance to the argument. This
choice fails to provide evidence that by itself would help decide whether the hypothesized
bacterial origin of oil actually supports the inference that oil reserves are greater than is
currently assumed.
D. This suggests that bacteria have been found in some oil reserves; the potential importance
of this discovery is unclear.
E. Correct. This strengthens the argument: if it is true, then the greater abundance of
complex hydrocarbons from which it is hypothesized that oil can be derived through
bacterial action would predict much larger oil reserves than exist under most geologists'
current predictions.
The correct answer is E.
CR66561.01

221. Meteorologists say that if only they could design an accurate mathematical model of the
atmosphere with all its complexities, they could forecast the weather with real precision. But
this is an idle boast, immune to any evaluation, for any inadequate weather forecast would
obviously be blamed on imperfections in the model.
Which of the following, if true, could best be used as a basis for arguing against the author's
position that the meteorologists' claim cannot be evaluated?
A. Certain unusual configurations of data can serve as the basis for precise weather
forecasts, even though the exact causal mechanisms are not understood.
B. Most significant gains in the accuracy of the relevant mathematical models are
accompanied by clear gains in the precision of weather forecasts.
C. Mathematical models of the meteorological aftermath of such catastrophic events as
volcanic eruptions are beginning to be constructed.
D. Modern weather forecasts for as much as a full day ahead are broadly correct about 80
percent of the time.
E. Meteorologists readily concede that the accurate mathematical model they are talking
about is not now in their power to construct.
Argument Evaluation
Which one of the following would provide the best basis for arguing against the author's
reasoning?
Meteorologists claim that the design of a mathematical model that would accurately capture
all the complexities of weather dynamics would enable great precision in weather forecasting.
However, according to the skeptical reasoning given, the meteorologists' claim cannot be
evaluated, because any inaccuracies in weather forecasting would be attributed to
shortcomings in the model.
It is important to consider that with the incremental improvement of capabilities for
collection and analysis of data, including new types of data, model construction would
improve to more accurately reflect weather dynamics. However, it would remain true that
random factors affecting weather may continue to reduce accuracy of forecasts.
A. This suggests that weather forecasting accuracy can be attained under certain unusual
conditions, even in the absence of understanding complex factors that affect weather.
Nevertheless, what is at issue in the skeptical reasoning given is how, or whether, overall
forecasting accuracy can be attained under all conditions. Furthermore, it is unclear
whether the meteorologists' aspiration to great precision and accuracy in weather
forecasting can even be evaluated.
B. Correct. This tells us that significant but incremental improvements in the accuracy of
mathematical models result in gradual improvements in the accuracy of weather
forecasting—even if wholly accurate and precise forecasts are never attained. This would
allow evaluation of any progress in modeling and forecasting weather.
C. Volcanic eruptions can affect weather but they do not rank as major ongoing causes of
weather phenomena. The reasoning given refers to forecasting of weather under all
conditions, whether the meteorologists' ideal is attainable or can even be evaluated.
D. This suggests that current weather forecasting falls significantly short of the forecasting
accuracy that the meteorologists mentioned aspire to. This idea reinforces the skeptical
reasoning that suggests the meteorologists' ideal is not amenable to evaluation and may
not even be attainable.
E. This does not convey a reason for thinking the meteorologists' claim can reasonably be
subjected to evaluation; furthermore, it seems to provide some support for the skeptical
reasoning given.
The correct answer is B.
CR38561.01

222. The lobbyists argued that because there is no statistical evidence that breathing other
people's tobacco smoke increases the incidence of heart disease or lung cancer in healthy
nonsmokers, legislation banning smoking in workplaces cannot be justified on health
grounds.
Of the following, which is the best criticism of the argument reported above?
A. It ignores causes of lung cancer other than smoking.
B. It neglects the damaging effects of smoke-filled air on nonsmokers who are not healthy.
C. It fails to mention the roles played by diet, exercise, and heredity in the development of
heart disease.
D. It does not consider the possibility that nonsmokers who breathe smoke-filled air at work
may become more concerned about their health.
E. It does not acknowledge that nonsmokers, even those who breathe smoke-filled air at
work, are in general healthier than smokers.
Argument Evaluation
Among the answer choices given, which one describes the most significant flaw in the
reasoning, given the information provided?
Note that not all nonsmokers are healthy in every respect. This raises the possibility that
tobacco-smoke exposure of some nonsmokers—those who already have some medical
condition—could either worsen existing illnesses or cause new ones such as lung cancer or
heart disease.
The risks in such exposure could be significantly greater for those unhealthy nonsmokers
than for the healthy nonsmokers.
A. This choice is outside the scope of the argument. The argument addresses the issue of
illnesses that could be caused by exposure to other people's smoke in the workplace. In
this context, cancer-causing factors other than smoking are irrelevant.
B. Correct. The information provided does not mention health risks to unhealthy
nonsmokers that exposure to other people's tobacco smoke in the workplace might cause.
C. This choice is outside the scope of the argument. The argument addresses the issue of
illnesses that could be caused by exposure to other people's smoke in the workplace. It
does not address general risk factors that contribute to anybody's risk of getting heart
disease.
D. The argument does not consider whether nonsmokers might become concerned about
their health risks in workplaces where they breathe smoke. The argument is about health
risks, not about workers' attitudes to health risks.
E. The argument suggests that healthy nonsmokers are not unduly affected by cigarette
smoke in the workplace. While possibly true, this claim is not a valid criticism of the
argument as stated. The argument has no explicit comparison of the health levels of
smokers and nonsmokers.
The correct answer is B.
CR78561.01

223. Since 1978 when the copyright law was changed, books that are less than fifty years old must
not be photocopied without the publisher's permission. Thus, any book that has been
photocopied since 1978 without the publisher's permission must be at least fifty years old.
The reasoning above exhibits a flaw similar to one in which of the following?
A. Any member of the solar system must be either a planet or a moon, so if an asteroid is
neither a planet nor a moon, it must not be a member of the solar system.
B. Anyone who rides a city bus must buy a bus pass, and since Demetrios has a bus pass, he
must be riding on a city bus.
C. A driver who turns right must signal, so any driver who did not signal must not have
turned right.
D. Anyone who legally crosses a national boundary must have a passport; thus anyone who
does not have a passport cannot legally cross a national boundary.
E. Any wage earner residing in the state must pay state taxes, so since Blodwen pays state
taxes, she must be resident in the state.
Argument Evaluation
The flaw in which one of the five arguments presented is most similar to the flaw in the
given argument?
The conclusion of the given argument is adequately supported by its premises only if
supplemented by a highly implausible assumption: Since the 1978 copyright law came into
force, no book published since 1978 has been photocopied without the publisher's
permission.
However, this would be unreasonable to assume: we have no reason to assume that the
existence of a rule implies compliance with that rule.
A. This argument contains no flaw; its conclusion follows logically from its supporting
information.
B. This argument is flawed because it is possible that Demetrios is not riding on a city bus
but yet still owns a bus pass. However, this is not an argument regarding compliance or
non-compliance with a rule.
C. Correct. This argument has the same structure as the given argument. For the argument
to work, it must be supplemented with the following highly implausible assumption: No
driver who fails to signal turns right. That assumption is implausible because the
existence of a rule is not always followed by compliance with that rule.
D. The conclusion drawn here follows logically from the premise on which it is based.
Therefore, the argument is not flawed.
E. This argument is flawed: it is possible that Blodwen pays state taxes yet is not a resident
in the state. The condition “liable to pay state tax” applies to all residents of the state, but
we do not know that this applies only to residents of the state. That is, the class of those
“liable to pay state tax” may in fact be larger than the class of those “resident in the state.”
In any case, the structure of this argument is significantly different from the given
argument.
The correct answer is C.

CR10661.01

224. In the United States, injuries to passengers involved in automobile accidents are typically
more severe than in Europe, where laws require a different kind of safety belt. It is clear from
this that the United States needs to adopt more stringent standards for safety belt design to
protect automobile passengers better.
Each of the following, if true, weakens the argument above EXCEPT:
A. Europeans are more likely to wear safety belts than are people in the United States.
B. Unlike United States drivers, European drivers receive training in how best to react in the
event of an accident to minimize injuries to themselves and to their passengers.
C. Cars built for the European market tend to have more sturdy construction than do cars
built for the United States market.
D. Automobile passengers in the United States have a greater statistical chance of being
involved in an accident than do passengers in Europe.
E. States that have recently begun requiring the European safety belt have experienced no
reduction in the average severity of injuries suffered by passengers in automobile
accidents.
Argument Evaluation
The argument suggests that passengers involved in automobile accidents in the United States
typically are more seriously injured than those in Europe. Furthermore, in Europe, a
different safety belt design is used. The argument suggests that these European-style safety
belts are more protective against serious injury than those used in the United States.
Furthermore, it suggests that the United States would therefore benefit by adopting more
stringent design standards for safety belts.
To clarify, the rate of severe injuries would indicate, for example, the number of seriously
injured passengers per 100,000 passengers involved in automobile accidents. Note that this
rate does not depend on the total number of passengers involved in automobile accidents.
However, many other factors could provide an alternative explanation for these differences
in rate of severe injury. The question stem asks us which answer choice does NOT weaken
the argument; in other words, we must find a factor among the answer choices that does
NOT account for this difference in the rate of severe injury. Effectively, we will be looking for
the answer that has no bearing on the rate of severe injury.
Which one of the statements given does NOT weaken the argument?
A. This choice weakens the argument. It suggests the possibility that the difference in rates
of severe injury is due to the number of people who actually wear safety belts in the U.S.
versus in Europe. This rate is, of course, irrespective of the functionality of the belts
themselves. In other words, the effectiveness of the belt design is irrelevant if the belt is
not being worn in the first place.
B. This choice weakens the argument. Training to understand how to minimize injury,
rather than a safety belt design difference, may be a primary factor accounting for the
lower severe injury rate in Europe.
C. This choice weakens the argument. The fact that cars constructed in Europe are more
sturdy may account for the lower severe injury rate in Europe rather than the difference
in the types of safety belts used.
D. Correct. This choice does not weaken the argument. The higher likelihood that one is
involved in an automobile accident in the U.S. actually has no bearing on the higher rate
of severe injury among passengers who are involved in automobile accidents. That is, the
rate itself is a proportion of the total number of passengers involved in accidents rather
than the number itself. This rate would remain the same whether 10 accidents or 10
million accidents occurred.
E. This answer choice suggests that even implementation within the United States of the
European safety belt design does not seem to change serious injury rates. This implies, of
course, that some other factor is likely responsible for the differences in rates of serious
injury between the U.S. and Europe.
The correct answer is D.
CR60661.01

225. A country's Aeronautics Board (AB) employs inspectors who make routine annual
inspections of all aircraft. On inspecting Azura Airlines' airplanes in December, they reported
considerably more violations of AB rules this year, compared to a year ago. This fact explains
why Azura had more accidents this year, compared to last year.
Which of the following, if true, would cast most doubt on the conclusion in the passage?
A. Some aviation experts in other countries consider certain AB rules to be too lax and too
easy to get around.
B. Azura's routes are no more dangerous than are those of most other airlines.
C. The AB increased the length and rigor of its inspections this year, compared to last year.
D. Prior to last year Azura had an excellent safety record with very few accidents.
E. In both years the AB report on Azura did not include violations on airplanes owned by
Azura but leased by another airline.
Argument Evaluation
During the Aeronautic Board (AB)'s annual inspection, it found more violations among
Azura Airlines' airplanes this year than last. The argument suggests that this increase in the
number of violations provides an explanation for why Azura Airlines experienced more
accidents this year than last. We must choose the statement that most weakens the
conclusion.
A. This statement is outside the scope of the argument; aviation experts in foreign countries'
opinion of AB has no bearing on the AB and its evaluation of Azura Airlines.
B. This statement is outside the scope of the argument; even if Azura's routes are more
dangerous than its competitors', the relationship between Azura's routes and other
airlines' routes has no bearing on the AB and its evaluation of Azura Airlines.
C. Correct. This statement provides an alternative reason that the number of violations for
Azura Airlines would have increased. That is, if the AB conducted more extensive and
more rigorous inspections this year, then it is very likely that the number of violations
found by the AB would increase. In other words, if more points are tested or existing tests
are made stricter, then the increase in rigor of the testing would presumably lead to an
increase in violations.
D. This statement is outside the scope of the argument; the argument only concerns last year
as compared to this year. Azura Airlines' record prior to last year is irrelevant to this.
E. This information does not apply to the argument at hand; the argument only concerns the
planes that were in fact counted both years. If we could take this information to imply
that even more Azura-owned airplanes violate safety standards than previously thought,
this would actually strengthen the argument rather than weaken it.
The correct answer is C.
CR13661.01

226. The more frequently employees take time to exercise during working hours each week, the
fewer sick days they take. Even employees who exercise only once a week during working
hours take less sick time than those who do not exercise. Therefore, if companies started
fitness programs, the absentee rate in those companies would decrease significantly.
Which of the following, if true, most seriously weakens the argument above?
A. Employees who exercise during working hours occasionally fall asleep for short periods of
time after they exercise.
B. Employees who are frequently absent are the least likely to cooperate with or to join a
corporate fitness program.
C. Employees who exercise only once a week in their company's fitness program usually also
exercise after work.
D. Employees who exercise in their company's fitness program use their working time no
more productively than those who do not exercise.
E. Employees who exercise during working hours take slightly longer lunch breaks than
employees who do not exercise.
Argument Evaluation
The argument suggests that the greater the amount of time employees take to exercise during
working hours, the fewer sick days these employees take. This is true even of employees who
exercise just once a week during work hours.
Based on this evidence, the argument concludes that companies that start fitness programs
will see a significant decrease in the absentee rate.
The question asks us to identify a serious weakness in the argument. The argument as stated
does not address the following fundamental problem: the same employees who are
frequently absent may also fail to make use of such a fitness program. If that is true,
therefore, the evidence provided would give us little reason to believe that those who are
currently frequently absent would be absent less often if such a program were implemented.
A. This answer choice may call into question the advisability of starting such a fitness
program. That, however, is not what the argument's conclusion is about. The argument's
conclusion states that company-run fitness programs will reduce absentee rates, but this
choice tells us nothing about what effect, if any, such fitness programs would have on
absentee rates.
B. Correct. The argument assumes that exercising during office hours will lead employees
to be absent from work less frequently. However, this choice indicates that those who are
frequently absent are the least likely to make use of a company fitness program. That
could mean that these employees will use such fitness programs rarely, if at all. If so, we
would have far less reason to believe that there would be a significant decline in the
absentee rate if such a program were implemented.
C. This answer choice is outside the scope of the argument: we are not given any information
about how exercise outside of work affects absentee rates.
D. This answer choice is outside the scope of the argument: the argument's conclusion is
about absentee rates; the argument does not concern productivity.
E. This answer choice is outside the scope of the argument: there is no information to
suggest any correlation between the length of lunch breaks and the use of fitness
programs or the length of lunch breaks and rates of absence.
The correct answer is B.
Answer Explanations Verbal Reasoning
Critical Reasoning
Construction/Plan
CR67370.01

227. Distressed by his own personal tragedies, the Roman philosopher Cicero once asked himself
whether a wise person should try to achieve the Stoic ideal of complete emotionlessness.
Cicero reasoned that, however desirable the goal may be, a wise person could never attain it,
since emotions are not simply irrational urges. They are, rather, a product of one's estimate
of the goodness and badness of the events, people, and actions one witnesses.
Which of the following is an assumption required by Cicero's reasoning?
A. Wise people inevitably evaluate at least some of the things they observe.
B. Irrationality makes evaluation of what one observes impossible.
C. Wisdom precludes attempting to attain what one cannot.
D. If evaluations are based only on reason, then they are inaccurate.
E. A wise person will not evaluate what cannot be directly observed.
Argument Construction
The question asks us to identify an assumption required by Cicero's reasoning.
Cicero reasoned that a wise person could never attain the goal of complete emotionlessness,
because emotions are not merely irrational urges but a product of one's estimate of the
goodness and badness of events, people, and actions one witnesses.
Why would Cicero have thought that this is a good reason to believe that a wise person could
never be completely emotionless? He must have thought that wise people cannot completely
avoid making evaluations of the goodness and badness of at least some of the events, people,
and actions they witness. If he thought they could completely avoid making such evaluations,
the reason he gave for his conclusion would not have supported it.
A. Correct. As indicated above, Cicero's argument requires that wise people evaluate at
least some of the things that they observe.
B. Cicero's argument does not require this assumption. It makes sense to think that wise
people would not be irrational. However, this assumption suggests that it is possible to
avoid evaluations of what one observes, which is not helpful to Cicero's reasoning.
C. If Cicero's conclusion is true, then this assumption helps support the claim that wise
people should not attempt to attain the goal of complete emotionlessness. But the
question does not ask you to identify an assumption that would allow you to infer that
wise people should not attempt to attain that goal. Instead, the question asks you to
identify an assumption that is required to infer that wise people cannot attain that goal.
Whether you should try to do something is a different issue from whether you can do
something.
D. Cicero's argument is not about the accuracy of one's evaluations; rather, it assumes that
wise people will inevitably make such evaluations.
E. Cicero's argument is based on a premise about evaluations of the observed, not about
evaluations of what is impossible to observe.
The correct answer is A.
CR49770.01

228. First discovered several years ago in North American lakes and rivers, the northern
snakehead is a nonnative fish with no local predators. To keep the northern snakehead's
population from growing, for the past three years wildlife officials have been paying
recreational fishers for each northern snakehead they catch. In this way, the officials hope to
stop the northern snakeheads from eliminating rare native fish species.
To evaluate the likelihood that the wildlife officials' plan will succeed, it would be most useful
to determine which of the following?
A. Whether the northern snakehead's population in local lakes and rivers could be reduced
by introducing predators from its native habitat
B. How local population numbers of rare native fish species have been changing since the
wildlife officials started paying recreational fishers to catch northern snakeheads
C. Whether the fish species on which the northern snakehead preys in regions to which it is
native and in which it is abundant have become significantly depleted in recent decades
D. What total number of northern snakehead have been caught by recreational fishers since
the wildlife officials began paying for them
E. Whether rare native fish species in the region face any threats to their survival other than
the proliferation of northern snakehead
Evaluation of a Plan
In hopes of preventing the nonnative species northern snakehead from eliminating rare
native fish species, wildlife officials have for the last three years been paying recreational
fishers for each snakehead they catch. The northern snakehead has no predators in the area.
This question requires us to identify information that would be useful for determining
whether the officials' plan will succeed.
Note that the plan has already been in effect. Nevertheless, we are given no information as to
how well the plan has succeeded so far. To determine whether it is responsible to keep paying
these fishers to catch northern snakehead, it is vital to understand whether the fishers' work
up to this point has had any observable effect.
That is, it would be helpful to anyone who wants to determine whether the officials' plan is
likely to succeed to have information about how the numbers of rare native fish species have
been changing during that time.
A. This information would be useful for determining whether there may be alternative ways
of reducing the northern snakehead population. However, it is not useful for determining
whether the plan in question is likely to be successful.
B. Correct. As noted above, this information would be helpful in assessing whether the
officials' plan is likely to succeed. If the numbers of rare native fish species have stayed
constant or even declined throughout the period that the plan has been in place, it seems
unlikely that the plan will ultimately be successful.
C. This information may provide a small amount of information as to how great a threat the
northern snakehead might pose, but not much. For one thing, in its native region, the
snakehead likely preys on different species from those in the region in question. More
importantly, however, is the fact that this information is simply not useful to determining
whether the officials' plan is likely to work in the region in question.
D. This might be useful for determining whether fishers have been motivated by the plan.
This could potentially be useful for determining whether the plan will work; after all, the
plan will not likely work if very few northern snakeheads are actually removed.
Nevertheless, it is not as useful as noting the changes in the native fish population since
the plan first went into effect.
E. Determining whether there are other threats to the rare native fish species would tell us
whether the plan, if successful, is sufficient to save the rare fish species. Still, it is not
useful in determining whether that plan is likely to be successful.
The correct answer is B.
CR51080.01

229. Scientist: A greenhouse gas, for example, carbon dioxide, forms a transparent layer that
traps solar heat beneath it in the earth's atmosphere. Atmospheric levels of carbon dioxide
are currently increasing, causing the climate to warm—an effect that is predicted by at least
one computer model of the greenhouse effect. But the warming that has occurred is a great
deal less than what would be expected based on the model. Therefore, _______.
Which of the following most logically completes the scientist's argument?
A. better measurements of atmospheric levels of carbon dioxide are needed
B. the definition of “greenhouse gas” should probably be reconsidered
C. there are factors besides the increase in greenhouse-gas emissions contributing to the
warming of the climate
D. the computer model of the greenhouse effect must be incorrect in some respect
E. the likely consequences of any warming of the climate are unlikely to be much less
damaging than predicted
Argument Construction
This question requires us to choose the option that most logically completes the scientist's
argument.
The scientist's argument points out that at least one computer model has predicted that the
amount of warming the atmosphere would by now have experienced as a result of the
greenhouse effect would be significantly greater than what has actually occurred.
Because what precedes the blank is the word therefore, we must choose the statement that is
most strongly supported by the information given.
A. It is reasonable to infer from the given information that some aspect of the computer
model is incorrect in some way. One way that it might be incorrect is that the
measurements of atmospheric levels of carbon dioxide used are inaccurate. Nevertheless,
there is not sufficient information to infer that this is the specific flaw in the model.
B. The given information provides no reason for us to reconsider the definition of
“greenhouse gas.” Perhaps there would be a reason to do so if no warming had occurred
whatsoever. However, the information tells us that warming has in fact happened, just
not as much as predicted.
C. If there are factors in addition to greenhouse-gas emissions that lead to atmospheric
warming, then presumably temperatures would have risen more than predicted, not less.
D. Correct. If the model were correct, its predictions would be confirmed by the data
regarding warming. However, this has not occurred. It is therefore reasonable to infer
that, in some way, the computer model is incorrect.
E. This does not follow from the argument: given the findings, it seems more reasonable to
think that the likely consequences of any atmospheric warming would in fact be less
damaging than predicted.
The correct answer is D.

CR09090.01

230. Beets and carrots are higher in sugar than many other vegetables. They are also high on the
glycemic index, a scale that measures the rate at which a food increases blood sugar levels.
But while nutritionists usually advise people to avoid high-sugar and high-glycemic-index
foods, despite any nutritional benefits they may confer, they are not very concerned about
the consumption of beets and carrots.
Which of the following, if true, would best explain the nutritionists' lack of concern?
A. Foods with added sugar are much higher in sugar, and have a larger effect on blood sugar
levels, than do beets and carrots.
B. Most consumption of beets and carrots occurs in combination with higher-protein foods,
which reduce blood sugar fluctuations.
C. Beets and carrots contain many nutrients, such as folate, beta-carotene, and vitamin C, of
which many people fail to consume optimal quantities.
D. The glycemic index measures the extent to which a food increases blood sugar levels as
compared to white bread, a food that is much less healthy than beets and carrots.
E. Nutritionists have only recently come to understand that a food's effect on blood sugar
levels is an important determinant of that food's impact on a person's health.
Argument Construction
This question requires us to identify an explanation for nutritionists' lack of concern
regarding consumption of beets and carrots, despite the fact that they have higher sugar
levels than many other vegetables and rate high on the glycemic index.
Normally, nutritionists would advise people against consuming foods that rate high on the
glycemic index, whatever their other nutritional benefits.
To explain why nutritionists might hold such apparently conflicting positions, consider
whether there might be something notable about the consumption of beets and carrots that
makes their consumption an exception to the rule.
A. The fact that some foods may be even more problematic than beets and carrots does not
explain nutritionists' lack of concern about beets and carrots.
B. Correct. This provides a reason that consumption of beets and carrots may be, at least
within the suggested context, exempt from nutritionists' concern about consumption of
high-sugar, high-glycemic-index foods. Remember, nutritionists' general concern arises
from the effects such foods have on those who consume them. If we suggest that beets
and carrots are usually consumed with high-protein foods that counteract these
unwanted effects, we have good reason not to be concerned about their consumption, at
least in regard to their sugar level and glycemic index.
C. The nutritionists advise people to avoid foods high in sugar and high on the glycemic
index despite any nutritional benefits they may confer. Therefore, the fact that beets and
carrots have such nutritional benefits does not explain why nutritionists would not be
concerned about consuming beets and carrots, given that they both high in sugar and rate
high on the glycemic index.
D. Whatever measure the glycemic index is based on, we know that beets and carrots are
high on that index. Furthermore, we know that nutritionists usually advise people to
avoid high-sugar and high-glycemic-index foods, despite any nutritional benefits they
may confer. Therefore, the fact that beets and carrots are more nutritious than white
bread does not in any way explain the nutritionists' lack of concern.
E. The fact that nutritionists have only recently become concerned about high-sugar and
high-glycemic-level foods does not explain their particular lack of concern regarding beets
and carrots.
The correct answer is B.

CR36601.01

231. Ozone in the stratosphere blocks deadly ultraviolet rays from the sun, but
chlorofluorocarbons (CFCs) in aerosols and other products have thinned this protective
layer. Evidence of this is the ozone hole that forms over the South Pole every Antarctic spring
as temperatures drop below –78°C, the temperature at which ozone depletion occurs.
Measurements of the ozone hole taken at various times this spring show that, compared with
the same times the previous year, its area diminished by four million square kilometers.
Nevertheless, scientists have not concluded that the ozone layer is recovering.
Which of the following would, if true, provide the strongest reason for the scientists' reaction
to the measurements?
A. The ozone hole has steadily grown in size every year for the past decade except this year.
B. The length of time that the ozone hole persists fluctuates from year to year.
C. As a result of international treaties, CFCs have been completely banned for several years.
D. Weather patterns allowed unusual amounts of warm air to mix into the polar regions this
year.
E. Human-made CFCs retain their ability to destroy ozone molecules for seventy-five to one
hundred years.
Argument Construction
This question requires us to identify which of the answer choices would provide the strongest
reason to support the scientists' skepticism about whether the ozone layer is recovering.
This skepticism exists despite the fact that measurements of the ozone hole taken at various
times during the Antarctic spring have shown that the hole has diminished significantly from
its size at the same times the previous year.
One major reason to be skeptical would be if there were some factor other than a recovery of
the ozone layer that could reasonably account for the diminished size of the hole.
A. This does not give much reason to be skeptical that the ozone layer is recovering.
Certainly one would not want to be too hasty in declaring a recovery after noting a trend
of growth in the ozone hole's size. Nevertheless, something must account for this year's
divergence from the trend. It seems reasonable to conclude, barring the discovery of some
other factor that would explain the change, that a recovery in the ozone layer could be a
factor in this year's divergence.
B. Each of the measurements was smaller than at the same time in the previous year. This
fact would seem to indicate that fluctuations in the length of time the ozone hole persists
do not justify the scientists' skepticism.
C. The fact that CFCs—which led to the thinning of the ozone layer—have been banned
counts against the scientists' skepticism rather than supports it.
D. Correct. The hole forms when the temperature drops below –78°C during the Antarctic
spring. If much of the area where the hole appears was significantly warmer than the
previous year—perhaps above –78°C—there exists a reason other than a recovery in the
ozone layer that explains the smaller size of this year's hole in the ozone. This would
support the scientists' skepticism.
E. Even if CFCs retain their ability to destroy ozone molecules for many decades, something
must account for the decrease in the size of the hole. Nothing in the passage indicates
whether the amount of CFCs in the atmosphere has been increasing or decreasing. If it
has been decreasing for a long time, then this fact is compatible with a belief that the
ozone layer is recovering.
The correct answer is D.

CR29111.01

232. The recycling of municipal solid waste is widely seen as an environmentally preferable
alternative to the prevailing practices of incineration and of dumping in landfills. Recycling is
profitable, as the recycling programs already in operation demonstrate. A state legislator
proposes that communities should therefore be required to adopt recycling and to reach the
target of recycling 50 percent of all solid waste within 5 years.
Which of the following, if true, most seriously calls into question the advisability of
implementing the proposal?
A. Existing recycling programs have been voluntary, with citizen participation ranging from
30 percent in some communities to 80 percent in others.
B. Existing recycling programs have been restricted to that 20 percent of solid waste that,
when reprocessed, can match processed raw materials in quality and price.
C. Existing recycling programs have had recurrent difficulties finding purchasers for their
materials, usually because of quantities too small to permit cost-effective pickup and
transportation.
D. Some of the materials that can be recycled are the very materials that, when incinerated,
produce the least pollution.
E. Many of the materials that cannot be recycled are also difficult to incinerate.
Evaluation of a Plan
To answer this question, consider what information would call the advisability of
implementing this proposal into question.
A state legislator proposes that communities be required to target recycling 50 percent of all
solid waste within five years. This legislator argues for the plan in part on the basis that
recycling programs already in operation are profitable.
To weaken this argument, consider whether there might be a reason that significantly
changing the amount or types of solid waste currently disposed of would make currently
profitable recycling programs unprofitable.
A. This does not cast much doubt on the advisability of implementing the plan. If some
communities manage to get as many as 80 percent of their citizens to participate, the goal
of recycling 50 percent of all solid waste within certain communities seems attainable.
B. Correct. This suggests that increasing the percentage of solid waste that is recycled to 50
percent may result in a significant amount of recycled, reprocessed material of inferior
quality. If this material cannot match processed materials in quality and price, this may
make recycling programs no longer profitable.
C. If the problem is a result of the small quantities, increasing the quantities of materials
could alleviate the problem. Therefore, this does not cast doubt on the advisability of
implementing the plan.
D. Even if some of these materials produce little pollution, they nevertheless may produce
some pollution. It may still be beneficial to reduce or eliminate this pollution.
E. This indicates that these materials will most likely need to go into landfills. That does not
call into question the advisability of recycling those materials that can be recycled.
The correct answer is B.

CR30721.01

233. Biologist: Species with broad geographic ranges probably tend to endure longer than species
with narrow ranges. The broader a species' range, the more likely that species is to survive
the extinction of populations in a few areas. Therefore, it is likely that the proportion of
species with broad ranges tends to gradually increase with time.
The biologist's conclusion follows logically from the above if which of the following is
assumed?
A. There are now more species with broad geographic ranges than with narrow geographic
ranges.
B. Most species can survive extinctions of populations in a few areas as long as the species'
geographic range is not very narrow.
C. If a population of a species in a particular area dies out, that species generally does not
repopulate that area.
D. If a characteristic tends to help species endure longer, then the proportion of species with
that characteristic tends to gradually increase with time.
E. Any characteristic that makes a species tend to endure longer will make it easier for that
species to survive the extinction of populations in a few areas.
Argument Construction
This question asks us to find an assumption that allows the biologist's conclusion to be
logically drawn if made in conjunction with the premises of the biologist's argument.
The biologist claims that species with broader ranges are more likely to survive the extinction
of populations in certain areas than are species with narrow ranges. The biologist concludes
that over time the proportion of species with broader ranges will probably increase.
Note that the argument as it stands is not logically valid: it is possible that if species that now
have broad ranges survive the extinction of populations with narrow ranges, the proportion
of species with broad ranges could still decline. That is, decimation of populations in certain
areas may in fact cause the ranges of species that now have broad ranges to shrink in size,
thereby becoming narrow ranges.
Were this to happen at a faster pace than the extinction of species that currently have narrow
ranges, the proportion of species with broad ranges would decline rather than increase. The
correct answer to this question must rule out this possibility.
A. This choice does not rule out the possibility that the proportion of species that have broad
ranges would decline.
B. This choice does not rule out the possibility that the proportion of species that have broad
ranges would decline.
C. This choice does not rule out the possibility that the proportion of species that have broad
ranges would decline. In fact, this assumption helps to support the claim that as certain
populations of a species that once had a broad range die out, that species' range could
narrow.
D. Correct. This assumption rules out the possibility described above; furthermore, it rules
out any other possibility that allows the biologist's conclusion to be false even if the
premises were true.
E. This choice does not rule out the possibility that the proportion of species that have broad
ranges would decline.
The correct answer is D.

CR79731.01

234. Letter to the editor: If the water level in the Searle River Delta continues to drop, the rising
sea level will make the water saltier and less suitable for drinking. Currently, 40 percent of
the water from upstream tributaries is diverted to neighboring areas. To keep the delta's
water level from dropping any further, we should end all current diversions from the
upstream tributaries. Neighboring water utilities are likely to see higher costs and
diminished water supplies, but these costs are necessary to preserve the delta.
Which of the following would, if true, indicate a serious potential weakness of the suggested
plan of action?
A. Desalination equipment would allow water from the delta to be used for drinking even it
if became saltier.
B. Water level is only one factor that affects salinity in the delta.
C. The upstream tributaries' water levels are controlled by systems of dams and reservoirs.
D. Neighboring areas have grown in population since the water was first diverted from
upstream tributaries.
E. Much of the recent drop in the delta's water level can be attributed to a prolonged
drought that has recently ended.
Argument Evaluation
This question requires us to find a potential weakness of the plan suggested. That is, the
question requires us to find a reason that ending all current diversions from the Searle
River's upstream tributaries might not be a good idea.
Note that the plan is designed to keep the delta's water level from dropping further. While
there are costs to executing this plan, the letter claims that these costs are necessary to
preserve the delta.
The letter begins with a discussion of how the dropping water levels in the delta will lead to
saltier and less potable water, but it is unclear what role this plays in the letter's argument.
That is, we do not know whether this is the primary reason for the concern about the delta's
water levels. Given the information provided, it could be just one of many concerns. Given
the letter says that the costs are necessary to preserve the delta, it appears to be just one
among multiple concerns.
A. If the sole reason for wanting to keep the delta's water level from dropping further is to
ensure that the river can provide drinking water, then this answer choice might suggest a
weakness in the plan. That is, the plan itself would be unnecessary. Note, however, that
the main reason given for the plan is to preserve the delta, not to ensure drinkability of
the water. Bearing this in mind, there may still be reasons to carry out the plan even if the
claim made in this answer choice is true.
B. The plan does not necessarily need to be able to solve the problem of increased salinity on
its own. Even if it cannot, the plan may be an important part of solving the problem.
Furthermore, there may be other reasons besides salinity for implementing the plan. This
choice, therefore, does not give us a good reason to believe that the plan is not necessary.
C. This choice gives us a potential explanation of how the upstream water has been diverted.
It does not, however, present a weakness in the proposed plan.
D. This choice suggests a reason that there may be costs to implementing the proposed plan;
most clearly, the growing population will need water that it could get from the river.
Nevertheless, the letter indicates that these costs are necessary to preserve the delta.
E. Correct. This gives us reason to think that the water levels may actually fix themselves.
That is, the level will not continue to decline even if the plan is not carried out. The letter
indicates that the specific reason to carry out the plan is to prevent the water level from
dropping any further. So if the plan turned out to be unnecessary for preventing such a
drop in water level, then the costs of the plan would have made the plan itself
undesirable.
The correct answer is E.
CR47931.01

235. Researchers conditioned a group of flies to associate a particular odor with a weak electric
shock. Twenty-four and forty-eight hours later the researchers conducted tests on the flies,
both individually and in groups, to determine whether the flies retained the conditioning.
When tested individually, the flies were significantly less likely to avoid areas marked with
the odor. The researchers hypothesized that in the presence of the odor, a fly that retains the
conditioned association gives off an alarm signal that arouses the attention of any
surrounding flies, retriggering the association in them and thereby causing them to avoid the
odor.
The researchers' hypothesis requires which of the following assumptions?
A. The flies do not give off odors as alarm signals.
B. Flies that did not avoid the odor when tested individually were not merely following other
flies' movements when tested in a group.
C. Flies that did not avoid the odor when tested individually were less likely than the other
flies to avoid the odor when tested in a group.
D. Prior to their conditioning, the flies would likely have found the odor used in the
experiment to be pleasant.
E. An electric shock was used during the flies' conditioning and during the later tests.
Argument Construction
This question asks us for an assumption required by the hypothesis.
The researchers noted that the flies that had been conditioned to associate a particular odor
with an electric shock were much less likely to avoid the odor when they were tested
individually than when they were tested as a part of a group.
Based on this fact, the researchers hypothesized that the flies that had retained the
conditioning would give off an alarm when they detected the odor. This alarm would then
retrigger the association among the other flies, leading them to avoid the odor.
We must consider whether there is some other fact that could explain why the flies were
more likely to respond to the odor in a group than when they are alone. If there were, that
alternative explanation would severely call into question the researchers' hypothesis. The
hypothesis therefore requires that any such alternative explanation be false.
A. It is possible that the flies do in fact give off odors as an alarm signal. Because the nature
of the flies' alarm signal is not specified by the hypothesis, this is entirely consistent with
the hypothesis and would not undermine it. Therefore, the hypothesis does not require
that flies do not give off odors as alarm signals.
B. Correct. Suppose that the flies that did not avoid the odor when tested individually were
more likely to avoid the odor when in groups. This may be simply because these flies were
following the movements of the flies that were triggered. That is, the signal did not cause
the other flies' reactions. Instead, the movements of the triggered flies did. It follows that
this possibility must be ruled out in order for the hypothesis to be plausible, and this
answer choice does precisely that.
C. The hypothesis would actually be more plausible if this statement were false. That is, the
hypothesis would be more plausible if the flies that did not avoid the odor when tested
individually were just as likely, when they were tested in a group, to avoid the odor as the
other flies in the group.
D. The hypothesis is perfectly compatible with the assumption that, prior to their
conditioning, the flies were entirely indifferent to the odor.
E. The principle of association—that the shock is associated with the odor even when the
shock itself is not present—actually requires that this statement be false.
The correct answer is B.

CR02741.01

236. Kayla: Many people are reluctant to shop in our neighborhood because street parking is
scarce. The city plans to address this by adding parking meters with time limits that ensure
that parking spaces are generally available. But this plan will surely backfire—shoppers
dislike paying at parking meters, so most will probably drive to other neighborhoods to shop
at malls with free parking.
Which of the following, if true, would be the most logically effective rebuttal a proponent of
the city's plan could make to Kayla's objection?
A. Most shoppers dislike hunting for scarce street parking spaces much more than they
dislike paying for metered parking spaces.
B. The city could post signs with street parking time limits to ensure that parking spaces
become available without forcing shoppers to pay at meters.
C. Currently, most shoppers in the neighborhood drive only occasionally to shop at malls in
other neighborhoods.
D. The neighborhood already contains a parking lot where shoppers must pay to park.
E. The nearby malls with free parking have no parking time limits to help ensure that
parking spaces in their lots become available.
Evaluation of a Plan
Which one of the answer responses provides the best rebuttal to Kayla's objection?
Kayla tells us that a city plans to install parking meters on streets where shoppers in a
shopping neighborhood try to park. The meters would have time limits designed to ensure
increased availability of parking.
Nevertheless, Kayla believes the plan will have the opposite effect since people dislike paying
for parking and are likely to shop instead at a mall where parking is free.
It is important to determine whether shoppers are likely to be deterred from shopping at this
mall because they dislike paying to park at time-
limited meters.
A. Correct. Shoppers currently try to park on the streets where meters will be installed.
These shoppers find searching for a parking space increasingly difficult. It is possible that
shoppers actually dislike searching for parking spaces more than they dislike paying for
parking spaces. If this is the case, there is no reason to believe that these customers will
leave the neighborhood to shop elsewhere.
B. This answer choice suggests a potential alternative to installing parking meters. This does
not provide a rebuttal to Kayla's objection.
C. While many shoppers do not often drive to other neighborhoods to shop, this could
change once the meters were installed.
D. We are given no information concerning the cost or availability of parking in the parking
lot, or its proximity to the shopping portion of the neighborhood.
E. We are given no precise information about the relative availability of parking spaces at
the nearby malls. However, given Kayla's concern, it can reasonably be inferred that there
is no significant shortage of free parking spaces at these malls.
The correct answer is A.
CR78551.01

237. A new handheld device purports to determine the severity of concussions by reading the
brain's electrical signals and comparing them to a database of 15,000 scans compiled at a
brain research lab. The device is intended to help doctors decide whether an athlete who has
received a blow to the head during a competition should be sent back into the game.
Which of the following would it be most useful to establish in order to evaluate the
effectiveness of the device for its intended purpose?
A. Whether the database of brain scans will regularly be updated with new scans
B. Whether by use of this device doctors will be able to make a sound decision about
whether to allow an athlete back into the competition before it ends
C. Whether the device will be endorsed by a large number of medical professionals
D. Whether the database includes scans of non-injured athletes in the same game as the
injured athlete
E. Whether team doctors have until now been mistaken in their assessments of whether an
athlete can safely continue to play
Argument Evaluation
Which one of the answer choices would most help in determining whether the device is
effective for its intended purpose?
A handheld scanning device has been developed to read brain signals and determine the
severity of a concussion. The intended purpose of this device is to help doctors decide
whether an athlete who has received a blow to the head during competition can be allowed
back into the game.
A. If the database is regularly updated, it might increase the effectiveness of the device in the
future. This is not relevant to answering whether the device will help doctors make
medical decisions during games.
B. Correct. If the answer is yes, then the device is effective for its intended purpose. If the
answer is no, then the device is not effective for its intended purpose: to help doctors
make sound decisions about whether athletes who have suffered a head blow can safely
resume play during a game.
C. Endorsement by medical professionals might help marketing of the device, but it is not
directly relevant to deciding whether the device is technically suitable for its intended
purpose. Simply being a medical professional does not necessarily make one an expert on
such issues; that is, medical professionals in general are not involved in deciding whether
athletes who have received a head blow in a game can safely resume play in the game.
D. In order to be technically useful, the database would have to be representative of
functioning brains in a variety of activities. This includes both normally functioning
brains and abnormally functioning brains. Note that even if this condition is met, it would
not be sufficient to decide on whether the device is adequate for its intended purpose.
E. Previous mistakes by team doctors implies a genuine need for the device. However, this
issue is separate from the issue of whether the device itself can help doctors make sound
decisions. That is, while such a device may be needed, it is possible that this particular
device still does not effectively help doctors make the decision to send athletes back into
the game.
The correct answer is B.
CR30461.01

238. Mashika: We already know from polling data that some segments of the electorate provide
significant support to Ms. Puerta. If those segments also provide significant support to Mr.
Quintana, then no segment of the electorate that provides significant support to Mr.
Quintana provides significant support to Mr. Ramirez.
Salim: But actually, as the latest polling data conclusively shows, at least one segment of the
electorate does provide significant support to both Mr. Quintana and Mr. Ramirez.
Among the following statements, which is it most reasonable to infer from the assertions by
Mashika and Salim?
A. At least one segment of the electorate provides significant support neither to Mr.
Quintana nor to Mr. Ramirez.
B. At least one segment of the electorate provides significant support to Ms. Puerta but not
to Mr. Quintana.
C. Each segment of the electorate provides significant support to Ms. Puerta.
D. Each segment of the electorate provides significant support to Mr. Quintana.
E. Each segment of the electorate provides significant support to Mr. Ramirez.
Argument Construction
Which one of the five answer choices is most reasonable to infer from the information given
by Mashika and Salim?
The information given allows us to make various deductive inferences. First, if each segment
that significantly supports Ms. Puerta also significantly supports Mr. Quintana, then it
follows, based on Mashika's statement, that no segment of the electorate that significantly
supports Mr. Quintana also supports Mr. Ramirez. In other words, to support Ms. Puerta
implies not supporting Mr. Ramirez; therefore, if the segments supporting Mr. Quintana
include all the segments supporting Ms. Puerta, there can be no segments that support both
Mr. Ramirez and Mr. Quintana.
However, Salim tells us that there is such a voter segment: at least one segment significantly
supports both Mr. Quintana and Mr. Ramirez. From this, it follows that the hypothesis if
those segments also provide support to Mr. Quintana must be untrue; in other words, there
exists at least one segment of the electorate that significantly supports Ms. Puerta but not
Mr. Quintana.
A. We are told that there is a segment that significantly supports both Mr. Quintana and Mr.
Ramirez, but this provides no basis for concluding that there is also a segment that
significantly supports neither of the two.
B. Correct. The previous explanation indicates that at least one segment supports Ms.
Puerta but not Mr. Quintana.
C. If support for Ms. Puerta and support for Mr. Ramirez are mutually exclusive, then this
must be false given that we know that at least one segment supports Mr. Ramirez.
D. The information provided indicates that this is false: there is at least one segment that
significantly supports Ms. Puerta but not Mr. Quintana.
E. If support for Ms. Puerta and support for Mr. Ramirez are mutually exclusive, then this
must be false given that we know that at least some segments support Ms. Puerta.
The correct answer is B.

CR98461.01

239. The proportion of manufacturing companies in Alameda that use microelectronics in their
manufacturing processes increased from 6 percent in 1979 to 66 percent in 1990. Many labor
leaders say that the introduction of microelectronics is the principal cause of the great
increase in unemployment during that period in Alameda. In actual fact, however, most of
the job losses were due to organizational changes. Moreover, according to new figures
released by the labor department, there were many more people employed in Alameda in the
manufacturing industry in 1990 than in 1979.
Which of the following, if true, best reconciles the discrepancy between the increase in
unemployment and the increase in jobs in the manufacturing industry of Alameda?
A. Many products that contain microelectronic components are now assembled completely
by machine.
B. Workers involved in the various aspects of the manufacturing processes that use
microelectronic technology need extensive training.
C. It is difficult to evaluate numerically what impact on job security the introduction of
microelectronics in the workplace had before 1979.
D. In 1990 over 90 percent of the jobs in Alameda's manufacturing companies were filled by
workers who moved to Alameda because they had skills for which there was no demand in
Alameda prior to the introduction of microelectronics there.
E. Many workers who have retired from the manufacturing industry in Alameda since 1979
have not been replaced by younger workers.
Argument Construction
Which one of the five answer choices given best reconciles the increase in unemployment
with the increase in manufacturing employment?
In Alameda, the use of microelectronics in manufacturing increased elevenfold within 11
years. Unemployment in Alameda increased during that period but—contrary to what labor
leaders argue—most of the job losses were due to organizational changes. According to the
latest official statistics, many more people were employed in manufacturing in Alameda at
the end of that period.
One idea to consider is whether manufacturing that involves use of microelectronics in
equipment requires employees with a different mix of skills from those traditionally used by
employees carrying out manual tasks associated with manufacturing.
A. This choice provides an example of automated manufacturing, but the given information
does not specifically mention manufactured products that contain microelectronic
components.
B. The given information tells us that the increased unemployment was due to
organizational changes. But the additional information here is not a sufficient basis for
attributing the increase in unemployment to gaps in training.
C. This simply suggests that there may be some inaccurate data underlying claims about
employment trends in manufacturing in Alameda prior to 1979. But the question
concerns trends from 1979 to 1990.
D. Correct. This suggests that an influx of new workers to Alameda with the new skills
needed in manufacturing may in fact have displaced other employees. These people may
still live in Alameda yet be jobless.
E. This information is not sufficient to reconcile the increase in unemployment among those
previously employed in manufacturing with the increase in total manufacturing jobs.
Retirement of employees from certain types of occupations does not increase
unemployment in those occupations.
The correct answer is D.

CR00561.01

240. The retinas of premature babies are not fully developed at birth. Because their retinas are so
delicate, premature babies sometimes lose their sight. Methods for preventing this
syndrome, which is called retinopathy of prematurity, have improved, but the proportion of
premature babies who lose their sight because of this syndrome has increased.
Which of the following, if true, best reconciles the apparent discrepancy described above?
A. When premature babies are born, their retinas are developmentally unprepared to deal
with light and air in the environment outside the womb.
B. The oxygen that must be administered to premature babies at birth can sometimes have a
damaging effect on the babies' retinas, but the oxygen is now administered in less
damaging concentrations than it used to be.
C. The effects of retinopathy of prematurity can be reduced by controlling the exposure of
premature babies to light and oxygen, but this method cannot completely prevent the
syndrome.
D. The improvement of methods to prevent retinopathy of prematurity has been a gradual
process, and there is still a need for further knowledge.
E. Improved medical technology is saving the lives of premature babies who would
previously have died, but these babies have even more delicate retinas than do other
premature babies and are more apt to lose their sight.
Argument Construction
What could reconcile the fact that, even though technology has improved the likelihood of
saving premature babies' sight, a greater percentage of premature babies are losing their
sight than in the past?
Retinopathy of prematurity (ROP) can lead to blindness in babies born prematurely.
However, methods for preventing ROP have improved. Given that, it is puzzling that the
proportion of premature babies who go blind due to ROP has increased.
It is important to carefully distinguish among three different groups: the proportion of
premature babies who survive; the proportion who have ROP; and the proportion of those
who have ROP who become blind as a result.
A change in the proportion of premature babies that have ROP does not necessarily imply a
similar change in the proportion of ROP cases that lead to blindness. Thus, even with an
increased survival rate of premature babies and improved prevention of ROP, a greater
proportion of babies with ROP can go blind.
For example, suppose the severity of ROP at birth among surviving premature babies is
greater. Then we could expect that a greater proportion of ROP cases would lead to
blindness, absent a measure to prevent this occurring.
A. This choice does not explain why a greater proportion of ROP cases lead to blindness; it
simply suggests some of the conditions in prematurity that can cause retinopathy.
B. This choice does not resolve the apparent contradiction described; it merely suggests one
factor that can damage premature babies' retinas.
C. This choice does not resolve the apparent contradiction; it merely indicates how some
severe consequences of ROP may be moderated.
D. This choice does not resolve the apparent contradiction; it merely indicates that there is
more for medical science to discover about how to prevent or treat ROP.
E. Correct. Among the ROP cases that currently occur, a significant number of them are
more severe than in the past. This is an indirect result of the increased survival rate of
premature babies, which is also due to technological advances. The greater severity of the
ROP among those babies who have it could easily explain the greater proportion of ROP
cases that lead to blindness.
The correct answer is E.
CR60561.01

241. Although elementary school children have traditionally received considerable instruction in
creating visual art, there has been no such instruction in music. Consequently, in contrast to
the situation for visual art, most people as adults do not recognize the artistic intentions of
composers. To remedy this situation, a few educators now recommend teaching elementary
school students to compose music.
Which of the following, if true, is the strongest basis for arguing that implementation of the
recommendation will not lead to the desired result?
A. Few elementary school students are likely to create superior compositions.
B. Traditional education facilitates the appreciation of visual art, but not the recognition of
the artistic intentions of artists.
C. More people report that they enjoy music than report that they enjoy visual art.
D. Some composers have had little formal instruction in composition.
E. The recommendation is based on the results of a controlled longitudinal study conducted
in three schools within a single city.
Evaluation of a Plan
Which answer choice most suggests that implementing the recommendation will not lead to
the desired result?
Most elementary school children receive education in creating visual art but no education in
composing music. Although most adults recognize the artist's intentions in a work of visual
art, the same is not true for most adults' ability to recognize the composer's intentions in a
piece of music. Based on these facts, it is advocated that elementary school children be taught
to compose music.
It is important to note that we are not told that most adults are able to recognize an artist's
intentions specifically because of elementary-level instruction in creating visual art.
A. The goal of the recommendation is not for elementary school students to create superior
compositions in music.
B. Correct. This indicates that traditional education in visual art—including, presumably,
such education for elementary school students—does not generally result in ability to
recognize the artistic intentions of artists. By analogy, the recommended education in
music composition would be unlikely to lead to a recognition of the artistic intentions of
composers.
C. Ability to enjoy music or visual art does not necessarily imply the ability to recognize the
artistic intentions of visual artists or composers.
D. Composers' formal instruction is entirely irrelevant to the recommendation as well as to
whether the recommendation achieves its intended purpose.
E. This suggests that the recommendation is based on scientifically collected data. That does
not suggest that the recommendation would fail to achieve its intended purpose.
The correct answer is B.

CR47561.01

242. Pharmaceutical companies spend more than ever on research and development; yet the
number of new drugs patented each year has dropped since 1963. At the same time, profits—
at constant 1963 dollars—for the industry as a whole have been steadily increasing.
Which of the following, if true, is the single factor most likely to explain, at least in part, the
three trends mentioned above for money spent, drugs patented, and profits made?
A. Government regulations concerning testing requirements for novel drugs have become
steadily more stringent.
B. Research competition among pharmaceutical companies has steadily intensified as a
result of a general narrowing of research targets to drugs for which there is a large
market.
C. Many pharmaceutical companies have entered into collaborative projects with leading
universities, while others have hired faculty members away from universities by offering
very generous salaries.
D. The number of cases in which one company's researchers duplicated work done by
another company's researchers has steadily grown.
E. The advertising budgets of the major pharmaceutical companies have grown at a higher
rate than their profits have.
Argument Construction
Of the five factors indicated in the answer choices, which one most helps to explain all three
of the trends?
The information provided about the pharmaceutical industry indicates three long-term
trends since 1963: increased research spending, increased profits, and reduction in the
number of patents granted.
Note that in a case where the pharmaceutical industry produced fewer—yet more lucrative—
new drugs as a result of its increased research investment, its profits might increase even if
fewer new patents were granted.
A. Although the stringency of government regulations could help explain increased research
spending and the granting of fewer patents, it fails to explain why profits would increase
in the long term.
B. Correct. This helps explain all three factors. It suggests that each company has had to
boost its research spending, but with a narrowing of focus to produce mass-market drugs
with enormous potential for profit. This narrowing of focus could explain why fewer
drugs are being patented.
C. Collaboration between pharmaceutical companies and leading universities would not
help explain all three trends. It could indicate that the pharmaceutical companies have
reduced their investment in their in-house research infrastructure. That is, perhaps these
companies find it less expensive to farm out research to universities where a powerful
research infrastructure already exists. Even if this could help explain increased research
spending, nothing indicates that it would help explain the trends in profits or patents.
D. Companies doing similar or duplicative research could indicate certain inefficiencies in
research spending, which could help explain the research-spending trend and the trend
regarding patents. Nevertheless, it still fails to explain the trend of increasing profits.
E. This does not explain the trend regarding research spending or the trend regarding
patents.
The correct answer is B.
CR97561.01

243. Under the agricultural policies of Country R, farmers can sell any grain not sold on the open
market to a grain board at guaranteed prices. It seems inevitable that, in order to curb the
resultant escalating overproduction, the grain board will in just a few years have to impose
quotas on grain production, limiting farmers to a certain flat percentage of the grain acreage
they cultivated previously.
Suppose an individual farmer in Country R wishes to minimize the impact on profits of the
grain quota whose eventual imposition is being predicted. If the farmer could do any of the
following and wants to select the most effective course of action, which should the farmer do
now?
A. Select in advance currently less profitable grain fields and retire them if the quota takes
effect.
B. Seek long-term contracts to sell grain at a fixed price.
C. Replace obsolete tractors with more efficient new ones.
D. Put marginal land under cultivation and grow grain on it.
E. Agree with other farmers on voluntary cutbacks in grain production.
Plan Evaluation
Of the plans described in the five answer choices and equally available to the farmer, which
one would be the most effective to pursue?
It is expected that the grain board of Country R, which purchases surplus grain production
from farmers at guaranteed prices, will, within a few years, impose quotas on each farmer's
grain production in order to limit overproduction.
This plan will limit each farmer to a flat percentage of the grain acreage previously cultivated.
The quota will be calculated based on pre-existing grain acreage (presumably averaged over a
few years). Therefore, it would make the most sense for the farmer to boost grain acreage for
the next few years, even if some of the acreage increase involves using land not optimal for
grain production.
A. Selecting less profitable land now would make sense if no other course of action did.
However, it would still entail some immediate reduction in profits: the land in question is
currently less profitable, not unprofitable.
B. Long-term fixed-price contracts would presumably ensure the farmer's profitability from
grain cultivation. But that might not occur if the total cost of agricultural inputs for grain
cultivation were to significantly increase without the contracts safeguarding against such
a case. Perhaps the most important factor, however, is that such long-term contracts
could significantly limit the farmer's ability to profit from future upward trends in market
demand for grain.
C. We are given no information to help us gauge how machinery obsolescence and major
investment in new machinery might affect the profits from grain cultivation.
D. Correct. Since any quotas issued in a few years will be calculated as a percentage of the
farmer's pre-existing grain-production acreage, the farmer would benefit from increasing
his or her grain-production acreage even if some of the new acreage is suboptimal for
grain cultivation.
E. To agree with other farmers on voluntary cutbacks might help forestall or at least delay
the introduction of grain quotas by the grain board. However, it could have much the
same effect as quotas even if it is sufficient to pre-empt mandatory quotas. Furthermore,
it would carry the risk that some farmers would defect from any agreement if they
perceived an advantage in doing so.
The correct answer is D.
CR69561.01

244. Exports of United States wood pulp will rise considerably during this year. The reason for the
rise is that the falling value of the dollar will make it cheaper for paper manufacturers in
Japan and Western Europe to buy American wood pulp than to get it from any other source.
Which of the following is an assumption made in drawing the conclusion above?
A. Factory output of paper products in Japan and Western Europe will increase sharply
during this year.
B. The quality of the wood pulp produced in the United States would be adequate for the
purposes of Japanese and Western European paper manufacturers.
C. Paper manufacturers in Japan and Western Europe would prefer to use wood pulp
produced in the United States if cost were not a factor.
D. Demand for paper products made in Japan and Western Europe will not increase sharply
during this year.
E. Production of wood pulp by United States companies will not increase sharply during this
year.
Argument Construction
Which one of the statements gives an assumption on which the argument depends?
The argument claims that exports of U.S. wood pulp will increase this year. Support for this
claim is provided by the suggestion that as a result of the falling value of the dollar, paper
manufacturers in Japan and Western Europe will be able to purchase wood pulp (the raw
material for paper) more cheaply from the United States than from any other source.
This would be true because, if the dollar prices of U.S. wood pulp did not increase, firms in
Europe and Japan would pay a smaller dollar-equivalent of their own currencies.
Note that this argument can easily fail: if U.S. wood pulp does not meet the minimum quality
requirements of any paper manufacturers in Japan or Western Europe, then those
manufacturers will purchase elsewhere. It follows that the reasoning depends on assuming
that U.S. wood pulp does in fact meet those quality standards.
A. Even if factory output of paper products did not increase this year, manufacturers in
Japan and Europe might import more U.S. wood pulp this year as raw materials for next
year's production.
B. Correct. The predicted increase would likely not occur unless U.S. wood pulp met the
manufacturers' minimum-quality standards.
C. The reasoning does not have to assume that cost is the sole factor. However, it clearly
assumes that cost is an important factor. Nevertheless, it does not make any assumption
regarding what the relative importance of cost and other factors might be. For this
reason, it neither assumes nor implies what might happen in the case that cost is not a
significant factor.
D. The reasoning does not need to assume that no such sharp increase will occur this year.
In fact, if such an increase were to occur, it would be even more likely that exports of U.S.
wood pulp would increase this year if U.S. wood pulp became internationally more
competitive on price this year.
E. Given the prediction that U.S. wood-pulp exports will increase, there is no reason to
assume no sharp increase in U.S. wood pulp production during this year. Such an
increase might even be likely if exports were to rise considerably. Therefore, option E is
not assumed by the argument.
The correct answer is B.

CR79561.01

245. A company's personnel director surveyed employees about their satisfaction with the
company's system for awarding employee performance ratings. The survey data indicated
that employees who received high ratings were very satisfied with the system. The personnel
director concluded from these data that the company's best-performing employees liked the
system.
The personnel director's conclusion assumes which of the following?
A. No other performance rating system is as good as the current system.
B. The company's best-performing employees received high ratings.
C. Employees who received low ratings were dissatisfied with the system.
D. Employees who receive high ratings from a performance-rating system will like that
system.
E. The company's best-performing employees were motivated to perform well by the
knowledge that they would receive performance ratings.
Argument Construction
Which one of the following states an assumption on which the personnel director's
conclusion depends?
The assumption we seek must provide a needed logical connection between the given
information and the conclusion drawn. From the information given, it seems reasonable to
think that the employees very satisfied with the system also liked the system. Furthermore,
we are told that the employees who received high ratings were very satisfied with the system.
It follows that if the company's best-performing employees also received high ratings, then
these same people—the best-performing employees—were very satisfied with the system.
This would make it reasonable to conclude that the best-performing employees liked the
system.
A. The argument does not address the issue of whether the existing performance-rating
system is the best available. Rather, it draws a conclusion about the existing performance-
rating system based on specific data generated by the system.
B. Correct. Assuming that the company's best-performing employees received high ratings
enables the personnel director's conclusion to be logically drawn.
C. This choice describes an association between ratings and satisfaction, rather than an
association between performance and ratings, or between performance and satisfaction.
The information given in the argument is compatible with the claim that all employees
were at least somewhat satisfied with the system.
D. This choice assumes that employees who are rated highly by a system will like that
system, but that assumption is more general than anything assumed in the argument.
This choice addresses employees' attitudes to a performance-rating system based on the
ratings they receive under that system. However, it does not address any association
between ratings, performance, and attitudes to the system.
E. Employees' satisfaction with a performance-rating system might well play a role in
motivating employees. However, the issue of motivation figures neither explicitly nor
implicitly in the reasoning of the personnel director.
The correct answer is B.
CR00661.01

246. There are fundamentally two possible changes in an economy that will each cause inflation
unless other compensating changes also occur. These changes are either reductions in the
supply of goods and services or increases in demand. In a pre-banking economy the quantity
of money available, and hence the level of demand, is equivalent to the quantity of gold
available.
If the statements above are true, then it is also true that in a pre-banking economy
A. any inflation is the result of reductions in the supply of goods and services
B. if other factors in the economy are unchanged, increasing the quantity of gold available
will lead to inflation
C. if there is a reduction in the quantity of gold available, then, other things being equal,
inflation must result
D. the quantity of goods and services purchasable by a given amount of gold is constant
E. whatever changes in demand occur, there will be compensating changes in the supply of
goods and services
Argument Construction
This question asks us which of the statements is most strongly supported by the
information in the argument. Given no other relevant changes, two factors can cause
inflation: a reduction in market supply of goods and services or an increase in market
demand. The argument also indicates that the total quantity of money available—or, in a pre-
banking economy, the quantity of gold available—determines market demand. Therefore, in
a pre-banking economy, an increase in the quantity of gold available will increase demand. In
a situation where supply remains constant, this increases demand for this fixed supply,
thereby raising prices. In other words, increasing the quantity of gold in a pre-banking
economy will cause inflation.
A. While this may be true in certain cases, it is not the argument made in the passage. The
passage indicates that certain instances of inflation are caused by increased demand
stimulated by an increase in available money (or gold).
B. Correct. According to the information in the passage, if the quantity of available gold in
a pre-banking economy increases while supply of goods and services remains unchanged,
demand for goods and services will increase relative to supply. This imbalance raises
prices for the supply; that is, it causes inflation.
C. This answer suggests the opposite of the information in the passage. While the
information in the passage indicates that an increase in the quantity of available gold may
cause inflation, this choice suggests that a reduction in the available amount of gold will
cause inflation.
D. This suggestion is contrary to the information in the passage: the passage suggests that in
a pre-banking economy, the total available amount of gold determines the amount that a
good or service will cost. This answer choice suggests that the total available amount of
gold is irrelevant to the cost of given goods or services.
E. The passage nowhere indicates that economies will compensate for changes in demand by
changing available supply. This suggestion may or may not be true in real-world terms,
but there is no information in the passage to support it.
The correct answer is B.
CR20661.01

247. Clearbell Telephone provides slow-dialing (SD) service to customers for a low fee, and fast-
dialing (FD) service to other customers who pay a somewhat higher fee. FD technology,
however, is so efficient that it costs Clearbell substantially less per average call to provide
than does SD. Nonetheless, accountants have calculated that Clearbell's profits would drop if
it provided FD to all its customers at the current low-fee rate.
Assume that installation costs for FD are insignificant if the customer already has SD service.
Which of the following, if true about Clearbell, best explains the results of the accountants'
calculation?
A. The extra revenue collected from customers who pay the high fee is higher than the extra
cost of providing SD to customers who pay the low fee.
B. The low fee was increased by 6 percent last year, whereas the higher fee was not increased
last year.
C. Although 96 percent of customers regard FD service as reliable and more convenient than
SD, fewer than 10 percent of them choose to pay the higher fee for FD service.
D. The company's competitors generally provide business customers with FD service at low-
fee rates.
E. Profits rose slightly each month for the first three months after FD was first offered to
customers, then fell slightly each month for the succeeding three months.
Argument Construction
The argument suggests that fast-dialing (FD) service costs Clearbell Telephone less to deliver
per call than does slow-dialing (SD) service, which Clearbell delivers at a lower fee. There are
no significant extra costs such as installation to switch to FD if a customer is already a user of
SD. Nevertheless, accountants expect Clearbell's profits to decrease if the company were to
provide the high-priced FD service at the lower SD rate. Our goal here will be to find a
reason for this apparent contradiction.
A. Correct. This answer provides a plausible reason for the contradiction. Ultimately, we do
not know the amount of extra fee that FD customers pay relative to SD customers. If the
higher FD fees make up a substantial portion of the company's revenues, then it is very
possible that the proposed change would reduce revenues significantly enough to lower
profits. Remember, the SD service actually costs more for Clearbell Telephone. Therefore,
the balance to find is whether the current higher FD fees generate more revenue than the
money saved by eliminating SD service and instead providing FD services at the low fee.
B. This statement provides information about how Clearbell's current prices were set.
However, it provides no information as to how the proposed changes might affect profits.
C. This statement does not explain the results of the accountants' calculations. First,
customers' preference is irrelevant to the accountants' results. Second, if we were to
assume that 10 percent is a small figure, it is still possible that FD fees are great enough to
offset the extra costs Clearbell incurs by providing SD service. Third, since the cost to
Clearbell is less per call using FD service, Clearbell's profits may in fact increase if all
customers were to be given FD service.
D. This statement is outside the scope of the argument. First, practices of Clearbell's
competitors have no bearing on the accountants' calculations. Second, we have no way to
determine what the change in Clearbell's profits and its competitive position might be if
Clearbell were to provide FD service at the low-fee rate to its business customers. Third,
given the facts provided in the argument, it is entirely possible that Clearbell already gives
preferential rates for FD service to its business customers.
E. This information does not help explain the results of the accountants' calculations. Simply
because the changes in profit and the introduction of FD service happened at roughly the
same time, we cannot assume that one caused the other. That is, it is possible that these
fluctuations in profit are due to normal, perhaps seasonal, fluctuations in profits.
Therefore, these fluctuations would not necessarily negatively impact Clearbell's overall
level of profitability.
The correct answer is A.

CR23661.01

248. Manufacturers sometimes discount the price of a product to retailers for a promotion period
when the product is advertised to consumers. Such promotions often result in a dramatic
increase in amount of product sold by the manufacturers to retailers. Nevertheless, the
manufacturers could often make more profit by not holding the promotions.
Which of the following, if true, most strongly supports the claim above about the
manufacturers' profit?
A. The amount of discount generally offered by manufacturers to retailers is carefully
calculated to represent the minimum needed to draw consumers' attention to the
product.
B. For many consumer products the period of advertising discounted prices to consumers is
about a week, not sufficiently long for consumers to become used to the sale price.
C. For products that are not newly introduced, the purpose of such promotions is to keep the
products in the minds of consumers and to attract consumers who are currently using
competing products.
D. During such a promotion retailers tend to accumulate in their warehouses inventory
bought at discount; they then sell much of it later at their regular price.
E. If a manufacturer fails to offer such promotions but its competitor offers them, that
competitor will tend to attract consumers away from the manufacturer's product.
Argument Evaluation
Which of the answer choices provides the strongest evidence for the claim that the
manufacturers could indeed make more profit by not holding promotions?
To promote a product during a period when it is advertised to consumers, manufacturers
sometimes sell the product to retailers at a discounted price. This often results in a large
boost in manufacturers' sales to retailers. However, we are told that manufacturers could
make more profit by not offering the promotions.
The manufacturers' total profits on the product increase during the period of reduced-price
sales to retailers; note that the price reduction results in a dramatic increase in the volume
of sales to retailers.
Nevertheless, it is possible that this temporary increase in sales volume would reduce future
sales volume to retailers at non-discounted prices. In such a case, it would be more likely that
the manufacturers' overall profits on the product would be reduced.
A. It is reasonable for manufacturers to calculate the minimum needed to draw attention to
a product if they wished to minimize the costs of the product promotion. However, this
information gives little if any support for the claim that manufacturers could make more
profit by not discounting prices to retailers.
B. The argument does not provide any information to suggest that the length of the
promotion affects manufacturers' profits. According to the passage, many promotions last
a short time, so consumers do not come to routinely expect a lower price on a product and
thereby avoid purchasing it at the higher post-promotion price. However, this new
information gives little if any support for the claim that manufacturers could make more
profit by not discounting prices to retailers.
C. This choice implies that the manufacturer would risk making less overall profit, not more,
if these promotions were not held.
D. Correct. The retailer profits by purchasing large volumes of the product at a
manufacturer's discounted price and selling it to consumers at the higher post-promotion
price. It follows that the increase in sales at the discount might in fact detract from non-
discount sales. As noted above, this gives the strongest support for the claim about the
manufacturers' profits.
E. This suggests that sales promotions are essential for manufacturers to compete in
relevant markets. However, it gives little if any support for the claim made about
manufacturers' profits.
The correct answer is D.
CR33661.01

249. Advertisement: Today's customers expect high quality. Every advance in the quality of
manufactured products raises customer expectations. The company that is satisfied with the
current quality of its products will soon find that its customers are not. At MegaCorp,
meeting or exceeding customer expectations is our goal.
Which of the following must be true on the basis of the statements in the advertisement
above?
A. MegaCorp's competitors will succeed in attracting customers only if those competitors
adopt MegaCorp's goal as their own.
B. A company that does not correctly anticipate the expectations of its customers is certain
to fail in advancing the quality of its products.
C. MegaCorp's goal is possible to meet only if continuing advances in product quality are
possible.
D. If a company becomes satisfied with the quality of its products, then the quality of its
products is sure to decline.
E. MegaCorp's customers are currently satisfied with the quality of its products.
Argument Construction
This question requires us to identify a statement that can be inferred on the basis of the
statements made in the advertisement.
The advertisement states that every improvement in the quality of manufactured products
leads to increased expectations among customers, yet that MegaCorp has a goal of meeting or
exceeding its customers' expectations.
It follows that MegaCorp will continually be able to meet this goal only if it is possible for it
to continue improving the quality of its products.
A. The information in the advertisement provides support for the idea that customers are
more likely to buy products that are of the highest available quality. However, there is
nothing in the advertisement to indicate that the only way for any of MegaCorp's
competitors to produce the highest quality product would be to adopt MegaCorp's goal.
B. There is nothing in the advertisement to support this claim. A company could, for
instance, improve the quality of a product in ways that are different from what customers
expect.
C. Correct. To meet or exceed ever-increasing demands for improved quality requires that
continuing advances in product quality are in fact possible.
D. The advertisement suggests that a company that is satisfied with the quality of its
products can, at the very least, maintain the current quality of its products. There is
nothing to suggest that the quality of these products will actually decline.
E. The passage provides no evidence that MegaCorp has been successful at reaching its goal
of meeting or exceeding customer expectations.
The correct answer is C.
Answer Explanations Verbal Reasoning
Sentence Correction
Communication
SC21011.01

250. The prime lending rate is a key rate in the economy: not only are the interest rates on most
loans to small and medium-sized businesses tied to the prime, but also on a growing number
of consumer loans, including home equity loans.
A. not only are the interest rates on most loans to small and medium-sized businesses tied to
the prime, but also on
B. tied to the prime are the interest rates not only on most loans to small and medium-sized
businesses, but also on
C. the interest rates not only on most loans to small and medium-sized businesses are tied to
the prime, but also
D. not only the interest rates on most loans to small and medium-sized businesses are tied to
the prime, but also on
E. the interest rates are tied to the prime, not only on most loans to small and medium-sized
businesses, but also
Logical predication; Parallelism
The sentence tells us that the interest rates on two classes of loans are affected by the prime
lending rate. The structure not only . . . but also . . . , consisting of two adverbial modifiers,
conveys this.
The position of the two modifiers is crucial in order to know what each modifies.
Furthermore, it is essential that the two things modified be parallel.
A. In this choice, not only are the interest rates . . . but also on is not parallel.
B. Correct. This choice uses a correctly parallel form not only on . . . but also on . . .
C. In this choice, the interest rates not only on . . . but also is not parallel.
D. In this choice, not only the interest rates on . . . are . . . is idiomatically incorrect: when
not only begins an independent statement, the order of subject and verb in the not only
part must be inverted as in not only are . . .
E. In this choice, the interest rates not only on . . . but also is not parallel.
The correct answer is B.

SC83811.01

251. Lacking information about energy use, people tend to overestimate the amount of energy
used by equipment, such as lights, that are visible and must be turned on and off and
underestimate that used by unobtrusive equipment, such as water heaters.
A. equipment, such as lights, that are visible and must be turned on and off and
underestimate that
B. equipment, such as lights, that are visible and must be turned on and off and
underestimate it when
C. equipment, such as lights, that is visible and must be turned on and off and
underestimate it when
D. visible equipment, such as lights, that must be turned on and off and underestimate that
E. visible equipment, such as lights, that must be turned on and off and underestimate it
when
Rhetorical construction; Agreement
This sentence, as worded, has a subject-verb agreement problem: the noun equipment, not
lights, serves as the subject of the verb are; because equipment is singular, is should be used.
Furthermore, the sentence would read more clearly if visible equipment were used rather
than equipment . . . that [is] visible. This would create a less wordy sentence that has a more
parallel structure, in which visible equipment is contrasted with unobtrusive equipment.
A. This version is flawed for the reasons given above.
B. This version has the same subject-verb agreement problem as in A. Furthermore, it is
unclear what the referent of the pronoun it is here. Finally, this version, like A, is wordy,
and its structure is not parallel.
C. This version correctly uses the singular verb is. However, as in choice B, the pronoun it
has no clear referent. Finally, this version, like A and B, is wordy, and its structure is not
parallel.
D. Correct. This version is free of subject-verb agreement errors and its structure is
parallel.
E. Although this version is free of subject-verb agreement errors and appropriately uses a
parallel structure, it retains the problem from choices B and C: the pronoun it has no
clear referent.
The correct answer is D.
SC37561.01

252. Evidence of some shifts in the character of violence on television is emerging from a new
study of 500 television programs by the Center for Media and Public Affairs, a nonprofit
research center in Washington, D.C., a study that is underwritten by a number of educational
institutions.
A. programs by the Center for Media and Public Affairs, a nonprofit research center in
Washington, D.C., a study that is underwritten by a number of educational institutions
B. programs by the Center for Media and Public Affairs, a nonprofit research center in
Washington, D.C., and it is underwritten by a number of educational institutions
C. programs underwritten by a number of educational institutions and conducted by the
Center for Media and Public Affairs, a nonprofit research center based in Washington,
D.C.
D. programs, a study underwritten by a number of educational institutions and conducted
by the Center for Media and Public Affairs, a nonprofit research center in Washington,
D.C.
E. programs, a study conducted by the Center for Media and Public Affairs, a nonprofit
research center based in Washington, D.C., and it is underwritten by a number of
educational institutions
Logical predication; Rhetorical construction
The sentence gives three pieces of information: that a new study suggests the nature of
violence on television is changing; that it was funded by educational institutions; and that it
was conducted by the Center for Media and Public Affairs, a nonprofit research center.
Our task is to order this information logically and grammatically. The most obvious defect in
the given sentence is that the long phrase a study . . . institutions describing how the study
was funded is placed too far from the first mention of the study (a new study), with too much
distracting information in between.
A. This choice is incorrect for the reasons stated above.
B. In this choice, it is ambiguous: it could, for example, refer to the evidence or the new
study.
C. In this choice, the modifying phrases underwritten by . . . and conducted by . . . appear to
modify programs.
D. Correct. This choice clearly organizes the information in the sentence, with no
ambiguity.
E. The pronoun it is ambiguous: it could refer to the new study or the evidence.
The correct answer is D.
SC48461.01

253. Judge Lois Forer's study asks why do some litigants have a preferred status over others in the
use of a public resource, the courts, which in theory are available to all but in fact are
unequally distributed among rich and poor.
A. do some litigants have a preferred status over others in the use of a public resource, the
courts, which in theory are available to all but in fact are unequally distributed among
B. some litigants have a preferred status over others in the use of a public resource, the
courts, which in theory are available to all but in fact are unequally distributed between
C. do some litigants have a preferred status over another in the use of a public resource, the
courts, in theory available to all but in fact are unequally distributed among
D. some litigants have a preferred status to another in the use of a public resource, the
courts, in theory available to all but in fact not equally distributed between
E. does one litigant have a preferred status over the other in the use of a public resource, the
courts, in theory available to all but in fact they are not equally distributed among
Diction; Parallelism
The word do between why and some is unnecessary here. It would be used if we were to
present the question in quotation marks, for example: “Why do some litigants . . . ?”
However, when simply reporting that a person has asked the question, we simply say X asks
why some litigants.
Additionally, the sentence appears to be attempting to say that the rich have better access to
courts than the poor. Given that there are only two groups, the appropriate preposition is
between (used for two and only two things) rather than among (used for three or more
things). That is, the sentence would more appropriately read unequally distributed between
rich and poor.
A. This choice is flawed for the reasons indicated above.
B. Correct. This version of the sentence uses the appropriate preposition between and
clearly conveys the intended idea.
C. This choice has the same flaws as choice A. Additionally, remember that the sentence
attempts to communicate that some litigants have preferred status over some others
rather than simply over one specific person. For this reason, over another is incorrect;
the correct wording would be over others. Finally, the wording of the final clause is
unacceptable; which in theory are available to all but in fact are unequally distributed is
preferable.
D. The correct preposition to use after preferred status is over, not to. Additionally, as in
choice C, others rather than another would be correct. Finally, the sentence would be
more clearly worded if unequally rather than not equally had been used.
E. As in choice A, the use of does is unnecessary. Note also that the question regarding the
preferred status in the use of a public resource is not about two particular litigants, but
about litigants in general. Therefore, one litigant . . . over the other does not clearly
convey the intended meaning. Furthermore, the use of does is unnecessary, and the final
clause would be better worded which in theory are available to all but in fact are
unequally distributed between than what is found here.
The correct answer is B.

SC30561.01

254. During an ice age, the buildup of ice at the poles and the drop in water levels near the
equator speed up the earth's rotation, like a spinning figure skater whose speed increases
when her arms are drawn in.
A. like a spinning figure skater whose speed increases when her arms are drawn in
B. like the increased speed of a figure skater when her arms are drawn in
C. like a figure skater who increases speed while spinning with her arms drawn in
D. just as a spinning figure skater who increases speed by drawing in her arms
E. just as a spinning figure skater increases speed by drawing in her arms
Logical predication; Parallelism
Each of the incorrect answer choices makes a faulty comparison.
A. Grammatically, the comparison here is between the buildup of ice . . . and the drop in
water levels . . . on one hand and a spinning figure skater on the other. However, the
appropriate comparison would be either between the figure skater and earth, or between
the skater's rotation and the earth's rotation.
B. Grammatically, the comparison here is between the buildup of ice . . . and the drop in
water levels . . . on one hand and the increased speed of a figure skater on the other.
However, the appropriate comparison would be between the increased speed of earth's
rotation and the increased speed of a figure skater. Furthermore, note that this answer
choice nowhere suggests that the skater is spinning.
C. Grammatically, as in choice A, the comparison here is between the buildup of ice . . . and
the drop in water levels . . . on one hand and a spinning figure skater on the other.
However, the appropriate comparison would be either between the figure skater and
earth, or between the skater's rotation and earth's rotation.
D. When just as is used to mean in the same way as, it must link two independent clauses.
The clause beginning who increases speed . . . is a dependent clause.
E. Correct. In this version just as is appropriately used to create a comparison between the
way the buildup of ice at the poles, along with the drop in water levels at the equator,
increases the speed of earth's rotation and the way a figure skater increases the speed of
her spin by drawing in her arms
The correct answer is E.

SC01561.01

255. Added to the increase in hourly wages requested last July, the railroad employees are now
seeking an expanded program of retirement benefits.
A. Added to the increase in hourly wages requested last July, the railroad employees are now
seeking an expanded program of retirement benefits.
B. Added to the increase in hourly wages which had been requested last July, the employees
of the railroad are now seeking an expanded program of retirement benefits.
C. The railroad employees are now seeking an expanded program of retirement benefits
added to the increase in hourly wages that were requested last July.
D. In addition to the increase in hourly wages that were requested last July, the railroad
employees are now seeking an expanded program of retirement benefits.
E. In addition to the increase in hourly wages requested last July, the employees of the
railroad are now seeking an expanded program of retirement benefits.
Logical predication; Agreement
The phrase Added to the increase in hourly wages . . .
modifies what immediately follows the comma, the railroad employees. This would mean
that the railroad employees themselves are being added to the increase in hourly wages,
which is absurd.
The idea the sentence is intended to convey is that the railroad employees are now seeking to
have an expanded program of retirement benefits added to the increase in hourly earnings
requested last July.
The correct answer will be the choice that most clearly, idiomatically, and grammatically
expresses this idea.
A. As indicated above, the phrase added to the increase in hourly wages requested last July
. . . incorrectly modifies the railroad employees.
B. The phrase Added to the increase in hourly wages which had been requested last July
modifies the railroad employees, which is not what is intended.
C. Unlike the prior two choices, this choice does not make any modification errors. However,
seeking an expanded program . . . added is not correct; seeking to have an expanded
program . . . added would be acceptable. In the other answer choices, seeking most
nearly means trying to acquire. In this choice, however, because it is used in conjunction
with another verb (added), seeking most nearly means trying. When used in this sense,
seeking should be followed by an infinitive.
D. Like C, but unlike A and B, this choice does not have any modification errors. However, it
has an agreement flaw: the noun the increase in hourly wages is singular, so the verb
must be was requested rather than were requested.
E. Correct. This version clearly and correctly conveys the intended idea.
The correct answer is E.

SC21561.01

256. The use of gravity waves, which do not interact with matter in the way electromagnetic waves
do, hopefully will enable astronomers to study the actual formation of black holes and
neutron stars.
A. in the way electromagnetic waves do, hopefully will enable
B. in the way electromagnetic waves do, will, it is hoped, enable
C. like electromagnetic waves, hopefully will enable
D. like electromagnetic waves, would enable, hopefully
E. such as electromagnetic waves do, will, it is hoped, enable
Rhetorical Construction; Diction
While the use of hopefully, as used in the sentence as written, to mean I hope that or let's
hope that, is well established, this use is considered to be informal. For this reason, it is
generally avoided in writing.
When it is used in this way in writing, however, it must be used at the beginning of a
sentence or at the beginning of an independent clause, and be set off by the use of commas.
In any case, the use of hopefully in this sentence is incorrect: in a technical sense, it implies
that the enabling of the astronomers will be hopeful.
A. The use of hopefully here is incorrect, for the reasons given above.
B. Correct. The use of it is hoped (as opposed to hopefully) here properly conveys the
intended meaning of the sentence. Furthermore, the use of in the way appropriately
conveys, unlike the answer choices using like or such as, that what is being compared is
the interaction, or lack thereof, of gravity waves, on one hand, and electromagnetic waves,
on the other, with matter.
C. The use of like incorrectly implies that electromagnetic waves are a type of matter.
Furthermore, the use of hopefully is incorrect, for the reasons given above.
D. The use of like incorrectly implies that electromagnetic waves are a type of matter.
Furthermore, the use of hopefully is incorrect, for the reasons given above.
E. The use of such as is inappropriate here: first, such as must be followed by a noun, or list
of nouns, rather than a noun followed by a verb, as in electromagnetic waves do; second,
the use of like incorrectly implies that electromagnetic waves are a type of matter.
The correct answer is B.
SC61561.01

257. Many of them chiseled from solid rock centuries ago, the mountainous regions of northern
Ethiopia are dotted with hundreds of monasteries.
A. Many of them chiseled from solid rock centuries ago, the mountainous regions of
northern Ethiopia are dotted with hundreds of monasteries.
B. Chiseled from solid rock centuries ago, the mountainous regions of northern Ethiopia are
dotted with many hundreds of monasteries.
C. Hundreds of monasteries, many of them chiseled from solid rock centuries ago, are
dotting the mountainous regions of northern Ethiopia.
D. The mountainous regions of northern Ethiopia are dotted with hundreds of monasteries,
many of which are chiseled from solid rock centuries ago.
E. The mountainous regions of northern Ethiopia are dotted with hundreds of monasteries,
many of them chiseled from solid rock centuries ago.
Logical predication; Verb form
This sentence should indicate that hundreds of monasteries were chiseled from solid rock.
As written, however, the phrase at the beginning of the sentence modifies the noun that
immediately follows the comma: therefore, as worded, the sentence states that the
mountainous regions of northern Ethiopia were what was chiseled from solid rock.
A. This choice is incorrect for the reasons described above.
B. This choice, like A, has an incorrect placement of its modifying phrase. As a result, it too
says that the mountainous regions are what were chiseled from solid rock. The regions
themselves were not chiseled; it was the monasteries.
C. The present continuous are dotting suggests that the dotting is something that is actively
occurring at this moment. It would be more idiomatically appropriate to use the present
tense dot.
D. The chiseling took place centuries ago. This requires the past were chiseled rather than
the present tense are chiseled.
E. Correct. This version of the sentence clearly states the intended meaning: many of the
monasteries that dot the mountainous regions of northern Ethiopia were chiseled from
solid rock centuries ago.
The correct answer is E.

SC81561.01

258. Plausible though it sounds, the weakness of the hypothesis is that it does not incorporate all
relevant evidence.
A. Plausible though it sounds, the weakness of the hypothesis
B. Even though it sounds plausible, the weakness of the hypothesis
C. Though plausible, the hypothesis' weakness
D. Though the hypothesis sounds plausible, its weakness
E. The weakness of the hypothesis which sounds plausible
Logical predication; Rhetorical construction
The phrase plausible though it sounds modifies the noun that comes immediately after the
comma, namely, the weakness of the hypothesis. As a result, the sentence says that the
weakness itself is plausible. It is reasonably clear, however, that the sentence is intended to
indicate that the hypothesis sounds plausible, not that the weakness does. Additionally, the
referent of the pronoun it is ambiguous. The referent could be either weakness or hypothesis.
A. This choice suffers from the above errors.
B. This choice also inappropriately says that the weakness is what is plausible.
C. This choice also inappropriately says that the weakness is what is plausible
D. Correct. This version appropriately describes the hypothesis itself as plausible.
E. The phrase which sounds plausible should be set off with commas, or else which should
be replaced with that. Furthermore, this choice fails rhetorically in that it does not covey,
as it should, how surprising it is that the plausible-sounding hypothesis has this
weakness.
The correct answer is D.

SC32561.01

259. In despite of the steady population flow out from rural areas into urban clusters, nearly 5
million farm households are still in Japan out of a total population of some 116 million
people.
A. In despite of the steady population flow out from rural areas into urban clusters, nearly 5
million farm households are still in Japan
B. In spite of the steady population flow out from rural areas into urban clusters, nearly 5
million farm households are still in Japan
C. Despite the steady population flow from rural areas into urban clusters, Japan's farm
households are still nearly 5 million
D. Despite the steady population flow from rural areas to urban clusters, there are still
nearly 5 million farm households in Japan
E. In Japan, despite the steady population flow out from rural areas into urban clusters, still
there are nearly 5 million farm households
Rhetorical construction; Diction
In despite of is idiomatically incorrect; either in spite of or despite is acceptable. Also, nearly
5 million farm households are still in Japan is stated confusingly; the idea is not that the
farm households are leaving Japan itself, but rather that people are leaving rural areas. The
sentence would be more appropriately phrased if it said there are still 5 million farm
households in Japan.
A. This choice is flawed for the reasons given above.
B. In spite of is correct, but, as in A, nearly 5 million . . . are still in fails to convey the
intended meaning clearly.
C. Japan's farm households are still nearly 5 million is confusingly phrased. As worded, it
seems to indicate that there are 5 million people on farms in Japan rather than that the
number of farm households in Japan is 5 million. The intended meaning would be better
conveyed if it said there are still nearly 5 million farm households in Japan.
D. Correct. This choice conveys the intended idea clearly and is grammatically correct.
E. The time indicator still should be placed after the verb are. That is, the appropriate
sentence should read there are still nearly . . . As worded here, still could be read in a way
that does not indicate time, but rather means something like nevertheless, which would
be redundant, given the earlier occurrence of despite.
The correct answer is D.
SC52561.01

260. Financial uncertainties from the accident at Three Mile Island may prove even more
deterring to the nuclear industry than political opposition is.
A. from the accident at Three Mile Island may prove even more deterring to the nuclear
industry than political opposition is
B. from the accident at Three Mile Island may prove to be even more serious a deterrent to
the nuclear industry than political opposition
C. from the accident at Three Mile Island may prove to be an even more serious deterrent to
the nuclear industry than political opposition
D. resulting from the accident at Three Mile Island may prove to be an even more serious
deterrent to the nuclear industry than is political opposition
E. resulting from the accident at Three Mile Island may prove even more deterring to the
nuclear industry than political opposition
Grammatical construction; Parallelism
First, deterring to the nuclear industry is not correct, as deter is a transitive verb and
requires a direct object. Furthermore, while prove does not have to be followed by to be, here
to be is required for parallelism with political opposition is. Finally, even though financial
uncertainties from is not strictly incorrect, financial uncertainties resulting from would be
clearer.
A. This choice is flawed for the reasons above.
B. As worded, this choice can be read as comparing the degree to which the financial
uncertainties are a deterrent with the degree to which the financial uncertainties are
political opposition. To fix this, we must insert the verb is either before or after political
opposition. Furthermore, the sentence would be clearer if it said resulting from. Finally,
even more serious a deterrent would more appropriately read an even more serious
deterrent.
C. The comparison is again faulty as in choice B. Also, the sentence would be clearer if it said
resulting from.
D. Correct. This choice clearly conveys the intended meaning. While it is possible to argue
that is would be better placed after political opposition, it is technically correct in either
place. Realistically, this issue is minor compared to those in each of the other choices.
E. Deterring to the nuclear industry is not correct, as deter is a transitive verb and requires
a direct object.
The correct answer is D.

SC72561.01

261. Remembered almost as an epic among America's 12,000 Bosnian Muslims is the digging of
Chicago's subway tunnels in the early 1900s, one of the proudest of family legends.
A. Remembered almost as an epic among America's 12,000 Bosnian Muslims is the digging
of Chicago's subway tunnels in the early 1900s, one of the proudest of family legends.
B. Almost an epic among America's 12,000 Bosnian Muslims is the digging in the early
1900s of Chicago's subway tunnels, one of the proudest of family legends.
C. Digging Chicago's subway tunnels in the early 1900s, America's 12,000 Bosnian Muslims
remember it almost as an epic and it is the one of the proudest of family legends.
D. America's 12,000 Bosnian Muslims remember almost as an epic the digging of Chicago's
subway tunnels in the early 1900s, one of the proudest of family legends.
E. One of the proudest of family legends, remembered almost as an epic among America's
12,000 Bosnian Muslims, is the digging of Chicago's subway tunnels in the early 1900s.
Logical predication; Rhetorical construction
An epic is a work of art or legend conveying heroic deeds. Therefore, presumably what is
meant to be remembered almost as an epic is the family legend relating to the digging of
Chicago's subway tunnels in the early 1900s. However, as it is worded, the sentence states
that the digging itself is remembered almost as an epic.
Furthermore, it is not immediately clear what one of the proudest of family legends
modifies. Whenever possible, it is best to place a phrase close to what it modifies.
A. This choice is incorrect for the reasons described above.
B. This choice does not describe the family legend as almost an epic. Instead, it describes
the digging of the subways that way. Additionally, as worded, the sentence seems to say
that Chicago's subway tunnels themselves are one of the proudest of family legends.
C. In this choice, the phrase digging Chicago's subway tunnels in the early 1900s modifies
America's 12,000 Bosnian Muslims. However, it was an earlier generation who dug the
tunnels, not the 12,000 Bosnian Muslims currently living in America. Additionally, as in
previous choices, this version of the sentence also describes the digging rather than the
legend as being remembered almost as an epic.
D. This choice also incorrectly describes the digging of the subway tunnels, not the family
legend, as being almost as an epic.
E. Correct. This version clearly states that it is the family legend that is remembered
almost as an epic.
The correct answer is E.
SC92561.01
262. Like the one reputed to live in Loch Ness, also an inland lake connected to the ocean by a
river, inhabitants of the area around Lake Champlain claim sightings of a long and narrow
“sea monster.”
A. Like the one reputed to live in Loch Ness, also an inland lake connected to the ocean by a
river, inhabitants of the area around Lake Champlain claim sightings of a long and
narrow “sea monster.”
B. Inhabitants of the area around Lake Champlain claim sightings of a long and narrow “sea
monster” similar to the one reputed to live in Loch Ness, which, like Lake Champlain is an
inland lake connected to the ocean by a river.
C. Inhabitants of the area around Lake Champlain claim sightings of a long and narrow “sea
monster” similar to Loch Ness's, which, like Lake Champlain, is an inland lake connected
to the ocean by a river.
D. Like Loch Ness's reputed monster, inhabitants of the area around Lake Champlain, also
an inland lake connected to the ocean by a river, claim sightings of a long and narrow "sea
monster.”
E. Similar to that reputed to live in Loch Ness, inhabitants of the area around Lake
Champlain, also an inland lake connected to the ocean by a river, claim sightings of a long
and narrow “sea monster.”
Logical predication; Rhetorical construction
As written, the phrase Like the one reputed to live in Loch Ness . . . compares the one to what
comes immediately after the comma directly after the river, namely inhabitants of the area
around Lake Champlain. That is surely not the intended comparison.
The intended meaning of the sentence, of course, is that the “sea monster” reputedly sighted
at Loch Ness is like the “sea monster” reputedly sighted at Lake Champlain.
An additional problem with this sentence is that the separation between one and “sea
monster” is too great; as a result, it is not clear what the word one refers to.
A. This choice suffers from the problems indicated above.
B. Correct. This choice clearly and efficiently conveys its intended meaning.
C. This version of the sentence appears to assert that the Loch Ness “sea monster” actually
exists. Furthermore, it inappropriately uses the phrase is an inland lake to modify Loch
Ness's [sea monster], not Loch Ness.
D. As in choice A, this version of the sentence inappropriately compares Loch Ness's reputed
monster to inhabitants of the area around Lake Champlain.
E. This version of the sentence inappropriately uses the phrase Similar to that reputed to
live in Loch Ness to modify inhabitants of the area around Lake Champlain.
The correct answer is B.

SC43561.01

263. A star will compress itself into a white dwarf, a neutron star, or a black hole after it passes
through a red giant stage, depending on mass.
A. A star will compress itself into a white dwarf, a neutron star, or a black hole after it passes
through a red giant stage, depending on mass.
B. After passing through a red giant stage, depending on its mass, a star will compress itself
into a white dwarf, a neutron star, or a black hole.
C. After passing through a red giant stage, a star's mass will determine if it compresses itself
into a white dwarf, a neutron star, or a black hole.
D. Mass determines whether a star, after passing through the red giant stage, will compress
itself into a white dwarf, a neutron star, or a black hole.
E. The mass of a star, after passing through the red giant stage, will determine whether it
compresses itself into a white dwarf, a neutron star, or a black hole.
Logical predication; Rhetorical construction
The sentence attempts to convey the idea that a star will compress itself into one of three
forms after it passes through the red giant stage: a white dwarf, a neutron star, or a black
hole. The sentence also indicates that which one of these three forms the star will compress
itself into is determined by the star's mass. As worded, these ideas are not clearly and
unambiguously conveyed. For example, it is not clear what the modifying phrase depending
on mass is supposed to modify. Because of its placement, it appears to modify red giant
stage; it should, however, modify star. Given this problem, the sentence fails to convey its
intended meaning clearly.
A. As indicated above, this version fails to convey its intended meaning clearly.
B. The referent of the pronoun its is unclear. Given that red giant stage is the only noun
before its, the sentence would seem to indicate that red giant stage is the referent, but the
meaning would be correct only if its refers to star. Furthermore, the modifying statement
depending on its mass appears to modify red giant stage rather than star.
C. The modifying phrase After passing through a red giant stage modifies a star's mass but
it should describe the star itself. Given that the mass is not what passes through the red
giant stage, this is incorrect.
D. Correct. In this version, it is clear that the sentence is saying that the star itself passes
through the red giant stage and that the star will ultimately compress itself into one of
the three listed options: white dwarf, neutron star, or black hole.
E. The pronoun it refers to the mass of a star, rather than a star, as intended. Likewise, the
star itself passes through the red giant stage, not its mass.
The correct answer is D.

SC53561.01

264. Although many art patrons can readily differentiate a good debenture from an undesirable
one, they are much less expert in distinguishing good paintings and poor ones, authentic art
and fakes.
A. much less expert in distinguishing good paintings and poor ones, authentic art and
B. far less expert in distinguishing good paintings from poor ones, authentic art from
C. much less expert when it comes to distinguishing good paintings and poor ones, authentic
art from
D. far less expert in distinguishing good paintings and poor ones, authentic art and
E. far less the expert when it comes to distinguishing between good painting, poor ones,
authentic art, and
Idiom; Parallelism
The preferred idiomatic form is distinguishing X from Y, rather than distinguishing X and Y.
A. This version uses the incorrect idiomatic form distinguishing X and Y.
B. Correct. This version uses the preferred idiomatic form distinguishing X from Y.
C. This version uses the incorrect idiomatic form distinguishing X and Y. Furthermore,
expert when it comes to is excessively wordy in comparison to expert in as seen in choices
A, B, and D.
D. This version uses the incorrect idiomatic form distinguishing X and Y.
E. Although the form distinguishing between X and Y is an acceptable alternative to
distinguishing X from Y, this version fails to capture that what art patrons have difficulty
distinguishing between are good paintings and poor ones on the one hand, and authentic
art and fakes on the other.
The correct answer is B.
SC83561.01

265. A site once used as an observatory by the Anasazi, ancient pueblo dwellers of New Mexico,
has been recently discovered where patterns of light and shadow were employed to establish
the precise limits of the positions of the Sun and Moon over a nineteen-year cycle.
A. A site once used as an observatory by the Anasazi, ancient pueblo dwellers of New
Mexico, has been recently discovered where patterns of light and shadow were employed
to establish the precise limits of the positions of the Sun and Moon over a nineteen-year
cycle.
B. A recently discovered site was once used as an observatory by the Anasazi, ancient pueblo
dwellers of New Mexico, where patterns of light and shadow were employed to establish
the precise limits of the positions of the Sun and Moon over a nineteen-year cycle.
C. At a recently discovered site once used as an observatory by the Anasazi, ancient pueblo
dwellers of New Mexico, patterns of light and shadow were employed to establish the
precise limits of the positions of the Sun and Moon over a nineteen-year cycle.
D. Patterns of light and shadow were employed to establish the precise limits of the positions
of the Sun and Moon over a nineteen-year cycle at a site that was recently discovered and
was once used by the Anasazi, ancient pueblo dwellers of New Mexico.
E. Patterns of light and shadow were employed to establish the precise limits of the positions
of the Sun and Moon over a nineteen-year cycle at a recently discovered place that the
Anasazi, ancient pueblo dwellers of New Mexico, once used the site as an observatory.
Logical predication; Rhetorical construction
The most reasonable interpretation of this sentence is that there is a recently discovered site
that was used as an observatory by the Anasazi, who were ancient pueblo dwellers of New
Mexico. At this observatory, patterns of light and shadow were employed to establish the
precise limits of the Sun and Moon over a nineteen-year cycle.
The correct answer choice will clearly convey this idea. However, the sentence as worded
does not do so. There is no clear indication of when the patterns of light and shadow were
employed for the indicated purpose.
A. This choice is incorrect for the reason indicated above.
B. This choice suggests that the entire clause where patterns of light and shadow were
employed to establish the precise limits of the positions . . . actually modifies New Mexico
rather than observatory.
C. Correct. This choice eliminates the above flaws; it clearly and cleanly conveys the
intended idea.
D. This choice does not clearly convey the idea that the Anasazi used the site as an
observatory, nor that they employed patterns of light and shadow for the purpose
indicated.
E. This choice does not clearly convey the idea that the Anasazi employed patterns of light
and shadow for the purpose indicated. Furthermore, at a recently discovered place that
the Anasazi . . . once used the site as an observatory unnecessarily inserts the site, which
is redundant (with place) and renders the sentence grammatically incorrect.
The correct answer is C.

SC93561.01

266. The cathedrals of the Middle Ages were community centers just as much as they were purely
religious edifices; and they were structures that represented a city's commitment to a public
realm, the opposite of being a private one.
A. community centers just as much as they were purely religious edifices; and they were
structures that represented a city's commitment to a public realm, the opposite of being a
private one
B. community centers as much as purely religious edifices; they were structures
representing a city's commitment to a public realm, as opposed to private
C. community centers as well as purely religious edifices; they were structures that
represented a city's commitment to a public realm, not private ones
D. as much community centers as purely religious edifices, structures that represented a
city's commitment to a public realm, as opposed to a private one
E. as much community centers as they were purely religious edifices, structures representing
a city's commitment to a public realm, opposite of a private one
Rhetorical construction; Diction
The given sentence conveys the idea that the medieval cathedrals were community centers as
well as religious edifices. Therefore, these cathedrals represented cities' commitment to a
public realm.
A. In the sentence as written, the referent of the pronoun they is unclear. Furthermore, the
sentence is unnecessarily wordy: just as much as would be better written as much as, and
the word being is unnecessary.
B. In this choice, the referent of the pronoun they is unclear. Furthermore, the phrase as
opposed to private is misplaced; it would be correct if placed immediately before realm.
C. In this choice, the plural phrase not private ones refers to the singular a public realm.
D. Correct. This choice avoids using a pronoun with no clear referent and correctly
expresses the comparison between a public realm, as opposed to a private one, where
both noun phrases are singular.
E. This choice uses opposite of a private one to refer to a public realm, unreasonably
suggesting that a public realm cannot coexist with a private realm.
The correct answer is D.
SC14561.01

267. The newspaper story accurately recounted the history of the colonial mansion, that it
contained thirteen rooms, and that it had a reputation for being a haunted house.
A. mansion, that it contained thirteen rooms, and that it had a reputation for being a
haunted house
B. mansion, that it contained thirteen rooms, and that it had a reputation of being haunted
C. mansion, that the mansion contained thirteen rooms, and said that it had a reputation for
being haunted
D. mansion, said that it contained thirteen rooms and had a reputation for being a haunted
house
E. mansion and said that the mansion contained thirteen rooms and had the reputation of
being haunted
Logical predication; Grammatical construction
The given sentence consists of three disjointed pieces; the second and third each begin with
that. The verb recounted appropriately takes history as its object. Note that the two that-
clauses that also appear, inappropriately, to be objects of the verb recount. Furthermore, the
noun history and these that-clauses are not parallel.
Note that these that-clauses work better as objects of the verb said rather than the verb
recount. The correct answer option will need to better integrate the two that-clauses into the
sentence as a whole. A common way in which different thoughts are made into one sentence
is by use of and. Choice E succeeds in creating an integrated sentence using and to combine
three different thoughts.
A. In this choice, the two that-clauses inappropriately appear to be objects of the verb
recount.
B. In this choice, the two that-clauses inappropriately appear to be objects of the verb
recount.
C. In this choice, the first that-clause inappropriately appears to be the object of the verb
recount.
D. In this choice, the referent of the pronoun it is unclear. Furthermore, had a reputation
for being a haunted house is wordy and would be better phrased had the reputation of
being haunted as in choice E.
E. Correct. This choice correctly uses the that-clause as the object of the verb said and uses
the appropriate, less-wordy predicate had the reputation of being haunted.
The correct answer is E.
SC24561.01

268. An archaeological excavation at what might have been a workshop where statues were
reproduced yielded 1,532 fragments of human figures, including 7 intact statues.
A. what might have been a workshop where statues were reproduced yielded 1,532
fragments of human figures, including
B. what might have been a workshop where statues were reproduced yielded 1,532
fragments of human figures and
C. the site of a possible workshop where statues were reproduced yielded 1,532 fragments of
human figures and
D. the site of a possible workshop where statues were reproduced yielded 1,532 fragments of
human figures, including
E. the site of a possible workshop where statues might have been reproduced yielded 1,532
fragments of human figures, including
Logical predication; Grammatical construction
The sentence tells us that an excavation yielded fragments of human figures as well as 7
intact statues. However, as written, the sentence implies that these 7 intact statues were
actually fragments. This contradiction is presumably not intended.
A. This choice is incorrect because it incorrectly implies that the 7 intact statues are
fragments.
B. Correct. The use of the word might makes clear that the suggestion that the structure
was a workshop and that statues were reproduced there is just a hypothesis.
C. This choice is incorrect because it suggests, nonsensically, that the statues were
reproduced in a possible workshop.
D. This choice is incorrect because it suggests, nonsensically, that the statues were
reproduced in a possible workshop. Furthermore, it incorrectly implies that the 7 intact
statues are fragments.
E. This choice is incorrect because it suggests, nonsensically, that the statues were
reproduced in a possible workshop. Furthermore, it incorrectly implies that the 7 intact
statues are fragments.
The correct answer is B.
SC34561.01

269. Sophisticated laser-guided land graders can now flatten uneven farmland almost perfectly so
as not to waste rainwater in runoff down sloping fields.
A. so as not to waste rainwater
B. so that rainwater is not wasted
C. so that there is no wasted rainwater
D. and thereby not waste rainwater
E. and there is no rainwater wasted
Logical predication; Rhetorical construction
The sentence is meant to indicate that the purpose of flattening uneven farmland is to
prevent the waste of rainwater in runoff.
Note that purpose can be expressed in various ways, including by the use of a clause
beginning with so that.
A. In this choice, the implicit subject of the verb waste is the graders. That is, it suggests
that these machines would themselves waste water unless they flattened land.
B. Correct. This choice avoids errors found in the other choices and contains no other
errors.
C. This choice is incorrect because it unnecessarily uses the wordy there is . . . form; it would
be more concise to use the passive-voice verb form no rainwater is wasted, as in choice
B.
D. As in choice A, this choice suggests that the graders themselves might waste rainwater.
E. This choice omits the element of purpose conveyed by so that in choice B: and fails to
convey purpose. As in choice C, the wordy form there is . . . is used.
The correct answer is B.
SC54561.01

270. Because there is not a linguistic census in France, as there is for Britain, there is difficulty in
estimating the number of speakers of Breton, a Celtic language.
A. Because there is not a linguistic census in France, as there is for Britain, there is difficulty
in estimating
B. Because there is no linguistic census in France, unlike Britain, it is difficult to estimate
C. Unlike Britain, there is no linguistic census in France, and that fact makes for difficulty in
estimating
D. There is not a linguistic census in France, as there is for Britain, a fact making for
difficulty in the estimation of
E. There is no linguistic census in France, as there is in Britain, a fact that makes it difficult
to estimate
Logical predication; Rhetorical construction; Parallelism
The sentence suggests that because France, unlike Britain, has no linguistic census, it is
difficult to estimate how many people in France speak Breton.
A. This choice is incorrect because in France is not parallel with for Britain. Furthermore,
this choice uses the wordy construction not a linguistic census as opposed to the clearer
no linguistic census as in choice E.
B. This choice is incorrect because in France is not parallel with unlike Britain.
C. This choice is incorrect because in France is not parallel with unlike Britain.
D. This choice is incorrect because in France is not parallel with for Britain. Furthermore,
this choice uses the wordy construction not a linguistic census as opposed to the clearer
no linguistic census as in choice E. Furthermore, the construction making for difficulty in
the estimation of unnecessarily turns the adjective difficult and the verb estimate into
nouns.
E. Correct. This choice uses the parallel in France and in Britain and correctly uses the
comparison structure there is no X as there is Y.
The correct answer is E.
SC65561.01

271. When adjusted for body weight, children of various age groups in the United States have a
caffeine intake that ranges from 36 to 58 percent of the average amount consumed by adults.
A. children of various age groups in the United States have a caffeine intake that ranges from
36 to 58 percent of the average amount consumed by adults
B. the caffeine intake of children of various age groups in the United States ranges from 36
to 58 percent of the average amount consumed by adults
C. various age groups of children in the United States range in caffeine intake from 36 to 58
percent of that consumed by the average adult
D. in the United States, children of various age groups have a caffeine intake that ranges
from 36 to 58 percent of the average adult's consumption
E. in the United States, the caffeine intake of children in various age groups ranges from 36
to 58 percent of that consumed by the average adult
Logical predication; Rhetorical construction
In a study quantifying caffeine intake averages among children in the United States, an
adjustment in the children's caffeine-intake averages was made for body weight, which varies
with age group.
A. In this choice, the modifying phrase adjusted for body weight is incorrectly applied to
children.
B. Correct. The modifying phrase adjusted for body weight correctly applies to the caffeine
intake of children of various age groups. Furthermore, the caffeine intake of children is
correctly compared with the average amount consumed by adults.
C. In this choice, the modifying phrase adjusted for body weight is incorrectly applied to
various age groups of children.
D. In this choice, the modifying phrase adjusted for body weight is incorrectly applied to
children of various age groups.
E. The word that in the phrase of that consumed by the average adult actually refers to
caffeine intake. This creates a nonsensical redundancy suggesting that the caffeine intake
itself is what is consumed.
The correct answer is B.
SC06561.01

272. When bitter managerial conflicts plague a small company, conflicts that in the past might
have led to dissolution of the business, executives are likely to turn to outside professional
counselors to help resolve disagreement.
A. conflicts plague a small company, conflicts that in the past might have led to dissolution
of the business, executives are likely to
B. conflicts plague a small company, conflicts that might have in the past led to its
dissolution, executives likely will
C. conflicts plague a small company, which in the past it might have led to the business's
dissolution, executives are liable to
D. conflicts, which in the past might have led to dissolution of the business, plague a small
company, executives are liable to
E. conflicts, which in the past might have led to its dissolution, plague a small company,
executives tend to
Logical predication; Rhetorical construction
The sentence is in order as it stands; the best answer choice is option A. The other four
answer choices have significant errors.
A. Correct. This choice is the best answer. It contains neither the errors in the other choices
nor any other errors.
B. The placement of the adverbial phrase in the past creates redundancy because have
already indicates a past tense. Placing the phrase in the past preceding might, as in choice
A, indicates that the adverbial phrase correctly modifies the entire verb might have led.
But likely will . . . is common in speech and somewhat informal; in a formal writing
context, are likely to, as in choice A, is better.
C. This construction appears to make it the subject and which the object of the verb led, but
the pronoun it has no logically plausible referent. Liable is sometimes used informally as
a synonym of likely, but in formal written English, it is typically used only where the
potential outcome is undesirable. In this sentence, likely is rhetorically a better choice.
D. The phrase the business is inappropriate given that no business has, by this point, been
mentioned. The verb plague is too far removed from its subject conflicts, making the
sentence awkward and difficult to read.
E. The possessive adjective its is meant to refer to a small company and could only do so if a
company had already been mentioned. The verb plague is too far removed from its
subject conflicts, making the sentence awkward and difficult to read.
The correct answer is A.
SC17561.01

273. A natural response of communities devastated by earthquake or flood is to rebuild on the


same site, overlooking the possibility that the forces that caused it could be repeated.
A. overlooking the possibility that the forces that caused it could be repeated
B. overlooking the possibility that the forces causing it could be repeated
C. overlooking that the forces that caused the disaster could also cause another one
D. without considering that the forces causing the disaster could be repeated
E. without considering that the forces that caused the disaster could also cause another such
disaster
Rhetorical construction; Logical predication
Communities hit by an earthquake or a flood naturally desire to rebuild in the same place,
yet sometimes fail to consider that the forces that caused the disaster could cause another,
similar disaster.
A. In this choice, possibility is redundant with could. Furthermore, this statement
incorrectly suggests that the forces . . . could be repeated rather than that the disaster
itself could be repeated. Note also that the pronoun it has no clear referent.
B. This statement incorrectly suggests that the forces . . . could be repeated rather than that
the disaster itself could be repeated.
C. This choice uses the idiomatically incorrect form overlooking that. This choice is
confusing also because overlooking has a physical meaning that can apply, for example,
to a site but does not fit with overlooking that.
D. This statement incorrectly suggests that the forces . . . could be repeated rather than that
the disaster itself could be repeated.
E. Correct. This choice clearly conveys the meaning of the sentence and has none of the
flaws mentioned above.
The correct answer is E.

SC67561.01

274. Avalanches at Rogers Pass in Glacier National Park killed more than 200 people between
1885 and 1910, but they are now controlled if not prevented; cannons are fired at the slopes
to make snow masses fall before they become dangerous.
A. Avalanches at Rogers Pass in Glacier National Park killed more than 200 people between
1885 to 1910, but they
B. More than 200 people have been killed by avalanches between 1885 and 1910 at Rogers
Pass in Glacier National Park, but they
C. Between 1885 and 1910, more than 200 people were killed by avalanches at Rogers Pass
in Glacier National Park, but they
D. More than 200 people have been killed by avalanches at Rogers Pass in Glacier National
Park between 1885 and 1910, but such avalanches
E. Avalanches at Rogers Pass in Glacier National Park killed more than 200 people between
1885 and 1910, but such avalanches
Logical predication; Verb form
In the sentence as written, the referent of the pronoun they is unclear. It may look, at first, as
if the referent is the noun phrase Avalanches . . . 1910. However, this does not work as
intended: obviously, avalanches that occurred at the turn of the nineteenth century cannot
now be controlled. It is therefore clear that they does not refer correctly.
A. In this choice, the pronoun they does not refer correctly.
B. In this choice, the pronoun they does not refer correctly. Furthermore, it unnecessarily
uses the passive form people . . . killed by avalanches.
C. In this choice, the pronoun they does not refer correctly. Furthermore, it unnecessarily
uses the passive form people . . . killed by avalanches.
D. In this choice, the verb form have been killed suggests a recent event that perhaps
continues to the present. It would be inappropriate to use this to refer to events that
happened near the turn of the nineteenth century. Furthermore, it unnecessarily uses the
passive form people . . . killed by avalanches.
E. Correct. This choice avoids the unclear pronoun and uses the active form glaciers . . .
killed, which produces a simple and rhetorically effective sentence. Furthermore, the
phrase such avalanches refers, as intended, to actual or possible avalanches in more
recent times than those mentioned in the sentence.
The correct answer is E.
SC77561.01

275. Because 70 percent of the people of India use wood as their sole fuel, ten million acres of
forest have been lost there since 1960, resulting in wood now costing eight times as much to
collect and distribute than in 1960.
A. resulting in wood now costing eight times as much to collect and distribute than
B. resulting in wood now costing eight times as much to collect and distribute as
C. resulting in wood now costing eight times as much to collect and distribute than it did
D. and as a result wood now costs eight times as much to collect and distribute as it did
E. and wood now costs eight times as much as a result to collect and distribute than
Idiom; Grammatical construction
Logically, the sentence attempts to say that the loss of forest has increased the price of wood.
In the sentence as written, as well as choices B and C, the participle resulting modifies ten
million acres of forest rather than the loss.
Note that any of the choices containing the comparison error as much . . . than are incorrect.
This disqualifies A, C, and E. That leaves only B and D for consideration.
A. This choice is incorrect for the reasons stated above. Furthermore, it is unclear what role
costing plays: it could be either an adjective form of the verb to cost or a noun form of to
cost.
B. This choice is incorrect because the participle resulting modifies ten million acres of
forest rather than the loss. Furthermore, it is unclear what role costing plays: it could be
either an adjective form of the verb to cost or a noun form of to cost.
C. This choice is incorrect because the participle resulting modifies ten million acres of
forest rather than the loss. Furthermore, it is unclear what role costing plays: it could be
either an adjective form of the verb to cost or a noun form of to cost.
D. Correct. This clearly expresses the consequence that the loss of forest has increased
prices of wood and uses the correct comparison as much . . . as.
E. This choice is incorrect because it uses the incorrect comparison as much . . . than.
The correct answer is D.
SC09561.01

276. In a crowded, acquisitive world, the disappearance of lifestyles such as those once followed
by southern Africa's Bushmen and Australia's Aboriginal people, requiring vast wild spaces
and permitting little accumulation of goods, seem inevitably doomed.
A. requiring vast wild spaces and permitting little accumulation of goods, seem inevitably
doomed
B. requiring vast wild spaces and permitting little accumulation of goods, seems to be
inevitably doomed
C. which require vast wild spaces and permit little accumulation of goods, seem to be
inevitably doomed
D. lifestyles that require vast wild spaces and permit little accumulation of goods, seem
inevitable
E. lifestyles requiring vast wild spaces and permitting little accumulation of goods, seems
inevitable
Logical predication; Agreement
In the sentence as written, the singular subject the disappearance of . . . and the plural verb
seem do not agree in number. It does not make sense to say that the disappearance is
inevitably doomed; presumably the sentence intends to suggest that the lifestyles themselves
are inevitably doomed or perhaps that the disappearance of . . . these lifestyles is inevitable.
A. For the reasons explained previously, the sentence as written is incorrect.
B. This choice fixes the verb error: the disappearance of . . . and seems agree in number.
However, it retains the problem that the disappearance is what is described as inevitably
doomed.
C. In this sentence, the subject the disappearance of . . . and the verb seem do not agree in
number. Furthermore, the pronoun which incorrectly refers to Aboriginal people rather
than to lifestyles.
D. In this sentence, the subject the disappearance of . . . and the verb seem do not agree in
number.
E. Correct. This sentence is well formed. The singular subject the disappearance agrees in
number with the singular verb seems. Furthermore, it is clearly the disappearance that
seems inevitable.
The correct answer is E.
SC42561.01

277. Before Colette, the female writers of France had been aristocrats, from Mme de Lafayette to
Anne de Noailles; there were no Jane Austens or Brontë sisters, perhaps because there were
almost no clergymen's daughters.
A. were no Jane Austens or
B. were not Jane Austens or
C. was not Jane Austen nor the
D. was not a Jane Austen or the
E. was no Jane Austen or no
Idiom; Diction
The sentence attempts to convey that, before Collette, female writers in France were
members of the aristocracy. Furthermore, the sentence suggests that the explanation of why
there were no non-aristocratic writers—as there had been in England—may be because there
were almost no clergymen's daughters.
The sentence exemplifies the non-aristocratic writers of England by reference to Jane Austen
and the Brontë sisters. However, the sentence uses these authors to indicate a particular type
of author. That is, the point is not to say, for example, that Jane Austen herself did not write
in France at the time, but rather to say that no author like Jane Austen wrote in France at the
time.
This is conveyed idiomatically by saying there were no Jane Austens or Brontë sisters, which
the sentence correctly uses to indicate “there were no authors of the Jane Austen or Brontë
sister type.” Several of the answer choices do not present this idiom correctly.
A. Correct. As indicated above, the sentence is clear and idiomatically correct.
B. Were not Jane Austens or Brontë sisters is not idiomatically correct.
C. Was not Jane Austen nor the Brontë sisters is not idiomatically correct.
D. Was not a Jane Austen or the Brontë sisters is not idiomatically correct.
E. Was no Jane Austen or no Brontë sisters is not idiomatically correct.
The correct answer is A.
SC73561.01

278. Chinese public buildings erected under a construction code of the Sung dynasty have
withstood earthquakes well because the white cedar used has four times the tensile strength
of steel and the timber frame, incorporating many joints and few nails, is flexible.
A. used has four times the tensile strength of steel and the timber frame, incorporating
B. used in them has four times the tensile strength of steel has and the timber frame,
incorporating
C. that was used in them has four times the tensile strength steel has, and the timber frame,
incorporating
D. that was used has four times as much tensile strength as steel, and the timber frame
incorporates
E. that was used has four times the tensile strength steel does, and the timber frame
incorporates
Logical predication; Grammatical construction
The sentence presents two reasons the Chinese buildings built under the code described have
withstood earthquakes: the white cedar used in them has four times the tensile strength of
steel, and the timber frame, which incorporates many joints and few nails, is flexible. The
sentence is correct as worded because it clearly specifies these reasons.
A. Correct. As stated above, this version clearly conveys the intended meaning.
B. The construction has four times the tensile strength of steel has is idiomatically incorrect.
The correct construction is has four times the tensile strength of steel.
C. The comma after has breaks up the sentence in such a way that only the great tensile
strength of white cedar is offered as an explanation of the buildings' ability to withstand
earthquakes, rather than both white cedar's tensile strength and the timber frame's
flexibility. The statement regarding the timber frame's flexibility is mentioned simply as a
separate fact not falling under the scope of because.
D. This choice presents a similar problem to the one in choice C: the comma after steel
breaks up the sentence in such a way that only the great tensile strength of white cedar is
offered as an explanation of the buildings' ability to withstand earthquakes, rather than
both white cedar's tensile strength and the timber frame's flexibility. Furthermore, the
timber frame incorporates many joints and few nails, is flexible is ungrammatical.
E. The construction has four times the tensile strength steel does is incorrect. The correct
construction is has four times the tensile strength of steel. Furthermore, the comma after
does creates the same problem that the initial commas in choices C and D cause,
suggesting that the only factor allowing the buildings to withstand the earthquakes is the
white cedar's tensile strength. Finally, the sentence ends with the same ungrammatical
construction as in choice D: the timber frame incorporates many joints and few nails, is
flexible.
The correct answer is A.
SC28561.01

279. Some historians of science have argued that science moves forward not so much because of
the insights of great thinkers but because of more mundane developments, such as improved
tools and technologies.
A. because of the insights of great thinkers but because of
B. because of the insights of great thinkers as the results of
C. because of the insights of great thinkers as because of
D. through the insights of great thinkers but through
E. through the insights of great thinkers but results from
Idiom; Parallelism
The sentence contains a comparison introduced by not so much. However, the comparison is
executed with correct idiomatic usage only in one of the answer options.
As written, the comparison not so much . . . but . . . is incorrect. This is also true in choices D
and E.
A. This is incorrect for the reason stated above.
B. In this choice, because of is not parallel with the results of.
C. Correct. This uses the comparison not so much . . . as . . . correctly and correctly makes
because of parallel with because of.
D. In this choice, because of is not parallel with through the insights of.
E. In this choice, because of is not parallel with through the insights of.
The correct answer is C.
Answer Explanations Verbal Reasoning
Sentence Correction
Grammar
SC27561.01

280. Indoor air pollution can threaten the health of closely confined farm animals and the
workers who tend them and perhaps as well impairs the quality of such farm products like
eggs, poultry, and pork.
A. perhaps as well impairs the quality of such farm products like
B. perhaps as well impairs the quality of such farm products as
C. perhaps also impairs the quality of such farm products like
D. may also impair the quality of such farm products like
E. may also impair the quality of such farm products as
Agreement; Diction
In the sentence as written, the phrase as well is used incorrectly as a substitute for also.
Furthermore, the incorrect construction such . . . like is used rather than such . . . as.
A. This choice is incorrect for the reasons mentioned above.
B. This choice incorrectly uses as well as a substitute for also.
C. This choice correctly uses also, but it uses the incorrect construction such . . . like.
D. This choice correctly uses also, but it uses the incorrect construction such . . . like.
E. Correct. This choice correctly uses the construction such . . . as and correctly uses also
instead of as well.
The correct answer is E.
SC12811.01

281. Carbon-14 dating reveals that the megalithic monuments in Brittany are nearly 2,000 years
as old as any of their supposed Mediterranean predecessors.
A. as old as any of their supposed
B. older than any of their supposed
C. as old as their supposed
D. older than any of their supposedly
E. as old as their supposedly
Diction; Grammatical construction
The sentence suggests that, based on carbon-14 dating evidence, the megalithic monuments
in Brittany are 2,000 years older than certain other monuments previously believed to
predate them.
The construction 2,000 years as old as . . . fails to convey this age difference; the sentences
using this construction are nonsensical and can be firmly eliminated as possibilities.
A. This choice uses the nonsensical construction 2,000 years as old as.
B. Correct. This choice uses the correct construction older than and correctly uses the
adjective supposed to modify the noun predecessors.
C. This choice uses the nonsensical construction 2,000 years as old as.
D. This choice uses the correct construction older than. However, it incorrectly uses the
adverb supposedly to modify the adjective Mediterranean rather than using supposed as
an adjective modifying predecessors.
E. This choice uses the nonsensical construction 2,000 years as old as. Furthermore, it
incorrectly uses the adverb supposedly to modify the adjective Mediterranean rather
than using supposed as an adjective modifying predecessors.
The correct answer is B.

SC68461.01

282. Some biographers have not only disputed the common notion that Edgar Allan Poe drank to
excess but also questioned whether he drank at all.
A. have not only disputed the common notion that Edgar Allan Poe drank to excess but also
questioned whether he drank
B. not only have disputed the common notion that Edgar Allan Poe drank to excess but also
over whether he drank
C. have disputed not only the common notion that Edgar Allan Poe drank to excess but also
whether he may not have drunk
D. not only have disputed the common notion that Edgar Allan Poe drank to excess but also
questioned whether or not he had drunk
E. have disputed the common notion not only that Edgar Allan Poe drank to excess but also
questioned whether he may not have drunk
Parallelism; Idiom
All the possible answer choices here use the construction not only . . . but also . . . We can
easily eliminate the choices that use the construction inappropriately. The construction is
used appropriately only if what immediately follows not only and but also are grammatically
parallel. For example, if what immediately follows not only is an adjectival phrase, then what
immediately follows but also must also be an adjectival phrase. If what immediately follows
not only is a verb phrase, then what immediately follows but also must also be a verb phrase.
And so on. The sentence here correctly uses the not only . . . but also . . . construction: what
immediately follows not only is a verb phrase (disputed the common notion . . . ), and what
immediately follows but also (questioned whether he drank at all) is a verb phrase as well.
A. Correct. This choice uses not only . . . but also . . . appropriately, employing
appropriately parallel structures.
B. This choice does not use not only . . . but also . . . appropriately, because what
immediately follows not only is not grammatically parallel with what immediately follows
but only.
C. This choice does not use not only . . . but also . . . appropriately, because what
immediately follows not only is not grammatically parallel with what immediately follows
but only.
D. This choice does not use not only . . . but also . . . appropriately, because what
immediately follows not only is not grammatically parallel with what immediately follows
but only.
E. This choice does not use not only . . . but also . . . appropriately, because what
immediately follows not only is not grammatically parallel with what immediately follows
but only.
The correct answer is A.
SC90561.01

283. The large populations and impressive cultural achievements of the Aztecs, the Mayas, and
the Incas could not have come about without corn, which was not only nutritious but also
was able to be dried, transported, and stored for long periods.
A. which was not only nutritious but also was able to be
B. which not only was nutritious but also could be
C. which was not only nutritious but also it could be
D. not only nutritious but it could also be
E. not only nutritious but also able to be
Parallelism; Grammatical construction
To answer this question, we need to understand the correct use of the construction not only .
. . but also . . . . We can easily eliminate the choices that use the construction inappropriately.
Note that whatever immediately follows not only and but also must be grammatically
parallel.
In this sentence, what immediately follows not only is an adjective (nutritious), but what
immediately follows but also is a verb phrase (was able to be dried, . . . ). Therefore, this
sentence does not use not only . . . but also . . . correctly, because what follows not only is not
grammatically parallel with what follows but only.
Furthermore, could be dried is more idiomatically correct than was able to be dried, as is
used here.
A. This choice is incorrect for the reasons discussed above.
B. Correct. This choice uses not only . . . but also … correctly. What immediately follows
not only is a verb phrase (was nutritious), as is what immediately follows but also (could
be dried, . . . ). Therefore, this choice has an appropriate, grammatically parallel
structure.
C. This choice does not use not only . . . but also . . . correctly, because what follows not only
is not grammatically parallel with what follows but only. What immediately follows not
only is an adjective (nutritious), whereas what immediately follows but also is an
independent clause (it could be dried, . . . ).
D. This choice uses a slight variation on the not only . . . but also construction. It is
idiomatically acceptable to separate the but and also; for instance, rather than writing but
also could be, you could correctly write but could also be. This variation, however should
function in essentially the same way as not only . . . but also does. Grammatically parallel
structures must be associated with both not only and but . . . also . . . But that is not the
case in this choice: what is associated with not only is an adjective (nutritious), but what
is associated with but . . . also . . . is an independent clause (it could also be dried, . . . ).
Therefore, this choice is not correct.
E. This choice does not use not only . . . but also . . . correctly, because what follows not only
is not grammatically parallel with what follows but also. What immediately follows not
only is an adjective, whereas what follows but also is a verb phrase. Furthermore, could
be dried is more idiomatically correct than able to be dried, as is used here.
The correct answer is B.

SC91561.01

284. The Rorschach test is gaining new respect as a diagnostic tool because it takes only one hour
to expose behavior and thought processes that may be unlikely to emerge in other procedures
or weeks of ordinary interviewing.
A. that may be unlikely to emerge in other procedures or weeks of ordinary interviewing
B. whose emergence is unlikely in other procedures or weeks of ordinary interviews
C. that might not emerge in other procedures or in weeks of ordinary interviews
D. that may not emerge under other procedures or weeks of ordinary interviews
E. likely not to emerge during weeks of ordinary interviewing or in other procedures
Parallelism; Rhetorical construction
As worded, it is unclear whether other modifies both procedures and weeks or procedures
only. To clarify that other does not modify both procedures and weeks, the correct phrasing
here would be in weeks. Also, the construction that may be unlikely to emerge is needlessly
wordy; that might not emerge is preferable.
A. This choice is flawed for the reasons given above.
B. This choice is also unclear as to what other is intended to modify. Also, whose is generally
restricted to use with people (or, more rarely, animals) rather than with abstractions such
as behavior and thought processes.
C. Correct. This choice clearly expresses the intended idea.
D. Although may and might are often interchangeable, this is not true of may not and might
not. This is because may not can be interpreted to mean are not permitted to. This is of
course not the intended meaning here; might not cannot be interpreted in this way, and
so would be preferable.
E. In this choice, the meaning is unclear. The word other can be interpreted as being
contrasted to ordinary interviewing rather than to the Rorschach test; however, it is clear
that the sentence intends for these other procedures to be contrasted with the Rorschach
test. Furthermore, this version expresses unlikeliness more strongly than the other
versions do: something that is likely not to happen is something that has a very low
probability of occurring, while something that is unlikely to happen has a relatively
higher chance of occurring.
The correct answer is C.
SC82561.01

285. The overall slackening of growth in productivity is influenced less by government regulation,
although that is significant for specific industries like mining, than the coming to an end of a
period of rapid growth in agricultural productivity.
A. the coming to an end of
B. the ending of
C. by the coming to an end of
D. by ending
E. by the end of
Parallelism; Diction
As worded, the sentence is not parallel: less by must be made parallel by using than by.
Thus, the sentence needs to read is influenced less by . . . than by. For this reason, the first
two answer choices can be ruled out.
To choose among the remaining three options, consider which of the coming to an end of,
ending, and the end of best conveys the most likely intended meaning: that the overall
slackening of growth in productivity is largely influenced by the fact that a period of rapid
growth in agricultural productivity is coming to an end.
A. As noted above, this version is not parallel; it lacks the less by . . . than by construction.
B. This version is also not parallel; it lacks the appropriate less by . . . than by construction.
C. Correct. This version correctly uses the less by . . . than by construction. It clearly
expresses the most likely intended meaning, discussed above.
D. This version sounds as if it is saying that the period of rapid growth of agricultural
productivity is intentionally being ended. This deviates from the intended meaning of the
sentence.
E. We can understand this choice to say that the slackening of growth in productivity is
largely influenced by the final part of the period of rapid growth in agricultural
productivity. This also deviates from the intended meaning.
The correct answer is C.

SC13561.01

286. It may someday be worthwhile to try to recover uranium from seawater, but at present this
process is prohibitively expensive.
A. It may someday be worthwhile to try to recover uranium from seawater
B. Someday, it may be worthwhile to try and recover uranium from seawater
C. Trying to recover uranium out of seawater may someday be worthwhile
D. To try for the recovery of uranium out of seawater may someday be worthwhile
E. Recovering uranium from seawater may be worthwhile to try to do someday
Verb form; Rhetorical construction
This sentence is well-formed. It appropriately uses the construction to try to rather than the
common yet idiomatically inappropriate to try and. It also uses the idiomatically correct
construction recover . . . from.
A. Correct. This choice is correct for the reasons discussed above.
B. This choice uses the idiomatically incorrect construction to try and.
C. This choice suggests that what is prohibitively expensive is trying to recover uranium . . .
rather than the process of recovering uranium itself.
D. To try for is idiomatically incorrect, as is the recovery . . . out of.
E. To try to do—as opposed to to attempt, for example—is awkward and unnecessarily
wordy.
The correct answer is A.

SC23561.01

287. The spraying of pesticides can be carefully planned, but accidents, weather conditions that
could not be foreseen, and pilot errors often cause much larger deposits of spray than they
had anticipated.
A. weather conditions that could not be foreseen, and pilot errors often cause much larger
deposits of spray than they had
B. weather conditions that cannot be foreseen, and pilot errors often cause much larger
deposits of spray than
C. unforeseeable weather conditions, and pilot errors are the cause of much larger deposits
of spray than they had
D. weather conditions that are not foreseeable, and pilot errors often cause much larger
deposits of spray than
E. unforeseeable weather conditions, and pilot errors often cause much larger deposits of
spray than they had
Parallelism; Rhetorical construction
The only plural nouns that could potentially be the referent of they in this sentence are
weather conditions and pilot errors. However, neither of these makes logical sense within
the context of the sentence.
For the purposes of this sentence, it is clearer to use the impersonal than anticipated rather
than than they had anticipated. This is because it is not important who is anticipating the
conditions under which the pesticides are sprayed. In other words, the statement is
universal; the point is that these are unknowable conditions, rather than that some particular
group failed to anticipate correctly the size of spray deposits.
A. As indicated above, there is no reasonable referent for the pronoun they.
B. Correct. The idea is conveyed clearly and cleanly.
C. They has no reasonable referent.
D. Weather conditions that are not foreseeable is an awkward construction. Either
unforeseeable weather conditions or weather conditions that cannot be foreseen would
be preferable.
E. They has no reasonable referent.
The correct answer is B.
SC33561.01

288. To read of Abigail Adams' lengthy separation from her family, her difficult travels, and her
constant battles with illness is to feel intensely how harsh life was even for the so-called
aristocracy of Revolutionary times.
A. To read of
B. Reading about
C. Having read about
D. Once one reads of
E. To have read of
Parallelism; Verb form
A common device used to associate two verbs is structured by using the following parallel
form: [infinitive phrase] is [infinitive phrase]. A well-known example of this “To know her is
to love her.” This sentence correctly follows this parallel form.
A. Correct. This choice is well expressed, correctly following the device discussed above.
B. This choice does not correctly follow the device discussed above.
C. This choice does not correctly follow the device discussed above.
D. This choice does not correctly follow the device discussed above.
E. This choice uses an infinitive phrase, but it is not parallel. The simple infinitive to feel is
mirrored by to have read, which is not parallel. This fails to convey properly the
association between the two verbs.
The correct answer is A.

SC74561.01

289. In the traditional Japanese household, most clothing could be packed flatly, and so it was
not necessary to have elaborate closet facilities.
A. flatly, and so it was not necessary to have elaborate closet facilities
B. flat, and so elaborate closet facilities were unnecessary
C. flatly, and so there was no necessity for elaborate closet facilities
D. flat, there being no necessity for elaborate closet facilities
E. flatly, as no elaborate closet facilities were necessary
Diction; Rhetorical construction
Although the word flat is a standard adjectival form, it functions in this context as the more
appropriate adverb, indicating the manner of packing clothing. That is, stated correctly, the
clothing would be packed flat.
Note, however, that the adverb flatly is correctly used in different contexts, such as he flatly
denied it.
A. This choice incorrectly uses the adverb flatly. Furthermore, the phrase it was not
necessary is unnecessarily wordy.
B. Correct. This choice correctly uses the adverb flat and implies that the flat-packing of
clothing led to the absence of closet facilities.
C. This choice incorrectly uses the adverb flatly. Furthermore, there was no necessity for is
unnecessarily wordy; it would be better stated were unnecessary, as in choice B.
D. This choice incorrectly implies that clothing could be packed flat because closet facilities
were unnecessary. Rather, the correct answer must imply that the flat-packing of clothing
made closet facilities unnecessary.
E. This choice incorrectly uses the adverb flatly. It also incorrectly implies that clothing
could be packed flat because closet facilities were unnecessary.
The correct answer is B.

SC75561.01

290. Many states, in search of industries that are clean, fast-growing, and pay good wages to
skilled workers, are trying to attract high-technology industries.
A. clean, fast-growing, and pay
B. clean, grow fast, and that pay
C. clean and fast-growing and that pay
D. clean and grow fast, paying
E. clean, fast-growing, and paying
Parallelism; Verb form
The sentence lists three characteristics of the industries that many states most desire to
attract. The answer choices each provide ways to conjoin these characteristics within the
sentence.
Normally, a series of three nouns or adjectives would have a comma following each one, with
and preceding the third. Note that this method requires adequate parallelism among the
three items.
As written, the third of these characteristics is conveyed by a verb form, either pay or
paying. However, the first of the three items is the adjective clean, which is of course not
parallel with the verb pay.
A. This choice is incorrect for the reasons stated above.
B. The adjective clean is not parallel with pay.
C. Correct. Both clean and fast-growing are adjectives and are therefore parallel; they can
correctly be conjoined by and. This choice creates a sentence with the relative clause that
pay . . . workers. Notice that the sentence has another relative clause that are clean and
fast-growing. This relative clause parallels the other one. Using and to conjoin these two
clauses, each modifying industries, is logical and correct.
D. The adjective clean is not parallel with the verb grow.
E. The adjective clean is not parallel with the verb form paying.
The correct answer is C.

SC95561.01
291. Much of the hope for continued improvement of the economy lies in the projection of
increasing consumer spending this year.
A. projection of increasing consumer spending
B. projection of consumers increasing spending for
C. projected consumer spending increase
D. consumer spending that is projected to increase
E. increase in consumer spending that is projected for
Agreement; Rhetorical construction
Consumer spending helps fuel the economy; when consumer spending increases, the
economy tends to improve. The given sentence suggests that there is hope for continued
improvement in the economy resulting from a projected increase in consumer spending this
year.
Which one of the five answer options best conveys this idea?
A. This choice focuses on projection rather than increase. Furthermore, the role of
increasing is ambiguous: it can either be read as an adjective modifying spending or as a
noun with spending as its object.
B. In this choice, the preposition of incorrectly governs the phrase consumers increasing
spending.
C. This choice correctly focuses on the increase in consumer spending as a whole. However,
it strings together too many adjectives; to make the sentence clearer and more readable, it
would be preferable to state increase first as in choice E.
D. This choice incorrectly focuses on what may be simply one portion of consumer spending:
that which is expected to increase.
E. Correct. With this choice, it is clear that the underlined portion of the sentence focuses
on increase. Use of the preposition for also clarifies that the projected increase is
attributed to this year.
The correct answer is E.

SC46561.01

292. Rejecting its argument that the Masters Dog Training Club's primary aim was to teach
people to train dogs, the court ruled the club ineligible for tax exemption as an educational
group.
A. Rejecting its argument that the Masters Dog Training Club's primary aim was to teach
people to train dogs, the court ruled the club ineligible for tax exemption as an
educational group.
B. In rejecting the Masters Dog Training Club's argument that their primary aim was to
teach people to train dogs, the court ruled the club ineligible to be exempted of taxes as an
educational group.
C. Rejecting the argument that the primary aim of the Masters Dog Training Club was to
teach people to train dogs, the court ruled that the club was ineligible for exemption from
taxes as an educational group.
D. The club was not to be exempted of taxes as an educational group, ruled the court by
rejecting its argument that the primary aim of the Masters Dog Training Club was
teaching people to train dogs.
E. The court ruled the Masters Dog Training Club not eligible to be exempted from taxes as
an educational group, rejecting the argument that the primary aim was to teach people to
train dogs.
Rhetorical construction; Diction
As written, the possessive adjective its fails to make clear whether it is to the court or to the
club that the argument mentioned is being attributed.
Note that choices B and D are immediately disqualified by their use of the incorrect
preposition of immediately following exempted. The correct usages are: exemption from,
exempt from, and exempted from.
A. This choice is incorrect for the reasons mentioned above.
B. This choice uses the idiomatically incorrect form exempted of.
C. Correct. This choice uses the idiomatically correct form exemption from and conveys the
meaning of the sentence in a clear, concise way.
D. This choice uses the idiomatically incorrect form exempted of.
E. This choice uses the idiomatically incorrect form ruled the . . . club rather than the
idiomatically correct ruled that . . . Furthermore, the phrase not eligible to be exempted is
wordy and would be more clearly stated ineligible for exemption as in choice C.
The correct answer is C.

SC56561.01

293. Sartre, an inadvertent guru, had an opinion on everything, painfully considered, elaborately
reasoned, often changed.
A. often changed
B. and it was usually changed
C. that was often changed
D. changing often
E. one he often changed
Parallelism; Diction; Logical predication
The sentence is intended to predicate frequent change of Sartre's opinions. We need to find
the answer choice that most effectively does so.
Note that this modifier in option A, consisting of the adverbial modifier often and the verbal
adjective changed, is most closely parallel to the two preceding predicates.
A. Correct. This choice is correct for the reasons stated above.
B. In this choice the pronoun it could refer either to opinion or to everything, and the force
of usually is unclear. Furthermore, this choice is not parallel in structure to the preceding
modifiers painfully considered and elaborately reasoned.
C. In this choice, the relative pronoun that normally has the closest preceding noun,
pronoun, or noun phrase as its referent. However, in this context, everything is clearly
not the intended referent. Furthermore, this choice is not parallel in structure to the
preceding modifiers painfully considered and elaborately reasoned.
D. This choice is not parallel in structure to the preceding modifiers painfully considered
and elaborately reasoned. This option also unnecessarily inverts the adjective and
adverb, creating a different order from the order in the previous two modifiers.
E. This choice is not parallel in structure to the preceding modifiers painfully considered
and elaborately reasoned.
The correct answer is A.

SC96561.01

294. One analyst of the liquor industry estimated that this year a few liquor stores have
experienced declining sales of up to fifty percent but predicted that the industry as a whole
will maintain a volume of sales fairly close to last year.
A. declining sales of up to fifty percent but predicted that the industry as a whole will
maintain a volume of sales fairly close to last year
B. declines in sales of up to fifty percent but predicted that the industry as a whole would
have maintained a volume of sales fairly close to last year
C. up to fifty percent in declining sales but predicted that the industry as a whole would
maintain a volume of sales fairly close to last year's
D. sales declines of up to fifty percent but predicted that the industry as a whole would
maintain a volume of sales fairly close to last year's
E. declines up to fifty percent of sales but predicted that the industry as a whole will have
maintained a volume of sales fairly close to last year's
Verb form; Rhetorical construction
The sentence reports one analyst's views about the liquor industry's sales performance this
year: there will be large declines in sales in a few liquor stores, and this year's sales volume
will be close to last year's sales volume.
A. This choice is incorrect because it refers to sales themselves rather than to declines in
sales. Furthermore, it compares a volume of sales with a year.
B. This choice incorrectly uses would have maintained after the simple past predicted.
C. The phraseology have experienced up to fifty percent is incorrect, since a percent cannot
be experienced. The phraseology up to fifty percent in declining sales fails to indicate
clearly that sales declined by fifty percent from the previous year.
D. Correct. This choice correctly refers to declines, uses the correct would maintain, and
correctly compares this year's sales volume with last year's sales volume.
E. This choice incorrectly uses will have maintained after the simple past predicted.
The correct answer is D.

SC08561.01
295. Television programs developed in conjunction with the marketing of toys, which was once
prohibited by federal regulations, are thriving in the free market conditions permitted by the
current Federal Communications Commission.
A. Television programs developed in conjunction with the marketing of toys, which was once
prohibited by federal regulations, are
B. Television programs developed in conjunction with the marketing of toys, a practice that
federal regulations once prohibited, is
C. Developing television programs in conjunction with the marketing of toys, as once
prohibited by federal regulations, is
D. Federal regulations once prohibited developing television programs in conjunction with
the marketing of toys, but they are
E. Federal regulations once prohibited developing television programs in conjunction with
the marketing of toys, but such programs are
Logical Predication; Agreement
The sentence is meant to indicate that federal programs once prohibited the development of
television programs in conjunction with the marketing of toys, but such programs are now
thriving in the free market conditions permitted by the Federal Communications
Commission.
As worded, however, which was once prohibited . . . modifies the marketing of toys. This is
clear for two reasons: first, the phrase immediately follows the marketing of toys; second,
the verb was is singular and the noun Television programs is plural and must therefore be
associated with the singular noun marketing of toys.
This indicates that the marketing of toys was once prohibited by federal regulations, but
surely that is not what the sentence is supposed to mean. Rather,
the sentence should assert that the development of television programs in conjunction with
the marketing of toys was prohibited.
A. This choice is incorrect for the reason discussed above.
B. The singular verb is is incorrect. It is the Television programs that are thriving, so the
plural verb are is needed.
C. This option has at least two problems. First, surely the sentence is intended to indicate
that the television programs themselves are thriving rather than that the process of
developing such programs is. Second, it is unclear what the awkwardly worded as once
prohibited by federal regulations is intended to modify. It could mean either that the
development of television programs in conjunction with the marketing of toys was
prohibited or that the marketing of toys itself was prohibited.
D. The intended referent of the pronoun they is unclear: it could be either Federal
regulations or Television programs.
E. Correct. This choice best conveys the intended meaning.
The correct answer is E.

SC98561.01

296. Statisticians studying the health effects of uranium mining on Navajo communities have
found others besides miners and millworkers to be affected; birth defects, children's cancer,
and altered birth ratios of males and females are much higher in mining than in non-mining
communities.
A. children's cancer, and altered birth ratios of males and females are much higher
B. cancer among children, and altered male and female ratios at birth are much higher
C. cancer among children, and altered birth ratios of males and females occur much more
frequently
D. altered birth ratios of males and females, and children's cancer are much higher
E. altered male and female ratios at birth, and cancer among children occur much more
Diction; Logical predication
Statisticians have been studying the frequencies of various health outcomes in uranium
mining areas in Navajo communities. They have found that certain kinds of bad outcomes
occur much more frequently in mining communities than in non-mining communities.
We need to pick the answer choice that best expresses this finding. The use of the predicate
higher is appropriate when applied to rates of illness. However, it is not appropriate when
applied to medical conditions (except as an informal shorthand), e.g., a claim that birth
defects . . . are much higher. That is, it is unclear what a higher birth defect might specify.
The meaning of altered male and female ratios is unclear, as opposed to altered birth ratios
of males and females.
A. In this choice, the statement altered birth ratios . . . are much higher is unclear.
B. In this choice, the statement altered . . . ratios at birth are much higher is unclear.
C. Correct. The idea of the sentence is expressed clearly and the phrase occur much more
frequently clearly specifies altered birth ratios.
D. In this choice, it is unclear what is higher: it could be birth ratios of males and females,
children's cancer, or both.
E. In this choice, the modifier at birth implies, nonsensically, that the ratios were altered at
the time of birth. Furthermore, male and female ratios implies that the ratios themselves
are of male or female gender.
The correct answer is C.

SC19561.01

297. The effect of the earthquake that caused most of Port Royal to sink into the Caribbean was
like the eruption that buried ancient Pompeii: in each case a slice of civilization was instantly
frozen in time.
A. The effect of the earthquake that caused most of Port Royal to sink into the Caribbean
was
B. As the result of an earthquake, most of Port Royal sank into the Caribbean; the effect was
C. In its effects, the sinking of most of Port Royal into the Caribbean was the result of an
earthquake
D. The earthquake that caused most of Port Royal to sink into the Caribbean was, in its
effects,
E. Most of Port Royal sank into the Caribbean because of an earthquake, the effect of which
was
Logical predication; Parallelism
The sentence is meant to indicate that an earthquake caused most of Port Royal to sink into
the Caribbean. However, the sentence as stated has two major flaws:
First, as the sentence is worded, the phrase that caused most of Port Royal to sink into the
Caribbean modifies The effect of the earthquake. This implies that the effect is what caused
most of Port Royal to sink, whereas it was the earthquake itself that caused most of Port
Royal to sink.
Second, the comparison is faulty: the effect is compared to the eruption. Presumably, the
earthquake itself is what is meant to be compared to the eruption.
A. This choice suffers from the two major flaws discussed above.
B. This choice attaches the modifier as the result of an earthquake to most of Port Royal.
This implies, confusingly, that most of Port Royal was the result of an earthquake.
Furthermore, the sentence improperly compares the effect to the eruption, whereas the
earthquake itself is what should be compared to the eruption.
C. This choice places the modifying phrase in its effects immediately before the sinking,
suggesting that we are discussing the effects of the sinking rather than the effects of the
earthquake.
D. Correct. This choice properly compares the earthquake to the eruption that buried
Pompeii.
E. This choice improperly compares an effect of the earthquake to the eruption that buried
Pompeii.
The correct answer is D.
SC58461.01

298. Since the 1930s aircraft manufacturers have tried to build airplanes with frictionless wings,
shaped so smoothly and perfectly that the air passing over them would not become turbulent.
A. wings, shaped so smoothly and perfectly
B. wings, wings so smooth and so perfectly shaped
C. wings that are shaped so smooth and perfect
D. wings, shaped in such a smooth and perfect manner
E. wings, wings having been shaped smoothly and perfectly so
Diction; Logical predication
The sentence as written is not stated clearly. For example, it is unclear what is shaped so
smoothly and perfectly; it could be either the airplanes or the wings. Upon considering the
likely intended meaning, along with a review of the other answer choices, it seems most likely
that the phrase is intended to modify frictionless wings.
Next, we need to consider whether it makes more sense to say that the wings are smooth or
shaped smoothly. This issue can be determined only by considering the most plausible
intended meaning rather than by considering just the grammar of the sentence. The idea of
being smoothly shaped would seem already to be part of the idea of being shaped perfectly,
and therefore it is redundant. On the other hand, having a surface that is very smooth would
be crucial in a quest to make an airplane wing frictionless; for this reason, it makes sense to
point out that the wings themselves are smooth.
A. This choice is incorrect for the reasons stated above.
B. Correct. Given the repetition of the word wings, it is clear that what follows modifies
wings and not airplanes. Describing the wings as smooth and perfectly shaped clearly
conveys what is most likely intended.
C. In this version, note that shaped is modified. For this reason, proper grammar requires
that the adverbs smoothly and perfectly are needed rather than the adjectives smooth and
perfect. However, as discussed previously, the intended meaning of the sentence would
be better conveyed if we used the adjective smooth to modify wings rather than the
adverb smoothly to modify shaped.
D. It is initially unclear what is being described as having been shaped in such a smooth and
perfect manner; it could be either the airplane or its wings. Even if that were clarified,
however, the sentence would be flawed: the sentence is intended to describe the wings
themselves, not the manner in which the wings were shaped. That is, this wording could
be interpreted as referring to the nature of the wings themselves or to the process through
which they were made. This ambiguity makes the meaning of the sentence unclear.
E. This version is very awkwardly worded, mainly because of having been. Furthermore, this
version describes the act of shaping the wings. Because the sentence is instead meant to
describe the wings themselves, this choice is incorrect.
The correct answer is B.

SC29561.01

299. A study of children of divorced parents found that ten years after the parents' divorce,
children who had been under six years of age at the time of the settlement were not
preoccupied, nor even very curious, about the reasons that led to their parents' divorces.
A. not preoccupied, nor even very curious, about the reasons that led to their parents'
divorces
B. not preoccupied with, or even very curious about, the reasons for their parents' divorce
C. neither preoccupied, nor even very curious, with the reasons that led to their parents
divorce
D. neither preoccupied with the reasons that led to their parents' divorces or even very
curious about them
E. neither preoccupied with the reasons that their parents divorced nor even very curious
about it
Diction; Parallelism
The correct idioms are preoccupied with and curious about. In the sentence as it is written,
because of the placement of the commas immediately after preoccupied and curious, the
preposition about is associated not only with curious, which is idiomatically correct, but also
with preoccupied, which is not correct.
Furthermore, not . . . nor is idiomatically incorrect. The correct forms are neither . . . nor and
not . . . or.
A. As discussed above, this choice is flawed in its use of both preoccupied about and not . . .
nor.
B. Correct. This choice is correct. Both preoccupied with and not . . . or are idiomatically
correct.
C. This choice uses the incorrect construction curious with, rather than the correct curious
about. On the other hand, its uses of neither . . . nor and preoccupied with are correct.
D. This choice uses the incorrect idiomatic form neither . . . or.
E. This choice incorrectly uses the singular pronoun it to refer to the plural reasons. On the
other hand, its use of preoccupied with, curious about, and neither . . . nor are all correct.
The correct answer is B.
SC49561.01

300. When Medicare was enacted in 1965, it was aimed at the prevention of a catastrophic illness
from financially destroying elderly patients.
A. at the prevention of a catastrophic illness from financially destroying elderly patients
B. at being a preventive against catastrophic illness financially destroying elderly patients
C. at preventing a catastrophic illness from financially destroying the elderly patient
D. to prevent a catastrophic illness financially destroying an elderly patient
E. to prevent elderly patients being financially destroyed by a catastrophic illness
Verb Form; Diction
Two issues can be identified here: first, we must determine what form follows the phrase it
was aimed, used to describe a goal or intended purpose; second, we must determine what
preposition (if any) connects prevent or its cognates with the thing that is actually prevented.
A. This choice uses the idiomatically correct was aimed at. However, the word prevention
incorrectly takes both the preposition of and the preposition from; this makes the
sentence awkward and ambiguous. Furthermore, using the noun prevention makes the
sentence wordier than choices using some form of the verb to prevent.
B. This choice uses the idiomatically correct was aimed at. However, using the noun a
preventative makes the sentence wordier than cases using some form of the verb to
prevent.
C. Correct. The phrase at preventing is correct in context: it describes Medicare's intended
purpose. Furthermore, the preposition from correctly indicates a consequence that has
been prevented.
D. This choice uses the idiomatically incorrect was aimed to. Furthermore, it fails to use the
idiomatically correct prevent from.
E. This choice uses the idiomatically incorrect was aimed to. Furthermore, it fails to use the
idiomatically correct prevent from.
The correct answer is C.
Appendix Answer Sheet
Quantitative Reasoning
Problem Solving
Counting/Sets/Series
1.
2.
3.
4.
5.
6.
7.
8.
9.
10.
11.
12.
Equalities/Inequalities/Algebra
13.
14.
15.
16.
17.
18.
19.
20.
21.
22.
23.
24.
25.
26.
27.
28.
29.
30.
31.
32.
Geometry
33.
34.
35.
36.
37.
38.
39.
40.
41.
42.
Rates/Ratios/Percent
43.
44.
45.
46.
47.
48.
49.
Value/Order/Factors
50.
51.
52.
53.
54.
55.
56.
57.
58.
59.
60.
61.
62.
63.
64.
65.
66.
67.
Data Sufficiency
Counting/Sets/Series
68.
69.
70.
71.
72.
73.
74.
75.
76.
Equalities/Inequalities/Algebra
77.
78.
79.
80.
81.
82.
83.
84.
85.
86.
87.
88.
89.
90.
91.
92.
93.
94.
95.
96.
97.
98.
99.
100.
101.
102.
103.
104.
105.
Geometry
106.
107.
108.
109.
110.
111.
112.
113.
114.
115.
116.
117.
118.
119.
120.
121.
122.
123.
124.
Rates/Ratios/Percent
125.
126.
127.
128.
Value/Order/Factors
129.
130.
131.
132.
133.
134.
135.
136.
137.
138.
139.
140.
141.
142.
143.
144.
145.
146.
147.
148.
149.
150.

Verbal Reasoning
Reading Comprehension
151.
152.
153.
154.
155.
156.
157.
158.
159.
160.
161.
162.
163.
164.
165.
166.
167.
168.
169.
170.
171.
172.
173.
174.
175.
176.
177.
178.
179.
180.
181.
182.
183.
184.
185.
186.
187.
188.
189.
190.
191.
192.
193.
194.
195.
196.
197.
198.
199.
200.
Critical Reasoning
Analysis/Critique
201.
202.
203.
204.
205.
206.
207.
208.
209.
210.
211.
212.
213.
214.
215.
216.
217.
218.
219.
220.
221.
222.
223.
224.
225.
226.
Construction/Plan
227.
228.
229.
230.
231.
232.
233.
234.
235.
236.
237.
238.
239.
240.
241.
242.
243.
244.
245.
246.
247.
248.
249.
Sentence Correction
Communication
250.
251.
252.
253.
254.
255.
256.
257.
258.
259.
260.
261.
262.
263.
264.
265.
266.
267.
268.
269.
270.
271.
272.
273.
274.
275.
276.
277.
278.
279.
Grammar
280.
281.
282.
283.
284.
285.
286.
287.
288.
289.
290.
291.
292.
293.
294.
295.
296.
297.
298.
299.
300.
GMAT™ Official Advanced Questions Online Index
GMAT™ Official Advanced Questions Online Index
ARE YOU USING BOTH THE BOOK AND THE ONLINE QUESTION BANK TO STUDY?
If so, use the following index to locate a question from the online question bank in the book.
To locate a question from the online question bank in the book: Every question in the
online question bank has a unique ID, called the Practice Question Identifier or PQID, which appears
above the question number. Look up the PQID in the table to find its problem number and page
number in the book.

PQID Question # Page


CR00561.01 240 179
CR00661.01 246 182
CR02531.01 213 166
CR02741.01 236 177
CR03001.01 209 164
CR03161.01 218 168
CR04161.01 219 169
CR05941.01 216 167
CR09090.01 230 174
CR09461.01 220 169
CR10661.01 224 171
CR13661.01 226 172
CR15380.01 207 163
CR20521.01 211 165
CR20661.01 247 182
CR21041.01 214 166
CR23661.01 248 183
CR29111.01 232 175
CR30370.01 204 161
CR30461.01 238 178
CR30721.01 233 176
CR31410.01 201 160
CR33661.01 249 183
CR36441.01 215 167
PQID Question # Page
CR36601.01 231 175
CR38561.01 222 170
CR45650.01 203 161
CR46521.01 212 165
CR47561.01 242 180
CR47931.01 235 177
CR49770.01 228 173
CR51080.01 229 174
CR53140.01 202 160
CR53870.01 206 162
CR60561.01 241 180
CR60661.01 225 172
CR61021.01 210 164
CR66561.01 221 170
CR66590.01 208 163
CR67370.01 227 173
CR69561.01 244 181
CR70870.01 205 162
CR78551.01 237 178
CR78561.01 223 171
CR79561.01 245 182
CR79731.01 234 176
CR87051.01 217 168
CR97561.01 243 181
CR98461.01 239 179
DS00502.01 142 38
DS01451.01 80 23
DS01951.01 82 23
DS02871.01 86 24
DS05502.01 122 33
DS05541.01 132 36
DS06110.01 77 22
DS06351.01 69 20
DS06402.01 141 38
DS07402.01 121 33
PQID Question # Page
DS08402.01 94 26
DS08602.01 147 39
DS12402.01 118 32
DS16291.01 113 30
DS16402.01 75 21
DS17602.01 144 39
DS18041.01 110 30
DS18602.01 105 28
DS19350.01 68 20
DS21891.01 135 37
DS24931.01 78 22
DS26402.01 93 26
DS27602.01 76 22
DS28402.01 95 26
DS29831.01 108 29
DS29931.01 127 35
DS32402.01 138 37
DS33551.01 133 36
DS34010.01 126 34
DS34402.01 120 32
DS35210.01 106 29
DS36141.01 131 36
DS37571.01 111 30
DS37602.01 145 39
DS38302.01 136 37
DS38602.01 148 40
DS41402.01 72 21
DS44402.01 140 38
DS45771.01 112 30
DS46402.01 150 40
DS47602.01 102 28
DS47661.01 84 24
DS48302.01 90 25
DS48391.01 88 25
DS48602.01 124 34
PQID Question # Page
DS49302.01 117 31
DS50502.01 97 27
DS50571.01 85 24
DS51402.01 73 21
DS52402.01 139 38
DS53402.01 119 32
DS53541.01 128 35
DS53841.01 79 22
DS54402.01 74 21
DS56971.01 87 24
DS57602.01 103 28
DS58302.01 116 31
DS59851.01 70 20
DS61791.01 114 31
DS64402.01 92 26
DS65291.01 134 36
DS67410.01 125 34
DS67602.01 104 28
DS69402.01 96 26
DS70061.01 83 23
DS73402.01 149 40
DS76602.01 100 27
DS76851.01 81 23
DS77302.01 115 31
DS81502.01 98 27
DS84302.01 89 25
DS85100.01 129 35
DS85502.01 143 38
DS86602.01 101 28
DS87602.01 123 33
DS88111.01 107 29
DS89302.01 91 25
DS92931.01 109 29
DS94502.01 99 27
DS95491.01 71 20
PQID Question # Page
DS95850.01 130 35
DS97602.01 146 39
DS99302.01 137 37
PS01661.01 53 18
PS03502.01 22 11
PS03551.01 13 9
PS04502.01 25 12
PS04851.01 51 17
PS06502.01 28 12
PS07602.01 12 9
PS12502.01 49 17
PS15302.01 36 14
PS15402.01 11 9
PS17302.01 43 16
PS17402.01 62 19
PS18302.01 38 15
PS18871.01 15 10
PS20502.01 20 11
PS22502.01 41 15
PS23502.01 23 11
PS24831.01 2 7
PS24851.01 52 17
PS25302.01 58 18
PS28602.01 66 19
PS30402.01 60 19
PS30502.01 21 11
PS35302.01 16 10
PS35461.01 33 13
PS35502.01 26 12
PS36302.01 59 18
PS37402.01 63 19
PS40502.01 9 8
PS41471.01 14 9
PS41661.01 54 18
PS43481.01 45 16
PQID Question # Page
PS43661.01 55 18
PS45461.01 5 8
PS45502.01 27 12
PS47302.01 17 10
PS47402.01 44 16
PS54110.01 1 7
PS55471.01 56 18
PS56271.01 34 13
PS56302.01 46 16
PS56441.01 50 17
PS56502.01 29 12
PS57302.01 37 14
PS57402.01 40 15
PS58602.01 31 13
PS61551.01 3 7
PS65402.01 7 8
PS66402.01 61 19
PS66602.01 65 19
PS68602.01 32 13
PS75571.01 35 14
PS76302.01 47 16
PS76402.01 39 15
PS77602.01 30 13
PS78302.01 18 10
PS78602.01 67 20
PS79302.01 19 10
PS83502.01 64 19
PS85402.01 8 8
PS88602.01 42 16
PS92751.01 4 7
PS92981.01 57 18
PS93502.01 24 12
PS95302.01 6 8
PS95402.01 48 17
PS96602.01 10 9
PQID Question # Page
RC04200.01 Passage 145
RC04200.01-10 156 145
RC04200.01-20 157 145
RC04200.01-30 158 146
RC04200.01-40 159 146
RC04200.01-50 160 146
RC04200.01-60 161 146
RC39461.01 Passage 149
RC39461.01-10 166 149
RC39461.01-20 167 149
RC39461.01-30 168 150
RC39461.01-40 169 150
RC39461.01-50 170 150
RC39461.01-60 171 150
RC39461.01-70 172 150
RC49461.01 Passage 151
RC49461.01-10 173 151
RC49461.01-20 174 151
RC49461.01-30 175 152
RC49461.01-40 176 152
RC49461.01-50 177 152
RC59461.01 Passage 153
RC59461.01-10 178 153
RC59461.01-20 179 153
RC59461.01-30 180 154
RC59461.01-40 181 154
RC59461.01-50 182 154
RC59461.01-60 183 154
RC59461.01-70 184 155
RC59461.01-80 185 155
RC60500.01 Passage 147
RC60500.01-10 162 147
RC60500.01-20 163 147
RC60500.01-30 164 147
RC60500.01-40 165 148
PQID Question # Page
RC62100.01 Passage 143
RC62100.01-10 151 143
RC62100.01-20 152 143
RC62100.01-30 153 144
RC62100.01-40 154 144
RC62100.01-50 155 144
RC69461.01 Passage 156
RC69461.01-10 186 156
RC69461.01-20 187 156
RC69461.01-30 188 156
RC69461.01-40 189 157
RC69461.01-50 190 157
RC69461.01-60 191 157
RC69461.01-70 192 157
RC69461.01-80 193 157
RC79461.01 Passage 158
RC79461.01-10 194 158
RC79461.01-20 195 158
RC79461.01-30 196 158
RC79461.01-40 197 159
RC79461.01-50 198 159
RC79461.01-60 199 159
RC79461.01-70 200 159
SC01561.01 255 185
SC06561.01 272 190
SC08561.01 295 195
SC09561.01 276 191
SC12811.01 281 192
SC13561.01 286 193
SC14561.01 267 189
SC17561.01 273 190
SC19561.01 297 195
SC21011.01 250 184
SC21561.01 256 185
SC23561.01 287 193
PQID Question # Page
SC24561.01 268 189
SC27561.01 280 192
SC28561.01 279 192
SC29561.01 299 196
SC30561.01 254 185
SC32561.01 259 186
SC33561.01 288 194
SC34561.01 269 189
SC37561.01 252 184
SC42561.01 277 191
SC43561.01 263 188
SC46561.01 292 194
SC48461.01 253 185
SC49561.01 300 196
SC52561.01 260 186
SC53561.01 264 188
SC54561.01 270 190
SC56561.01 293 194
SC58461.01 298 196
SC61561.01 257 186
SC65561.01 271 190
SC67561.01 274 191
SC68461.01 282 192
SC72561.01 261 187
SC73561.01 278 192
SC74561.01 289 194
SC75561.01 290 194
SC77561.01 275 191
SC81561.01 258 186
SC82561.01 285 193
SC83561.01 265 188
SC83811.01 251 184
SC90561.01 283 193
SC91561.01 284 193
SC92561.01 262 187
PQID Question # Page
SC93561.01 266 189
SC95561.01 291 194
SC96561.01 294 195
SC98561.01 296 195
WILEY END USER LICENSE AGREEMENT
Go to www.wiley.com/go/eula to access Wiley’s ebook EULA.

You might also like